0% found this document useful (0 votes)
377 views

1700 Files Explained by Dr. Khalid Saifullah (Question 1 To Question 1707)

1. A 67-year-old man presented with left-sided ptosis and constricted pupil, loss of pain and temperature sensation on the right side of his body and left side of his face after a stroke. This suggests damage to the medulla oblongata in the brainstem, known as lateral medullary syndrome. 2. Lateral medullary syndrome causes a range of symptoms due to ischemia in the lateral part of the medulla, including dysphagia, slurred speech, ataxia, and Horner syndrome-like symptoms such as miosis and ptosis. 3. The medulla is responsible for motor and sensory functions as well as controlling the vagus and

Uploaded by

ashrafholail
Copyright
© © All Rights Reserved
We take content rights seriously. If you suspect this is your content, claim it here.
Available Formats
Download as DOCX, PDF, TXT or read online on Scribd
0% found this document useful (0 votes)
377 views

1700 Files Explained by Dr. Khalid Saifullah (Question 1 To Question 1707)

1. A 67-year-old man presented with left-sided ptosis and constricted pupil, loss of pain and temperature sensation on the right side of his body and left side of his face after a stroke. This suggests damage to the medulla oblongata in the brainstem, known as lateral medullary syndrome. 2. Lateral medullary syndrome causes a range of symptoms due to ischemia in the lateral part of the medulla, including dysphagia, slurred speech, ataxia, and Horner syndrome-like symptoms such as miosis and ptosis. 3. The medulla is responsible for motor and sensory functions as well as controlling the vagus and

Uploaded by

ashrafholail
Copyright
© © All Rights Reserved
We take content rights seriously. If you suspect this is your content, claim it here.
Available Formats
Download as DOCX, PDF, TXT or read online on Scribd
You are on page 1/ 371

1700 files explained by Dr. Khalid Ans. The key is C. Ectopic ACTH.

[The patient is
smoker and probably developed squamous cell
Saifullah (Question 1 to question
lung cancer which is working as a tumour
1707) producing ectopic ACTH causing pigmentation.
Resulting raised cortisole is leading to diabetes

1.A 65yo man presents with painless and hypokalemia]. Adrenocorticotropic


hematuria, IVU is normal, prostate is hormone (ACTH) --- pituitary gland ----stimulates
mildly enlarged with mild frequency. What the release of cortisol from the cortex of the
is the most appropriate next step? adrenal gland.
a. US Abdomen Hyperpigmentation in primary adrenal cortex
b. Flexible cystoscopy insufficiency: Addison disease. ACTH is thought to
c. MRI have a powerful activating effect on melanocyte-
d. Nuclear imaging stimulating hormone receptors so that high levels
e. PSA spur production of melanin and subsequent
discoloration of the skin or mucosa.
Ans. The key is B. Flexible cystoscopy. [Painless
hematuria in an elderly (here 65 years old man)
indicates carcinoma bladder for which flexible
2. A 44yo woman has lost weight over
cystoscopy is done.
12 months. She has also noticed
Here (BEP -Benign Enlargement of Prostate = episodes where her heart beats
BPH- Benign prostatic hyperplasia) is not
rapidly and strongly. She has a
advanced to cause hemorrhage. There is mild
enlargement of prostate and mild symptoms of regular pulse rate of 90bpm. Her
prostration and hemorrhage is unlikely at this ECG shows sinus rhythm. What is
initial stage of BEP which makes Bladder cancer the most appropriate inv to be
as the likely cause of painless hematuria.It is also done?
less likely to be prostate cancer as symptoms of a. Thyroid antibodies
prostration are mild (indicates disease is not b. TFT
advanced) and no constitutional or other features
c. ECG
suggesting cancer and moreover bleeding in
d. Echocardiogram
cancer prostate is much more less common].
e. Plasma glucose

Ans. The key is B. TFT. [The patient has


2.A 74yo smoker presented to his GP with paroxysmal atrial fibrillation That is why there is
cough and SOB. Exam revealed no arrhythmia in between attacks. From the given
pigmentation of the oral mucosa and also option TFT is the appropriate test as
over the palms and soles. Tests show that thyrotoxycosis is a leading cause of paroxysmal
he is diabetic and hypokalemic. What is atrial fibrillation and this ladies weight loss also
makes thyrotoxycosis as the probable cause
the most probable dx?
a) Pseudocushing syndrome here].
b) Conns disease
c) Ectopic ACTH
d) Cushings disease
e) Hypothyroidism 3.A 79yo anorexic male complains of thirst
and fatigue. He has symptoms of
frequency, urgency and terminal dribbling. d. Fluoxetine
His urea and creatinine levels are high. His e. Phenelzine
serum calcium is 1.9 and he is anemic. His Ans. The key is C. Citalopram. [Among SSRIs
BP is 165/95 mmHg. Sertraline is the drug of choice for ischemic heart
What is the most probable dx? disease. Next choice is citalopram (as it is often
a. BPH related to torsades de pointes). If SSRI cannot be
used Mirtazapine is recommended as next
b. Prostate carcinoma
antidepressant].
c. Chronic pyelonephritis
d. Benign nephrosclerosis Lofepramine is a tricyclic antidepressant.

Dosulepin is a tricyclic antidepressant that


interacts
Ans. The key is B. Prostate Carcinoma. Fluoxetine is a type of antidepressant known as
an SSRI (selective serotonin reuptake inhibitor)
Explanation for Question no. 4:
Phenelzine is an antidepressant (monoamine
First to say in this case (almost all features goes in oxidase inhibitor)
favour of prostatic carcinoma like- frequency,
Sertraline is an antidepressant that belongs to a
urgency and terminal dribbling are features of
group of drugs called selective serotonin reuptake
prostatism; Age, anorexia and anaemia are inhibitors (SSRIs).
constitutional features of carcinoma prostate and
it would be accurate presentation if it was Mirtazapine, is an atypical antidepressant.
hypercalcaemia. But given calcium level is of 5.A 67yo man after a stroke, presents with
hypocalcaemic level and it is the main cause of
left sided ptosis and constricted pupil. He
discrepancy of this question). Renal failure can be
also has loss of pain and temp on the right
an association of malignant disease and can cause
high BP. Thirst is a feature of hypercalcaemia side of his body and left side of his face.
(here may be erroneously calcium level is given in Which part of the brain is most likely
hypocalcaemic level ; probably a bad recall). affected?
Prostate biopsy is the confirmatory diagnosis and A. Frontal cortex
others like PSA is suggestive. B. Cerebellum
C. Pons
***There are some suggestion that Renal Failure
D. Medulla
may be the cause of hypocalcemia.
E. Parietal cortex

4.A 64yo man has recently suffered from


Ans. The key is D. Medulla. [The name of the
an MI and is on aspirin, atorvastatin and
condition is “Lateral medullary syndrome”
ramipril. He hasbeen having trouble [ipsilateral Horner syndrome and contralateral
sleeping and has been losing weight for loss of pain and temperature sense].
the past 4 months. He doesn’t feel like
doing anything he used to enjoy and has
stopped socializing. He says he gets tired Lateral medullary syndrome(Wallenberg's
easily and can’t concentrate on anything. syndrome) is a neurological disorder causing a
What is the most appropriate tx? range of symptoms due to ischemia in the lateral
a. Lofepramine part of the medulla oblongata in the brainstem.
b. Dosulepin The ischemia is a result of a blockage most
c. Citalopram
commonly in the vertebral artery or the posterior achalasia where dysphagia to solid suggest
inferior cerebellar artery. stricture. Also gross dilatation of oesophagus with
smooth narrowing at lower end is seen in
Common symptoms with lateral medullary
achalasia. In achalasia dysphagia is usually
syndrome may include difficulty swallowing, or
described as progressive].
dysphagia. This can be caused by the involvement
of the nucleus ambiguus, as it supplies the vagus
and glossopharyngeal nerves. Slurred speech
(dysarthria) and disordered vocal quality 3. A man undergoes a
(dysphonia) are also common. The damage to the pneumonectomy. After surgery,
cerebellum or the inferior cerebellar peduncle invs show hyponatremia. What
can cause ataxia. Damage to the
could be the cause of the
hypothalamospinal fibers disrupts sympathetic
nervous system relay and gives symptoms that biochemical change?
are similar to the symptoms caused by Horner a. Removal of hormonally active
syndrome – such as miosis, anhidrosis and partial tumor
ptosis. b. Excess dextrose
c. Excess colloid
d. Excessive K+
Palatal myoclonus, the twitching of the muscles e. Hemodilution
of the mouth, may be observed due to disruption
of the central tegmental tract. Other symptoms
include: hoarseness, nausea, vomiting, a decrease Ans. The key is A. Removal of harmonically active
in sweating, problems with body temperature tumour. [Ectopic ACTH secreting tumour causes
sensation, dizziness, difficulty walking, and hypernatremia and body's homeostatic
difficulty maintaining balance. Lateral medullary mechanism try to lower the level of high sodium
syndrome can also cause bradycardia, a slow and do a lesser degree though sodium remains in
heart rate, and increases or decreases in the hypernatremic level or even it may be normal
patient’s average blood pressure (this question does not mention any preoperative
hypernatremia). Removal of that tumour results
in negative sodium balance for time being which
results hyponatremia while gradually it tends to
rise again to normal level].
6.A 60yo man presents with dysphagia
and pain on swallowing both solids and
liquids. A barium meal shows gross
dilatation of the esophagus with a smooth
narrowing at the lower end of the
esophagus. What is the SINGLE most likely 7.A pregnant lady came with pain in her
cause of dysphagia? calf muscle with local rise in temp to the
a. Achalasia antenatal clinic. What tx should be
b. Myasthenia gravis started?
c. Esophageal carcinoma a) Aspirin
d. Esophageal web b) LMWH
e. Systemic sclerosis c) Paracetamol
d) Cocodamol
Ans. The key is A. Achalasia. [Dysphagia for both e) Aspirin and heparin
solid and liquid or prominently liquid suggest
Ans. The key is B. LMWH. [Injections with low Ans. The key is D. Prostate carcinoma. [Age,
molecular weight heparin (LMWH) are usually nocturia, urgency and dribbling points towards
used to treat pregnant women with DVT. LMWH prostate pathology. Pain of lower back and hip
is an anticoagulant, which means it prevents the points towards bony metastases from prostate
blood clot getting bigger. It does not affect the cancer. Blood test for PSA; Prostate biopsy; MRI
developing baby (www.nhs.uk)].  (if initial biopsy is negative, to decide repeat
biopsy).
Treatment options:

8.A 53yo female presents with an acute 1. Active treatment [i) radical prostatectomy ii)
painful hot knee joint. She is a known case radical radiotherapy iii) hormone therapy iv)
of RA. On examination, the knee is red, brachytherapy v) pelvic radiotherapy vi)
orchidectomy
tender and swollen. The hamstring
muscles are in spasm. Her temp is 38.5C 2. Active surveillance
and BP is 120/80mmHg. 3. Watchful waiting
What is the SINGLE best next inv? 4. Palliative care (Source: NICE)].
A. Joint aspiration for cytology and
culture and sensitivity 12. An 18yo female has peri-orbital
B. Joint aspiration for positive blisters. Some of them are crusted, others
birefrengent crystals secreting pinkish fluid. What is the most
C. Joint aspiration for negative likely dx?
birefrengent crystals a. Shingles
D. Blood culture b. Chicken pox
E. Serum uric acid c. Varicella
d. Rubella
Ans. The likely key is A. Joint aspiration for
e. Measles
cytology and culture and sensitivity. [Any
chronically arthritic joint is predisposed to Ans. The key is A. Shingles. [Here ophthalmic
infection. Moreover chronic use of steroid in Rh. division of trigeminal nerve is involved. Typically
arthritis is one of the important predisposing shingles are unilateral].
factor. In this age group likely organism is
Staphylococcus. In younger age group Neisseria
gonorrhea is more common].
13. A 29yo lady who is a bank manager is
referred by the GP to the medical OPC due
to a long hx of tiredness and pain in the
9.An 80yo man presented with pain in his
joints. An autoimmune screen result
lower back and hip. He also complains of
showed smooth muscle antibodies
waking up in the night to go to the
positive. What is the most appropriate
washroom and has urgency as well as
next inv?
dribbling. What is the most likely dx?
a. ECG
a. BPH
b. TFT
b. Prostatitis
c. LFT
c. UTI
d. Serum glucose
d. Prostate carcinoma
e. Jejunal biopsy
e. Bladder carcinoma
Ans. The key is C. LFT. [A case of autoimmune
hepatitis. Autoimmune hepatitis is an uncommon
cause of chronic hepatitis which if untreated can Ans. The key is C. Small cells with round nucleus
lead to cirrhosis. However with treatment outlook and scant indistinct cytoplasm (Lobular
is very good. Smooth muscle antibody is positive carcinoma)
in autoimmune hepatitis. Definitive investigation
is liver biopsy. Treated with steroid [start with
high dose prednisolone]. Azathioprine is 16. A 22yo man has a reduced conscious
commonly added with steroid to reduce its dose level and a fixed dilated pupil after being
as steroid has more side effects than
involved in a MVC(motor vehicle crash) .
azathioprine].
Choose the single most appropriate
option?
a. Facial nerve
14. A 5yo with recurrent chest pain, finger b. Oculomotor nerve
clubbing with offensive stool. Choose the c. Olfactory nerve
single most likely investigation? d. Optic nerve
a. Endomyseal/Alpha glidin antibody e. Trigeminal nerve
b. Sweat test Ans. The key is B. Oculomotor nerve. [3rd nerve
c. Barium meal damage can cause fixed dilated pupil].
d. ECG
e. Glucose tolerance test

Ans. The key is B. Sweat test. [Recurrent chest


17. A man with suspected active TB wants
pain from frequent lung infections including to be treated at home. What should be
pneumonia or bronchitis."Clubbing" of the fingers done to prevent the spread of disease?
is a classic features of Cystic Fibrosis, although a. Immediate start of the tx with Anti-TB
not present in many patients. The digestive drugs
enzymes are not being produced, food is not b. All family members should be
adequately digested (malabsorption) and excess immediately vaccinated with BCG vaccine
fat and protein is lost in the stools, making them c. Patient should be isolated in a negative
bulky, oily, smelly and difficult to flush away]. pressure chamber in his house
d. Universal prevention application protocol

15. A clinical picture of breast cancer Ans. The key is D. Universal prevention
application protocol.
originated from the mammary duct.
Biopsy was done and there were
neoplastic cells found. Choose the 18. A 7yo child is brought to the ED with a
histological picture of the cancer? 1 day hx of being listless. On examination,
a. Neoplastic cells are arranged in small the child is drowsy with an extensive non-
clusters occupying a space between blanching rash. What advice would you
collagen bundles (Seirrhous carcinoma) give the parents?
b. Spindle cell neoplasms with margins,
which infiltrate adjacent structure, fat a. All family members need antibiotic
invaded (Breast sarcoma) therapy
c. Small cells with round nucleus and scant b. Only the mother should be given
indistinct cytoplasm (Lobular carcinoma) rifampicin prophylaxis
c. All family members need isolation
d. All family members should be given precursors of adenocarcinomas and follow a
rifampicin prophylaxis predictable cancerous temporal course unless
interrupted by treatment. They can be either
Ans. The key is D. All family members should be pedunculated or sessile. Polyps are generally
given rifampicin prophylaxis. [Meningococcal asymptomatic but may occasionally ulcerate and
disease. Diagnosis is done with blood or CSF PCR. bleed; uncommonly, they may result in
Initial prehospital management: Benzyl penicillin obstruction if very large.
or cefotaxime].
Adenomas are divided into 3 subtypes based on
histologic criteria, as follows: (1) tubular, (2)
tubulovillous, and (3) villous. According to World
19. A 47yo man has a temp of 39C and is
Health Organization (WHO) criteria, villous
delirious. He has developed blisters mainly adenomas are composed of greater than 80%
on his trunk, which appeared a few hours villous architecture. Tubular adenomas are
ago. He is well and not on any encountered most frequently (80-86%).
medications. He last travelled 5 months Tubulovillous adenomas are encountered less
ago to Italy. Which of the following is the frequently (8-16%), and villous adenomas are
most likely dx? encountered least frequently (5%).
a. Shingles
Villous adenomas are associated more often with
b. Chicken pox
larger adenomas and more severe degrees of
c. Pemphigoid
dysplasia. These adenomas occur more
d. Bullous pemphigus frequently in the rectum and rectosigmoid,
although they may occur anywhere in the colon.
Ans. The key is B. Chicken pox. [Adults more
They generally are sessile structures that appear
commonly develop a more generalized brain
as velvety or cauliflowerlike projections. See the
inflammation ("encephalitis") whose symptoms
images below.
may include delirium and seizures. Incubation
period of chicken-pox is 10-21 days. So this travel Note that the vast majority of patients are
history is not significant]. asymptomatic and have unremarkable laboratory
findings

The most common presenting symptom is


20. A 64yo pt has been having freq occult/overt bleeding (hematochezia) with an
episodes of secretory diarrhea, which is anemia, which may be microcytic. Nonspecific
extremely watery, with large amts of symptoms include diarrhea, constipation, and
mucus. A dx of villous adenoma was made flatulence. Pencil-thin stools, if truly present,
after endoscopy. What electrolyte would be secondary to large distal adenomas or
abnormality is most likely in this patient? frank carcinomas.
a. Hyperkalemia
Villous adenomas rarely cause a secretory
b. Hypernatremia
diarrhea syndrome. The tumor usually is located
c. Hyponatremia
at the rectosigmoid or rectum and often is 3-4 cm
d. Hypokalemia in diameter. Stool volumes of 350-3000 mL are
reported and may cause hypovolemia and
Ans. Key not given. Correct key is both C and D!
metabolic imbalances.
[Villous adenoma can cause both hyponatremia
and hypokalemia]. A full colonoscopy is the accepted procedure of
choice .The literature supports the use of NSAIDs.
Adenomatous polyps are, by definition,
neoplastic. Although benign, they are the direct
21. A pt with an acute gout attack came c. Replace morphine with oral
to the ED. What drug should be given to hydromorphone
relieve symptoms? d. Replace morphine with oxycodone
e. Subcutaneous morphine
a. NSAIDs
b. Allopurinol Ans. The key is D. Replace morphine with
c. Ibuprofen oxycodone.

Ans. The key is A. NSAIDs. [Oral NSAIDs 24. A 40yo woman notices increasing
commenced immediately and continue for 1 – 2 lower abdominal distention with little/no
weeks; Colchicine can be effective alternative but pain. On examination, a lobulated cystic
is slower to work than NSAIDs. Intra articular mass is felt and it seems to be arising
corticosteroids are highly effective in acute gouty from the pelvis. What is the most
monoarthritis.
appropriate inv?
22. A pt was lying down on the operating
a. CA 125
table in a position with his arms hanging b. CA 153
down for 3 hours. Soon after he woke up, c. CA 199
he complains of numbness and weakness d. CEA
in that hand and has limited wrist e. AFP
movement/wrist drop and sensory loss
Ans. The key is A. CA 125. [Ovarian ca is the likely
over dorsum of that hand, weakness of diagnosis for which tumour marker is CA 125].
extension of the fingers and loss of
sensation at the web of the thumb. What
structure is likely to be damaged? 25. A resident of a nursing home
a. Radial nerve presented with rashes in his finger webs
b. Median nerve and also on his abdomen, with complaints
c. Ulnar nerve of itching which is severe at night. He was
d. Axillary nerve dx with scabies. What the best tx for his
e. Suprascapular nerve condition?
Ans. The key is A. Radial nerve. [Here arm hanging a. 0.5% permethrin
down compressing the radial nerve at the spiral b. Doxycycline
groove is the cause of given scenario]. c. 5% permethrin
d. Reassure
e. Acyclovir
23. A pt who was previously on 120mg
Ans. The key is C. 5% permethrin. [Scabies
slow release oral morphine has had his
outbreaks in nursing homes and cases of crusted
dose increased to 200mg. He is still in scabies may require combination therapy
significant pain. He complains of consisting of topical application of permethrin
drowsiness and constipation. What is the and 2 oral doses of ivermectin at 200 mcg/kg
next step in the management? (administered 1 wk apart)].

a. Increase slow release morphine dose


b. Fentanyl patch
26. A 34 yo alcoholic is found passed out d. Arteriography
in front of a local pub. The ambulance e. 24h ECG
crew informs you that he was sweating
Ans. The key is c. [A case of TIA. Probable cause
when they found him and there were cans carotid artery narrowing. Treated with Aspirin
of cider lying empty around him. What is 300 mg daily for 2 weeks then aspirin + modified
the initial stage of investigation? release dipiridamole daily [NICE guidelines].
a. Capillary blood sugar
b. CT head 29. A man complains of loss of sensation
c. MRI head in his little and ring finger.
d. ABG
e. MCV
Which nerve is most likely to be involved?
a. Median nerve
Ans. The key is A. Capillary blood sugar. [Alcohol
b. Ulnar nerve
induced hypoglycemia can present as this case].
c. Radial nerve
27. A young boy fell on his outstretched d. Long thoracic nerve
hand and has presented with pain around e. Axillary nerve
the elbow. He has absent radial pulse on Ans. The key is B. Ulner nerve. [Compression of
the affected hand. What is the most likely ulner nerve at the elbow, known as cubital tunnel
dx? syndrome, causes numbness in the 5th (pinky)
finger, along the half (lengthwise) of the 4th (ring)
a. Dislocated elbow
finger closest to the 5th finger, and the back half
b. Angulated supracondylar fx
of the hand over the 5th finger].
c. Undisplaced fx of radial head
d. Posterior dislocation of shoulder 30. A young man complains of double
vision on seeing to the right. Which nerve
Ans. The key is B. Angulated supracondyllar fx.
is most likely to be involved?
[Damage or occlusion of the bracheal artery is the
cause of absent radial pulse. Often closed a. Left abducens
reduction results in restoration of normal b. Right abducens
anatomy and correction of occlusion of bracheal c. Left trochlear
artery and establishes circulation again but in few d. Right trochlear
instances open reduction is required to fix the
e. Right oculomotor
occluded artery].
Ans. The key is B. Right abducens. [Diplopia on
seeing to right indicates right lateral rectus palsy
28. A 65yo woman presented with which is supplied by right abducent nerve].
transient arm and leg weakness as well as 31. A 45yo man keeps having intrusive
a sudden loss of vision in the left eye. Her thoughts about having dirt under the bed.
symptoms resolved within the next couple He can’t keep himself from thinking about
of hours. What is the most appropriate these thoughts. If he tries to resist, he
next investigation? starts having palpitations. What is the
a. CT brain most likely dx?
b. Echo
a. OC personality
c. Doppler USG
b. OCD
c. Schizophrenia 34. A 32yo man presented with painless
d. Panic disorder hematuria. He is hypertensive but the rest
e. Phobia of the exam is unremarkable. What is the
most likely dx?
Ans. The key is B. [Here patients thoughts are
a. Polycystic kidneys
obsession and though no compulsive act is
b. Ca bladder
described (like repeated cleansing of dirt) but his
c. Ca prostate
nature of thought like inability to resist the
thinking or getting palpitation on trying to avoid d. TTP
thinking can be regarded as compulsion of e. HUS
thought. CBT 1st line. SSRIs].
Ans. The key is A. Polycystic kidneys. [Painless
haematuria at an younger age with hypertension
is suggestive of polycystic kidney disease. Renal
32. A 33yo man presents with an itchy ultrasound is used to diagnose the condition].
scaly annular rash on his thigh after a
walk in the park. Which of the
following drugs will treat his condition? 35. A 45yo female complains of pain in the
inner side of her right thigh. She was dx
a. Erythromycin with benign ovarian mass on the right.
b. Doxycycline Which nerve is responsible for this pain?
c. Penicillin a. Femoral nerve
d. Amoxicillin b. Obturator nerve
c. Iliohypogastric nerve
Ans. The key is B. Doxycycline. [Itchy scaly annular
d. Ovarian branch of splanchic nerve
rash after a walk in the park indicates erythema
e. Pudendal nerve
migrans caused by the spirochete Borrelia
Burgdorferi transmitted by bite of pinhead-sized Ans. The key is B. [The Obturator nerve is
ixodes ticks leading to lyme disease]. responsible for the sensory innervation of the
skin of the medial aspect of the thigh].
33. A pt with cerebral mets has polyuria
and polydipsia. What part of the brain
would be affected?
36. A 37yo lady strongly believes that a
a. Cerebral cortex famous politician has been sending her
b. Cerebellum flowers every day and is in love with her.
c. Diencephalon However, this is not the case.
d. Pons
e. Medulla What is the most likely dx?

Ans. The key is C. Diencephalon. [Diencephalon is a. Erotomania


the caudal (posterior) part of the forebrain, b. Pyromania
containing the epithalamus, thalamus, c. Kleptomania
hypothalamus, and ventral thalamus and the d. Trichotillomania
third ventricle. Hypothalamus produce ADH and e. Grandiosity
hens lesion of diencephalon (hypothalsamus) may
produce cranial diabetes insipidus. Ans. 1. The key is A. Erotomania. [Erotomania is a
type of delusion in which the affected person
believes that another person, usually a stranger,
high-status or famous person, is in love with d. Multiple immature granulocytes with
them]. blast cells
e. Numerous blast cells
Pyromania is an impulse control disorder in
which individuals repeatedly fail to resist Ans. The key is B. Excess of mature lymphocytes.
impulses to deliberately start fires, in order to [Dx is CLL. Age of patient (usually above 50 yrs),
relieve tension or for instant gratification. lymhadenopathy and splenomegaly, appearance
of lymphocytes (mature lymphocytes – but
functionally not normal). Repeated chest
Kleptomania is the inability to refrain from the infection points towards abnormal function of
urge to steal items. lymphocytes against infection].
Trichotillomania is an impulse disorder
characterized by the compulsive urge to pull
out one's hair, leading to noticeable hair loss 39. A lady presents with itching around
and balding. the breast and greenish foul smelling
Grandiosity refers to an unrealistic sense of discharge from the nipple. She had a
superiority. similar episode before. What is the most
likely dx?
a. Duct papilloma
37. A 3yo child has been brought with
b. Duct ectasia
facial lacerations. On examination he has
c. Breast abscess
some cuts over his right cheek and under
d. Periductal mastitis
the eye. The GCS on initial evaluation is e. Mammary duct fistula
15. What is the appropriate next inv?
a. Skull XR Ans. The key is B. Duct ectasia. [Duct ectasia of
b. Facial XR the breast or mammary duct ectasia or plasma
c. CT scan cell mastitis is a condition in which the lactiferous
d. MRI duct becomes blocked or clogged. This is the
e. Observation most common cause of greenish discharge.
Mammary duct ectasia can mimic breast cancer.
Ans. The key is B. Facial X-ray. [Normal GCS makes It is a disorder of peri- or post-menopausal age].
intracranial lesion less likely. As there is facial
injury to exclude any facial bone fracture we can
do facial X-ray]. 40. A young male whose sclera was noted
to be yellow by his colleagues has a hx of
taking OTC drugs for some pain. Tests
 38. A 73yo woman has lymphadenopathy showed raised bilirubin, ALT and AST
and splenomegaly. She feels well but has normal. The provocation test with IV
had recurrent chest infections recently. nicotinic acid is positive and produces
Choose the single most likely blood film further rise in the serum bilirubin levels.
findings? What is the most likely dx?
a. Acute hepatitis
a. Atypical lymphocytes
b. Drug hypersensitivity
b. Excess of mature lymphocytes
c. Gilberts syndrome
c. Plasma cells
d. Acute pancreatitis
Ans. The key is C. Gilbert’s syndrome. [Only a. Pulmonary embolism
unconjugated bilirubin is increased but not the b. Cardiac tamponade
liver enzymes. Also positive nicotinic acid c. Pericardial effusion
provocation test is in its favour]. d. Hemothorax
e. Pneumothorax

41.A 24yo biker has been rescued after Ans. The key is B. Cardiac tamponade. [chest is
being trapped under rocks for almost 12h. clear, so there is no pneumothorax or pleural
effusion. Muffled heart sound is due to fluid in
He complains of reddish brown urine. His
pericardial space, low BP from reduced chamber
creatinine is 350umol/L and his urea is
expansion due to pericardial fluid’s pressure and
15mmol/L. What is the most important restricted right heart expansion causes raised
step in the management of this patient? JVP].
a. Dialysis
b. IV NS 44. A 50yo pt is admitted for elective
c. IV dextrose herniorraphy. Which of the following
d. IV KCl options will lead to a postponement of the
e. Pain relief operation?
Ans. Key is B. IV NS. [It is a case of rhabdomyolysis a. SBP 110mmHg
which is initially treated with IV NS]. b. MI 2 months ago
c. Hgb 12g/dl
d. Pain around hernia
42. A 74yo man who has been a smoker e. Abdominal distention
since he was 20 has recently been dx with
Ans. The key is B. MI 2 months ago. [After MI
SCLC. What serum electrolyte picture will
elective surgery should not be done before 6
confirm the presence of SIADH? months post MI, as operation in earlier than this
time has significant increase in mortality].
a. High serum Na, low serum osmolarity,
high urine osmolarity
45. A 32yo woman of 39wks gestation
b. Low serum Na, low serum osmolarity,
attends the antenatal day unit feeling very
high urine osmolarity
unwell with sudden onset of epigastric
c. Low serum Na, high serum osmolarity,
pain associated with nausea and
high urine osmolarity
vomiting. Her temp is 36.7C. Exam: she is
d. High serum Na, low serum osmolarity,
low urine osmolarity
found to have RUQ tenderness. Her blood
e. High serum Na, high serum osmolarity,
results show mild anemia, low platelets,
low urine osmolarity elevated liver enzymes and hemolysis.
What is the most likely dx?
Ans. . The key is B. Low serum Na, low serum a. Acute fatty liver of pregnancy
osmolarity, high urine osmolarity. b. Acute pyelonephritis
c. Cholecystitis
43. A man brought into the ED after being d. HELLP syndrome
stabbed in the chest. Chest is bilaterally e. Acute hepatitis
clear with muffled heart sounds. BP is
60/nil. Pulse is 120bpm. JVP raised. What Ans. The key is D. HELLP syndrome. [The main
treatment is to deliver the baby as soon as
is the most likely dx?
possible [as early as after 34 weeks if multisystem Ans. 1. The key is D. Subdural hematoma. [In
disease is present]. elderly head injury usually leads to subdural
hematoma even if head injury is minor or trivial
and extradural hematoma in elderly is extremely
46. A woman comes with an ulcerated
uncommon even in more severe head injury.
lesion 3 cm in the labia majorum. What is Management: 1st line: Evacuation by barr hole
the lymphatic drainage of this area? craniostomy. 2nd line: Craniotomy if the clot is
organized].
a. External iliac
b. Superficial inguinal LN
c. Para-aortic
49. A 25yo female complains of
d. Iliac
e. Aortic intermittent pain in her fingers. She
describes episodes of numbness and
Ans. Key is B. Superficial inguinal LN. burning of the fingers. She wears gloves
47. A man post-cholecystectomy whenever she leaves the house. What is
presented with jaundice, fever and dark the most probable dx?
urine. What is the most diagnostic inv? a. Kawasaki disease
a. ERCP b. Takayasu arteritis
b. USG Abdomen c. Buerger’s disease
c. CT Scan d. Embolism
d. MRCP e. Raynaud’s phenomenon
e. MRI

Ans. The key is A. ERCP [Post operative US will not Ans. The key is E. Raynaud’s phenomenon.
give good results. We shall not go for ercp first as [Intermittent nature points towards some triggers
it has complications like pancreatitis. Acceptable and wearing of gloves during going out indicates
options are CT, MRI and MRCP among which most cold weather. Also female sex makes the
easiest and less time consuming but with very diagnosis of Raynaud’s phenomenon more likely].
good test result is CT scan. So CT is most
appropriate! But as the Question wants most
diagnostic it is ERCP (though not practical)!!! The 50. A 22yo lady has been unwell for some
diagnosis here is choledocolithiasis with
time. She came to the hospital with
cholangitis].
complaints of fever and painful vesicles in
48. A 79yo stumbled and sustained a her left ear. What is the most probable
minor head injury 2 weeks ago. He has dx?
become increasingly confused, drowsy
and unsteady. He has a GCS of 13. He a. Acne
takes warfarin for Afib. What is the most b. Herpes zoster
likely dx? c. Chicken pox
a. Extradural hemorrhage d. Insect bite
b. Cerebellar hemorrhage e. Cellulitis
c. Epidural hemorrhage
Ans. The key is B. Herpes Zoster. [This is a case of
d. Subdural hemorrhage
Herpes zoster oticus and if facial nerve is also
e. Subarachnoid hemorrhage involved then it is called Ramsay Hunt syndrome].
51. A 5yo girl had earache and some e. Pyridoxine
yellowish foul smelling discharge, Ans. The key is A. Folic acid. [Frequently
perforation at the attic and conductive associated with neural tube defect].
hearing loss. She has no past hx of any ear
infections. What is the most appropriate 53. A 23yo woman has been having pain
at the base of her thumb, the pain is
dx?
reproduced when lifting her 3 month old
a. Acute OM baby or changing diapers and also with
b. OM with effusion forceful abduction of the thumb against
c. Acquired cholesteatoma resistance. What is the likely cause?
d. Congenital cholesteatoma a. Avascular necrosis of scaphoid
e. Otitis externa b. Trigger finger.
c. De Quervain’s tenosynovitis

Ans. The key is c. Acquired cholesteatoma.


[Acquired cholesteatomas develop as a result of Ans. The key is c. De Quervain’s tenosiovitis. [Can
chronic middle ear infection and are usually be diagnosed by Finkelstein’s test:
associated with perforation of the tympanic
membrane at the attic (mass is seen in attick with The physician grasps the thumb and the hand is
perforation at pars flaccida- in contrast to medial ulnar deviated sharply. If sharp pain occurs along
to tympanic membrane which is in congenital). the distal radius (top of forearm, about an inch
Clinical presentation usually consists of below the wrist), de Quervain's syndrome is
conductive hearing loss, often with purulent likely].
discharge from the ear]. 

54. A 6m child presents with fever and


In congenital:
cough. His mother has rushed him to the
• mass medial to the tympanic membrane
• normal tympanic membrane ED asking for help.O/E: temp=39 'C and
• no previous history of ear discharge, the child is feeding poorly. Dx?
perforation or ear surgery.
a. Bronchiolitis
[discrepancy like no previous infection is the only b. Asthma
point in favour of congenital here and may be due c. Bronchitis
to a bad recall!!!]
Ans. The key is A. Bronchiolitis. [Management: 1.
Oxygen inhalation 2. Nasogastric feeding. DON’T
USE: i) bronchodilator ii) steroid iii) antibiotics
52. A female with T1DM would like to
routinely. [OHCS, 9th edition, page-160]
know about a deficiency of vitamins in
pregnancy that can be harmful. A
deficiency of which vitamin can lead to 55. A 75yo man collapsed while walking in
teratogenic effects in the child? his garden. He recovered fully within 30
a. Folic acid mins with BP 110/80 mmHg and regular
b. Vit B12 pulse of 70bpm. He has a systolic murmur
c. Thiamine on examination. His activities have been
d. Riboflavine.
reduced lately which he attributes to old Q.1. What is the key.
Q.2. What drugs should you use in i) tobacco
age. What is the definitive diagnostic inv
abuse and in ii) alcohol abuse?
that will assist you with his condition?
a. ECG Ans. 1. Key is d. Methadone. (used in opiate
b. Echo abuse)
c. 24h ECG monitoring
Ans. 2. i) tobacco abuse: a) bupropion ii) alcohol:
d. 24h BP monitoring
a) acamprosate b) disulfirum
e. Prv CIN
58. A 16m child presents with drooling,
Ans. B. Echo. [Aortic stenosis – more likely in
elderly. And hypertrophic cardiomyopathy – less sore throat and loss of voice. He has fever
likely in this age, presentation may be in an with a temp of 38.2C. What is your next
earlier age]. step towards management?
a. Direct pharynoscopy
b. Call ENT surgeon
56. A 35yo man with a hx of
c. Call anesthesiologist
schizophrenia is brought to the ER by his d. IV fuilds
friends due to drowsiness. On examination e. Start antibiotics
he is generally rigid. A dx of neuroleptic
malignant syndrome except: Q. 1. What is the key?
Q. 2. What is the diagnosis?
a. Renal failure Q. What is the urgent management?
b. Pyrexia
c. Elevated creatinine kinase Ans. 1.The key is C. Call anesthesiologist. [As an
d. Usually occurs after prolonged tx expert to intubate].
e. Tachycardia
Ans. 2. Diagnosis is Acute epiglottitis.

Ans. 3. In given case urgent intubation is needed


Ans. The key is D. Usually after prolonged tx. to secure airway to prevent blockage of
[Malignant neuroleptic syndrome usually occurs respiration.
SOON after start or in increasing dose of
antipsychotic drugs! All others mentioned are
seen in this syndrome like renal failure, pyrexia, 59. A 62yo woman complains of
elevated creatinine kinase, tachycardia etc]. unsteadiness when walking. On
examination she has pyramidal weakness
of her left lower limb and reduced pain
57. A 33yo drug addict wants to quit. She and temp sensation on right leg and right
says she is ready to stop the drug abuse. side of trunk up to the umbilicus. Joint
She is supported by her friends and family. position sense is impaired at her left great
What drug tx would you give her? toe but is normal elsewhere. She has a
a. Benzodiazepines
definite left extensor plantar response and
b. Diazipoxide
the right plantar response is equivocal.
c. Lithium
Where is the lesion?
d. Methadone
e. Disulfiram
a. Left cervical cord Ans. The key is A. Haemochromatosis.
b. Midline mid-thoracic cord [Hemochromatosis itself is a cause of
c. Right mid-thoracic cord hepatocellular carcinoma and associated with
d. Left mid-thoracic cord raised level of ferritin. Serum ferritin levels
e. Left lumbo-sacral plexus elevated higher than 200 mcg/L in
premenopausal women and 300 mcg/L in men
and postmenopausal women indicate primary
iron overload due to hemochromatosis, especially
Q. 1. What is the key?
when associated with high transferrin saturation
Q. 2. What is the name of this condition?
and evidence of liver disease. Ferritin
concentration higher than 1000 mcg/L suggests
Ans. 1. The key is d. Left mid-thoracic cord. liver damage with fibrosis or cirrhosis].

Ans. 2. Brown-sequard syndrome. [In brown-


sequard syndrome paralysis and loss of
proprioception on the same (or ipsilateral) side as 62. A woman has electric pains in her face
the injury or lesion, and loss of pain and that start with the jaw and move
temperature sensation on the opposite (or upwards. Her corneal reflexes are normal.
contralateral) side as the lesion]. What is the most likely dx?
a. Atypical face pain
b. Trigeminal neuralgia
60. A 26yo man presents to ED with c. Tempero-mandibular joint dysfunction
increasing SOB on left side and chest pain. d. GCA
He has been a heavy smoker for the past 4 e. Herpes zoster
years. He doesn’t have any past med hx.
Ans. 1. Key is b. Trigeminal neuralgia. [Electric
What is the likely dx? pains in her face that starts with the jaw and
a. Pulmonary embolism
moves upwards [this description indicates
b. MI neurological pain of trigeminal nerve. In
c. Asthma trigeminal neuralgia transient loss of corneal
d. Pleural effusion reflexs are seen just after attacks but in between
e. Pneumothorax attacks corneal reflexes are quite normal].

Ans. The key is e. Pneumothorax. [Increased


shortness of breath and chest pain with no past
medical history favours the dx of pneumothorax.
63. A 32yo man presented with slow
Heavy smoking or tobacco is a risk factor for progressive dysphagia. There is past hx of
spontaneous pneumothorax]. retro-sternal discomfort and he has been
treated with prokinetics and H2 blockers.
What is the probably dx?
61. A pt with hepatocellular ca has raised a. Foreign body
b. Plummer vinson syndrome
levels of ferritin. What is the most
c. Pharyngeal puch
probable cause?
d. Peptic stricture
a. Hemochromatosis e. Esophageal Ca
b. A1 antitrypsin def
c. Cystic fibrosis Ans. 1. The key is D. Peptic stricture. [Progressive
dysphagia to mostly solid is suggestive of peptic
stricture which is supported here by the use of
prokinetic drugs and H2 blockers which are used on examination shows a perforated nasal
for reflux oesophagitis]. septum. What is the most likely dx?
a. Marijuana OD
b. Cocaine OD
64. A 56yo man comes with hx of right c. Heroin OD
sided weakness & left sided visual loss. d. Alcohol OD
Where is the occlusion? e. CO poisoning
a. Ant meningeal artery
b. Mid meningeal artery
c. Mid cerebral artery Q. 1. What is the key?
d. Carotid artery Q. 2. What are the points that favours the
e. Ant cerebral artery diagnosis in given question?
f. Ant communicating artery Q. 3. What are other important findings?

Ans. 1. Key is B. Cocaine overdose.


Q. 1. What is the key?
Q. 2. How will you differentiate between middle Ans. 2. Points in favour: i) Tachyrhythmia ii) High
cerebral artery occlusion from anterior cerebral fever iii) perforated nasal septum iv)
artery occlusion? unconsciousness

Ans. 3. Other findings of cocaine toxicity: i)


Ans. 1. The key is d. Carotid artery. [Carotid artery Psychiatric: anxiety, paranoia ii) Tachypnoea iii)
divides to internal and external carotid of which Increased energy and talking rapidly iv) Dilated
internal continues as middle cerebral ultimately. pupils. Also: [rhabdomyolysis, metabolic acidosis,
But just before it becomes middle cerebral convulsion].
internal carotid gives rise to ophthalmic branch.
So middle cerebral occlusion may give partial
visual loss but not complete mono-ocular 66. A 56yo pt whose pain was relieved by
blindness. For complete mono-ocular blindness oral Morphine, now presents with
occlusion should be proximal to ophthalmic progressively worsening pain relieved by
artery i.e. either in internal carotid or more increasing the dose of oral morphine.
proximally to carotid artery].
However, the pt complains that the
Ans. 2. increased morphine makes him drowsy
and his is unable to carry out his daily
i) Middle cerebral artery occlusion:
paralysis or weakness of contralateral activities. What is the next step in his
face and arm (faciobracheal). Sensory management?
loss of the contralatera face and arm. a. Oral oxycodone
b. Oral tramadol
ii) Anterior cerebral artery occlusion: c. PCA
paralysis or weakness of the contralateral d. IV Fentanyl
foot and leg. Sensory loss at the
e. Diamorphine
contralateral foot and leg.

65. A young college student is found in his


dorm unconscious. He has Ans. The key is oral oxycodon.
tachyarrhythmia and high fever. He also
seems to be bleeding from his nose, which
67. A 30yo man presents with a 5cm neck Ans. 2. The diagnosis is opiate overdose.
mass anterior to the sternocleido-mastoid Ans. 3. Points in favour are: i) reduced
muscle on the left side in its upper third. consciousness ii) RR 8/min (12<) iii) hypotension
He states that the swelling has been (here lower normal) iv) miosis v) needle track
treated with antibiotics for infection in the marks on his arms.
past. What’s the most likely cause?
Methadone as an essential medicine for the
a. Branchial cyst
management of opioid dependence.
b. Parotitis
c. Pharyngeal pouch Naloxone is a medicine that rapidly reverses an
d. Thyroglossal cyst opioid overdose, used to treat known or
e. Thyroid swelling suspected opioid overdose.

69. A 30yo man and wife present to the


reproductive endocrine clinic because of
Q. 1. What is the key?
infertility. The man is tall, has bilateral
Q. 2. Justify your answer.
gynecomastia. Examination of the testes
reveals bilateral small, firm testes. Which
Ans. 1. The key is A. Branchial cyst. of the following inv is most helpful in dx?
Ans. 2. i) Branchial cyst is anterior triangular a. CT of pituitary
lump. [parotid is also anterior triangular lump but b. Chromosomal analysis
parotitis regresses with appropriate treatment i.e. c. Measure of serum gonadotropins
becomes normal in size]. ii) pharyngeal pouch is d. Measure of serum testosterone
posterior triangular lump. iii) Thyroglossal is
midline lump. iv) thyroid swelling moves with Q. 1. What is the key?
Q. 2. What is the diagnosis?
swallowing].
Q. 3. What are the points in favour of your
diagnosis?

68. An 18yo man is rushed into the ER by


Ans. 1. The key is B. Chromosomal analysis.
his friends who left him immediately
before they could be interviewed by staff. Ans. 2. The diagnosis is Klinefelter’s syndrome.
He is semiconscious, RR=8/min, (xxy)
BP=120/70mmHg, pulse=60bpm. He is Ans. 3. The points in favour are: i) Infertility ii) Tall
noted to have needle track marks on his stature iii) Bilateral gynaecomastia iv) Bilateral
arms and his pupils are small. What is the small firm testes.
single best initial tx?
a. Insulin
b. Naloxone 70. An 18yo female just received her A-
c. Methadone Level results and she didn’t get into the
d. Gastric lavage university of her choice. She was brought
Q. 1. What is the key? into the ED after ingestion of 24
Q. 2. What is the diagnosis? paracetamol tablets. Exam: confused and
Q. 3. What are the points in favour of the tired. Initial management has been done.
diagnosis? Inv after 24h: normal CBC, ABG = pH7.1,
PT=17s, Bilirubin=4umol/L,
Ans.1. The key is B. Naloxone.
creatinine=83umol/L. What is the next Ans. 2. Alcohol consumption [Alcoholism can
step in management? cause urinary retention but it is a less common
a. Observation for another 24h cause of retention].
b. Refer to psychologist
c. Give N-Acetylcysteine
d. Discharge with psychiatry referral
e. Liver transplantation 72. In CRF, main cause of Vit D deficiency
is the failure of:

Q. 1. What is the key? a. Vit D absorption in intestines


Q. 2. What are the indications of this b. 25 alpha hydroxylation of Vit D
management? c. Excess Vit D loss in urine
d. 1 alpha hydroxylation of Vit D
Ans. 1. The key is E. Liver transplantation. e. Availability of Vit D precursors

Ans. 2. King's College Hospital criteria for liver Ans. The key is D. 1 alpha hydroxylation of Vit D.
transplantation in paracetamol-induced acute
liver failure. 73. Pt with puffiness of face and rash
arterial pH <7.3 or arterial lactate >3.0 mmol/L showing cotton wool spots on fundoscopy.
after adequate fluid resuscitation, OR What’s the dx?
if all three of the following occur in a 24-hour
a. Macular degeneration
period:
Creatinine >300 μmol/L. b. Hypertensive retinopathy
PT >100 seconds (INR >6.5). c. Diabetic background
Grade III/IV encephalopathy. d. Proliferative diabetic retinopathy
e. SLE

Q. 1. What is the key?


***71. A 75yo alcoholic presents with a Q. 2. Why there is puffyness of face?
mass up to umbilicus, urinary dribbling, Q. 3. Why there is cotton wool spots on
incontinence, and clothes smelling of fundoscopy? What is the most common ocular
ammonia. What is the next step in manifestation of SLE?
management?
a. Urethral catheter Ans. 1. The key is SLE.
b. Suprapubic catheter
c. Antibiotics Ans. 2. Puffiness is due to lupus nephritis.
d. Condom catheter Ans. 3. SLE, can involve the retina. The classic
e. Nephrostomy lesion of SLE is a white fluffy appearing lesion
within the retina known as a cotton wool spot.
The most common ocular manifestation in SLE is
Q. 1. What is the key? Keratoconjunctivits sicca.
Q. 2. What is the cause of this retention?

Ans. 1. The key is A. Urethral catheter.


74. A 35yo man presents with progressive
breathlessness. He gave a hx of
polyarthralgia with painful lesions on the
shin. CXR: bilateral hilar
lymphadenopathy. What’s the most likely a. b. KUB XR
dx? b. Pelvic US
a. Bronchial asthma c. Rectal exam
b. Cystic fibrosis d. Abdominal US
c. Sarcoidosis
d. Bronchiectasis
e. Pneumonia Q. 1. What is the key?
Q. 2. What is the diagnosis?
Q. 1. What is the key? Q. 3. What are the points given here in favour of
Q.2 . What is the specific name of this condition? your diagnosis?
What is the triad?

Ans. 1. The key is E. Abdominal US.


Ans. 1. The key is C. Sarcoidosis. Ans. 2. The diagnosis is Abdominal aortic
Ans. 2. Lofgren syndrome. The triad is i) Erythema aneurism.
nodosum ii) Bilateral hilar lymphadenopathy iii) Ans. 3. Points in favour of AAA are i) hypertension
Arthralgia. ii) abdominal pain iii) back pain iv) expansile
abdominal mass lateral and superior to the
Sarcoidosis is a disease characterized by the umbilicus.
growth of tiny collections of inflammatory cells
(granulomas) in any part of your body — most
commonly the lungs and lymph nodes. But it can 77. A 55yo man has had severe pain in the
also affect the eyes, skin, heart and other organs.
right hypochondrium for 24h. The pain
75. A child presents with clean wound, but comes in waves and is accompanied by
he has never been immunized as his nausea. Nothing seems to relieve the pain.
parents were worried about it. There is no He feels hot and sweaty but has normal
contraindication to immunization, what is temp. What is the most appropriate next
the best management? inv?
a. Full course of DTP a. US Abdomen
b. 1 single injection DT b. ERCP
c. 1 single injection DTP c. MRCP
d. Only Ig d. Serum amylase
e. Antibiotic e. UGI endoscopy

Ans. The key is A. Full course of DTP. Q. 1. What is the key?


Q. 2. What is the diagnosis?
Q. 3. What are the points in favour of your
diagnosis?
76. A 65yo HTN man presents with lower
abdominal pain and back pain. An
expansive abdominal mass is palpated Ans. 1. The key is A. US abdomen.
lateral and superior to the umbilicus. Ans. 2. The diagnosis is biliary colic.
What is the single most discriminating
Ans. 3. Points in favour- i) severe right
inv?
hypochondrial pain. ii) colicky nature of the pain
a. Laparascopy
(comes in waves) iii) nausea iv) absence of fever
iv) absence of jaundice. [OHCM, 9th edition, page-
637].
Ans. 3.

 Iron-deficiency anaemia.
78. A 67yo man has deteriorating vision in
 Folate deficiency or vitamin B12
his left eye. He has longstanding COPD deficiency.
and is on multiple drug therapy. What
single medication is likely to cause this  Bleeding, resulting from low vitamin K.
visual deterioration?  Oedema, which occurs in protein/calorie
a. B2 agonist malnutrition.
b. Corticosteroid
c. Diuretic 80. A 35yo male is bitterly annoyed with
d. Theophylline people around him. He thinks that people
are putting ideas into his head. What is
the single most likely dx?
Q. 1. What is the key? a. Thought block
Q. 2. What is the cause of deteriorating vision? b. Thought insertion
c. Thought broadcasting
d. Thought withdrawal
Ans. 1. The key is B. Corticosteroid.
e. Reference
Ans. 2. Prolonged corticostiroids [also topical i.e.
eye drop] can cause cataract. Q. 1. What is the key?
Q. 2. In which disease you will find this feature?
Ans. 1. The key is B. Thought insertion.

79. A woman who returned from abroad Ans. 2. It is seen in schizophrenia.


after 3 weeks of holiday complains of
severe diarrhea of 3 weeks. She also
81. A 10yo girl presents with hoarseness
/
developed IDA Iron deficiency anemia and of the voice. She is a known case of
folic acid def. What condition best bronchial asthma and has been on oral
describes her situation? steroids for a while. What is the most
a. Jejunal villous atrophy likely cause of hoarseness?
b. Chronic diarrhea secretions a. Laryngeal candidiasis
c. Malabsorption b. Infective tonsillitis
d. Increased catabolism c. Laryngeal edema
e. Increased secretions of acid d. Allergic drug reaction
e. Ludwigs angina

Q. 1. What is the key? Q. 1. What is the key?


Q. 2. What are the points in favour? Q. 2. What is the reason for this condition?
Q. 3. What are the signs of deficiency may be
present? Ans. 1. The key is A. Laryngeal candidiasis.
Ans. 1. The key is C. Malabsorption.
Ans. 2. Steroids predisposes to fungal infection.
Ans. 2. Diarrhoea, IDA and folic acid deficiency.
82. A lady with breast cancer has Half of this calculated volume is given in the first
undergone axillary LN clearance. She 8 hours and the other half is given over the
develops arm swelling after being stung following 16 hours.
by a bee. What is the most likely
mechanism responsible for the swelling?
a. Lymphedema 84. A 54yo man has recently been dx with
b. Cellulitis moderate depression. He has hx of MI and
c. Hypersensitivity reaction is suffering from insomnia. What is the
d. DVT drug of choice for him?
e. Fluid retention a. Citalopram
b. Lofepramine
Q. 1. What is the key? c. ECT
Q. 2. What is the reason for this condition? d. Haloperidol
e. Diazepam
Ans. 1. The key is A. Lymphoedema.

Ans. 2. Reason is compromised lymphatic


Ans. Key is A. Citalopram. [Sertraline is the drug
drainage of arm due to axillary LN clearance. So if
there is any inflammation or selling, lymph of choice in post MI as citalopram may be a risk
drainage is compromised further giving rise to factor for precipitating torsades-de-pointes. But if
swelling of limb of that operated side. sertraline is not in option second choice is
citalopram and where SSRIs are not suitable next
choice is mirtazapine].
83. A 34yo pt presents with 50% partial
thickness burns. What should be the most
appropriate management? 85. A man presented with cellulitis and
a. IV fluids calculated from the time of swelling. He was started on flucloxacillin.
hospital arrival What other medication do you want to
b. IV fluids calculated from the time of burn add?
c. No IVF a. Vancomycin
d. IV dextrose stat b. Penicillin
e. Burns ointment c. Metronidazole
d. Ceftriaxone
e. Amoxicillin
Q. 1. What is the key? Q. 1. What is the key?
Q. 2. How the calculation of fluid is made? Q. 2. Please mention why?

Ans. 1. The key is B. IV fluids calculated from the Ans. 1. The key is B. Penicillin.
time of burn.
Ans. 2. cellulitis is usually caused by
Ans. 2. Resuscitation fluids required in the first 24 staphylococcus and streptococcus. To cover both
hours from the time of injury. Flucloxacillin (for staphylococcus) and Penicillin
For adults: 3 ml (in partial thickness burn) (to cover streptococci) should be prescribed.
of Hartmann’s solution/kg body weight/% total

Body surface area.


86. A 24yo college student presents with abnormal valves: aortic or mitral valve disease,
nausea, vomiting, headache, neck tricuspid valve in IV drug users, prosthetic valves.
stiffness and a fever of 38.4C. What is the A hypochondriac is someone who lives with the
most appropriate empirical antibiotic to fear that they have a serious, but undiagnosed
be started? medical condition, even though diagnostic tests
a. Ceftriaxone show there is nothing wrong with them
a. Ceftriaxone 88. A 45yo man with posterior gastric
b. b. Penicillin ulcer presented with severe excruciating
c. Gentamicin pain which subsided after conservative
d. Tazobactam treatment. 10 days later he developed
e. Meropenem swinging pyrexia. US shows a collection in
the peritoneum. What will be the most
likely location of the collection?
Ans. The key is A. Ceftriaxone. [Cfftriaxone is the a. Hepatorenal puch
drug of choice in hospital management. Probable b. Left paracolic gutter
dx is meningitis. [In OHCM-Cefotaxime <55yrs and c. Subphrenic
Cefotaxime + Ampicillin if age >55yrs (pre- d. Pelvic cavity
hospital)].
e. Lesser sac

Ans. The key is E. Lesser sac.


87. A man with prosthetic heart valve
89. A 23yo lady was prescribed with
underwent hemicolectomy and after some
azithromycin 1gm for her chlamydial
days complains of left hypochondriac pain,
pelvic infection. She has got a new
fever and has a systolic murmur. What is
boyfriend for the last 2 months. She has
the next inv to ascertain the cause of HF?
a. CT recently started contraception to avoid
b. Blood culture conception. Which of the following
c. ECG contraception method will be affected by
d. MRI azithromycin?
e. Radioactive thyroid scan a. Barrier
b. IUCD
Q. 1. What is the key? c. POP- progestogen-only pill
Q. 2. What is the diagnosis? d. COCP - combined oral contraceptive pill
Q. 3. Why have you made this diagnosis?
Q. 4. What are the important risk factors for this Ans. None of them! Before it was thought that
condition? hepatic enzyme inhibitor drugs may affect COCP
but later it was established that actually there is
Ans. 1. The key is B. Blood culture. no such significant effect. Only drugs like
refumpicin, rifabutin etc. can cause this. [For
Ans. 2. The diagnosis is infective endocarditis. exam purpose if you have to choose one please
choose D. COCP/
Ans. 3. Fever + new murmur = endocarditis until
proven otherwise.

Ans. 4. Important risk factors: dermatitis, IV 90. An 11yo boy is being checked by the
injections, renal failure, organ transplantation, diabetic specialist nurse. His HbA1c was
DM, post operative wond. Risk factors for
high and he has been skipping meals 92. A man has reducible bulge below the
recently. He has been unhappy at school. pubic tubercle, and on occlusion of the
Which single member of the clinical team deep inguinal ring, cough impulse is
would you refer him to next? present. What is the most likely dx?
a. GP a. Direct inguinal
b. Pediatrician b. Indirect inguinal
c. Dietician c. Femoral
d. Clinical psychologist d. Spigelian
e. Lumbar

Q. 1. What is the key?


Ans. The key is D. Clinical psychologist. [Skipping
Q. 2. What are the points in favour of your
meals, unhappy at school these are psychological answer?
issues. So he should be referred to clinical
psychologist].
Ans. 1. The key is C. Femoral hernia.
Ans. 2. It is just below the pubic tubercle that is
just below the inguinal ligament.
91. A 35yo man who has served in the
army presents with lack of interest in
enjoyable activities and feeling low. He 93. A 48yo woman is admitted to ED with
doesn’t feel like reading the news or a productive cough and moderate fever.
watching movies as he believes there is She has often central chest pain and
violence everywhere. What is the most regurgitation of undigested food most
appropriate first line therapy? times but doesn’t suffer from acid reflux.
a. Citalopram These symptoms have been present for
b. Lofepramine the last 3.5 months which affects both
c. CBT food and drink. A CXR shows an air-fluid
d. Chlordiazepoxide level behind a normal sized heart. What is
e. Desensitization the most likely dx?
a. Pharyngeal pouch
Q. 1. What is the key?
b. Hiatus hernia
Q. 2. What is the diagnosis?
c. Bulbar palsy
Q. 3. What is the first line treatment?
d. Achalasia
e. TB
Ans. 1. The key is C. CBT

Ans. 2. The diagnosis is mild depressive illness.


Q. 1. What is the key?
Ans. 3. In mild depressive illness CBT is preferred
Q. 2. What are the points in favour?
option.

Cognitive behavioral therapy (CBT) usually lasts Ans. 1. The key is D. Achalasia.
for between 5 and 20 sessions, with each session
lasting 30 to 60 minutes, CBT works by changing Ans. 2. Points in favour: Aspiration pneumonia
people's attitudes and their behavior by focusing due to retained food and fluid in oesophagus. In
on the thoughts, images, beliefs and attitudes. achalasia usually there is no acid reflux.
Dysphagia for both food and drink. Air-fluid level
behind heart.
Why it is not hiatus hernia? Ans. Differentiating Q. 1. What is the key?
point:-i) In hiatus hernia usually you will get Q. 2. What is the type of the given case?
associated GORD ii) Also in hiatus hernia there Q. 3. What are the points in favour of mentioned
may be nausea or vomiting. type?

Why it is not pharyngeal pouch? Ans. In


pharyngeal pouch there will be halitosis. Ans. 1. The key is C. Migraine.

Ans. 2. It is migraine without aura.

Ans. 3. Criteria of migraine without aura: ≥5


94. A 64yo man has been waking up in the
headaches lasting 4-72 hours + nausea/vomiting
middle of the night to go to the bathroom.
(or photo/phono-phobia) + any 2 of: i) unilateral
He also had difficulty in initiating ii) pulsating iii) worsen by routine activity [OHCM,
micturition and complains of dribbling. A 9th edition, page-462].
dx of BPH was made after a transrectal US
guided biopsy and the pt was prepared for
a TURP(Transurethral resection of the prostate). 96. A 35yo man presented with
What electrolyte abnormality is highly hematuria, abdominal swelling and has a
likely due to this surgery? BP of 190/140. What is the most
a. Hypokalemia
diagnostic inv?
b. Hypocalcemia a. Cystoscopy
c. Hyperkalemia b. USG
d. Hyponatremia c. CT
e. Hypernatremia d. Renal biopsy
e. Urine analysis
Q. 1. What is the key?
Q. 2. Why this happens?
Q. 1. What is the key?
Q. 2. What is the diagnosis?
Ans. 1. The key is D. Hyponatremia. Q. 3. What will be the USG findings to establish
diagnosis in given case?
Ans. 2. Absorption of fluid used for bladder
irrigation to flush out blood clots and IV fluids all
may lead to hypervolaemia and dilutional Ans. 1. The key is B. USG.
Ans. 2. The diagnosis is ADPKD(Autosomal
hyponatremia.
dominant polycystic kidney disease)
Ans. 3. In given case patients age is 35. So the
USG diagnostic criteria is: Age 18 – 39 yrs>3
95. A 56yo lady has developed severe unilateral or, bilateral cysts, 40 – 59 yrs >2 cysts in
right sided headache which worsens each kidney, >60 yrs >4 cysts in each kidney.
whenever she comes to bright light since [OHCM, 9th edition, page- 312].
the last 4 days. She feels nauseated, but
doesn’t vomit. What is the most likely dx? 97. A young man is brought to the ED
a. SAH after an RTA. His GCS on initial evaluation
b. Brain tumor is 6. What is the most appropriate next
c. Migraine step?
d. Cluster headache a. CT
e. Subdural headache b. MRI
c. IV fluids
d. Skull XR Ans. 1. The key is D. Retinitis pigmentosa.
e. Secure airway
Ans. 2. i) It is not angle closure glaucoma as angle
Ans. The key is E. Secure airway. closure glaucoma occurs usually after the age of
50; In open angle glaucoma visual loss is not
evenly gradual rather occurs a bit suddenly at its
later part. It is not cataract as cataract occurs
98. A 65yo man presented with frank
usually in elderly. In macular degeneration near
hematuria. He has no other urinary blindness does not occur rather causes inability to
symptoms. What is the most appropriate identify face or cannot read small prints;
next step that will lead to the dx? otherwise peripheral vision is not that depressed.
a. IVU In keratitis will be pain, redness, photophobia and
b. US Abdomen vision is ok.
c. Cystoscopy
d. Mid-stream urine for culture
e. Transrectal US 100. A 27yo lady has had an
uncomplicated pregnancy so far. She
came to the hospital 2h ago after her
Q. 1. What is the key? water broke. The midwife is looking at her
Q. 2. What is the diagnosis? now. She has regular contractions. P.V
Q. 3. What are the reasons for this diagnosis?
Q. 4. If there is painless haematuria in young (say
exam revealed 2cm dilated cervix. Vital
25-30yrs) what diagnosis will come first? signs are normal.
What stage of labour is she in?
a. Second stage
Ans.1. Key is C. Cystoscopy.
b. First stage
Ans. 2. Bladder cancer. c. Latent stage
d. Third stage
Ans. 3. Age 65, asymptomatic haematuria.
e. Active phase
Ans. 4. ADPKD [at the beginning there is very few
or no symptoms].
Ans. The key is B. First stage starts with softening
of cervix with start of opening of cervix and ends
99. A 30yo woman had a gradual when cevix is fully dilated (i.e. 10 cm dilated).
decrease of visual acuity since the last 3 [Question is asking stage of labour].
years. Now she has a disability due to very
low vision. What’s the dx?
a. Glaucoma 101. A 2yo boy fell off his tricycle and hurt
b. Cataract his arm. He got up to start crying, but
c. Macular degeneration before there was any sound, he went pale,
d. Retinitis pigmentosa unconscious and rigid. He recovered after
e. Keratitis 1-2 mins but remained pale. After an hour
he was back to normal. His mother says
she was afraid he was going to die, and
Q. 1. What is the key? that he had a similar episode 3 months
Q. 2. Why it is not the other given D/D s? prior after falling down some steps. What
single inv is indicated?
a. CT head Ans. The key is B. Cervical/vaginal trauma.
b. EEG [complication of forceps delivery].
c. CBC
104. A 50yo man has had anterior
d. None
resection of the rectum for carcinoma. He
e. Skeletal survey
expressed concerns about control of post-
Q. 1. What is the key? op pain in discussions with the
Q. 2. What is the diagnosis? anaesthetist before surgery. What is the
best management strategy?
Ans. 1. The key is D. None. a. Oral diclofenac
Ans. 2. Diagnosis is breath holding spell. b. Oral codeine
c. IM morphine
d. IM dihydrocodeine
102. A 29yo woman had just delivered a e. Ondansetron oral
still born vaginally, following a major Ans. The key is C. IM morphine. [Some post
placental abruption. Choose the single operative pain is severe pain which needs strong
most likely predisposing factor for opioid analgesic].
developing PPH in this lady?
a. Retained product 105. A 73yo male presents with enlarged
b. DIC cervical nodes. He has had recurrent
c. Fibroid uterus infections over the last year. His
d. Uterine infection conjunctiva is pale. Choose the single cell
e. Large placental site type you will find on the blood film.
a. Granulocyte without blast cells
Q. 1. What is the key? b. Myelofibroblasts
Q. 2. What are the causes of this condition here?
c. Plasma cells
d. Mature lymphocytes
Ans. 1. The key is B. DIC.

Ans. 2. Pregnancy itself is a risk factor for DIC. Q. 1. What is the key?
Placental abruption is a more common cause of Q. 2. What is the diagnosis?
DIC. Q. 3. What are the points in favour of your
diagnosis?
Other causes of pregnancy related DIC are:
eclampsia, retention of a dead fetus, amniotic Ans. 1. The key is D. Mature lymphocytes.
fluid embolism, retained placenta or bacterial
Ans. 2. The diagnosis is CLL.
sepsis.
Ans. 3. It is CLL because of his age (73 yrs),
103. A 28yo woman has delivered with cervical lymphadenpathy, recurrent infections
rotational forceps after an 8h labor and (mature but functionally defective lymphocytes),
3h second stage. Choose the single most and pale conjunctiva (anaemia).
likely predisposing factor for PPH for this
pt? 106. A 45yo lady has 10m hx of SOB. She
a. Atonic uterus is found to have irregularly irregular pulse
b. Cervical/vaginal trauma and loud P2 with fixed splitting and
c. Retained product ejection systolic murmur in left 2nd ICS.
d. Preterm labor What is the probable dx?
e. Uterine infection a. TOF
b. ASD 109. A 33yo male involved in a street fight
c. VSD presents with bruises and deformity in the
d. PDA upper part of his leg. XR shows fx of the
e. CoA neck of fibula. What is the single most
associated nerve injury?
Q. 1. What is the key?
a. Sciatic nerve
Q. 2. What is the diagnosis?
Ans. 1. The key is B. Atrial septal defect. b. Gluteal nerve
c. Musculocutaneous nerve
Ans. 2. Diagnosis is ASD with atrial fibrillation. [i) d. Lateral peroneal nerve
atrial fibrillation = irregularly irregular pulse. ii) e. Tibial nerve
ASD = SOB, fixed splitting with loud P2, ESM in
f. Femoral nerve
pulmonary area]. This picture is typical. One
should not misdiagnose SOB, ESM in pulmonary Ans. The key is D. Lateral peroneal nerve. [Lateral
area and loud P2 as pulmonary hypertension peroneal nerve is other name of superficial
(though in elderly this can develop with ASD). peroneal nerve].
107. A 5m baby present with recurrent 110. A 35yo man presents with hx of
vomiting. Mother noticed some of the dyspepsia. H.Pylori antibodies are
vomitus is blood stained. Choose the negative. No improvement is seen after
single most likely inv? 1m of tx. What is the next step?
a. Upper GI endoscopy a. Urea breath test
b. Barium meal b. Gastroscopy
c. US c. CT
d. Colonoscopy d. MRI
e. CT abdomen

Ans. The key is A. upper GI endoscopy. Q. 1. What is the key?


Q. 2. What may be the D/D here?
108. A 76yo is treated with HTN. He Q. 3. At this age what are the indications of this
suffers from pain and redness at the MTP procedure?
joint of his right big toe. Which of the Ans. 1. Gastroscopy.
following anti-HTN cause this symptoms? Ans. 2. Not responding to treatment D/D is: i)
a. Losartan
Jollinger Elison syndrome ii) Ca stomach
b. Bendroflumethiazide
c. Ramipril Ans. 3. Indications of gastroscopy in a 35 yo man
d. Bisoprolol (for man of age <50): i) Acute symptoms with H/O
e. Verapamil previous episode (PUD) ii) Alarm features [weight
loss, anaemia, vomiting, hematemesis and
Q. 1. What is the key? melaena, dysphagia, palpable abdominal mass],
Q. 2. What is the diagnosis? fear of cancer, evidence of organic disease.
Q. 3. What is the cause of the disease?

Ans. 1. The key is B. Bendroflumethiazide


111. A 15yo male has bilateral ankle
Ans. 2. Diagnosis is acute gout. edema. His BP=110/70mmHg and
urinalysis shows protein++++. What is the
Ans. 3. Thiazide diuretics may cause
hyperuricemia and thus cause or precipitate gout. most likely dx?
a. HUS
b. IgA nephropathy c. Mitral valve prolapse
c. Membranous GN d. Pulmonary stenosis
d. Minimal change GN e. Mitral valve stenosis
e. Nephrotic syndrome
Ans. The kay is E. Mitral valve stenosis. [Mitral
Q. 1. What is the key? stenosis is the most common valvular
Q. 2. What are the points in favour of your complication of rheumatic fever].
diagnosis?
Q. 3. What is the treatment? 114. A 37yo woman presents with fatigue.
Ans. 1. The key is D. Minimal change disease. Exam: angular stomatitis, no koilonychea.
Choose the single cell type you will find on
Ans. 2. Points in favour: i) Age 15 ii) Ankle
oedema iii) Normotension iv) Heavy proteinuria. the blood film.
a. Macrocytes
Ans. 3. Treatment of choice is steroid b. Microcytes
(prednisolon). Failure of steroid or frequent c. Granulocytes wthout blast cells
relapse (>3) cyclophosphamide. d. Blast cells

Q. 1. What is the key?


112. A 28yo man has developed a red, Q. 2. What is the cause here?
Q. 3. What are the points in favour of mentioned
raised rash on trunk after playing football. cause?
His PMH shows he had childhood asthma. Ans. 1. The given key is A. Macrocytes.
The rash is becoming increasingly itchy.
What is the most appropriate tx? Ans. 2. The cause here is VIT. B12 or folate
a. Oral chlorpheneraime deficiency.
b. Oral amoxicillin Ans. 3. Points in favour of Vit. B12 or folate
c. IM adrenaline deficiency: i) fatigue (anaemia) ii) angular
d. Nebulized salbutamol stomatitis (can be seen in Vit. B12 or folate
e. Histamine deficiency) iii) absence of koilonychea is against
IDA.
Q. What is the key?
Q. 2. What is the diagnosis? 115. A 4yo boy with a febrile convulsion
Ans. 1. The key is A. Oral chlorpheneramine. lasting eight minutes has been given IV
Ans. 2. Diagnosis is Atopy (allergy). lorazepam to control them. What is the
single most likely serious side effect?
a. Amnesia
b. Anaphylactic shock
c. Apnea
d. Bronchospasm
113. A 72yo man has been advised to
e. Cardiac arrhythmia
have antibiotic prophylaxis for some years
now before dental tx. He has never Ans. The key is C. Apnoea. [Respiratory
experienced chest pain. Three weeks ago, depression may occur following lorazepam
he noticed breathlessness on exertion and administration].
for one week he had orthopnea. His pulse
is normal. What is the most probable dx?
a. Aortic regurgitation 116. A 4wk girl has been dx of having
b. Ischemic mitral regurgitation breast milk jaundice. She is otherwise
well. What is the single most appropriate Followings are the indications of adrenaline in
management? anaphylaxis:
a. Continue breastfeeding 1. Horseness of voice
b. Exchange transfusion 2. Wheeze
c. Increase fluid intake 3. Shortness of breath
d. Phototherapy 4. Shock
e. Stop breastfeeding 5. Stridor
6. Swelling of the tongue and cheek
7. Facial swelling

Q. 1. What is the key?


Q. 2. What is breast milk jaundice? 118. A term baby born to a 30yo woman
Q. 3. What type of hyperbilirubinemia occurs in of blood group A-ve develops severe
breast milk jaundice? jaundice within the first 24h of birth.
Q. 4. What is the cause of this jaundice?
What is the most likely dx?
Ans. 1. The key is A. Continue breast feeding. a. Hereditary spherocytosis
b. G6PD
Ans. 2. If jaundice lasts past the first week of life c. ABO incompatibility
in a breastfed baby who is otherwise healthy, the d. Rh incompatibility
condition may be called "breast milk jaundice." e. Physiological jaundice
Ans. 3. Unconjugated hyperbilirubinaemia.
Ans. The key is D. Rh incompatibility.
Ans. 4. Cause of breast milk jaundice: factors in a
mother's milk that help a baby absorb bilirubin
from the intestine. 119. A 4yo girl is found to have bounding
pulse and continuous machinery murmur.
What is the most probable dx?
117. A 12yo girl when playing in the a. TOF
garden accidentally stepped on a hive and b. ASD
was bitten several times. She has c. VSD
numerous wheals on her body and d. PDA
complains of severe itching. What is the e. CoA
single most appropriate management?
a. Oral antihistamine Ans. The key is D. PDA. [Continuous mechinary
b. IV antihistamine murmur is well known feature of PDA].
c. IM adrenaline
d. Oral ciprofloxacin
e. Reassurance 120. A 12yo child with episodes of sudden
bluish discoloration and brief loss of
Ans. The given key is C. IM adrenaline which is a consciousness. O/E : clubbing, central
wrong key. The correct answer is A. Oral cyanosis, systolic thrill with systolic
antihistamine. [Intravenous antihistamines are ejection murmur in 2nd left ICS. What is
used as an adjunct to epinephrine in the
the most probable dx?
emergency treatment of anaphylaxis and
a. TOF
angioedema. Parenteral antihistamine is not
b. ASD
recommended in less severe allergic reaction
c. VSD
(other than anaphylaxis)].
d. PDA d. Exostosis
e. CoA e. Osteomyelitis

Ans. The key A. TOF. [TOF usually does not Q. 1. What is the key?
become symptomatic at birth or early infancy and Q. 2. What are the points in favour of your
given features (central cyanosis and clubbing with diagnosis?
murmur of right ventricular outflow obstruction Ans. Given key is E. Osteomyelitis which is a
i.e. ejection systolic murmur in 2nd left ICS) are wrong key. The correct answer is B. Septic
well known features of TOF]. [Tetralogy of fallot = arthritis.
1. VSD + 2. Overriding of the aorta + Right Ans. Points in favour of diagnosis: i) Pain in joints
ventricular outflow tract obstruction + Right (knee and hip). In osteeomyelitis there is no joint
ventricular hypertrophy]. pain but pain in other parts of bone like shaft. ii)
Fever iii) Painful restricted movement of joint.

121. An 8yo child who is tall for his age NB This controversial question was debated and
ultimately settled as septic arthritis by old
and has a refractory error for which he
plabbers.
wears glasses has presented with severe
crushing chest pain. What is the most 123. A man with anterior resection and
likely dx? end to end anastomosis done complains
a. Fragile X syndrome of severe pain in the chest and abdominal
b. Prader-willi syndrome distension. What is the most appropriate
c. DiGeorge syndrome inv likely to review the cause this
d. Marfans syndrome deterioration?
a. XR abdomen
Q. 1. What is the key?
b. Exploratory laparoscopy
Q. 2. What is the cause of this severe crushing
chest pain? c. CT
Q. 3. What are the most common cardiac d. US
abnormalitis found in this disease? e. Laparotomy

Ans. 1. The key is D. Marfans syndrome. Ans. The key is E. Laparotomy. It is a wrong key!
Right key is C. CT. [This is likely an anastomotic
Ans. 2. Cause of severe crushing chest pain may leak].
be aortic dissection.

Ans. 3. Most common cardiac abnormalities in


Marfans syndrome are: dilatation of the aorta
and mitral regurgitation.

122. A 4yo child presents with pain of


spontaneous onset in his knee of 2 days
duration. He has developed mild fever in
the 2nd day. He can walk but has a limp.
Exam: painful restriction in the right hip.
What is the most probable dx?
a. Osteosarcoma
b. Septic arthritis
c. TB arthritis
a rigid abdomen. What is the most likely
dx?
a. Biliary peritonitis
b. Ischemic colon
c. Pancreatic necrosis
d. Perforated diverticulum
e. Perforated peptic ulcer

Ans. The key is E. Perforated peptic ulcer. [NSAIDs


induced perforation. Points in favour- 1.
Prolonged use of NSAIDs, 2. Sudden onset of
severe abdominal pain, 3. Rigid abdomen].

126. A woman 5 days post-op for bilateral


salphingo-oopherectomy and abdominal
hysterectomy has developed abdominal
pain and vomiting a/w abdominal
distension and can’t pass gas. No bowel
sounds heard, although well hydrated.
What is the most appropriate next step?
124. Pt with hx of alcoholism, ataxic gait, a. XR abdomen
hallucinations and loss of memory. He is b. Exploratory laparoscopy
given acamprosate. What other drug can c. CT
you give with this? d. USG
a. Chlordiazepoxide
e. Barium enema
b. Thiamine
c. Diazepam Q. 1. What is the key?
d. Disulfiram Q. 2. What is the diagnosis?
e. Haloperidol Q. 3. What are the causes of it?
Q. 4. What is the management?
Q. 1. What is the key? Ans. 1. The key is A. X-ray abdomen.
Q. 2. What is the diagnosis?
Ans. 2. The diagnosis is paralytic ileas.
Q. What are the points in favour of diagnosis?
Ans. 1. The key is B. Thiamine. Ans. 3. Causes of paralytic ileus: i) electrolyte
Ans. 2. The diagnosis is Wernicke’s imbalance ii) gastroenteritis iii) appendicitis iv)
encephalopathy. pancreatitis v) surgical complications and vi)
certain drugs.
Ans. 3. Points in favour of diagnosis: i) history of
alcoholism ii) ataxic gait iii) hallucination iv) Ans. 4. Management of paralytic ileus: i) nil by
memory loss. mouth ii) nasogastric suction to alleviate the
distension and remove the obstruction.
125. A 35yo male builder presented with
127. A 30yo man complains of hoarseness
sudden onset of severe abdominal pain.
of voice. Exam: unilateral immobile vocal
He was previously fit and well other than
cord. What is the most probable dx?
taking ibuprofen for a long term knee
a. Graves disease
injury. On examination he is insevere pain,
b. Hematoma
pulse=110bpm, BP=110/70mmHg and has
c. Unilateral recurrent laryngeal nerve enough information for us to say that one of
injury these treatments is better than another].
d. External laryngeal nerve injury
Ans. 2. The diagnosis is post traumatic stress
e. Tracheomalacia disorder.
Ans. The key is C. unilateral recurrent laryngeal Ans. 3. Points in favour of PTSD: i) H/O stressor
nerve injury. (house fire a year ago) ii) Nightmares of the
stressor iii) Hyper arousal (very anxious and
128. A 38yo woman has delivered after an
inability to relax (leading to irritability) iv)
induced labor which lasted 26h. choose associated depression (poor sleep, tearful).
the single most likely predisposing factor
for postpartum hemorrhage? Note: Fluoxetin and peroxetin are the drugs of
a. Atonic uterus choice in PTSD. CBT is the non-pharmacological
b. Cervical/vaginal trauma treatment.
c. Rupture uterus 130. A 22yo woman with longstanding
d. Fibroid uterus constipation has severe ano-rectal pain on
e. Age of mother
defecation. Rectal exam: impossible due
Ans. The key is A. Atonic uterus. [Prolonged to pain and spasm. What is the most
labour is a risk factor for PPH secondary to atonic probable dx?
uterus]. a. Anal hematoma
b. Anal fissure
129. A 32yo woman in tears describing c. Anal abscess
constant irritability with her 2 small d. Protalgia fugax
children and inability to relax. She e. Hemorrhoids
describes herself as easily startled with
poor sleep and disturbed nightmares Ans. The key is B. Anal fissure.
following a house fire a year ago, while
the family slept. What is the single best
tx? 131. A 20yo student attends the OPD with
a. Rassurance complaint of breathlessness on and off,
b. Relaxation therapy cough and sputum. His sleep is disturbed
c. Quetiapine and skin is very dry in flexural areas of the
d. Lofepramine body. Exam: tachypnea, hyperresonant
e. Fluoxetine percussion and wheezing on auscultation.
What is the most likely dx?
Q. 1. What is the key a. Extrinsic allergic alveolitis
Q. 2. What is the diagnosis? b. Asthma
Q. 3. What are the points in favour of your
c. Wegener’s granulomatosis
diagnosis?
Ans. 1 The key is E. Fluoxetine. [NICE guidelines d. COPD
suggest that trauma-focussed psychological e. Cystic fibrosis
therapies (CBT or EMDR) should be offered
Q. What is the key?
before medication, wherever possible. As these
Q. What are the diagnostic criteria?
are not in option the best answer here is
Fluoxetine (SSRI). At present, there is evidence Ans. The key is B. Asthma.
that EMDR, psychotherapy, behaviour therapy
and antidepressants are all effective. There is not
Ans. 2. Diagnostic criteria of asthma: i) Airway years and has ectropion and contact bleeding
hyper-responsiveness to certain stimuli ii) smear is the best option here as in UK 1st smear is
Recurrent variable airflow limitation usually offered at 25 yrs.
reversible iii) presents as wheezing,
breathlessness, chest tightness and cough.
134. A 32yo had a normal vaginal delivery
132. A pt with thought disorder washes 10 days ago. Her uterus has involuted
hands 6x each time he uses the toilet. normally. Choose the single most likely
What is the best management? predisposing factor for PPH?
a. Psychodynamic therapy a. Retained product
b. CBT b. DIC
c. Antipsychotics c. Uterine infection
d. Refer to dermatology d. Von Willebrand disease
e. Reassure e. Primary PPH

Q. 1. What is the key?


Q. 2. What type of PPH it would be?
Q. 1. What is the key?
Q. 2. What is the diagnosis?
Ans. 1. The key is C. uterine infection.
Ans. 1. The key is B. CBT. [For OCD CBT is the 1 st
Ans. 2. Secondary PPH
treatment of choice and if fails comes drugs].

Ans. 2. The diagnosis is obsessive compulsive


135. A 37yo man slipped while he was
disorder.
walking home and fell on his out stretched
hand. He complains of pain in the right
arm. XR showed fx of the head of radius.
133. A 25yo woman presented to her GP What is the single most associated nerve
on a routine check up. Upon vaginal exam, injury?
she was fine except for finding of cervical a. Radial nerve
ectropion which was painless but mild b. Musculocutaneous nerve
contact bleeding on touch. What is the c. Median nerve
next management? d. Ulnar nerve
a. Endometrial ablation
Q. 1. What is the key?
b. Cervical smear (2nd line)
Q. 2. What is the root value?
c. Colposcopy Ans. 1. The key is A. Radial nerve. [At wrist, radial
d. Antibiotics (1st line) nerve injury cause finger drop with a normal wrist
e. Vaginal US and intact sensation].
f. Pack with gauze and leave to dry
Ans. 2. Root value of radial nerve: C5,6,7,8 and
Q. 1. What is the key? T1.
Q. 2. Points in favour correct of key.
Ans. The key is D. Antibiotics. This is a wrong key.
The correct key is B. Cervical smear. [Before 136. A butcher stabbed accidently his
antibiotic we have to take swab].
groin. He bled so much that the towel was
Ans. 2. Cervical smear is a screaning service which soaked in blood and BP=80/50mmHg,
follows its own schedule and can not be done pulse=130bpm. What % of circulatory
unless it is due or overdue. As the Lady is of 25 blood did he lose?
a. <15% blocker or rate limiting CCB; OHCM, 9th edition,
page-124].
a. b. 15-30%
b. 30-40% 138. A 78yo man is depressed after his
c. 40-50% wife’s death. He has been neglecting
d. >50% himself. His son found him in a miserable
state when he went to visit. The son can’t
Q. 1. What is the key?
Q. 2. What is the classification of blood loss deal with his father. What is the
according to vita sign? appropriate management?
Ans. 1. The key is C. 30-40% a. Voluntary admission to psychiatry ward
b. Hand over to social worker
Ans. 2. Classification:
c. Request son to move in with father
1. Class 1 up to 15% of blood volume lost: d. Send pt to care home
pulse <100; systolic BP normal; pulse
pressure normal; Respiratory rate 14-20; Ans. The key is A. Voluntary admission to
urine output greater than 30 ml/hour. psychiatry ward. [This is much too a controversial
question!! Patient is neglecting himself and is in a
2. Class 2 15%-30% blood volume lost: pulse measerable state. His son can’t deal with the
100-120; systolic blood pressure normal; patient! So it seems impossible for care home
pulse pressure decreased; respiratory staff to deal with him and social worker as well. If
rate 20-30; urine output 20-30 ml/hour. son move in with father will be of no benefit as
he can not deal with the patient. So voluntary
3. Class 3 30%-40% blood volume lost: pulse
admission to psychiatry ward is the only option to
120-140; systolic BP decreased; pulse
which we can look but still with doubt as whether
pressure decreased, respiratory rate 30-
he is able to understand or give consent for
40; urine output 5-15 ml/hr
voluntary admission. Even though A seems to be
4. Class 4, blood loss of greater than 40%: the likely option!!]
pulse rate >140; systolic BP decreased;
pulse pressure decreased’ respiratory 139. An old alcoholic presents with cough,
rate >35; urine output negligible. fever, bilateral cavitating consolidation.
What is the most probable cause?
a. Gram +ve diplococcic
137. A 67yo man presents with 1. b. Coagulase +ve cocci
palpitations. ECG shows an irregular d. Gram –ve cocci
rhythm and HR=140bpm. He is otherwise e. AFB
stable, BP=124/80mmHg. f. Coagulase –ve cocci
What is the most appropriate
management? Q. 1. What is the key?
Q. 2. What is the organism?
a. Bisoprolol
b. ACEi
c. Ramipril Ans. 1. The key is B. Coagulase +ve cocci. [The
d. Digoxin picture is of pneumonia and bilateral cavitating
consolidation favours staphylococcus as the
Ans. The key is A. Bisoprolol. [In acute AF (<48h) if causative agent].
stable rate control by verapamil
40-120mg/8hourly po or bisoprolol 2.5-5mg/d po. Ans. 2. Name of organism is Staphylococcus
In chronic AF (>48h) rate control with beta- aureus. [Both coagulase positive and coagulase
negative cocci are staphylococci. Gram positive occurs in mitral stenosis. Iii) Atrial fibrillation is a
diplococcic is pneumococcus and gram negative common association.
nisseria, moraxella catarrhalis and hemophilus
influenza. For AFB there should be low grade 142. A 60yo diabetic complains of pain in
evening fever with night sweats, weight loss, thigh and gluteal region on walking up the
anorexia etc]. stairs for the last 6 months. She is a heavy
smoker and has ischemic heart disease.
140. A 67yo man had successful What is the most appropriate dx?
thrombolysis for an inf MI 1 month ago a. Thromboangitis Obliterans
and was discharged after 5days. He is now b. Sciatica
readmitted with pulmonary edema. What c. DVT
is the most probable dx? d. Atherosclerosis
a. Aortic regurgitation e. Embolus
b. Ischemic mitral regurgitation
c. Mitral valve prolapse
d. Pulmonary stenosis Q. 1. What is the key?
e. Rheumatic mitral valve stenosis Q. 2. What are the points in favour?
Ans. 1. The key is D. Atherosclerosis.
Ans. The key is B. Ischaemic mitral regurgitation.
[ischaemic mitral regurgitation > raised Ans. 2. i) It is not sciatica as sciatica pain is worse
pulmonary capillary pressure > pulmonary when sitting. There may be weakness, numbness,
oedema]. [ Inferior myocardial infarction causes difficulty moving the leg or foot. A constant pain
left ventricular remodeling, which displaces on one side of the rear. A shooting pain that
posterior papillary muscle away from its normal makes it difficult to stand up. ii) It is not DVT as no
position, leading to ischemic mitral regurgitation].
swelling, warmth or redness of skin are there iii)
It is not thromboangitis obliterans as pulses are
141. A 60yo lady who had stroke 3 years ok, no colour change or reduced hair growth, no
ago now reports having increased ulceration or gangrene iv) not embolism as no
pain (rest pain), no numbness, no redness or
dyspnea on exertion and atrial fibrillation.
itching or rash, no ulceration of skin.
CXR: straight left border on the cardiac
silhouette. What is the most probable dx? 143. A 3yo child who looks wasted on
a. Aortic regurgitation examination has a hx of diarrhea on and
b. Ischemic mitral regurgitation off. The mother described the stool as
c. Mitral valve prolapse bulky, frothy and difficult to flush.
d. Pulmonary stenosis What is the single inv most likely to lead
e. Rheumatic mitral valve stenosis to dx?
a. Sweat chloride test
Q. 1. What is the key?
Q. 2. What are the points in favour of your b. Anti-endomysial antibodies
answer? c. LFT
d. US abdomen
Ans. 1. The key is E. Rheumatic mitral valve e. TFT
stenosis.
Q. 1. What is the key?
Ans. 2. Points in favour: i) Dyspnoea on exertion Q. 2. What is the diagnosis?
ii) Straight left border of the cardiac silhouette. To
straighten the left heart border it requires
underfilling of the left ventricle and aorta which Ans. 1. The key is B. Anti-endomysial antibody
Ans. 2. The diagnosis is celiac disease. [It is not Q. What is the diagnosis?
cystic fibrosis as lung problem is most commonly
seen in cystic fibrosis along with GI problem like Ans. 1. The key is D. MRI brain.
indigestion].
Ans. 2. Diagnosis is multiple sclerosis.

146. A 55yo male presents with malaise


144. A 45yo woman has had severe and tiredness. Exam: spleen approaching
epigastric and right hypochondrial pain RIF, no
for a few hours. She has a normal CBC, lymphadenopathy. Choose the single cell
serum ALP is raised, normal transaminase. type?
3 months ago she had a cholecystectomy a. Helmet shaped cell
done. What is the most appropriate inv? b. Sickle cell
a. US abdomen c. Granulocyte without blast cells
b. ERCP d. Blast cells
c. MRCP
d. CT abdomen Q. 1. What is the key?
e. Upper GI endoscopy Q. 2. What is the diagnosis?
Q. 3. What are the diagnostic features?
Q. 1. What is the key?
Q. 2. What is the diagnosis? Ans. 1. The key is C. Granulocyte without blast
Ans. 1. The key is B. ERCP. It is probably a wrong cells.
key and correct key should be C. MRCP. [Post
Ans. 2. The diagnosis is CML.
operative US of abdomen does not give good
result for hepatobiliary system. ERCP is invasive Ans. 3. Diagnostic features are i) increased
procedure and it has its considerable number of mature granulocytes ii) huge
complications like cholangitis, injury, pancreatitis splenomegaly.
etc. Among given options MRCCP is most
appropriate. We shall go for ERCP after making [* Helmet shaped cells (or shistocytes) =
the dx confirm. For this MRCP is preferred. If the hemolytic or Microangiopathic hemolytic anemia
question asks which is the “DEFINITIVE” or the * Sickle cell = sickle cell anemia
“MOST DIAGNOSTIC” then the option will be
ERCP]. * Blast cells (immature cells) = in acute leukemia].

Ans. 2. Diagnosis is choledocolithiasis.

145. A 53yo woman presented with pain Chronic myelogenous leukemia (CML), also
known as chronic myeloid leukemia, is a cancer of
in the eye, blurry vision and clumsiness for
the white blood cells. It is a form of leukemia
3 months. She has a hx of difficulty in
characterized by the increased and unregulated
swallowing and weakness in her right growth of myeloid cells in the bone marrow and
upper limb 2y ago. What is the the accumulation of these cells in the blood. CML
investigation of choice? is a clonal bone marrow stem cell disorder in
a. CSF analysis which a proliferation of mature granulocytes
b. EEG (neutrophils, eosinophils and basophils) and their
c. EMG precursors is found. It is a type of
d. MRI brain myeloproliferative neoplasm associated with a
e. Visual evoked response test characteristic chromosomal translocation called
the Philadelphia chromosome.
Q. What is the key?
e. Uterine artery embolization

147. A 6yo pt comes with easy bruising in Ans. The key is E. Uterine artery embolization.
different places when she falls. CBC: [Done by interventional radiologist expert in
WBC=25, Hgb=10.9, Plt=45. Her Paul arterial embolization technique. Particles are
Bunnel test +ve. What is the most likely placed in uterine artery to block circulation to
dx? uterine body. No operation or GA is required].
a. Glandular fever
149. A girl with hx of allergies visited a
b. ITP
friend’s farm. She got stridor, wheeze and
c. Trauma
erythematous rash. What is the most
d. NAI
appropriate tx?
e. Septicemia
a. 0.25ml IM adrenaline
Q. 1. What is the key? b. 0.25ml PO adrenaline
Q. 2. What are the lab. Values that suggests the c. 0.25ml IV adrenaline
diagnosis here? d. IV chlorphearamine

Ans. 1. The key is A. Glandular fever = infectious Ans. The key is A. 0.25 ml IM adrenaline
mononucleosis [Presence of stridor and wheeze are suggestive of
anaphilaxis and treatment option is IM
Ans. 2. Suggestive lab. Values: WBC=25 adrenaline].
(leucocytosis), Hgb=10.9 (usually patient is not
anaemic), Plt=45 (thrombocytopenia-leading to 150. A 5yo boy is referred to the hospital
easy bruising), Positive paul bunnel test. … and seen with his father who is worried
The Paul Bunnell test is used to screen for that he has been listless. He is not sure
infectious mononucleosis (IM), which is seen why his GP suggested he should come to
most commonly in adolescents and young adults the ED and is keen to get some tablets
…. mononuclear spot test or monospot test, a and go home. Exam: tired and irritable,
form of the heterophile antibody test,is a rapid swelling around eyes. Renal biopsy:
test for infectious mononucleosis due to Epstein– remarkable for podocyte fusion on EM.
Barr virus (EBV). It is an improvement on the What is the most probable dx?
Paul–Bunnell test. a. NAI
b. Myelodysplastic disease
148. A 41yo woman who has completed
c. HSP
her family, has suffered from extremely
d. Membranous GN
heavy periods for many years. No medical
e. Minimal change GN
tx has worked. She admits that she would
rather avoid open surgery. After Ans. The key is E. Minimal change
discussion, you collectively decide on a glomerulonephritis. [Podocyte fusion on electron
procedure that wouldn’t require open microscopy]
surgery or GA.
151. A 6yo boy is brought to the hospital
Select the most appropriate management
for a 3rd episode of sore throat in 1
for this case.
a. Endometrial ablation
month. He is found bleeding from gums
b. Hysterectomy
and nose and has pale conjunctiva.
c. Fibroid resection What’s the single cell type?
a. Clumped platelets
d. Myomectomy
b. Microcytes
c. Granulocyte without blast cells tension pneumothorax is more in stab wond and
d. Blast cells no tracheal deviation is controversial. This is
e. Mature lymphocytes probability of bad recall!!

Q. 1. What is the key? 153. A 44yo pt comes with right


Q. 2. What is the diagnosis? hemiparesis. Exam: left sided ptosis and
Q. 3. What are the points that favour diagnosis? left dilated pupil. What is the site of the
lesion?
Ans. 1. The key is D. Blast cells. a. Cerebral infarct
Ans. 2. The diagnosis is ALL b. Cerebellar infarct
c. Medulla oblongata
Ans. 3. Points in favour: i) Age-6yrs ii) recurrent
d. Pons
infection (sorethroat) due to neutrpenia and
e. Midbrain
abnormal lymphoblasts which cannot protect
from infection iii) thrombocytopenia causing gum Q. 1. What is the key?
and nose bleeding. Iii) anaemia (pale conjunctiva) Q. 2. What is the name of this condition?
due to reduced red cell production from marrow Ans. 1. The key is E. Midbrain.
occupation by blast cells. [Here debate came why
it is not aplastic anaemia? There is no risk factor Ans. 2. Weber syndrome [presence of ipsilateral
mentioned for this patient for aplastic anaemia. oculomotor nerve palsy and contralateral
There may be congenital aplastic anaemia but hemiparesis or hemiplagia].
again it would present earlier in life. So it goes Weber's syndrome, also known as superior
more with leukaemia but it cannot be confirmed alternating hemiplegia, is a form of stroke
unless we do bone marrow aspiration.] characterized by the presence of an ipsilateral
oculomotor nerve palsy and contralateral
152. A 23yo man has been stabbed in the
hemiparesis or hemiplegia.
back and has SOB. The trachea is not
deviated, he has engorged neck veins and 154. A 50yo man has a stab wound to his
absent breath sounds on the right. left anterior chest at the level of the 4th
What is the most appropriate dx? ICS. He has a BP = 80mmHg,
a. Tension pneumothorax pulse=130bpm. His neck veins are dilated
b. Cardiac tamponade and his heart sounds are faint. His trachea
c. Simple pneumothorax is central. What is the most appropriate
d. Hemothorax dx?
e. Pleural effusion a. Cardiac tamponade
b. Diaphragmatic rupture
Q. 1. What is the key?
c. Fractured ribs
Q. 2. What are the point in favour of your
answer? d. Tension pneumothorax
e. Traumatic rupture of aorta

Ans. 1. The key is A. Tension pneumothorax. Q. 1. What is the Key?


Q. 2. What are the points in favour of your
Ans. 2. Points in favour: i) Stab wound in the back answer?
ii) SOB iii) Engorged neck vein iv) Absent breath Ans. 1. Theakey is Cardiac tempoade.
sound.
Ans. 2. Points in favour: i) Systolic BP 80 mmHg ii)
These features are common for both hemothorax Pulse 130 bpm iii) Engorged neck vein iv) Faint
and tension pneumothorax and tracheal heart sounds v) Trachea is central.
deviation is common to both! But chance of
155. A 15yo boy has a soft painless E. DM
swelling in the left scrotum, blue in color
Q. 1. What is the key?
and can be compressed. What is the most Q. 2. Points that support your diagnosis.
appropriate next step?
a. Analgesia
b. Antibiotic Ans. 1. The key is B. Chronic ischaemia of the
c. Biopsy limb.
d. Immediate surgery Ans. 2. Intermittent claudication is a symptom not
e. Reassurance diagnosis. It is not buerger’s disease as buerger
occur in more younger heavy smoker (before the
Q. 1. What is the key? age of 50yrs), It is not DVT as dvt pain or
Q.2. What is the name of this condition?
tenderness is not of an intermittent claudication
pattern. Again in DM there is no intermittent
Ans. 1. The key is E. Reassurance. claudication.

Ans. 2. Name of the condition is Varicocele.

156. A 12yo pt presents with copious 158. An otherwise healthy 13yo boy
diarrhea. Exam: urine output=low, presents with recurrent episodes of facial
mucous membrane=dry, skin turgor =low. and tongue swelling and abdominal pain.
What is the most appropriate initial His father has had similar episodes.
management? What is the most likely dx?
a. Antibiotic a. C1 esterase deficiency
b. Antimotility b. HIV
c. Anti-emetic c. Mumps
d. Fluid replacement d. Sarcoidosis
e. Reassurance e. Sjogren’s syndrome

Q. 1. What is the key?


Q. 2. What is the diagnosis and why?
Q. 1. What is the key?
Q. 2. What is the name of this condition?
Ans. 1. The key is D. Fluid replacement. Q. 3. Why it is not acquired?

Ans. 2. Diagnosis is severe dehydration. Points in


favour: i) low urine output ii) dry mucous Ans. 1. The key is A. C1 esterase inhibitor
membrane and iii) low skin turgor. deficiency.

157. A 60yo smoker presents with cramp- Ans. 2. Hereditary angioedema.


like pain in the calves relived by rest and Ans. 3. Acquired angeoedema usually manifest
non-healing ulcers. O/E : cold extremities after the age of 40 yrs.
with lack of hair around the ankles, absent
distal pulses. What is the most probable
dx? 159. A 25yo had an LSCS 24h ago for fetal
A. Intermittent claudication distress. She now complains of
B. b. Chronic ischemia of the limbs intermittent vaginal bleeding.
C. Buerger’s disease Observations: O2 sat=98% in air,
D. DVT BP=124/82mmHg, pulse=84bpm,
temp=37.8C. The midwife tells you that different in levator ani syndrome, and may last
she had a retained placenta, which days instead of minutes… Most treatments for
required manual removal in the OT. proctalgia fugax (e.g., oral diltiazem, topical
glyceryl nitrate, nerve blocks) act by relaxing the
Choose the most appropriate C-Section
anal sphincter spasm.
complication in this case?
a. Retained POC
b. Aspiration pneumonitis
c. Endometritis 161. A 78yo male, DM and HTN, had a fall
d. Uterine rupture and since then is unable to walk. He
e. DIC presents with deformity and tenderness
over the right hip area. XR=fx of femur
neck. What is the single most associated
Q. 1. What is the key? nerve injury?
Q. 2. What are the points in favour? a. Sciatic nerve
b. Gluteal nerve
c. Lateral peroneal nerve
Ans. 1. The key is C. Endometritis.
d. Tibial nerve
Ans. 2. More handling of tissue like manual e. Femoral nerve
removal of placenta, intermittent vaginal bleeding
and raised temperature points toward infective
process like endometritis. Ans. The key is A. Sciatic nerve. [Sciatic nerve
injury though may occur but may not be very
common!]
160. A 30yo woman has brief episodes of
severe shooting pain in the rectum. Rectal
examination and flexible sigmoidoscopy 162. A 20yo man has a head on collision in
are normal. What is the most probable a car. On presentation he is breathless,
dx? has chest pain and fx of 5-7th rib. CXR
a. Anal hematoma confirms this.
b. Anal fissure What is the most appropriate initial action
c. Rectal carcinoma in this pt?
d. Proctalgia fugax a. Antibiotics
e. Piles b. Analgesia
c. O2 by mask
d. Physiotherapy
Ans. is D. Proctagia fugax [normal rectal e. Refer to surgeon
examination and flexible sigmoidoscopy excludes
other options].
Ans. The key is C. O2 by mask. [There was debate
in this forum that pain relief should be given first
Proctalgia fugax is anal pain that doesn't have a which will automatically relieve breathing
specific cause. This pain is usually caused by problem. But others told O2 first]. O2 first is the
intense muscle spasms in or around the canal of correct answer!
the anus. It's similar to another type of anal pain
called levator ani syndrome. The pain is slightly
Ans. The key is C. Serum blood glucose [it is also
possible that he may have taken drug, even
though first we have to do serum glucose as low
blood glucose can be very easily managed and it
needs urgent management to save life. If it is
excluded then we can look for other causes which
may be not fatal in short time as hypoglycaemia].

[http://www.urmc.rochester.edu/encyclopedia/
165. A 20yo man complains of recent
content.aspx?
onset of itching which followed a viral
contenttypeid=22&contentid=flailchest]
infection. There are numerous wheals of
all sizes on his skin particularly after he
has scratched it. These can last up to an
163. A 28yo man with complains of
hour. What is the most probable dx?
headache and nose bleeds also has pain in
a. Uremia
the lower limbs on exertion. O/E: radio-
b. Urticaria
femoral delay, cold legs with weak pulse c. Psychogenic itching
and mild systolic murmur with normal d. Atopic eczema
S1S2. What is the most probable dx? e. Primary biliary cirrhosis
a. TOF
b. ASD
c. VSD
Ans. The key is B. Urticaria.
d. PDA
e. CoA

166. A 75yo lady who had mitral valve


replacement 13 yrs ago has developed
Ans. The key is coarctation of aorta. [headache
and nosebleeds - >hypertension, pain in lower
recurrent
limb on exertion -> as reduced blood supply to leg breathlessness. Her husband has noticed
due to coarctation, radio-femoral delay, cold legs prominent pulsation in her neck. She
with week pulse, mid-systolic murmur are all complains of abdominal pain and ankle
features of coarctation of aorta]. swelling. What is the most probable dx?
a. Aortic regurgitation
b. Mitral regurgitation
164. A 23yo male has a tonic clonic c. Mitral stenosis
seizure whilst at college. His GCS is 12, d. Tricuspid regurgitation
BP=120/77mmHg, HR=99bpm. What is e. Pulmonary stenosis
the most appropriate inv for his
condition?
a. CT Ans. The key is D. Tricuspid regurgitation. [Points
b. MRI in favour: i) recurrent breathlessness – if the
c. Serum blood glucose cause is LV dysfunction, ii) prominent pulsation in
d. Serum drug levels the neck – giant v waves, iii) abdominal pain –
pain in liver on exertion, ankle swelling; These are
features of tricuspid regurgitation. Reference:- T2DM and BMI=35. What is the most
OHCM, 9th edition, page- 142]. likely cause of death?
a. MI
b. DM
167. A 45yo T1DM had an annual check c. HF
up. Ophthalmoscopy showed dot and blot d. PE
hemorrhage + hard exudate and multiple e. Renal failure
cotton wool spots. What is the next step in
Q. 1. What is the key?
management?
Q. 2. Why the patient’s death was unnoticed?
a. Reassurance and annual screening only
b. Urgent referral to ophthalmologist
c. Laser therapy
d. Non-urgent referral to ophthalmologist Ans. 1. The key is A. MI.
e. Nothing can be done Ans. 2. In diabetics MI become painless when the
patient develop autonomic neuropathy (till there
is no autonomic neuropathy diabetic patients will
Ans. The key is D. Non-urgent referral to feel MI pain). In this case the disease was
ophthalmologist. [It is pre-proliferative unnoticed as it was a painless attack.
retinopathy so non-urgent referral; If proliferative
(with neovascularization) urgent referral]. 170. A 38yo pt presented with tingling,
numbness, paraesthesia, resp stridor and
involuntary spasm of the upper
168. A 2m baby who has ambiguous extremities. She has undergone surgery
genitalia presents to the ED with for thyroid carcinoma a week ago. What is
vomiting. Labs: the most likely dx?
a. Thyroid storm
Na+=125mmol/L, K+=6mmol/L. What is
b. Hyperparathyroidism
the most likely dx?
c. Unilateral recurrent laryngeal nerve
a. Fragile X syndrome
b. Turners syndrome injury
d. External laryngeal nerve injury
c. Noonan syndrome
d. Congenital adrenal hyperplasia e. Hypocalcemia

Q. 1. What is the key?


Q. 1. What is the key?
Q. 2. What are the points in favour? Q. 2. What is the cause of this condition?
Q. 3. Why there is respiratory stridor?
Ans. 1. The key is D. Congenital adrenal
Ans. 1. The key is E. Hypocalcaemia.
hyperplasia
Ans. 2. Hypocalcaemia(Tetany) may be due to
Ans. 2. Points in favour: i) ambiguous genitalia ii)
accidental parathyroid gland removal during
salt wasting manifested as hyponatremia and
thyroidectomy.
hyperkalemia (In mild forms of salt-wasting
adrenal hyperplasia, salt wasting may not become Ans. 3. Laryngospasm is a feature in
apparent until an illness stresses the child). [here hypocalcaemia which may cause stridor.
hyperkalaemia inspite of vomiting is indicating
the disease].

169. A 40yo man collapsed at home and 171. A 50yo chronic smoker came to OPD
died. The GPs report says he suffered from with complaint of chronic productive
cough, SOB and wheeze. Labs: Ans. The key is C. Minimal change disease. [Points
CBC=increase in PCV. CXR >6ribs seen in favour: i) Age 15 yrs ii) Generalized oedema iii)
above the diaphragm in midclavicular line. Protein in urine +++ vi) Normal eGFR of 110
(Normal range- 90 to 120 mL/min)].
ABG=pO2 decreased. What is the most
likely dx? 174. A 72yo man is receiving
a. Interstitial lung disease chemotherapy for SCLC. He has his 4th tx
b. Wegener’s granulomatosis 8 days ago. He has a cough with some
c. Ca bronchi green sputum but feels well. Temp=37.6C.
d. COPD
Chest exam = few coarse crepitations in
e. Amyloidosis
the right base. HR=92bpm. CBC:
Q. 1. What is the key? Hgb=12.5g/dL, WBC=1.1, Neutrophils=0.6,
Q. 2. What are the points in favour? Plt=89. Sputum, urine and blood culture
Ans. 1. The key is D. COPD. sent to microbiology.
Ans. 2. Points in favour: i) Age 50 yrs ii) Chronic What is the most appropriate
smoker iii) Chronic productive cough, SOB and management?
Wheeze iv) Raised PCV (hematocrit)secondary to a. Broad spectrum antibiotics IV
chronic hypoxaemia v) Low set diaphragm and b. Broad spectrum antibiotics PO
widened horizontal ribs vi) Hypoxaemia on ABG. c. GCSF
d. Postpone tx until bacteriology results
172. A 44yo pt has sudden onset of available
breathlessness and stridor few minutes e. Reassure and send home
after extubation for thyroidectomy. The
pat had longstanding goiter for which he Q. 1. What is the key?
had the surgery. What is the most likely Q. 2. What is the Diagnosis?
Q. 3. What is the treatment of low WBC count?
dx? Ans. 1. The key is A. Broad spectrum antibiotics IV
a. Thyroid storm
b. Hematoma Ans. 2. The diagnosis is lower respiratory tract
c. Unilateral recurrent laryngeal nerve infection.
injury Ans. 3. GCSF(granulocyte-colony stimulating
d. External laryngeal nerve injury factor) subcutaneously. [it is the treatment of
e. Tracheomalacia chemotherapy induced leucopenia]

Ans. The key is tracheomalacia. [Prolonged 175. A 25yo woman with T1DM has
pressure over trachea by goiter is a cause of delivered a baby weighing 4.5kg. Her
tracheomalacia following thryroidectomy]. uterus is well contracted. Choose the
173. A 15yo boy presents with generalized single most likely predisposing factor for
edema. His urinalysis reveals protein +++, PPH from the options?
a. Atonic uterus
eGFR =110. What is the most likely dx?
a. IgA nephropathy b. Cervical/vaginal trauma
b. Membranous nephropathy c. Retained POC
c. Minimal change disease d. Large placental site
d. PSGN e. Rupture uterus
e. Lupus nephritis
Q. 1. What is the key?
Q. 2. Reason for your answer.
Ans. 1. The key is B. Cervical/vaginal trauma c. Proguanil
d. Quinine
Ans. 2. The baby is a big baby. If patient’s uterus
e. Artesonate
was not well contracted we would fear of atonic
uterus! But as uterus is well contracted it is not Q. 1. What is the key?
atonic uterus. Rather most likely cause is trauma Q. 2. What does Shuffner’s dot in RBC indicate?
dring delivery of this big baby.
Ans. 1. The key is B. Mefloquine. Probably wrong
176. A 23yo lady presents with headache. key as mefloquine can not eradicate hepatic
Exam: photophobia and generalized rash cycle! Primaquine is the drug that can eradicate
that doesn’t blanch on pressure. What hepatic cycle.
must be done immediately?
a. IV benzylpenicillin Ans. 2. Shuffners dot indicates, it is plasmodium
b. Isolate pt ovale or plasmodium vivex infestation.
c. Gown and mask 179. A 35yo woman had an uneventful lap
d. Blood culture chole 18h ago(uneventful laparoscopic
cholecystectomy). She has a pulse=108bpm,
temp 37.8C. There are signs of reduced air
Ans. The key is A. IV benzylpenicillin. entry at the right base but the CXR doesn’t
show an obvious abnormality. What is the
most appropriate management strategy?
177. A 4yo baby has generalized tonic- a. Cefuroxime PO
clonic seizure and fever of 39C. his mother b. Ceftriaxone IV
informs you that this has happened 3-4x c. Chlorpheniramine PO
before. What is the most likely dx? d. Chest physiotherapy
a. Febrile convulsion e. Reassure
b. Absence seizures
c. Epilepsy Q. 1. What is the key?
d. Partial complex seizure Q. 2. What is the diagnosis?

Ans: The key is C. Epilepsy. It is wrong key. Ans. 1. The key is D. Chest physiotherapy.
Correct key should be A. Febrile convulsion. Ans. 2. Atelactasis. [Atelectasis can be seen in
[Points in favour: seizers in presence of high chest x-ray but sometimes it becomes
fever. Same previous illness indicate seizers undetectable in chest x-ray as we are suspecting
during fever which goes against epilepsy. Though in this case].
usually febrile convulsion dosen’t tend to recur
but it can recur as well. Epilepsy has no relation 180. A 20yo pop star singer complains of
to fever but is due to pathology in brain]. inability to raise the pitch of her voice. She
178. A middle aged Asian presents with attributes this to the thyroid surgery she
underwent a few months back.
episodes of fever with rigors and chills for
last 1y. Blood film: ring form plasmodium What is the most likely dx?
a. Thyroid storm
with schaffners dots in RBCs.
b. Bilateral recurrent laryngeal nerve injury
What is the drug to eradicate this
c. Unilateral recurrent laryngeal nerve
infection? injury
a. Doxycycline
d. External laryngeal nerve injury
b. Mefloquine
e. Thyroid cyst
Ans. The key is D. External laryngeal nerve injury. facies and CXR shows absent thymic
[External laryngeal nerve functions to tense the shadow. What is the most likely dx?
vocal cords by activating the cricothyroid muscle, a. Downs syndrome
increasing pitch]. b. Fragile X syndrome
181. A 28yo woman at 39wk gestation is c. DiGeorge syndrome
d. Marfans syndrome
in labor. She develops abdominal pain and
HR=125bpm, BP=100/42mmHg, Q. 1. What is the key?
temp=37.2C and saturation=99%. Exam: Q. 2. What are the points in favour?
lower abdomen is exquisitely tender. Ans. 1. The key is C. DiGeorge syndrome.
CTG=prv normal, now showing reduced Ans. 2. Points in favour: i) Early age of onset ii)
variability and late deceleration develops abnormal facies iii) absent thymic shadow on
with slow recovery. She has had 1 prv LSCS Chest X-ray iii) history of recurrent infection
for a breech baby. Choose the most [Infections are common in children due to
appropriate CS complication for this lady? problems with the immune system's T-cell-
a. Endometritis mediated response that in some patients is due
b. UTI to an absent or hypoplastic thymus]. [in
c. Urinary tract injury newborne can be recognized by convulsions from
d. Pleurisy hypocalcaemia due to malfunctioning parathyroid
e. Uterine rupture glands and low level of parathyroid hormones].

Ans. The key is E. Uterine rupture. [Features of 184. A 30yo man presents with deep
shock, exquisitely tender lower abdomen with penetrating knife wound. He said he had
abdominal pain, previously normal CTG now TT when he left school. What will you do
becoming non-reassuring and previous history of for him now?
LSCS makes the diagnosis of uterine rupture]. a. Human Ig only
b. Human Ig and TT
182. An 8m infant presented with c. Full course of tetanus vaccine only
FTT(Failure to thrive) and constipation. d. Human Ig and full course of tetanus
Exam: large tongue and fam hx of vaccine
prolonged neonatal jaundice. What is the e. Antibiotic
most likely dx?
a. Downs syndrome Ans. The key is B. Human Ig and TT. It is a wrong
b. Fragile X syndrome key!! According to UK greenbook correct key is A.
c. Praderwilli syndrome Human Ig only.??????
d. DiGeorge syndrome
185. A 32yo previously healthy woman
e. Congenital hypothyroidism
has developed pain and swelling of both
Q. 1. What is the key? knees and ankles with nodular rash over
Q. 2. What are the points in favour? her shins. As part of the inv a CXR has
Ans.1. The key is E. Congenital hypothyroidism. been performed. What is the single most
Ans. 2. Points in fevour:i) FTT (failure to thrive) ii) likely CXR appearance?
constipation iii) macroglossia iv) prolonged a. Apical granuloma
neonatal jaundice. b. Bilateral hilar lymphadenopathy
c. Lobar consolidation
183. A 3m infant has presented with d. Pleural effusion
recurrent infections. He has abnormal e. Reticular shadowing in the bases
Q. 1. What is the key? c. Hydrocele
Q. 2. What is the name of this condition? What d. Teratoma
are the points in favour? e. Testicular torsion

Ans. 1. The key is B. bilateral hilar Ans. The key is A. Epididymal cyst. [the location of
lymphadenopathy. upper pole of the posterior part of testis is the
Ans. 2. The name is Lofgren’s syndrome. It is the common site for epididymal pathology].
triad of i) erythema nodosum ii) bilateral hilar
lymphadenopathy and iii) arthralgia. An epididymal cyst is a fluid-filled cyst found in
Lofgren's syndrome is an acute form of the epididymis, a coiled tube in the back of the
sarcoidosis characterized by erythema nodosum, testicle that stores and transports sperm. This
bilateral hilar lymphadenopathy (BHL), and creates a benign lump in the testicle and can be
polyarthralgia or polyarthritis. This syndrome is
common among Caucasians . very common with increasing age. An epididymal
cyst doesn't require treatment unless it causes
pain or discomfort, may need surgery.
186. A neonate’s CXR shows double
bubble sign. Exam: low set ears, flat
occiput. What is the most likely dx?
a. Downs syndrome
189. A young footballer has collapsed
b. Fragile X syndrome during a game. During initial evaluation:
c. Turner’s syndrome RR=14/min, pulse=88bpm,
d. DiGeorge syndrome BP=110/70mmHg. He seems to be
sweating and muttering some
Q. 1. What is the key? incomprehensible words. What is the most
Q. 2. What double bubble sign indicate? imp next step?
Ans. 1. The key is A. Down’s syndrome. a. CT
Ans. 2. Double bubble sign indicate duodenal b. MRI
atresia. c. Blood sugar
d. Body temp
187. A 19yo boy complains of itching on e. IV fluids
the site of insect bite. What is the single
most appropriate management? Ans. The key is C. Blood sugar. [Normal vital signs,
a. Penicillin oral sweating and collapse during exercise gives
b. Doxycycline oral suspicion of hypoglycemia which is very easy to
c. Oral antihistamine treat but fatal if left unrecognized. So most
d. Oral ciprofloxacin important next step is to check the blood sugar].
e. Reassurance 190. A 45yo waitress complains of pelvic
Ans. The key is C. Oral antihistamine. pain which worsens pre-menstrually and
on standing and walking. She also
188. A man presents with scrotal swelling, complains of post-coital ache. Select the
the swelling is cystic and is non-tender. It most likely cause leading to her
is located in the upper pole of the symptoms?
posterior part of the testis. What is the a. PID
most likely dx? b. Endometritiosis
a. Epididymal cyst c. Pelvic congestion syndrome
b. Testicular ca d. Adenomyosis
e. Premature ovarian failure c. Vocabulary consists of only 2 meaningless
words
Ans. The key is C. Pelvic congestion syndrome. d. She can’t make a sentence
[Women with pelvic congestion syndrome
e. None
experience a constant pain that may be dull and
aching, but is occasionally more acute. The pain is Q. 1. What is the key?
worse at the end of the day and after long Q. 2. How many words is told clearly by a 15
periods of standing, and sufferers get relief when month old child?
they lie down. The pain is worse during or after Ans. 1. The key is C. Vocabulary consists of only 2
sexual intercourse, and can be worse just before meaningless words.
the onset of the menstrual period. Cause is
Ans. 2. At 15 months the child can clearly say 5
attributed to female hormone estrogen induced
words and his first meaningful clear word he says
vascular (venous) changes. So it is usually seen in
at 12 months.
females].

Pelvic congestion syndrome (PCS) is a chronic 193. A 35yo lady who has been using IUCD
condition that occurs in women when varicose for one year now complains of pelvic pain
veins form below the abdomen within the pelvic and heavy painful periods. Select the most
region. Varicose veins are veins that become likely cause leading to her symptoms?
swollen, twisted, and lengthened as a result of a. PID
poor vein function, Pelvic congestion syndrome b. Endometriosis
occurs when varicose veins develop around the c. Adenomyosis
ovaries, similar to varicose veins that occur in the d. Fibroids
legs … Possible treatment options include: e. Asherman syndrome
Gonadotropin-releasing hormone drugs, which
block ovarian function and may relieve pain. Q. 1. What is the key?
Progestin hormone drugs, which may relieve Q. 2. What points favour your diagnosis?
pain. Procedures to shut off damaged veins Ans. 1. The key is A. PID.
(sclerotherapy, embolization) Ans. 2. The given picture may have D/D of PID or
191. A 37yo female had a fall with fibroid. As IUCD is a risk factor for PID, it is the
most likely diagnosis of given picture.
outstretched hand, presented with dinner
Menorrhagia for fibroids are usually painless].
fork deformity and tenderness over the
right arm. What is the single most
associated nerve injury?
a. Axillary nerve
b. Radial nerve
c. Musculocutaneous nerve
d. Median nerve 194. The dx cells of Hodgkin disease are:
e. Ulnar nerve
a. T-cells
Ans. The key is D. Median nerve. [Median nerve is b. R-S cells
the nerve injured in Colle’s fracture]. c. B-cells
d. Macrophages
192. A mother comes with her 15m child. e. Auer rods
Which of the following will bother you?
Ans. The key is B. R-S cells. [Diagnostic cell in
a. Shies away from strangers Hodgkins disease is Reed-Sternberg cells].
b. Can walk but not run
195. A 16yo girl is admitted after taking a
paracetamol OD 4 h ago. She has
consumed large amounts of alcohol. Her
plasma paracetamol conc is just below the
conc that would suggest tx. What should
be the tx option for her? 197. A young girl complains of episodic
a. Refer to psychiatry ward headaches preceded by fortification
b. Refer to medical ward spectra. Each episode last for 2-3 days.
c. N-acetylcystine During headache pt prefers quiet, dark
d. Serum plasma paracetamol room.
e. No further investigation What is the tx of choice for acute stage?
a. Paracetamol
Ans. The key is A. Refer to psychiatry ward. [Short
b. Aspirin
term acute alcohol consumption causes enzyme
c. Sumatriptan
inhibition as in present case and even then
paracetamol level is under tx level. So the d. Gabapentin
patients drug level is in safe side but e. Cafergot
simultaneous drug overdose and alcohol
Ans. The key is B. Aspirin. [OHCM, 9th Eition, page-
consumption needs psychiatric evaluation and
462 where NSAIDS like ketoprophen or
hence the option here is A].
dispersible aspirin 900 mg/6 hr are recommended
196. A 64yo woman has been on HRT for as treatment in acute stage. Dx is migraine with
9yrs. She had regular withdrawal bleeds aura].
until 3 yrs ago and since then has been
taking a no bleed prep. Recently she
noticed a brown vaginal discharge. 198. A 60yo pt recovering from a surgery
Choose the single most appropriate initial for toxic goiter is found to be hypotensive,
inv? cyanosed in the the RR. Exam: tense neck.
a. Cervical smear There is blood oozing from the drain.
b. High vaginal swab What is the most likely dx?
c. TFT a. Thyroid storm
d. Transvaginal US b. Reactionary hemorrhage
c. Secondary hemorrhage
Q. 1. What is the key? d. Primary hemorrhage
Q. 2. Why this test will be done? e. Tracheomalacia
Ans. 1. The key is D. Transvaginal US.

Ans. 2. To determine the endometrial thickness!


Ans. The key is B. Reactionary haemorrhage. [in
In a postmenopausal woman with vaginal
the recovery room, cyanosis, hypotension, tense
bleeding, the risk of cancer is approximately 7.3%
neck, woozing of blood from drain; all these goes
if her endometrium is thick (> 5 mm) and < 0.07%
in favour of reactionary haemorrhage].
if her endometrium is thin (≤ 5 mm).

In postmenopausal women without vaginal


bleeding, the risk of cancer is approximately 6.7% 199. A 33yo man is hit by a car. He loses
if the endometrium is thick (> 11 mm) and 0.002% consciousness but is found to be fine by
if the endometrium is thin (≤ 11 mm). the paramedics. When awaiting doctors
review in the ED he suddenly becomes 201. A 29yo teacher is involved in a tragic
comatose. What is the most likely dx? RTA. After that incident, he has been
a. SAH suffering from nightmares and avoided
b. Subdural hemorrhage driving on the motorway. He has been dx
c. Intracerebral hemorrhage with PTSD.
d. Extradural hemorrhage What is the most appropriate
management?
a. CBT Cognitive behavioral therapy
Ans. The key is D. Extradural haemorrhage. [Age b. Diazepam
33 (younger age), considerable head trauma, and c. Citalopram
lucid interval (present in both extradural and d. Dosalepin
subdural) are the points in favour]. e. Olanzepin
200. A 77yo male presents with hx of Ans. The key is A. CBT. [CBT is the treatment of
enuresis and change in behavior. Exam: choice in PTSD].
waddling gait. What is the most likely dx?
a. Subdural hemorrhage 202. A 5yo child presents with fever. He
b. Brain tumor looks pale. His parents say he always feels
c. Normal pressure hydrocephalus tired. On exam: orchidomegaly &
d. Psychotic depression splenomegaly. Labs: WBC=1.7, Hgb=7.1,
Plt=44.
Ans. The key is C. Normal pressure
What is the diagnosis?
hydrocephalus. [age (usually occurs in 60s or 70s),
a. ALL
loss of bladder control (enuresis), waddling gait
b. CLL
and behavior change are all features of normal
c. AML
pressure hydrocephalus].
d. CML
Normal-pressure hydrocephalus (NPH), also e. Hodgkins
called malresorptive hydrocephalus, is form of
communicating hydrocephalus in which excess Ans. The key is A. ALL. [normally in ALL CBC shows
cerebrospinal fluid (CSF) occurs in the ventricles, raised WBC, low RBC and low platelet; but it is
and with normal or slightly elevated also possible to all cell lines to be depressed, as is
cerebrospinal fluid pressure. As the fluid builds the presented case].
up, it causes the ventricles to enlarge and the
pressure inside the head to increase, compressing
203. A 6wk child is brought in with
surrounding brain tissue and leading to vomiting, constipation and decreased
neurological complications. The disease presents serum K+. What is the dx?
in a classic triad of symptoms, which are urinary
a. Pyloric stenosis
incontinence, dementia, and gait deviations… For
b. Duodenal atresia
suspected cases of NPH, CSF shunting is the first-
line treatment. The most common type used to c. Hirschsprung disease
treat NPH is ventriculoperitoneal (VP) shunts… No d. Achalasia cardia
medications are effective for primary NPH. e. Tracheo-esophageal fistula
Acetazolamide and other diuretics are not
Ans. The key is A. Pyloric stenosis. [why not
recommended except for limited use in patients
duodenal atresia? Pyloric stenosis is much more
who are not candidates for placement of a shunt.
commoner than duodenal atresia; in duodenal
atresia the vomitus should contain bile, which is respiratory alkalosis. If you continue breathing
not the case in pyloric stenosis]. and rebreathing in paper bag it will allow CO2
concentration to rise in paper bag and as you
204. A 17 yo girl had an episode of rebreath this again and again you will regain
seizure. Contraction of muscles started some washed out CO2 and thus relief to this
from around the interphalangeal joints, alkalosis].
Ans. 2. The girl may have anxiety disorder when it
which spread to the muscles of wrist and
precipitates leads to hyperventilation syndrome.
elbow.
X
Choose possible type of seizure?
a. Grand mal
Hyperventilation syndrome is a pattern of
b. Tonic clonic breathing where you breathe more quickly and
c. Myoclonic deeply than normal. This can be very
d. Absent upsetting. If it goes on for some time, it can
cause the carbon dioxide level in the blood to
Ans. The key is C. Myoclonic. [seizers associated get too low. This can lead to symptoms all
with contraction of specific muscle group is seen over the body.
in myoclonic seizers].
Symptoms that may occur along with
205. 46yo man, known case of chronic GN hyperventilation include:
presents to OPD. He feels well. BP =
140/90mmHg. Urine dipstick: protein ++,  lightheadedness
blood ++ and serum  numbness or tingling in the fingers
creatinine=106mmol/L. Which medication  a pounding heart
can prevent the progression of this dx?  a feeling that air is not getting into the
a. ACEi lungs
 a headache
b. Diuretics
 anxiety
c. Cytotoxic meds
d. Longterm antibiotics
e. Steroids

Ans. The key is A. ACEI. [renal impairment is


delayed by ACEI].

206. A 23 yo girl presented with perioral 207. A 25 yo woman has been feeling
paresthesia and carpopedal spasm 20 anxious and nervous for the last few
mins after a huge argument with her months. She also complains of
boyfriend. What is the next step for this palpitations and tremors. Her symptoms
pt? last for a few minutes and are very hard
a. SSRI to control. She tells you that taking
b. Diazepam alcohol initially helped her relieve her
c. Rebreath into a paper bag symptoms but now this effect is wearing
d. Propranolol off and she has her symptoms even after
e. Alprazolam drinking alcohol. What is the dx?
a. Panic disorder
Q. 1. What is the key? b. Depression
Q. 2. What is the likely diagnosis? c. OCD
Ans. 1. The key is C. Rebreathin in paper bag. d. Alcohol addiction
[hyperventilation causes CO2 washout and
e. GAD 210. A 2 month child with diarrhea and
vomiting for 6 days is brought in looking
Ans. The key is A. Panic disorder.
lethargic. What is the appropriate initial
208. A 2yo child is very naughty. His inv?
teacher complains that he is easily a. BUE
distracted. His parents say that he can’t b. Random blood sugar
do a particular task for a long time. He c. CBC
sometimes hurts himself and breaks many d. CXR
things. This causes many troubles at e. AXR
home. What is the dx? Ans. The key is A. BUE. [Electrolyte imbalance
a. ASD
should be checked in this baby presenting with
b. Dyslexia diarrhea and vomiting for considerable time].
c. ADHD
d. Antisocial personality disorder 211. A 72 yo man fell while shopping and
e. Oppositional defiant hurt his knee. His vitals are fine. He speaks
in a low voice and is very slow to give
Ans. The key is C. ADHD (Attention deficit answers. What is the most probable dx?
hyperreactive disorder). Age??? a. Alzheimers
b. Vascular demetia
c. TIA
d. Pseudo-dementia
209. A 79 yo lady who is otherwise well e. Picks dementia
recently started abdominal pain. She is
Q. 1. What is the key?
afebrile and complains that she passed air Q. 2. What are the points in favour?
bubbles during urination. A urethral
catheter showed fecal leakage in the Ans. 1. The key is A. Alzheimers.
urinary bag. What is the likely pathology? Ans. 2. Points in favour: i) age 72 yrs ii) fall iii) loss
a. Diuretics
or slowness of speech.
b. CD
c. Rectosigmoid tumor why not vascular? in vascular: i) confusion ii)
d. Large bowel perforation disorientation iii)loss of vision
e. UC why not pseudodementia? in pseudo i) onset is
short and abrupt ii associated depression

why not picks i) dementia and aphasia


Ans. The key is B. CD. [debate came that Crohn’s
disease cannot occur in 79 yrs but this is not the Why not TIA? In TIA complete resolution of
case! “Crohn’s disease can occur at any age, but is symptom!! But here symptoms are persistent.
most frequently diagnosed in people ages 15 - 35.
About 10% of patients are children under age 18”. 212. A 47 yo man met with a RTA. He has
[http://www.nytimes.com/health/guides/disease multiple injuries. Pelvic fx is confirmed. He
/crohns-disease/risk-factors.html]. So I think it has not passed urine in the last 4 hrs.
can occur in this age also and the feature like What is the next appropriate
fistula is a common association of CD]. management for this pt?
a. Urethral catheter
b. Suprapubic catheter
c. IV fluids
d. IV furosemide
e. Insulin

Q. 1. What is the key?


Q. 2. What is the reason of this management? 215. A 71 yo man presents with coarse
tremor. He is on some meds.
Ans. 1. The key is B. Suprapubic catheter. Which one can be the reason for the
Ans. 2. In pelvic fracture there is chance of
tremor?
a. Lithium
urethral rupture and hence displacement of
b. Diazepam
urethral catheter while try to place it.
c. Fluoxetine
213. A 49 yo pt presents with right d. Imipramine
hypochondriac pain. Inv show a big e. Haloperidol
gallstone. What is the most appropriate
Ans. The key is A. Lithium. Actually in therapeutic
management?
dose lithium causes fine tremor but in toxic dose
a. Lap Cholecystectomy
it causes coarse tremor. So the probable answer
b. Reassure
is lithium.
c. Low fat diet
d. Ursodeoxycholic acid 216. A young woman complains of
e. Emergency laparotomy diarrhea, abdominal cramps and mouth
ulcers. AXR shows distended transverse
Q. 1. What is the key?
Q. 2. Points in favour? colon with globet cell depletion on rectal
biopsy.
Ans. 1. The key is A. Lap Cholecystectomy. What is the most probable dx?
a. CD
Ans. 2. i) as symptomatic only reassurence is not
b. UC
appropriate ii) as big stone ursodyoxycholic acid is
c. Bowel Ca
less effective iii) less invasiv is preferred so
d. Bowel obstruction
laparoscopic rather than laparotomy.
e. IBS
214. In a man who is neglected and
Q. 1. What is the key?
alcohol dependent, with high suicidal risk,
Q. 2. What are points in favour?
Which factor can increase this risk Ans. 1. The key is B. UC.
further?
a. Alcohol dependence Ans. 2. In UC there is goblet cell depletion and
b. SSRI less mucous production in contrast with CD
c. S]moking where there may be goblet cell hyperplasia and
d. Agoraphobia mucous secretion is not reduced. Please note
aphthous ulcer can develop in both CD and UC.
e. Court involvement
217. After eating a cookie at a garden
Ans. The key is A. Alcohol dependence. This is
wrong key!! Correct key should be B. SSRI. [He is party, a child began to cough and went
already alcohol dependent. So further (additional) blue. The mother also noticed that there
risk factor is SSRI (SSRI has well known risk of were swollen patches on the skin. What is
developing suicidal ideation]. the dx?
a. Allergic reaction
b. Aspiration of food dormia basket.
c. Cyanotic heart disease Stones > 2cm/large/multiple/complex:
d. Trachea-esophageal fistula Percutaneous nephrolithotomy.
e. Achalasia cardia
220. A footballer has been struck in the
Ans. The key is A. Allergic reaction. groin by a kick and a presents with severe
pain and mild swelling in the scrotum.
218. A 70 yo man presents with balance What is the most appropriate next step?
difficulties, vomiting and nausea. a. USG
Which of the following is the best inv? b. Doppler
a. MRI cerebellum c. Exploratory surgery
b. CT cerebellum d. IV fluids
c. Skull XR e. Antibiotics
d. LP
e. Blood culture

Ans. The key is A. MRI cerebellum. [Balance Ans. The key is C. Exploratory surgery. [To exclude
difficulties, vomitin and nausea suggests torsion].
cerebellar lesion. In posterior fossa lesion MRI is
preferred].
221. A 47 yo ex-soldier suffers from low
219. A 2y pt presents with colicky pain mood and anxiety. He can’t forget the
which radiates from loin to groin. He images he faces before and has always
complains of similar episodes in the past. had flashbacks. He is not able to watch
Inv has been done and 7mm stone was the news because there are usually some
found in the ureter. reports about war. What is he suffering
What is the most appropriate from?
management? a. Depression
a. Percutaneous nephrolithiotomy b. PTSD
b. Open surgery c. Panic attack
c. Ureteroscopy or laser d. Agoraphobia
d. Conservative tx e. GAD
e. ESWL

Ans. The key is B. PTSD. [repeated flashbacks and


Q. 1. What is the key? tendency to avoid the thoughts of stressor is
Q. 2. What treatments are recommended for diagnostic of PTSD].
different sized stones?
Ans. 1. The key is E. ESWL. Probably a wrong key!
Upto 10mm stone conservative tx. So correct key
should be D. conservative tx. 222. A 36 yo woman has recently spent a
lot of money on buying clothes. She goes
Ans. 2. Stones < 5mm: pass spontaneously, out almost every night with her friends.
Increase fluid intake.
She believes that she knows better than
Stones 5mm-10mm /pain not resolving: medical
her friends, so she should choose the
expulsive therapy---> Nifedipine or
Tamsulosin(and/or prednisolone).
restaurant for eating out. She gave hx of
Stones 10mm-2cm: ESWL or Ureteroscopy using having low mood at 12 yo. What’s the dx?
a. Mania bark like cough is diagnostic of croup! So the
b. Depression correct key should be a. Corticosteroids. [high
c. Bipolar affective disorder fever, bark-like cough, drooling in a child suggest
d. Borderline personality disorder croup which is treated with corticosteroids].
e. Dysthymia

225. A 78yo lady on warfarin for atrial


Ans. The key is C. Bipolar affective disorder. fibrillation lives in a care home. She
[Initial depressive episode (may be befor a long) presents with hx of progressive confusion
followed by mania is bipolar affective disorder]. for three days. She was also noticed to
have bruises on her arms. INR = 7. What is
the most probable dx?
223. A 28 yo female presents with a 3m hx a. Alzheimers
of diarrhea. She complains of abdominal b. Delirium
discomfort and c. Subdural hemorrhage
passing stool 20x/day. Exam=febrile. d. Vascular dementia
Barium enema shows cobblestone e. Pick’s dementia
mucosa. What is the
most likely dx?
a. Ameoba Ans. The key is C. Subdural haemorrhage. [Age 78
b. Colon Ca yrs, living in a care home where unnoticed trivial
c. GE injury is possible (like fall), warfarine and high INR
d. CD is potential risk factor of subdural haemorrhage
e. UC suggested by bruises on arms also].

Ans. The key is D. CD. [Hx of diarrhea, abdominal


226. A 28 yo drug user presents to the ED
discomfort, and patient being febrile indicate gut with collapse and anuria. His serum K+ =
inflammation and cobblestone appearance on 7.5mmol/L. CXR = early pulmonary
barium enema is suggestive of CD]. edema. What is the next appropriate
management?
a. Urgent hemodialysis
224. A child is brought in with high grade b. IV calcium gluconate
fever, runny nose and bark-like cough. He c. IV insulin + dextrose
is also drooling. What is the most d. Furosemide
appropriate tx for this child? e. IV NS 0.9%
a. Corticosteroids
b. Paracetamol
c. Adrenaline nebulizer Ans. The key is B. IV calcium gluconate. [IV
d. IV antibiotics calcium gluconate isgiven to ]. Protectthe heart
e. Intubation under GA from lethal arrhythmia or cardiac arrest from high
potassium level and used to buy time while
definitive management is being taken. Actually
calcium gluconate don’t lower the potassium
Ans. The key is E. Intubation under GA. This is a
wrong key! In epiglottitis there is no cough and
level that much but protect the heart from with no motor or retinal symptoms and
arrhythmia]. completely reversible symptoms
 (D) At least one of the following: At
least 1 aura symptom occurring
227. A 32 yo woman suffers an episode of gradually over 5 minutes or more
and/or 2 or more symptoms occurring
severe occipital headache with vomiting in succession over 5 minutes or more,
and loss of consciousness. She is brought each aura symptom lasts more than 5
to the hospital where she is found to be minutes, but less than 60 minutes, at
conscious and completely alert. O/E: least 1 aura symptom is unilateral
normal pulse & BP. No abnormal  (E) Migraine without aura begins
neurological signs. during the aura or within 1 hour
What is the next step in management?
Treatment of acute attack is symptomatic. An
a. Admission for observation
acute attack of a basilar migraine is usually
b. CT brain
managed with NSAIDs and anti-emetics like
c. MRI head
prochlorperazine or metoclopramide. Triptans
d. Reassurance and discharge and ergotamines were excluded in the earlier
e. XR skull trials for the treatment of a basilar migraine and
have not been well studied. Aura symptoms
should be used as a signal for initiating early
Ans. The key is B. CT brain. [basilar migraine can acute treatment. Verapamil and topiramate are
cause severe headache and LOC. But there occurs the most commonly used medications for
no neurological deficit and on recovering from preventing migraine with brainstem aura.
unconsciousness becomes completely alert. But Propranolol has traditionally been avoided as it
to diagnose basilar migraine there should at least can affect the cerebral blood flow.
history of two migraine attacks with aura. As here
diagnostic criteria of basilar migraine is not
fulfilled we can not discharge the patient without
neuroimaging like CT or MRI].

Basilar migraine is now believed to be a type of


migraine with aura. Basilar migraine was initially
thought to have vascular pathology caused by 228. A 25 yo woman was brought to the
short-term spasms of the basilar artery. ED by her boyfriend. She has many
superficial lacerations on her forearm. She
The International Classification of headache
is so distressed and constantly says her
disorders outlined the following criteria for
the diagnosis of basilar migraine. boyfriend is going to end the relationship.
She denies trying to end her life.
 (A) Symptoms not attributed to What is the most likely dx?
another disorder a. Acute psychosis
 (B) At least 2 attacks that fulfill b. Severe depression
criteria C, D, or E c. Psychotic depression
 (C) Aura with more than one of the d. Borderline personality disorder
following symptoms: dysarthria, e. Schizophrenia
vertigo, tinnitus, hearing impairment,
diplopia, ataxia, decreased level of
consciousness, bilateral paresthesia,
Ans. The key is D. Borderline personality disorder. 231. After eating a cookie at a garden
[ Borderline personality disorder: Act impulsively party, a child began to cough and went
and develop intense but short-lived emotional blue. The mother also noticed that there
attachment to others. They are usually attention
were swollen patches on the skin.
seekers but not suicidal].
What is the initial management?
a. OTC antihistamine
b. Oxygen
229. A young woman was brought to the c. Bronchodilators
hospital. On exam she has low d. Epinephrine IM
temperature and tremor. She says when e. Nebulized epinephrine
she closes her eyes, she can see colors.
What drug has been used? Ans. The key is D. Epinephrine IM [anaphylaxis
a. Amphetamines with partially blocked airway].
b. LSD
c. Cocaine 232. A 63 yo female is noted to have left
d. Heroine pupil irresponsive to light and is dilated.
e. Ecstasy What is the most probably dx?
a. Pontine hemorrhage
b. Subdural hemorrhage
c. Cerebellar hemorrhage
Ans. The key is B. LSD. [LSD can cause colour in
d. Extradural hemorrhage
vision].Ly sergic acid diethylamide (LSD),also
known a hallucinogenic drug. Effects typically e. Subarachnoid hemorrhage
include altered thoughts, feelings, and awareness
Ans. [[[The key is D. Extradural hemorrhage. [This
of one's surroundings. Many users see or hear is 3rd nerve palsy. Now SAH, EDH and Subdural
things that do not exist. Dilated pupils, increased hematoma can cause raised intracranial pressure
blood pressure, and increased body temperature leading to 3rd nerve palsy. Let's look at
are typical. Effects typically begin within half an prevalence of them!
hour and can last for up to 12 hours. It is used Subdural hematoma, 7.35 cases per 100,000
mainly as a recreational drug or for spiritual population in those aged 70-79 years.
reasons SAH, affects 6-9 people per 100,000 of the
population per year.
So definitely subdural hematoma is much too
common than SAH. Again Chronic subdural
230. A lady comes in severe liver disease hematoma is seen in such an elderly patient and
and hematemesis. Her INR is >10. What acute subdural hematoma is unlikely. So though
should she be given? in elderly extradural hematoma is less common it
remains the most logical option in this question].
a. FFP [Opinion from neurosurgeon of this group].
b. Steroids
c. Whole blood
d. IV fluids
233. A 28yo business executive presents
e. Vit K
at the GP asking for some help because
she has been arguing with her boyfriend
frequently. She is worried about her
Ans. The key is A. FFP.
weight, and she thinks she may be fat. She
has been on a diet and lost 7 kgs in the
last 2 months on purpose. She is eating
less. She used to do a lot of exercise. Now
she says she’s feeling down, has some
235. A 37 yo man who has many
insomnia and feels tired and without
convictions and has been imprisoned
energy. She has not showed up at work.
many times has a hx of many unsuccessful
She is worried because recently she got a
relationships. He has 2 boys but doesn’t
loan to buy a luxury car. She can’t be
contact them.
fired. She complains about her low mood.
What is the most probable dx?
She thinks this is weird because she used a. Borderline personality disorder
to be extremely productive. She used to b. Schizophrenia
work showing an excellent c. Avoidant personality disorder
performance at the office. She even d. Histrionic personality disorder
received compliments from her boss. How, e. Antisocial behavior disorder
she says her boyfriend is angry because
her apartment is a chaos. Usually she
spends a lot of time cleaning it, even upto Ans. Ans. The key is E. Antisocial behavior
3 AM. She liked it to be perfect, but not disorder. [Antisocial personality disorder is a
it’s a mess. On exam: BMI=23, no other particularly challenging type of personality
signs. What is the most probably dx? disorder, characterised by impulsive,
a. Anorexia nervosa irresponsible and often criminal behavior].
b. Bipolar disease
c. Binge eating disorder
d. Hyperthyroidism 236. A 60 yo man has a pathological rib fx.
e. Schizophrenia He also complains of recurrent infection.
BMA is done.
Labs: Ca2+ = 3.9mmol/L and ALP =
Ans. The key is B. Bipolar disorder. [Now she is 127u/L. what type of cell would be found
depressed but before hypomanic which makes in abdundance in the marrow smear?
the likely dx of bipolar disorder]. a. Plasma cell
b. Myeloid cell
c. Bence-jones protein
234. A woman brought her husband d. Megakaryocytes
saying she wants the ‘thing’ on his e. Reticulocytes
forehead removed. The husband is
refusing tx saying it improves his thinking.
What is the most appropriate next step? Q. 1. What is the key.
a. Assess his mental capacity to refuse tx Q. 2. What is the diagnosis?
b. Remove lesion Q. 3. What are the points in favour of diagnosis?
c. Refer to ED
d. Mini-mental state exam Ans. 1. The key is A. Plasma cell.
e. Refuse surgery and send pt back
Ans. 2. The diagnosis of multiple myeloma.

Ans. 3. Points in favour: i) age 60 yrs ii)


Ans. The key is A. Assess his mental capacity to pathological rib fracture (from metastases) iii)
refuse treatment. recurrent infection (due to  B cell dysfunction
(manifested as hypogammaglobulinemia), Ans. The key is B. X-ray. [As there is no
numerical and functional abnormalities of T cells, neurological deficit we can exclude any fracture
and dysfunction of natural killer cells), iv) raised by x-ray first]. [Diagonal x ray means ,oblique
calcium level. view of cervical spine. By this view we can assess
facet joint arthopathy. This doesn't related to
RTA].
237. A child presents with blue marks on
the sclera, short stature and heart
murmur. What is the dx? 240. A young female who has many
a. Osteogenesis imperfect superficial lacerations was brought into
b. Hypopituitarism the ED by her boyfriend for superficially
c. VSD lashing her upper arm. She is adamant
d. Achondrogenesis and screaming that she is not suicidal but
e. Dwarfism scared her boyfriend wants to leave her.
What is the dx?
a. Acute psychosis
Ans. The key is A. Osteogenesis imperfecta. b. Severe depression
c. Obsessive
d. Bipolar
238. A 5month child can’t speak but e. Borderline personality
makes sounds. She can hold things with f. Schizophrenia
palm, not fingers. Can’t sit independently
but can hold her hand and sit when
propped up against pillows. How’s the Ans. The key is acute psychosis. Probably this is
child's development? wrong key! Correct key should be E. Borderline
a. Normal personality disorder. [Borderline personality
b. Delayed speech disorder (BPD), also known as emotionally
c. Delayed sitting unstable personality disorder, is a long term
pattern of abnormal behavior characterized by
d. Delayed motor development
unstable relationships with other people,
unstable sense of self, and unstable emotions.[3]
[4] There is often an extreme fear of
Ans. The key is A. normal abandonment, frequent dangerous behavior, a
feeling of emptiness, and self-harm].

239. A 27 yo woman has hit her neck in an


RTA without complains of tingling or 241. A 22yo woman was brought by her
motor loss. What is the next most boyfriend with multiple superficial
appropriate inv? lacerations. There are scars of old cuts on
a. MRI her forearms. She is distressed because he
b. XR wants to end the relationship. She denies
c. CT cervical suicide. What is the most likely dx?
d. Diagonal XR a. Acute psychosis
b. Borderline personality disorder
c. Severe depression
d. Schizoid personality 244. An 11m baby had an apnea event.
e. Psychotic depression The parents are worried that if something
like this happens in the future, how they
Ans. The key is B. Borderline personality.
[Borderline personality disorder (BPD), also
are to deal. Advise them about infant CPR.
a. Index and middle finger compression
known as emotionally unstable personality
b. Compression with palm of one hand
disorder, is a long term pattern of abnormal
behavior characterized by unstable relationships c. Compression with palm of two hands
with other people, unstable sense of self, and d. Compression with rescue breaths 30:2
unstable emotions.[3][4] There is often an e. Compression with rescue breaths 15:2
extreme fear of abandonment, frequent
dangerous behavior, a feeling of emptiness, and
self-harm]. Ans. The key is A. Index and middle finger
compression. D ?????

242. A 31yo single man lives with his 245. A teacher brings in a child who says
mother. He usually drives to work. He she fell down after hitting a table. On
always thinks when the traffic lights probing further, you decide that it was
change, his mother is calling him, so he most probably an absence seizure.
drives back home. What led you to this dx?
a. The child had not eaten since morning
What is the dx?
b. The child suddenly went blank and there
a. OCD
was up-rolling of eyes
b. GAD
c. The child started moving his fingers
c. Schizophrenia
uncontrollably before he fell
d. Bipolar
d. The child’s body became rigid and then
e. Cyclothymia
started to jerk

Ans. The key is C. Schizophrenia. [ delusion of


Ans. The key is B. The child suddenly went blank
reference - he thinks that the changing traffic
and there was up-rolling of eyes.
lights are giving message to him].

243. A 56yo woman is known case of 246. A man has discharge from his left ear
pernicious anemia. She refuses to take after a fight. Where is the discharge
hydroxycobalamin IM as she is needle shy. coming from?
She asks for oral medication. a. CSF
Why will oral meds be not effective? b. Inner ear
a. Intrinsic factor def c. Outer ear
b. Malabsorption d. Brain
c. Irritated gastric mucosa
d. Lack of gastric acidity
Ans. The key is A. CSF. [probable fracture base of
Ans. The key is A. Intrinsic factor def. [Vitamin skull].
B12 cannot be absorbed without intrinsic factor].
247. A 40 yo manic depressive is noted to takes longer than before to finish tasks as
have high serum levels of lithium and she must constantly wash her hands.
profound What is the most appropriate
hypokalemia. His GP had started him on management?
anti-HTNs. Choose the single most likely a. CBT
cause? b. SSRI
a. Verapamil c. ECT
b. Amiodarone d. Antipsychotics
c. Ranitidine e. Desensitization
d. Lithium
Q. 1. What is the key?
e. Thiazide
Q. 2. What is the diagnosis?

Ans. The key is E. Thiazide. [Thiazide was Ans. 1. The key is A. CBT.
prescribed for Hpt and when lithium was Ans. 2. The diagnosis is OCD.
prescribed its level increased due to thiazide and
thiazide also caused hypokalemia resulting the
given picture].
250. A 61yo man underwent a surgery in
which ileal resection had been done. He
complains of fatigue, headache, and heart
248. A 74yo man presents with weakness
racing. Labs: MCV=108fL, Hgb=8.9g/dL.
in his arm and leg from which he
What is the most likely dx?
recovered within a few days and short a. Vit B12 def
term memory loss. He has an extensor b. Iron def
plantar response. He has similar episodes c. Folate def
2 years ago and became unable to identify d. Hemolytic anemia
objects and to make proper judgment. e. Anemia of chronic disease
What is the most appropriate dx?
a. Alcoholic dementia Q. 1. What is the key?
b. Pick’s dementia Q. 2. What are the points in favour?
c. Huntington’s disease
d. Alzheimer’s disease Ans. 1. The key is Vit. B12 deficiency.
e. Vascular dementia
Ans. 2. Vit B12 is absorbed mostly in ileum. [As
ileal resection is done B12 is not absorbed leading
to megaloblastic anaemia].
Ans. The key is E. Vascular dementia.
[hemiparesis, memory impairment, extensor
planter reflex, inability to identify objects, poor
judgment are features of vascular dementia].
251. A 7yo is brought by his mother who
says that he was well at birth but has
been suffering from repeated chest and GI
249. A nurse comes to you saying that she infections since then. She also says that he
has recently developed the habit of is not growing well for this age.
washing her hands after every 15-20 mins. What is the likely condition of this child?
a. CF
She is unable to conc on her work and
b. SCID
c. Primary Tcell immunodeficiency
d. Primary Bcell immunodeficiency
e. Malabsorption

Q. 1. What is the key?


Q. 2. What are the points in favour?

Ans. 1. The key is A. cystic fibrosis.

Ans. 2. CF involved in production of sweat,


respiratory mucous, digestive fluid and mucous.
These secretion becomes thick than normal
predisposing to lung and GI infections since birth.

252. A 3yo child has a high temp for 4


days and he had not seen a doctor. Then
mother notices rashes on buccal mucosa
and some around the mouth. 253. A 70yo lady presents with fever for
What is the most appropriate dx? 3d and confusion. There is no significant
a. Measles
PMH(PAST MEDICAL HX). What is the
b. Roseola infectiosum
most probable dx?
c. Rubella
a. Delirium
d. Chicken pox
b. Hypoglycemia
e. Impetigo
c. Alzheimers
d. DKA

Ans. The key is B. Roseola infectiosum. It is a


wrong key! The correct key should be A. Measles!
Ans. The key is A. Delirium. Delirium is an acute
[As the rash developed after 4 days fever the dx is
confusional state and declined cognitive function
measles!].
which involves changes in arousal (hyperactive,
hypoactive or mixed), perceptual deficits, altered
sleep-wake cycle, and psychotic features such as
hallucinations and delusions.

254. An obese mother suffers from OSAS-


Obstructive sleep apnea. Which of the
following inv is best for her?
a. ABG
b. Overnight pulse-oximetry
c. Polysomnography
d. EEG
Ans. The key is B. Overnight pulse-oxymetry. [It is Q. 1. What is the key?
already a diagnosed case of OSAS. So no need for Q. 2. What are the points in favour?
reconfirmation with polysomnography. If like to
know the current status or monitor overnight
Ans. 1. The key is E. Tetralogy of Fallot. It is a
pulse oxymetry is good]. wrong key!! Correct answer is C. tricuspid atresia.
Polysomnography, also called a sleep study, is a Ans. 2. Points in favour: i) tachypnoea over first 2
test used to diagnose sleep disorders. wks of life ii)progressive cyanosis iii) poor feeding
iv) holosystolic murmur of VSD.
Polysomnography records your brain waves, the
TOF … pulmonary stenosis murmur … ejection
oxygen level in your blood, heart rate and
systolic at the left mid and upper sternal border
breathing, as well as eye and leg movements
during the study 257. A 29yo woman who was dx to have
migraine presents with severe onset of
occipital headache. She lost her
255. A 28yo business man came to the consciousness. CT=normal. Neurological
sexual clinic. He was worried that he has exam=normal.
HIV infection. 3 HIV tests were done and What is the most appropriate
all the results are negative. After a few management?
months, he comes back again and claims a. Repeat CT
that he has HIV. What is the dx? b. MRI
a. Somatization c. LP
b. Hypochondriac d. XR
c. Mancheusens e. No inv required
d. OCD
e. Schizophrenia
Q. 1. What is the key?
Ans. The key is B. Hypochondriac. [worry about
Q. 2. What is the diagnosis?
having a serious illness].
Q. 3. What are the points in favour?
hypochondriac is someone who lives with the
fear that they have a serious, but undiagnosed Ans. 1. The key is E. No investigation required.
medical condition, even though diagnostic tests
show there is nothing wrong with them Ans. 2. The diagnosis is basilar migraine.

Ans. 3. Points in favour i) history of migraine ii)


severe occipital headache iii) LOC iv) CT normal v)
256. A 6wk child presents with progressive neurological examination is normal.
cyanosis, poor feeding, tachypnea over
the first 2 wks of life and holosystolic 258. A 19yo man has been happier and
murmur. What is the most appropriate more positive than usual, with more
condition? energy than he has ever felt before for no
a. ASD particular reason. He has been getting
b. VSD more work done at the office today and
c. Tricuspid atresia has been socializing with his friends as
d. PDA usual.
e. TOF What is the most likely dx?
a. Atypical depression d. AML
b. Marked depression e. Polycythemia
c. Bipolar syndrome
d. Psychosis
e. Hypomania Q. 1. What is the key?
Q. 2. Why it is so diagnosed?
Q. 1. What is the key?
Q. 2. What are the points in favour of this Ans. 1. The key is A. CML.
diagnosis?
Ans. 2. Points in favour of CML: i) Age 57 years ii)
weight loss iii) abdominal discomfort iv) anaemia
Ans. 1. The key is E. Hypomania. v) fever vi) marked splenomegaly.
Ans. 2. i) elevated mood ii) more energy than
before iii) getting more work done at the office
(loss of inhibition). These features are common 261. A baby born at 34 weeks with a heart
for both mania and hypomania!! Then why it is murmur is kept in the incubator for almost
not mania? It is not mania as in mania you will get
psychotic symptoms like i) delusion of grandeur ii)
4 weeks. There is no murmur at discharge.
auditory hallucinations, which are absent here! What is the likely cause of this murmur?
a. PDA
b. TOF
259. A 35yo female attempts suicide 10x. c. Aneurysm of sinus of Valsalva
There is no hx of psychiatric problems and d. Aorto-pulmonary septal defect
all neurological exams are normal. What e. AVM
is the best tx?
a. Problem focused tx
b. CBT Q. 1. What is the key?
Q. 2. Why it was present in this baby?
c. Antipsychotic
Q. 3. If it is present after birth what is the
d. Antidepressant
management?
e. ECT

Ans. 1. The key is A. PDA.


Ans. The key is A. Problem focused tx. [patient is
not psychotic and with normal neurology! So she Ans. 2. As it is more common in premature baby!
may getting some problem in family life, finance, Ans. 3. Management:
job or somewhere like this which she is not able
to cope with and that is leading to her suicidal  Indomethacin closes the connection in the
thoughts]. majority of cases

260. A 57yo man presents with weight  if associated with another congenital heart
defect amenable to surgery then prostaglandin E1
loss, tiredness, fever and abdominal
is useful to keep the duct open until after surgical
discomfort. Exam: spleen palpable up to
repair.
the umbilicus. Labs: WBC=127, Hgb=8.7,
Plt=138.
What is the most likely dx?
262. A 6yo girl who has previously been
a. CML
well presented with a hx of tonic-clonic
b. AML
c. CLL
seizures lasting 4mins. Her mother
brought her to the hospital and she
appeared well. She is a febrile and didn’t
lose consciousness during the episode of Q. 1. What is the key?
seizure. She has no neurologic deficit. Q. 2. What is the diagnosis?
What is the most appropriate inv for her? Q. 3. What are the points in favour of your
a. ABG diagnosis?
b. Serum electrolytes
c. ECG Ans. 1. The key is A. ECT.
d. Blood glucose
Ans. 2. The diagnosis is post purtum psychosis.
Q. 1. What is the key?
Q. 2. What are the points in favour? Ans. 3. Points in favour: i) onset 3 weeks after
childbirth ii) Depressive symptoms (very low
mood, insomnia) iii) thought of harming her little
baby.
Ans. 1. The key is B. Serum electrolyte. Electroconvulsive therapy (ECT) is a medical
Ans. 2. In epilepsy patient becomes unconscious. treatment most commonly used in patients with
The child has no previous illness and she was severe major depression or bipolar disorder that
conscious during the episode of seizure. So has not responded to other treatments. ECT
electrolyte imbalance may be the cause. involves a brief electrical stimulation of the brain
while the patient is under anesthesia.

263. A 60yo woman was found by her son.


She was confused and had urinary
265. A 65yo woman presents with
incontinence. She has recovered fully after
headache. She also complains of dizziness
6h with no neurological complaints.
and tinnitus. She has recently realized she
What is the most likely dx?
has visual problems. There is hx of burning
a. Stroke
sensation in fingers and toes. O/E:
b. Vestibular insufficiency
c. TIA
splenomegaly, itchy after hot bath. Labs:
d. Intracranial hemorrhage
RBC=87, Hgb=31.9, Plt=796. What is the
dx?
a. CML
Ans. The key is C. TIA. b. CLL
c. Polycythemia vera
264. A 34yo woman presents 3 weeks d. Myelofibrosis
after childbirth. She has had very low e. NHL
mood and has been suffering from lack of
sleep. She also has thought of harming
her little baby. What is the most Q. 1. What is the key?
appropriate management for this pt? Q. 2. What are the points in favour?
a. ECT
b. CBT
Ans. 1. The key is C. Polycythemia vera.
c. IV haloperidol
d. Paroxethine Ans. 2. Points in favour: i) hyperviscosity
e. Amitryptiline symptoms (headache, dizziness, tinnitus, visual
problem) ii) pruritus, typically after a hot bath, This is usually with a "cytoreductive agent"
iii)splenomegaly iv) RBC=87, Hb=31.9, Plt=796. (hydroxyurea, also known as hydroxycarbamide).

Polycythemia vera is an uncommon The JAK2 inhibitor, ruxolitinib, is also used to


myeloproliferative neoplasm in which the bone treat polycythemia
marrow makes too many red blood cells. It may
also result in the overproduction of white blood
cells and platelets.

Most of the health concerns associated with 266. A 29yo male brought to ED in
polycythemia vera are caused by the blood being unconscious state. There is no significant
thicker as a result of the increased red blood cells.
past hx. Which of the following should be
It is more common in the elderly and may be
done as the initial inv?
symptomatic or asymptomatic. Common signs
a. CT
and symptoms include itching (pruritus), and
b. Blood glucose
severe burning pain in the hands or feet that is
c. ABG
usually accompanied by a reddish or bluish
coloration of the skin. Patients with polycythemia d. MRI
vera are more likely to have gouty arthritis.. e. CBC
Physical exam findings are non-specific, but may
include enlarged liver or spleen, plethora, or
gouty nodules.. Ans. The key is B. Blood glucose.
Common findings include an elevated
hemoglobin level and hematocrit, reflecting the
increased number of red blood cells; the platelet 267. A 45yo woman comes with red,
count or white blood cell count may also be swollen and exudating ulcer on the nipple
increased. The erythrocyte sedimentation rate and areola of right breast with palpable
(ESR) is decreased due to an increase in zeta lump under the ulcer.
potential. Because polycythemia vera results What do you think is causing this skin
from an essential increase in erythrocyte condition?
production, patients have normal blood a. Inflammatory cells releasing cytokines
oxygenation and a low erythropoietin (EPO) level. b. Infiltration of the lymphatics by the
Phlebotomy is the treatment, which often may be carcinomatous cells
combined with other therapies. c. Infiltration of the malignant skin cells to
the breast tissue
Low dose aspirin (75–81 mg daily) is often
prescribed. Research has shown that aspirin
reduces the risk for various thrombotic
Ans. The key is B. Infiltration of the lymphatics by
complications.
the carcinomatous cells.
Bone marrow transplants are rarely undertaken
in people with polycythemia.

Chemotherapy for polycythemia may be used, 268. A 20yo young lady comes to the GP
either for maintenance, or when the rate of for advice regarding cervical ca. she is
bloodlettings required to maintain normal worried as her mother past away because
hematocrit is not acceptable, or when there is of this. She would like to know what is the
significant thrombocytosis or intractable pruritus. best method of contraception in her case?
a. POP
b. Barrier method would be the most appropriate inv for
c. IUCD her?
d. COCP a. Hysterectomy
e. IUS b. Endometrial biopsy
c. CBC
d. High vaginal swab
Ans. The key is A. POP(progestogen-only pill). e. Coagulation profile
Probably wrong key! Correct key should be B.
Barrier method! [spermatozoa itself acts as a Ans. The key is E. Coagulation profile.
carcinogen!!! So barrier method is the best [Transvaginal US is normal i.e. no endometrial
protection from the given option!!]. hyperplasia, no fibroid or obvious cause for heavy
bleeding was found. So now most appropriate
investigation should be coagfulation profile].

269. A 66yo man, an hour after


hemicolectomy has an urine output of 272. A 60yo woman presented to OPD
40ml. However, an hour after that, no with dysphagia. No hx of weight loss or
urine seemed to be draining from the heartburn. No change in bowel habits.
catheter. While doing endoscopy there is some
What is the most appropriate next step? difficulty passing through the LES, but no
a. IV fluids other abnormality is noted.
b. Blood transfusion What is the single most useful inv?
c. Dialysis a. CXR
d. IV furosemide b. MRI
e. Check catheter c. Esophageal biopsy
d. Esophageal manometry
e. Abdominal XR
Ans. The key is E. Check catheter.
Q. 1. What is the key?
Q. 2. What is the diagnosis?
Q. 3. What is the treatment.
270. A 24yo pt presented with Ans. 1. The key is D. Oesophageal manometry.
anaphylactic shock. What would be the
Ans. 2. Achalasia cardia
dose of adrenaline?
a. 0.5ml of 1:1000 Ans. 3. i) Oral medication: Nitrates or CCB ii)
b. 0.5ml of 1:10000 Balloon dilatation of the spincter iii)
c. 1ml of 1:500 Oesophagomyotomy.
d. 5ml of 1:1000
e. 0.05ml of 1:100
273. A 24yo woman presents with deep
Ans the key is A. o.5 ml of 1:1000. [in cardiac
dyspareunia and severe pain in every
arrest 1 ml of 1:1000 iv].
cycle. What is the initial inv?
a. Laparoscopy
b. Pelvic US
271. A 44yo woman complains of heavy c. Hysteroscopy
bleeding per vagina. Transvaginal US was d. Vaginal Swab
done and normal. Which of the following
275. A 32yo female with 3 prv 1st
Q. 1. What is the key? trimester miscarriages is dx with
Q. 2. What is the likely diagnosis?
antiphospholipid syndrome.
Q. 3. What is the treatment?
Anticardiolipin antibodies +ve. She is now
Ans. 1. The key is B. Pelvic US. 18wks pregnant.
What would be the most appropriate
Ans. 2. The likely diagnosis is endometriosis.
management?
Ans. 3. Treatment: There is no cure for a. Aspirin
endometriosis, but a number of treatments may b. Aspirin & warfarin
improve symptoms. This may include pain c. Aspirin & heparin
medication [NSAIDs such as naproxen], hormonal d. Heparin only
treatments [COCP, or mirena], or surgery e. Warfarin only
[Surgical removal of endometriosis when other
measures fail]. Ans. The key is C. Aspirin & heparin.

Dyspareunia is the term for recurring pain in the 276. A 23yo presents with vomiting,
genital area or within the pelvis during sexual nausea and dizziness. She says her
intercourse. menstrual period has been delayed 4
weeks as she was stressed recently. There
are no symptoms present. What is the
274. A 38yo woman, 10d postpartum next appropriate management?
presents to the GP with hx of passing a. Refer to OP psychiatry
blood clots per vagina since yesterday. b. Refer to OP ENT
Exam: BP=90/40mmhg, pulse=110bpm, c. CT brain
temp=38C, uterus tender on palpation and d. Dipstick for B-hCG
fundus 2cm above umbilicus, blood clots + e. MRI brain
++.
Q. 1. What is the key?
Choose the single most likely dx.
Q. 2. What is the likely diagnosis?
a. Abruption of placenta 2nd to pre-
Ans. 1. The key is D. Dipstick for B-hCG.
eclampsia
b. Concealed hemorrhage Ans. 2. Likely diagnosis is pregnancy. [Features
c. Primary PPH like vomiting, nausea and dizziness are consistent
d. Secondary PPH with early pregnancy supported by delayed
e. Retained placenta menstruation].
f. Scabies 277. A 16yo girl came to the sexual clinic.
Q. 1. What is the key? She complains of painful and heavy
Q. 2. How the condition is defined? bleeding. She says she doesn’t a regular
Ans. 1. The key is D. Secondary PPH. cycle. What is the most appropriate
Ans. 2. Secondary PPH: Secondary PPH is defined
management?
a. Mini pill
as abnormal or excessive bleeding from the birth
b. Combined pill
canal between 24 hours and 12 weeks
postnatally. [www.rcog.org.uk/en/guidelines- c. IUS
research-services/guidelines/gtg52/]. d. Anti-prostoglandins
e. Anti-fibrinolytics
Ans. The key is B. Combined pill. Patients may, for example, believe that certain
news bulletins have a direct reference to them,
278. A 36yo man walks into a bank and that music played on the radio is played for them,
demands money claiming he owns the or that car licence plates have a meaning relevant
bank. On being denied, he goes to the to them.
police station to report this.
What kind of delusions is he suffering
from?
a. Delusion of reference
b. Delusion of control
c. Delusion of guilt 279. Which method of contraception can
d. Delusion of persecution
cause the risk of ectopic pregnancy?
e. Delusion of grandeur
a. COCP
Ans. The key is E. Delusion of grandeur. b. IUCD
Delusions of grandeur are one of the more c. Mirena
common ones. It's when you believe that you d. POP
have more power, wealth, smarts, or other grand
Ans. The key is B. IUCD.
traits than is true.
280. A woman has pernicious anemia. She
has been prescribed parenteral vitamin
Delusion of control: False belief that another B12 tx but she is needle phobic. Why is
person, group of people, or external force
oral tx not preferred for this pt?
controls one's general thoughts, feelings,
a. IM B12 is absorbed more
impulses, or behavior.
b. Intrinsic factor deficiency affects oral B12
Delusion of guilt or sin (or delusion of self- utilization
accusation): Ungrounded feeling of remorse or c. IM B12 acts faster
guilt of delusional intensity. d. IM B12 needs lower dosage
e. Pernicious anemia has swallowing
Delusion of mind being read: False belief that
other people can know one's thoughts. difficulties

Delusion of thought insertion: Belief that another Ans. The key is B. Intrinsic factor deficiency
thinks through the mind of the person. affects oral B12 utilization.

delusions of persecution. It's when you're


convinced that someone is mistreating,
conspiring against, or planning to harm you or
281. An old man comes to the doctor
your loved one. complaining that a part of this body is
rotten and he wants it removed. What is
the most likely dx?
An idea of reference—sometimes called a a. Guilt
delusion of reference—is the false belief that b. Hypochondriasis
irrelevant occurrences or details in the world c. Munchausen’s
d. Nihilism
relate directly to oneself… A delusion in which e. Capgras syndrome
the patient believes that unsuspicious
occurrences refer to him or her in person.
Ans. The key is D. Nihilism. [nihilism (medical episode of heavy drinking. His is not able
term is nihilistic delusion): parts of the body do to walk straight and is complaining of
not exist or are dead] double vision and is shouting obscenities
Guilt: an emotion that occurs when a person feels and expletives. What is the most likely dx?
that they have violated a moral standard. a. Korsakoff psychosis
b. Delirium tremens
Hypochondriasis:  worry about having a serious
c. Wernickes encephalopathy
illness. 
d. Tourettes syndrome
Munchausen’s: a psychiatric factitious e. Alcohol dependence
disorder wherein those affected feign disease,
illness, or psychological trauma to Ans. The key is C. Wernicke’s encephalopathy.
draw attention,sympathy, or reassurance to [triad of i) ophthalmoplegia, ii) ataxia iii)
themselves.  confusion].

Capgras syndrome: a delusion that a friend,


spouse, parent, or other close family member (or Wernicke encephalopathy (WE) is an acute
pet) has been replaced by an identical- neurological condition characterized by a clinical
looking impostor. triad of ophthalmoparesis with nystagmus, ataxia,
and confusion. This is a life-threatening illness
caused by thiamine deficiency, which primarily
affects the peripheral and central nervous
282. A 31yo woman who is 32weeks systems. Wernicke encephalopathy and
pregnant attends the antenatal clinic.
Korsakoff syndrome are different conditions that
Labs: Hgb=10.7, MCV=91. What is the often occur together. Both are due to brain
most appropriate management for this damage caused by a lack of vitamin B1. Lack of
pt? vitamin B1 is common in people who have
a. Folate supplement alcohol use disorder. It is also common in people
b. Ferrous sulphate 200mg/d PO whose bodies do not absorb food properly
c. Iron dextran
d. No tx req (malabsorption). Parenteral thiamine is used in
the acute treatment of Wernicke's

Ans. The key is D. No tx required. [According to


NICE, cut offs for iron supplements:
at booking (8-10 weeks)- if less than 11 
at 28 weeks and further- if less than 10.5
284. A 32yo woman of 39wks gestation
if less than these values=> give iron]. attends the antenatal day unit feeling very
unwell with sudden onset of epigastric
pain a/w nausea and vomiting. Temp
36.7C. Exam: RUQ tenderness. Bloods:
mild anemia, low plts, elevated LFT and
hemolysis.
283. A 47yo man who is a chronic What is the most likely dx?
alcoholic with established liver damage, a. Acute fatty liver of pregnancy
has been brought to the hospital after an b. Acute pyelonephritis
c. Cholecystitis
d. HELLP syndrome complaints of restricted joint movement
e. Acute hepatitis and severe pain in her affected joints.
What is the
Ans. The key is D. HELLP syndrome. [H=hemolysis,
EL=elevated liver enzyme, LP=low platelet count].
choice of drug?
a. Paracetamol
285. A 57yo woman presents with dysuria, b. Steroid
frequency and urinary incontinence. She c. NSAID
complains of dyspareunia. Urine culture d. Paracetamol+dihydrocoiene
has been done and is sterile. e. Pethadine
What is the most appropriate step?
Ans. The key is A. Paracetamol.
a. Oral antibiotics
b. Topical antibiotics 288. A 24yo 18wk pregnant lady presents
c. Topical estrogen with pain in her lower abdomen for the
d. Oral estrogen last 24h. She had painless vaginal
e. Oral antibiotics and topical estrogen bleeding. O/E : abdomen is tender, os is
Ans. The key is C. Topical estrogen. [There may be
closed.
UTI like symptoms and dyspareunia in atrophic What is the most probable dx?
vaginitis for which topical oestrogen can be used]. a. Threatened miscarriage
b. Inevitable miscarriage
c. Incomplete miscarriage
d. Missed miscarriage
e. Spontaneous miscarriage

Ans. The key is A. Threatened miscarriage.


286. A pt came to the ED with severe
[gestational age 18 weeks, lower abdominal pain,
lower abdominal pain. Vitals:
tender abdomen, closed os and painless vaginal
BP=125/85mmHg, Temp=38.9C. bleeding indicates threatened abortion].
Exam: abdomen rigid, very uncomfortable
during par vaginal. She gave a past hx of 289. A 2yo child playing in the garden had
PID 3 years ago which was successfully a clean cut. She didn’t have any
treated with antibiotics. vaccinations. Also, there is no
What is the appropriate inv? contraindication to vaccinations. Parents
a. US were worried about the vaccine side
b. Abdomen XR effects. What will you give?
c. CT a. Clean the wound and dress it
d. High vaginal
a. b. Give TT only
e. Endocervical swab
b. Give DPT only
Ans. The key is A. US. [Patient had previous PID. c. Give DPT and tetanus Ig
Current symptoms of severe cervical motion d. Give complete DPT vaccine course
tenderness with significant rise of temperature is
Ans. The key is E. Give complete DPT vaccine
very much suggestive of pelvic abscess].
course.
287. A pregnant woman with longterm hx
290. A 32yo female who has had 3 prv
of osteoarthritis came to the antenatal
miscarriages in the 1st trimester now
clinic with
comes with vaginal bleeding at 8wks. US of BP. In case of BP of 145/95 mmHg no
reveals a viable fetus. treatment for BP is needed. Ref: NICE guideline].
What would be the most appropriate 293. A 24yo lady who is 37wk pregnant
definitive management? was brought to the ED. Her husband says
a. Admit
a few hours ago she complained of
b. Aspirin
headache, visual disturbance and
c. Bed rest 2 weeks
abdominal pain. On arrival at the ED she
d. Cervical cerclage
has a fit.
e. No tx
What is the next appropriate
Ans. The key is B. Aspirin. [Early miscarriage is management for this pt?
more common in antiphospholipid syndrome and a. 4g MgSO4 in 100ml 0.9%NS in 5mins
treated with heparin or aspirin when become b. 2g MgSO4 IV bolus
pregnant]. c. 2g MgSO4 in 500ml NS in 1h
d. 4g MgSO4 IV bolus
291. A 6yo girl started wetting herself up
e. 10mg diazepam in 500ml 0.9%NS in 1h
to 6x/day. What is the most appropriate
tx? Ans. The key is A. 4g MgSO4 in 100ml 0.9%NS in
5mins [NICE]. [Dx is eclumpsia].
a. Sleep alarms
b. Desmopressin 294. What is the pathological change in
c. Reassure Barret’s esophagitis?
d. Behavior training a. Squamous to columnar epithelium
e. Imipramine b. Columnar to squamous epithelium
c. Dysplasia
Ans. The given key is B. Desmopressin. This is d. Metaplasia
wrong key! Correct key is D. Behavior training. e. Hyperplasia
[behavior training seems to be more appropriate.
Desmopressin is given for short term relief
generally and after alarm bells fail to control Ans. The key is A. Squamous to columner
symptoms, it is used in children above 7 yrs epithelium.
whereas given case is of a child of 6 yrs].
295. A 34yo male presents with hx of
292. A 27yo 34wk pregnant lady presents headache presents with ataxia,
with headache, epigastric pain and nystagmus and vertigo. Where is the site
vomiting. Exam: pulse=115, of the lesion?
BP=145/95mmHg, proteinuria ++. She a. Auditory canal
complains of visual disturbance. What is b. 8th CN
the best medication for the tx of the BP? c. Cerebellum
a. 4g MgSO4 in 100ml 0.9%NS in 5mins d. Cerebral hemisphere
b. 2g MgSO4 IV bolus e. Brain stem
c. 5mg hydralazine IV
d. Methyldopa 500mg/8h PO
e. No tx Ans. The key is C. Cerebellum. [Features
described are consistent with cerebellar lesion].
Ans. The given key is A. 4g MgSO4 in 100ml
0.9%NS in 5mins. It is a wrong key. Correct key is
E. No tx. [Here, question specifically asked for tx
296. A 24yo girl comes to the woman 299. A young girl presenting with fever,
sexual clinic and seeks advice for headache, vomiting, neck stiffness and
contraception. She is on sodium valproate. photophobia. She has no rashes. What is
a. She can’t use COCP the most appropriate test to confirm dx?
b. She can use COCP with extra precaution a. Blood culture
c. She can use COCP if anticonvulsant is b. Blood glucose
changed to carbamezapin. c. LP
d. She can use COCP with estrogen 50ug and d. CXR
progesterone higher dose e. CT
e. She can use COCP
Ans. The key is C. LP. [case of meningitis. LP will
confirm the diagnosis].
Ans. The key is E. She can use COCP. [sodium
valproate has no effect on cocp]
300. A 65yo HTN man wakes up in the
morning with slurred speech, weakness of
297. A 27yo lady came to the ED 10 days the left half of his body and drooling.
ago with fever, suprapubic tenderness and Which part of the brain is affected?
vaginal discharge. PID was dx. She has a. Left parietal lobe
been on the antibiotics for the last 10days. b. Right internal capsule
She presents again with lower abdominal c. Right midbrain
pain. Temp=39.5C. d. Left frontal lobe
what is the most appropriate next
Ans. The key is B. Right internal capsule. [As
management? symptoms are on left side lesion is on right side of
a. Vaginal swab the brain. So answer should be either b) right
b. Endocervical swab internal capsule or c) right midbrain. If it was
c. US midbraine there would have cranial nerve
d. Abdominal XR involvement. On the other hand given picture is
e. Laparoscopy very much consistent with lacunar infarction of
internal capsule!]
301. A 27yo presents with abdominal
Ans. The key is C. US. [Initial presentation was of pain, bleeding, vomiting and diarrhea. Her
PID. But recurrance of symptoms suggests LMP was 7wks ago. O/E: abdominal
resistant condition like abscess formation]. tenderness, BP=90/60mmHg.
298. An 18yo man complains of fatigue What is the next appropriate
and dyspnea, he has left parasternal management?
a. Immediate laparotomy
heave and systolic thrill with a harsh pan-
b. Laparoscopy
systolic murmur at left parasternal edge.
c. Salpingotomy
What is the most probable dx?
d. Salpingectomy
a. TOF
e. MTX
b. ASD
c. VSD Q. 1. What is the key?
d. PDA Q. 2. What is the diagnosis?
e. TGA Q. 3. Justify the key.

Ans. The key is C. VSD. Ans. 1. The key is A. Immediate laparotomy.


Ans. 2. The diagnosis is ruptured ectopic 305. A 10yo boy presents with nose bleed.
pregnancy . What measure should be taken to stop
Ans. 3. In ruptured ectopic pregnancy if there is
the bleeding?
shock we should go for immediate laparotomy.
a. Press base of the nose
302. A woman presents with complains of
b. Ice packs
abdominal pain, unsteadiness, numbness
c. Press soft parts of the nose
of lower limb and palpitations. All inv are d. Start tranexemic acid
normal. What is the dx? e. IV fluids
a. Manchausen
b. Somatization Ans. The key is C. Press soft parts of the nose.
c. Hypochondriac
d. Bipolar

Ans. The key is B. Somatization. [This is multiple,


recurrent, medically unexplained symptoms 306. An MI pt who is already on aspirin no
usually starting early in life. Usually patient longer smokes and his cholesterol, ECG,
presents with one symptom at a time.
echo and BP are normal. Choose the best
Investigations are normal].
option for him:
303. A 34yo African-caribbean man with a a. Give statin
hx of sarcoidosis has presented with b. Give statin+warfarin
bilateral kidney stones. What is the most c. Low cholesterol diet
likely cause for this pt’s stones? d. Statin+ACEi
a. Hypercalcemia
Ans. The key is D. Statin + ACEi.
b. Hyperuricemia
c. Diet [Offer all people who have had an acute MI
d. Recurrent UTIs treatment with the following drugs:
e. Hyperparathyroidism  ACE (angiotensin-converting enzyme)
inhibitor
Q. 1. What is the key?
Q. 2. Why this occurs?  dual antiplatelet therapy (aspirin plus a
Ans. 1. The key is A. Hypercalcemia. second antiplatelet agent)
Ans. 2.  Hypercalcemia in sarcoidosis is due to the  beta-blocker
uncontrolled synthesis of 1,25-dihydroxyvitamin
D3 by macrophages. 1,25-dihydroxyvitamin D3  statin. [2007, amended 2013] [NICE
leads to an increased absorption of calcium in the guideline].
intestine and to an increased resorption of
307. A 46yo man is being treated for a
calcium in the bone. 
pleural effusion. A chest drain has been
304. Which of the following is NOT a sited just below the 4th rib in the mid-
physiological change during pregnancy? axillary line on his right side.
a. Tidal volume 500ml What single structure is at particular risk
b. RBC vol 1.64L of injury?
c. Cardiac output 6.5L/min a. Arzygos vein
d. Uterus weight 1.1kg b. Diaphragm
e. ESR up by 4x c. Intercostal artery
d. Internal thoracic artery
Ans. The key is A. Tidal volume 500 ml.
e. Liver c. TB peritonitis
d. Pyogenic peritonitis
Ans. The key is C. Intercostal artery. [Most e. Neoplasm
vulnerable structure is intercostal nerve, then
intercostal artery then intercostals vein. As Q. 1. What is the key?
intercostal nerve is not in option intercostal Q. 2. How this diagnosis is made?
artery is the answer here]. Ans. 1. The key is B. Decompansated cirrhosis.

308. What advice would you give for the Ans. 2. If the patient experiences any of the
parents of a child with repeated UTI? serious problems described below his disease has
a. Surgery progressed from compensated cirrhosis to
b. Prophylactic antibiotics decompensated cirrhosis:
c. Increase fluids i) Bleeding varices (internal bleeding)
d. Toilet training
e. Laxatives ii) Ascites (fluid in the belly)

iii) Encephalopathy (confusion)


Ans. The given key is A. Surgery. This is a wrong
key. Correct option is B. Prophylactic antibiotic. iv) Jaundice (yellowing of eyes and skin).
[For repeated UTI prophylactic antibiotic should
be given].
311. A 15yo boy presents with testicular
309. A pt presents with complete anuria pain for 2days. There is no hx of trauma.
following prolonged hypotension and O/E: temp=38.5C, right hemi-scrotum
shock in a pt who bled profusely from a tenderness.
placental abruption. What is the most What is the single most appropriate
probable dx? management?
a. Post viral infection a. Give antibiotics
b. Acute papillary necrosis b. Give analgesia
c. Acute cortical necrosis c. Reassure
d. HUS d. US scrotum
e. Renal vein thrombosis e. Exploratory surgery

Q. 1. What is the key? Q. 1. What is the key?


Q. 2. What is the reason for this?
Ans. 1. The key is C. Acute cortical necrosis. Q. 2. What is the diagnosis?

Ans. 2. There are 2 reasons for this acute cortical Q. 3. What are the points in favour?
necrosis. i) significant diminished arterial
Ans. 1. The key is A. Give antibiotics.
perfusion of the kidneys due to spasm of the
feeding artery secondary to profuse bleeding Ans. 2. The diagnosis is epididymo-orchitis.
from placental abruption ii) DIC secondary to
Ans. 3. Points in favour: i) No history of trauma ii)
placental abruption.
testicular pain with fever points towards
310. An alcoholic 56yo man had ascetic epididymo-orchitis.
fluid analysis done which was found to be 312. A 58yo lady presented with urinary
yellow color. What is the most incontinence. She looks anxious for her
appropriate cause? condition. Urine culture is sterile. Her
a. Alcoholic hepatitis
b. Decompensated cirrhosis
urodynamic study is normal. What is the
next step? 316. A 28 yo female who delivered 6
2. Antibiotics weeks ago feels sad and has no interest to
3. Topical estrogen feeding the baby. She has been eating
4. Systemic estrogen poorly and having difficulty sleeping. She
5. Duloxetine
feels weak throughout the day and has
6. Pelvic floor exercise
stopped taking the baby out of the house.
Ans. The key is E. Pelvic floor exercise. She also says that the baby has evil eyes.
What is the most likely diagnosis?
313. A 45yo lady came to family planning a. Postpartum blues
clinic for contraception advice. She is not b. Postpaetum depression
keen to be pregnant for the next 3yrs. Her c. Postpurtum psychosis
recent US showed multiple small d. Schizophrenia
submucosal fibroid. What is the best e. Psychotic depression
method of contraception for her?
1. Etonogestrol Q. 1. What is the key?
2. COCP Q. 2. What are the points in favour?
3. IUS Ans. 1. The key is C. Postpartum psychosis.
4. POP
Ans. 2. Points in favour: i) features of depression:
5. IUCD
feels sad, poor eating, difficulty sleep, feeling
Ans. The key is C. IUS. [IUS gives 3-5 yrs long weak ii) delusional ideas: thinks baby has evil eyes
contraception. It also helps to shrink the fibroid]. and not taking the baby out of the house. These
points to postpartum psychosis. [Postpartum
314. A child presents with eczema. She psychosis starts within 2 wks (occasionally later)
was given two creams by the GP – of delivery and it can take 6 -12 months or more
emollient and steroid. What advice would to recover from postpartum psychosis].
you give her regarding application of the 317. A 44yo man presents with periorbital
cream? and pedal edema. 24h urine shows 8g of
a. Sparingly use both the cream
protein/d and serum
b. First use emollient, then steroid
cholesterol=7mmol/L. Renal biopsy results
c. Apply steroid then emollient
are awaited.
d. Mix emollient & steroid before use
What would be the most likely dx?
e. Emollient at night with steroid
a. Minimal change disease
Ans. The key is B. First use emollient, then a. b. Glomerulonephropathy
steroid. [emmolient 30 minutes before steroid].
b. Membranous glomerulonephropathy
c. FSGS
315. All the following drugs do not cause
d. IgA nephropathy
bronchoconstriction except?
e. Mesangiocapillary
1. Atenolol
2. Salbutamol
Ans. The given key is C. Membranous
3. Salmetrol
glomerulonephritis. [Some authority claims FSGS
4. Ipratropium bromide
5. Cocaine as more common cause of nephrotic syndrome].

318. A 53yo man presents complaining of


Ans. The key is A. Atenolol. [Cocaine can also
cause bronchoconstriction]. weight loss, lethargy, increasing
abdominal discomfort and gout for the e. Ca breast
past yr. Exam: spleen palpated 5cm below
Q. 1. What is the key?
left costal margin, no fluid wave. CBC: Q. 2. Please justify the key.
Hgb=10.5g/dL, WBC=200 – 85% Ans. 1. The key is B. Fat necrosis.
neutrophils, plts=100, Na+=140mmol/L,
Ans. 2. Fat necrosis usually occurs following
K+ =4mmol/L, create=151umol/L,
trauma or surgery. Given case is a fat necrosis of
urea=7mmol/L. Serum B12 increased.
breast as there is no discharge and there is a
Philadelphia chromosome +ve. bruise indicating prior trauma.
What is the most likely dx?
a. CML 321. A 67yo female who had undergone a
b. CLL radical mastectomy now comes with the
c. AML complaint of swelling and redness in her
d. ALL right upper limb.
e. Lymphoma Involvement of which of the following
structures explain these symptoms?
Q. 1. What is the key?
a. Epitrochlear LN
Q. 2. What are points in favour of this diagnosis?
b. Cephalic vein
Ans. 1. The key is A. CML. c. Subclavian artery
Ans. 2. Points in favour: i) wt loss ii) lethargy iii) d. Axillary group of LN
abdominal discomfort iv) splenomegaly v) gout e. Long thoracic nerve
[Elevated uric acid and vitamin B12 levels are
found in 25% of patients of CML]. A blood picture Ans. The key is D. Axillary group of LN. [Axillary
is suggestive and +ve Philadelphia chromosome is clearance compromise lymphatic flow and may
diagnostic. results in swelling of upper limb].
319. In a group of cancer pts, 10 died that
wasn’t treated while 5 died in the tx 322. A 50yo smoker and heavy drinker
group. Which statement is correct? presents with complaints of racing heart.
a. Absolute risk =10 A 24h ECG comes out normal. What is
b. Relative risk =10 your next step in management?
c. Relative risk =5 a. ECHO
d. Absolute risk=5 b. Reassure
e. Relative risk=2 c. Stress test

Ans. Given key is E. relative risk = 2. [RR= Number Ans. The key is B. Reassure. [Smoking and alcohol
of death in not treated group/number of death in excess can cause palpitation without any
treated group]. recognizable arrhythmia and for this no
treatment is required].
320. A 67yo woman has presented with
hard, irregular, poorly defined 5cm lump 323. A 47yo man comes to the GP with a
in her right breast. She has a bruise on the swelling in his left groin which disappears
surface and there is no discharge. on lying down. The swelling was bluish in
What is the most likely dx? color and felt like a bag of worms. He also
a. Fibroadenosis complains of a mass in the left loin along
b. Fat necrosis with hematuria occasionally.
c. Fibroadenoma What could be the possible dx?
d. Duct ectasia a. Left sided RCC
b. Varicosity 2nd to liver disease d. Left main stem, post descending artery
c. Testicular tumor e. Right coronary artery
d. UTI
Q. 1. What is the key?
e. IVC obstruction
Q. 2. Justify the key.
Q. 1. What is the key?
Q. 2. What is the condition described? Ans. 1. The key is E. Right coronary artery.
Q. 3. What is the link between these two Ans. 2. If the posterior discending artery is
conditions?
supplied by the circumflex artery then it is left
dominant and if posterior descending artery is
Ans. 1. The key is A. Left sided Renal cell
supplied by the right coronary artery then it is
carcinoma.
right dominant. As in 85% of population posterior
Ans. 2. Left sided varicocele. descending artery is supplied by right coronary
artery it is called the dominant that is right
Ans. 3. Most common secondary cause of left
coronary artery is dominant.
sided varicocele is RCC. Newly diagnosed
varicocele over the age of 40yrs are very much
suggestive of RCC. Varicocele is common on left
side as left testicular veins drain to the left renal
vein, while the right testicular vein drain directly 326. A 54 yo lady presents with sudden,
into IVC. severe pain in the left half of her skull. She
324. A man presents with muffled hearing also complains of pain around her jaw.
and feeling of pressure in ear with tinnitus What is the next likely step?
a. CT
and vertigo. He also complains of double
b. MRI
vision when looking to the right. What is
c. Fundoscopy
the most appropriate dx?
a. Meniere’s disease d. ESR
b. Acoustic neuroma e. Temporal artery biopsy
c. Acute labyrinthytis Q. 1. What is the key?
d. Meningioma Q. 2. What is the diagnosis?
e. Otosclerosis Q. 3. What are the points in favour of your
diagnosis?
Q. 1. What is the key? Ans. 1. The key is ESR.
Q. 2. Justify the key.
Ans. 2. The diagnosis is Giant cell arteritis or
Ans. 1. The key is B. Acoustic neuroma. temporal arteritis.

Ans. 2. Hearing loss, feeling of pressure in the ear Ans. 3. Points in favour: i) Age >50yrs ii) Female
with tinnitus, vertigo and involvement of cranial sex iii) Severe pain in the left half of skull iv) Pain
nerve i.e. right abducent nerve are suggestive of around the jaw (jaw claudication).
acoustic neuroma.
327. A teenage girl who was ‘fine’ until
325. In 85% of the population this artery is her boyfriend said he didn’t want the
dominant. What is the single most relationship anymore. She took 10 tablets
appropriate option? of paracetamol in front of his mother after
a. Left ant descending artery taking alcohol. What should you do?
b. Coronary sinus a. Refer to psychiatry
c. Circumflex artery b. Counselling
c. GP to sort out family issues 331. An old lady 72yo staying at a nursing
d. Return to work to relieve her anger home for a few years, a known HTN on
reg tx presented with sudden dysphagia
Ans. The key is A. Refer to psychiatry. [1o tablets
of paracetamol is not a life threatening toxic dose
while eating with drooling of saliva and
and simultaneous drug overdose and alcohol req urgent inv. What would be your next
consumption needs psychiatric evaluation]. step?
a. Ba swallow
328. A 6yo fell on outstretched hand while b. Chest CT
playing. He feels tender at the elbow but c. Endoscopy
otherwise well. What is the most likely dx? d. Laryngoscopy
a. Spiral fx e. CXR
b. Green stick fx f. Endoscopy with biopsy
c. Compound fx
d. Supracondylar fx  Ans. The key is C. Endoscopy. [Probable
e. Pulled elbow impacted food bolus (usually meat)which can be
visualized and removed with the aid of
Ans. The key is B. Green stick fracture. endoscopy].

329. A man has a BP of 160/90mmHg, 332. A man presents with outward


proteinuria++. KUB US are equally reduced deviation of his right eye and diplopia.
in size with smooth borders and normal Which nerve is affected?
pelvic calyceal system. a. Left trochlear
What is the cause of HTN in the pt? b. Left oculomotor
a. Chronic glomerulonephritis c. Right trochlear
b. Chronic pyelonephritis d. Right abducens
c. Bilateral renal artery stenosis e. Right oculomotor
d. Essential HTN
Ans. The given key is B. Left oculomotor! It is
e. Polycystic kidney
wrong key! As in oculomotor nerve lesion there is
Ans. The key is bilateral renal artery stenosis. This ipsilateral symptoms so the correct answer is E.
is probably a wrong key. The correct key should Right oculomotor.
be A. Chronic glomerulonephritis. [In bilateral
333. A 60yo pt who has had a MI a week
renal artery stenosis BP is very high].
back presents with dyspnea and
330. A lady presents with abdominal pain, pericardial rub. ECG shows ST elevation.
dysuria, dyspareunia and vaginal CXR: loss of margin at costo-vertebral
discharge. What si your next step? angle.
a. Laparoscopy What is the single most likely cause?
b. High vaginal swab a. Cardiac tamponade
c. Hysteroscopy b. Mitral regurge
d. Laparotomy c. Dressler’s syndrome
e. US d. Atrial fib
e. Emboli
Ans. The key is B. High vaginal swab. [Probable
diagnosis is PID].

Q. 1. What is the key?


Q. 2. Why it is not reinfarction as there is ST 336. A 34yo man after a car crash is in the
elevation? ED and deteriorating. His GCS has fallen
from 13 to 7. What is the most
Ans. 1. The key is C. Dressler’s syndrome. appropriate next step?
a. CT
Ans. 2. There is pericardial rub there is
b. Burr hole
pericarditis and in pericarditis there is widespread
c. MRI
ST elevation. So the condition is not new MI but
d. Intubation
Dressler’s syndrome.
e. IV fluids

334. A 12yo girl presented with tics, LOC,


no residual sign and no post-ictal phase. Ans. The key is D. Intubation. [ABC protocol].
EEG abnormality in temporal lobe. The girl
had a rapid recovery. 337. A pt with alternating swings or
What is the most probably dx? episodes from elation and depression had
a. Generalized tonic-clonic underwent tx and gotten better.
b. Myoclonic What medication needed to be continued
c. Partialgeneralized seizure so he can stay well?
d. Atonic seizure a. Anxiolytics
e. Febrile convulsion b. Mood stabilizers
c. Antidepressants
d. Antipsychotics
Ans. The key is C. Partial –> generalized seizure

Ans. The key is B. Mood stabilizers [bipolar


335. A 48yo woman who has been taking disorder treated with mood stabilizers].
medications for asthma for a long time
has now presented with decreasing vision.
What is the most probable cause for her 338. A 40yo male with pre-existing
decrease in vision? glumerulonephritis having proteinuria and
a. Inhaled salbutamol hematuria suddenly deteriorates and
b. Inhaled steroids presents with oliguria and serum
c. Aminophylline K+=7.8mmol/L, urea=13mmol/L,
d. Beta-blockers creat=342mmol/L, GFR=19mL/h.
e. Oral steroids The best management would be?
a. Calcium supplement
b. Calcium resonate enema 30g
Q. 1. What is the key?
Q. 2. Justify the key. c. 10units insulin with 50% dextrose
d. Nebulized salbutamol
e. 10ml of 10% calcium gluconate
Ans. 1. The key is E. Oral steroid.
f. Hemodialysis urgent
Ans. 2. Prolonged steroid use leads to cataract
formation.
Q. 1. What is the key?
Q. 2. Justify the key.
Addison’s disease was made. What is the
Ans. 1. The key is E. 10 ml of 10% calcium most likely electrolyte abnormality
gluconate. expected in this pt?
a. High Na+, Low K+
Ans. 2. To prevent cardiac arrhythmia. [Actually b. Low Na+, High K+
calcium gluconate neither shifts K+ to cells nor c. Low Na+, Low K+
reduces serum K+ level that much. It just prevents d. High Na+, High K+
cardiac arrest or life threatening cardiac
e. Low Na+, Normal K+
arrhythmia and buys time till definitive measures
are taken]. Ans. The key is B. Low Na+, High K+. [ with
Addison disease, the sodium, chloride, and
carbon dioxide levels are often low, while the
339. 34yo man was brought to the ED potassium level is high].
after a RTA. BP=50/0mmHg and chest wall
342. An 8yo returned from Spain with
not moving symmetrically, RR=34bpm.
severe pain in one ear. Exam: pus in
What would be initial action?
a. IV fluid infusion
auditory canal, tympanic membrane looks
b. Intubation and ventilation
normal. What is the tx option?
a. Gentamicin topical
c. CT chest
b. Amoxicillin PO
d. Transfer to ITU
c. Analgesia
d. Amoxicillin IV

Ans. The key is B. Intubation and ventilation [ABC Q. 1. What is the key?
protocol]. Q. 2. What is the diagnosis?
Ans. 1. The key is A. Gentamycin topical.

Ans. 2. Diagnosis is otitis externa.


340. A pt complains of SOB, wheeze,
cough and nocturnal waking. He has dry 343. A 6wk child is very sick-looking.
scaly shin with rashes that are itchy. What Bloods: Na+=124, K+=2.8. Dehydrated.
is the single most likely dx? What would you choose to resuscitate?
a. Scabies a. 0.18% NS + 4% dextrose + 20mmol KCl
b. Eczema b. 0.9% NS
c. Rheumatism c. 0.45% NS
d. Dermatitis d. 0.45% NS + 5% dextrose
e. Psoriasis e. 0.45% NS + 5% dextrose + 20 mmol KCl

Ans. The given key is E. But it is wrong key! The


correct key is B. 0.9% NS. Explanation:
Ans. The key is B. Eczema. [Asthma may be Rsuscitation is mostly done with 0.9% NS or
associated with atopy]. ringers lactate, or hartmans solution. Here is
hypokalemia. To treat hypokalaemia the cut off
value is below 2.5 mmol/L and absence of anuria
341. A 54yo woman has presented with during resuscitation. Maintenance is with fluid E.
episodes of abdominal ache, vomiting and
postural hypotension. She also has a dark
pigmentation of her skin. A dx of
344. A 68yo man gets repeated attacks of to get out of bed or getting up from
LOC and TIA. What is the most likely cause sitting. She is on some anti-HTN therapy
for this? with no other med prbs. What is the cause
a. Atrial fib of her fall?
b. Mitral stenosis a. CCB
c. Aortic stenosis b. Vertibrobasiliar insufficiency
d. HOCM c. Thiazide
e. Carotid artery stenosis d. Hypoglycemia
e. Infection

Ans. The key is E. Carotid artery stenosis.

Ans. The key is C. Thiazide. [It causes postural


345. Pt presented with hemoptysis 7d hypotension by volume depletion].
post-tonsillectomy. What is the next step?
a. Packing
b. Oral antibiotics + discharge 348. A 56yo woman with MS presents
c. Admit + IV antibiotics with drooping of the left side of her lips.
d. Return to theatre and explore She also has loss of sensation over her
e. Ice cream and cold fluids face, hearing impairment and some in-
coordination of her movements. What is
Ans. The key is C. Admit + IV antibiotic. [infection the most likely anatomical site affected?
a. Cerebellum
is a common cause of secondary haemorrhage.
Patient should be admitted to observe the course b. Cerebrum
of bleeding and treatment is given with IV c. Spinal cord
antibiotics]. d. Brain stem
e. Optic nerve

346. A child was admitted following a RTA


with initial GCS=15. Then during the night Ans. The key is D. Brain stem. Features of 5, 7, 8 th
the noticed GCS reduced to 13. What is cranial nerve and cerebellum involvement
the management? suggestive of brainstem lesion.
a. Refer to neuro-surgeon
b. IV fluids
c. Oxygen 349. A 68yo male presented with swelling
d. CT brain in the lower pole of the parotid gland for
e. Skull XR the last 10yrs. Exam: firm in consistency.
What’s the most probable dx?
a. Pleomorphic adenoma
Ans. The key is D. CT brain. [probable intracranial b. Adenolymphoma
haemorrhage]. c. Mikulicz’s disease
d. Parotiditis
347. A 57yo woman who is suffering from e. Frey’s syndrome
HTN, presented to the hospital with
complaints of recurrent falls when trying
Ans. The key is A. Pleomorphic adenoma. Ans. The key is A. Anti-HSV antibodies. [Genital
[Pleomorphic adenoma (most common) - also Herpes may be asymptomatic or may remain
called benign mixed tumour: is the most common dormant for months or even years. When
tumour of the parotid gland and causes over a symptoms occur soon after a person is infected,
third of submandibular tumours. They are slow- they tend to be severe. They may start as multiple
growing and asymptomatic, having a malignant small blisters that eventually break open and
potentiality]. produce raw, painful sores that scab and heal
over within a few weeks. The blisters and sores
may be accompanied by flu-like symptoms with
350. A 28yo shipyard worker was fever and swollen lymph nodes.
admitted for pain in calf while at work There are three major drugs commonly used to
which has been increasing over the last treat genital herpes symptoms: 
3m. There is no hx of HTN or DM but he is acyclovir (Zovirax), famciclovir (Famvir),
a smoker. Exam: loss of posterior tibial and valacyclovir(Valtrex). These are all taken in
and dorsalis pedis pulsation along with a pill form. Severe cases may be treated with the
non-healing ulcer at the base of the right intravenous (IV) drug acyclovir].
1st MTP joint. What is the most probably 352. A 53yo man presents with a
dx? longstanding hx of a 1cm lesion on his
a. Thromboangitis obliterans
arm. It has started bleeding on touch.
b. Sciatica
What is the most likely dx?
c. DVT
a. Basal cell carcinoma
d. Baker’s cyst
b. Kaposi’s sarcoma
e. Embolus
c. Malignant melanoma
d. Squamous cell carcinoma
Q. 1. What is the key? e. Kerathoacanthoma
Q. 2. What are the points in favour?
Ans. The key is D. Squamous cell carcinoma. [SSCs
Arises in squamous cells. SCCs may occur on all
Ans. The key is A. Thromboangitis obliterans. areas of the body including the mucous
Ans. 2. i) young age ii) smoker iii) pain in cuff iv) membranes and genitals, but are most common
loss of posterior tibial and dorsalis pedis pulsation in areas frequently exposed to the sun, such as
v) non-healing ulcer at the base of the right 1 st the rim of the ear, lower lip, face, balding scalp,
MTP joint all are suggestive of Buerger’s disease. neck, hands, arms and legs. SCCs often look like
scaly red patches, open sores, elevated growths
with a central depression, or warts; they may
crust or bleed. A tissue sample (biopsy) will be
351. A 35yo lady presents with painful examined under a microscope to arrive at a
ulcers on her vulva, what is the diagnosis. Squamous cell carcinomas detected at
appropriate inv which will lead to the dx? an early stage and removed promptly are almost
a. Anti-HSV antibodies always curable and cause minimal damage]. 
b. Dark ground microscopy of the ulcer
c. Treponema palladium antibody test 353. A 47yo man with hx of IHD complains
d. Rapid plasma regain test of chest pain with SOB on exertion over
e. VDRL the past few days. ECG normal, Echo=
increased EF and decreased septal wall
thickness. What is the most likely dx?
a. Dilated CM tachycardia, with difficulty identifying p
b. Constrictive pericarditis wave. What is the single most appropriate
c. Amyloidosis immediate tx?
d. Subacute endocarditis a. Administer fluid bolus
b. Administer oxygen
Ans. The key is A. Dilated CM. [In dilated
c. Oral beta-blockers
cardiomyopathy ejection fraction is decreased
d. Synchronized DC cardio-version
(but here increased which goes in favour of
e. Unilateral carotid sinus massage
constricted pericarditis). On the other hand
decreased septal wall thickness favours the Q. 1. What is the key?
diagnosis of dilated cardiomyopathy. So it seems Q. 2. Justify the key.
to be a bad recall!!]. Q. 3. What is the diagnosis?
354. An elderly pt who is known to have Ans. 1. The key is D. Synchrnized DC
DM presents to the hospital with cardioversion.
drowsiness, tremors and confusion. What
inv should be done to help in further Ans. As the patient is in probable hemodynamic
instability (suggested by cool peripheries) so we
management?
should go for DC cardioversion.
a. Blood sugar
b. ECG Ans. 3. Probable diagnosis is SVT.
c. Standing and lying BP
d. Fasting blood sugar
e. CT 357. A 7yo child presented with chronic
cough and is also found to be jaundiced
on examination. What is the most likely
Ans. The key is A. Blood sugar. dx?
a. Congenital diaphragmatic hernia
355. A 28yo pregnant woman with b. Congenital cystic adenematoid
polyhydramnios and SOB comes for an malformation
anomaly scan at 31 wks. US= absence of c. Bronchiolitis
gastric bubble. What is the most likely dx? d. RDS
a. Duodenal atresia e. Alpha 1 antitrypsin deficiency
b. Esophageal atresia
Q. 1. What is the key?
c. Gastrochiasis
Q. 2. Justify the key.
d. Exomphalos
Ans. 1. The key is E. Alpha 1 antitrypsin
e. Diaphragmatic hernia deficiency.

Ans. 2. Unexplained liver disease with respiratory


symptoms are very suggestive of AATD.
Ans. The key is B. Oesophageal atresia.

356. A 1m boy has been brought to the 358. A 35yo construction worker is dx with
ED, conscious but with cool peripheries indirect inguinal hernia. Which statement
and has HR=222bpm. He has been below best describes it?
a. Passes through the superficial inguinal
irritable and feeding poorly for 24h.
ring only
CXR=borderline enlarged heart with clear
b. Lies above and lateral to the pubic
lung fields. ECG=regular narrow complex
tubercle
c. Does not pass through the superficial consolidation in the left lung. What is the
inguinal ring next appropriate step?
d. Passes through the deep inguinal ring a. PPI IV
b. Alendronate
Ans. The key is D. Passess through the deep
c. IV antibiotics
inguinal ring.
d. Analgesic
359. A woman has numerous painful e. PPI PO
ulcers on her vulva. What is the cause?
Ans. Here is two key C. IV antibiotics and E. PPI
1. Chlamydia
2. Trichomonas PO. Correct key is C. IV antibiotics. [Pneumonia
3. Gardenella should be treated first].
4. HSV
363. A 66yo man has the following ECG.
5. EBV
Ans. The key is D. HSV. What is the most appropriate next step in
management?
360. A 72 yo man has been on warfarin
for 2yrs because of past TIA and stroke.
What is the most important complication
that we should be careful with?
a. Headache
b. Osteoporosis
c. Ear infection
d. Limb ischemia
e. Diarrhea a. Metoprolol
b. Digoxin
Ans. The given key is E. Diarrhoea which is
considered as a wrong key and A. Headache is the c. Carotid sinus massage
correct key. [Headache is the warning sign of d. Adenosine
hemorrhagic stroke]. e. Amiodarone.

361. A 55yo man has been admitted for Ans. The key is A. Metoprolol. [P waves are
elective herniorraphy. Which among the replaced by fibrillatory f-waves. Irregular R-R
following can be the reason to delay his intervals. Dx atrial fibrillation].
surgery? 364. A 22yo sexually active male came
a. Controlled asthma
with 2d hx of fever with pain in scrotal
b. Controlled atrial fib
area. Exam: scrotal skin is red and tender.
c. DVT 2yrs ago
What is the most appropriate dx?
d. Diastolic BP 90mmHg a. Torsion of testis
e. MI 2 months ago b. Orchitis
Ans. The key is E. MI 2 months ago [better go for c. Inguinal hernia
surgery 6 months post MI]. d. Epididymo-orchitis

362. A 65yo known case of liver ca and Q. 1. What is the key?


metastasis presents with gastric reflux Q. 2. How will you differentiate torsion from
epididymo-orchitis?
and bloatedness. On bone exam there is Ans. 1. The key is D. Epididymo-orchitis.
osteoporosis. He also has basal
Ans 2. In orchitis there should be fever, elevation d. Ureteric colic
of testes reduces pain (positive prehn sign), In e. Strangulated hernia
torsion testis lies at a higher level. In torsion
urinalysis negative but in orchitis it is positive.
Orchitis usually occurs in sexually active man. Q. 1. What is the key?
Q. 2. Abdominal pain radiating to groin, at which
365. A man on warfarin posted for
level of stone does it indicate?
hemicolectomy. As the pt is about to
undergo surgery. What option is the best
for him? Ans. 1. The key is D. Ureteric colic.
a. Continue with warfarin Ans. 2. It indicate stone at lower ureter. [i) Pain
b. Continue with warfarin and add heparin from upper ureteral stones tends to radiate to
c. Stop warfarin and add aspirin the flank and lumbar areas. ii) Midureteral calculi
d. Stop warfarin and add heparin cause pain that radiates anteriorly and caudally.
e. Stop warfarin This midureteral pain in particular can easily
mimic appendicitis on the right or acute
Ans. The key is D. Stop warfarin and add heparin. diverticulitis on the left. iii) Distal ureteral stones
366. A 65yo known alcoholic is brought cause pain that tends to radiate into the groin or
testicle in the male or labia majora in the female.
into hospital with confusion,
aggressiveness and
ophthalmoplegia. He is treated with
368. A 39yo coal miner who smokes,
diazepoxide.
drinks and has a fam hx of bladder cancer
What other drug would you like to
is suffering from BPH. The most important
prescribe?
a. Antibiotics risk factor for his bladder carcinoma is?
a. Fam hx
b. Glucose
b. Smoking
c. IV fluids
c. Exposure to coal mine
d. Disulfiram
d. BPH
e. Vit B complex
Q. 1. What is the key?
Q. 2. What are the risk factors for bladder cancer?
Ans. The key is E. Vitamin B complex. [confusion
and ophthalmoplegia points towards the Ans. 1. The key is B. smoking.
diagnosis of Wernicke’s encephalopathy].
Ans. 2. Risk factors of bladder cancer: i) Smoking
ii) Exposure to chemicals used in dye industry iii)
367. A 32yo woman has severe right sided Whites are more likely to develop bladder cancer
iv) Risk increases with age v) More common in
abdominal pain radiating into the groin
men vi) Chronic bladder irritation and infections
which has lasted for 3h. She is writhering
(urinary infections, kidney and bladder stones,
in pain. She has no abdominal signs. bladder catheter left in place a long time.) vii)
What is the most likely cause of her Personal history of bladder or other urothelial
abdominal pain? cancer viii) Family history ix) Chemotherapy or
a. Appendicitis radiotherapy x) Pioglitazone for more than one
b. Ruptured ectopic pregnancy year and certain herb xi) Arsenic in drinking water
c. Salpingitis xii) Low fluid consumption.
now cyanosed and unresponsive. What is
the 1st thing you would do?
369. A 34yo woman is referred to the
a. IV fluids
endocrine clinic with a hx of
b. Clear airway
thyrotoxicosis. At her 1st appointment she c. Turn pt and put in recovery position
is found to have a smooth goiter, lid lag d. Give 100% oxygen
and bilateral exophthalmos with puffy e. Intubate and ventilate
eyelids and conjunctival injection. She
wants to discuss the tx of her thyroid prb
as she is keen to become pregnant. What Ans. The key is B. Clear airway. [ABC protocol].
is the most likely tx you would advise?
a. 18m of carbimazole alone
b. 18m of PTU alone 372. A HTN male loses vision in his left
c. A combo od anti-thyroid drug an0d eye. The eye shows hand movement and a
thyroxine light shined in the eye is seen as a faint
d. Radioactive iodine light. Fundus exam: flame shaped
e. Thyroidectomy hemorrhages. The right eye is normal.
Q. 1. What is the key? What is the cause of this pts unilateral
Q. 2. What is the reason? blindness?
a. HTN retinopathy
Ans. 1. The key is B. 18m of PTU alone. b. CRA thrombosis
Ans. 2. Other drug option i.e Carbimazole is c. CRV thrombosis
teratogenic [can cause i) spina bifida ii) d. Background retinopathy
cardiovascular malformations, hypospadius etc] if e. Retinal detachment
become pregnant. PTU is on the other hand
relatively safe in pregnancy.
Ans. The key is A. HTN retinopathy [it is a wrong
key. Correct key is C. central retinal vein
370. A child living with this stepfather is
thrombosis].
brought by the mother with multiple
bruises, fever and fractures. What do you
suspect?
a. NAI
373. A mentally retarded child puts a
b. Malnutrition
green pea in his ear while eating. The
c. Thrombocytopenia carer confirms this. Otoscopy shows a
d. HIV green colored object in the ear canal.
What is the most appropriate single best
approach to remove this object?
a. By magnet
Ans. The key is A. NAI. [H/O living with stepfather,
b. Syringing
multiple bruises, fever and fractures are
c. Under GA
suggestive of NAI].
d. By hook
e. By instilling olive oil
371. A young man who was held by the
police was punched while in custody. He is
Ans. The key is Under GA. [Pea is not a magnetic carcinoma has bowel obstruction and
material and hence it cannot be removed by severe colic for 2h and was normal in
magnet, it will swell up if syringing is attempted, between severe pain for a few hours.
as hook placement is likely with risk of pushing
What is the most appropriate
the pea deeper it is not also suitable in a mentally
management?
retarded child, and olive oil is not of help in case
a. PCA (morphine)
of pea. So to avoid injury it is better to remove
b. Spasmolytics
under GA].
c. Palliative colostomy
d. Oral morphine
e. Laxatives
374. A pt presents with longstanding
gastric reflux, dysphagia and chest pain. Q. 1. What is the key?
On barium enema, dilation of esophagus Q. 2. Why we should go for this option?
with tapering end is noted. He was found
with Barrett’s esophagus. He had Ans. 1. The key is C. Palliative colostomy.
progressive dysphagia to solids and then Ans. 2. Cancer or chemotherapy induced
liquids. What is the single most obstructions are unlikely to respond to
appropriate dx? conservative management [NBM, IV fluid,
a. Achalasia nasogastric suction] and hence only analgesia will
b. Esophageal spasm not relieve it. So in such cases we have to go for
c. GERD palliative colostomy.
d. Barrett’s esophagus
377. A 70yo man admits to asbestos
e. Esophageal carcinoma
exposure 20yrs ago and has attempted to
Ans. The key is E. Oesophageal carcinoma. [there quit smoking. He has noted weight loss
is dilatation in oesophagus which is seen both in and hoarseness of voice. Choose the single
achalasia and carcinoma. Dysphagia to solid most likely type of cancer a.w risk factors
initaially is very much suggestive of carcinoma present.
and also barrett’s change is a clue to carcinoma] a. Basal cell carcinoma
b. Bronchial carcinoma
c. Esophageal carcinoma
375. A 48yo lady presents with itching, d. Nasopharyngeal carcinoma
excoriations, redness, bloody discharge e. Oral carcinoma
and ulceration around her nipple. What is
Q. 1. What is the key?
the most likely dx?
Q. 2. What are the conditions related to asbestos
a. Paget’s disease of the breast
exposure?
b. Fibrocystic dysplasia Ans. 1. The key is B. Bronchial carcinoma.
c. Breast abscess [Asbestos exposure is a risk factor for lung cancer
d. Duct papilloma and also has a synergistic effect with cigarette
e. Eczema smoke. Horseness can be from involvement of
recurrent laryngeal nerve].

Ans. 2. Conditions related to asbestos exposure: i)


Ans. The key is A. Paget’s disease of the breast. Pleural plaques (after a latent period of 20-40 yrs)
ii) Pleural thickening iii) Asbestosis (latent period
376. Pt with widespread ovarian
is typically 15-30 yrs) iv) Mesothelioma (prognosis which became more prominent and
is very poor) v) Lung cancer. painful on chewing. He also gave hx of
378. A 32yo woman had progressive sour taste in the mouth, the area is tender
decrease in vision over 3yrs. She is no dx on palpation. Choose the most probable
as almost blind. What would be the dx?
a. Chronic recurrent sialadenitis
mechanism?
b. Adenolymphoma
a. Cataract
c. Mikulicz’s disease
b. Glaucoma
d. Adenoid cystic carcinoma
c. Retinopathy
e. Sub-mandibular abscess
d. Uveitis
e. Keratitis
Ans. The key is A. Chronic recurrent sialadenitis.
Q. 1. What is the key ?
[pain, swelling, more pain on chewing,
Q. 2. Why you made this diagnosis?
tenderness, and submandibular region suggests
Ans. 1. The key is B. Glaucoma. This is wrong key!
diagnosis of submandibular chronic recurrent
Correct option is retinopathy.
sialadenitis, usually secondary to sialolithiasis or
Ans. 2. Cataract is unlikely at this age. Uveitis and stricture].
iritis doesn’t have such degree of vision loss and
iritis and anterior uveitis have pain, redness and
photophobia. Open angle glaucoma mostly occurs 381. ECG of an 80yo pt of ICH shows saw-
after the age of 50yrs. Answer should be tooth like waves, QRS complex of 80ms
retinopathy (example retinitis pigmentosa). duration, ventricular rate=150/min and
regular R-R interval. What is the most
379. A child during operation and porbable dx?
immediately after showed glycosuria, but a. Atrial fib
later his urine sugar was normal. Choose b. Atrial flutter
the most probable dx. c. SVT
a. Pre-diabetic state d. Mobitz type1 second degree heart block
b. Normal finding e. Sinus tachycardia
c. Low renal tubular threshold
d. DM
Ans. The key is B. Atrial flutter. [Saw-tooth like
waves, normal QRS complex of 80 ms (normal
Q. 1. What is the key? range 70-100 ms), ventricular rate of 150/min and
Q. 2. Why glycosuria occurred? regular R-R interval is diagnostic of atrial flutter].

Ans. 1. The key is B. Normal finding. 382. A 50 yo woman who was treated for
Ans. 2. Stress during operation can cause breast cancer 3 yrs ago now presents with
transient hyperglycemia causing glycosuria increase thirst and confusion. She has
secondary to stress induced rise of cortisole become drowsy now.
which becomes normal after some time. What is the most likely metabolic
abnormality?
a. Hypercalcemia
380. A pt presented with hx of swelling in b. Hyperkalemia
the region of the sub-mandibular region, c. Hypoglycemia
d. Hyperglycemia swelling in the stoma. What is the
e. Hypocalcemia management option for her?
a. Local exploration
b. Exploratory laparotomy
Q. 1. What is the key?
Q. 2. Justify the key c. Open laparotomy
d. Reassure

Ans. 1. The key is A. Hypercalcemia.


Ans. The key is A. Local exploration.
Ans. 2. Increased thirst, confusion, drowsiness
these are features of hypercalcemia. Any solid
organ tumour can produce hypercalcemia. Here 385. A 65yo female pt was given
treated Ca breast is the probable cause of tamoxifen, which of the following side
hypercalcemia. effect caused by it will concern you?
a. Fluid retention
b. Vaginal bleeding
383. A 29yo woman presents to her GP c. Loss of apetite
with a hx of weight loss, heat intolerance, d. Headache and dizziness
poor conc and palpitations. Which of the e. Anorgasm
following is most likely to be a/w dx of
Q. 1. What is the key?
thyroiditis a/w viral infection?
Q. 2. What is the reason to select this key?
a. Bilateral exophthalmos
b. Diffuse, smooth goiter
c. Reduced uptake on thyroid isotope scan Ans. 1. The key is B. Vaginal bleeding.
d. Positive thyroid peroxidase antibodies Ans. 2. Tamoxifen can promote development of
e. Pretibial myxedema endometrial carcinoma. So vaginal bleeding will
be of concern for us.
Q. 1. What is the key?
Q. 2. What is the diagnosis?
Q. 3. Whats are the points in favour?
386. A 39yo man with acute renal failure
presents with palpitations. His ECG shows
Ans. 1. The key is C. Reduced uptake on thyroid
isotope scan.
tall tented T waves and wide QRS
complex. What is the next best step?
Ans. 2. The diagnosis is De Quervain’s or subacute a. Dialysis
thyroiditis. b. IV calcium chloride
Ans. 3. Viral or subacute thyroiditis: diagnostic c. IV insulin w/ dextrose
criteria: i) Features of hyperthyroidism present. ii) d. Calcium resonium
Pain thyroid, not mentioned. iii) Investigations: e. Nebulized salbutamol
high esr (60-100) not mentioned, Reduced uptake
Q. 1. What is the key?
of radioactive iodine by the gland.
Q. 2. What is the diagnosis?
Q. 3. What is the significant of tall tented T waves
and wide QRS complex?
384. A lady, post-colostomy closure after
4days comes with fluctuating small Ans. 1. The key is B. IV calcium chloride (both IV
calcium gluconate or IV calcium chloride can be
used when there is ECG changes).
Ans. 2. The ECG changes are suggestive of anticonvulsants (phenytoin,barbiturates), aspirin,
Hyperkalemia. antituberculoids, and allopurinol.

Ans. 3. At potassium level of >5.5mEq/L occurs 389. A 60yo man presented with a lump in
tall tented T waves and at potassium level the left supraclavicular region. His
>7mEq/L occurs wide QRS complex with bizarre appetite is decreased and he has lost 5kg
QRS morphology.
recently. What is the most probably dx?
387. A 54yo pt 7 days after a total hip a. Thyroid carcinoma
replacement presents with acute onset b. Stomach carcinoma
breathlessness and raised JVP. Which of c. Bronchial carcinoma
d. Mesothelioma
the following inv will be most helpful in
e. Laryngeal carcinoma
leading to a dx?
a. CXR Ans. The key is B. Stomach carcinoma.
b. CTPA [Mentioned lump in the left supraclavicular
c. V/Q scan region is Vershow’s gland,  has long been
d. D-Dimer regarded as strongly indicative of the presence
e. Doppler US of legs of cancer in the abdomen, specifically gastric
cancer].
Q. 1. What is the key?
Q. 2. Justify the key. 390. A 64yo man has presented to the ED
Ans. 1. The key is B. CTPA. with a stroke. CT shows no hemorrhage.
Ans. 2. The patient has a +ve two level PE Wells ECG shows atrial fib. He has been
score (if it was negative we should do D-Dimer) thrombolysed and he’s awaiting
and there is no renal impairment or history discharge.
suggestive of allergy to contrast media (if these What prophylactic regimen is best for
present we should have go for VQ scan) the him?
investigation of choice is PTCA. NICE guideline. a. Warfarin
b. Heparin
388. A 7yo girl has been treated with
c. Aspirin
penicillin after sore throat, fever and
d. Statins
cough. Then she develops skin rash and
e. Beta blockers
itching. What is the most probable dx?
a. Erythema nodosum Ans. The key is A. Warfarine. [Atrial fibrillation:
b. Erythema multiforme post stroke- following a stroke or TIA warfarine
c. SJS should be given as the anticoagulant of choice.
d. Erythema marginatum NICE guideline].
e. Erythema gangernosum

Q. 1. What is the key?


391. A 54yo man after a CVA presents
Q. 2. What common drugs causes this to occur?
with ataxia, intention tremors and slurred
Ans. 1. The key is B. Erythema multiforme. speech. Which part of the brain has been
affected by the stroke?
Ans. 2. Common drugs causing erythma a. Inner ear
multiforme are: antibiotics
b. Brain stem
(including, sulphonamides, penicillin),
c. Diencephalon
d. Cerebrum
e. Cerebellum piloerection, arthralgia these are features of
heroin withdrawal].
Q. 1. What is the key?
Q. 2. What are the features of the condition? 394. A child presents with increasing
Ans. 1. The key is E. Cerebellum. jaundice and pale stools. Choose the most
Ans. 2. i) Ataxia ii) slurred speech or dysarthria iii) appropriate test?
dysdiodokokinesis iv) intention tremor v) a. US abdomen
nystagmus. b. Sweat test
c. TFT
d. LFT
e. Endomyseal antibodies
392. A 57yo man with blood group A Ans. The key is A. US abdomen. [This is a picture
complains of symptoms of vomiting, suggestive of obstructive jaundice. LFT can give
tiredness, weight loss and palpitations. clue like much raised bilirubin, AST and ALT not
Exam: hepatomegaly, ascites, palpable that high and raised alkaline phosphatase but still
left supraclavicular mass. What is the USG is diagnostic in case of obstructive jaundice].
most likely dx?
a. Gastric carcinoma
395. A 32yo man presents with hearing
b. Colorectal carcinoma loss. AC>BC in the right ear after Rhine
c. Peptic ulcer disease test. He also complains of tinnitus, vertigo
d. Atrophic gastritic and numbness on same half of his face.
e. Krukenburg tumor What is the most appropriate inv for his
condition?
Ans. The key is A. Gastric carcinoma. [i) blood a. Audiometry
group A is associated with gastric cancer ii) b. CT
vomiting, tiredness, weight loss are general c. MRI
features of gastric cancer iii) palpitation from d. Tympanometry
anemia of cancer iv) hepatomegaly (metastasis) e. Weber’s test
and ascites are late features of gastric cancer. v)
palpable left supraclavicular mass- is Vershow’s Ans. The key is C. MRI. [features are suggestive of
gland,  has long been regarded as strongly acaustic neuroma, so MRI is the preferred
indicative of gastric cancer]. option].

393. A 21yo girl looking unkempt, 396. A 56 yo lady with lung cancer
agitated, malnourished and nervous came presents with urinary retention, postural
to the hospital asking for painkillers for hypotension, diminished reflexes and
her abdominal pain. She is sweating, sluggish pupillary reaction.
shivering and complains of joint pain. What is the most likely explanation for her
What can be the substance misuse here? symptoms?
a. Alcohol a. Paraneoplastic syndrome
b. Heroin b. Progression of lung cancer
c. Cocaine c. Brain metastasis
d. LSD d. Hyponatremia
e. Ecstasy e. Spinal cord compression

Ans. The key is B. Heroin. [agitation, nervousness, Ans. The key is A. Paraneoplastic syndrome.
abdominal cramp, sweating, shivering and [Features given are well known features of
autonomic neuropathy which can be a result of on palpation and only one testis was
paraneoplastic syndrome]. normal in size and position.
397. An old woman having decreased What would be your next step?
a. Analgesia
vision can’t see properly at night. She has
b. Antibiotics
changed her glasses quite a few times but
c. Refer urgently to a surgeon
to no effect. She has normal pupil and
d. Reassurance
cornea.
e. Discharge with analgesics
What is the most likely dx?
a. Cataract Ans. The key is A. Analgesia. [According to some
b. Glaucoma US sites it is analgesia but no UK site support
c. Retinal detachment this!!! So for Plab exam the more acceptable
d. Iritis option is C. Refer urgently to a surgeon].
e. GCA
400. A child suffering from asthma
Ans. The key is B. Glaucoma. It is a wrong key. presents with Temp 39C, drooling saliva
Correct key should be A. Cataract. [Age and on to the mother’s lap, and taking oxygen
normal pupil and cornea are suggestive of by mask. What sign will indicate that he is
cataract. If it was glaucoma pupil would be a bit deteriorating?
dilated and/or oval in shape]. a. Intercostal recession
b. Diffuse wheeze
398. A pt comes with sudden loss of vision.
c. Drowsiness
On fundoscopy the optic disc is normal.
What is the underlying pathology? Ans. The key is A. Intercostal recession. This is
a. Iritis wrong key. Correct key is C. Drowsiness.
b. Glaucoma [Intercostal recession is a sign of severe asthma
c. Vitreous chamber but it can be seen at a lesser degree as well. So
d. Retinal detachment drowsiness is more appropriate answer].

Q. 1. What is the key? 401. A 12yo boy presents with painful


Q. 2. What are the causes of sudden painless loss swollen knew after a sudden fall. Which
of vision? bursa is most likely to be affected?
Ans. 1. The Key is D. Retinal detachment. a. Semimembranous bursa
Ans. 2. Causes of sudden painless loss of vision: b. Prepatellar bursa
c. Pretibial bursa
 Retinal detachment
d. Suprapatetaller bursa
 Vitreous haemorrhage
 Retinal vein occlusion Ans. The key is B. Prepatellar bursa. [A fall onto
 Retinal artery occlusion the knee can damage the prepatellar bursa. This
 Optic neuritis usually causes bleeding into the bursa sac causing
 Cerebrovascular accident
swellen painful knee. Prepatellar bursitis that is
caused by an injury will usually go away on its
own. The body will absorb the blood in the bursa
over several weeks, and the bursa should return
to normal. If swelling in the bursa is causing a
slow recovery, a needle may be inserted to drain
399. A child was woken up from sleep with
the blood and speed up the process. There is a
severe pain in the testis. Exam: tenderness
slight risk of infection in putting a needle into the thalamus ii) the subthalamus iii) the
bursa]. hypothalamus and iv) the epithalamus. Among
these the hypothalamus has crucial role in
402. A 61yo man has been referred to the causing urinary frequency].
OPD with frequent episodes of
breathlessness and chest pain a/w 404. A man is very depressed and
palpitations. He has a regular pulse miserable after his wife’s death. He sees
rate=60bpm. ECG=sinus rhythm. What is no point in living now that his wife is not
the most appropriate inv to be done? around and apologises for his existence.
a. Cardiac enzymes He refuses any help offered. His son has
b. CXR brought him to the ED. The son can’t deal
c. ECG with the father any more. What is the
d. Echo most appropriate next step?
e. 24h ECG a. Voluntary admission to psychiatry ward
b. Compulsory admission under MHA
Ans. The key is E. 24h ECG. c. Refer to social services
d. Alternate housing
e. ECT
Indications of 24 h ambulatory holter monitoring:
Ans. The key is B. Compulsory admission under
 To evaluate chest pain not reproduced
MHA. [This patient is refusing any help offered!
with exercise testing
 To evaluate other signs and And his son cannot deal with him anymore! In
symptoms that may be heart-related, this situation voluntary admission to psychiatry
such as fatigue, shortness of breath, ward is not possible and the option of choice is
dizziness, or fainting “compulsory admission under MHA”. The point
 To identify arrhythmias or palpitations here is the man has felt himself in danger by self
 To assess risk for future heart-related neglect].
events in certain conditions, such as
idiopathic hypertrophic cardiomyopathy, 405. A 31yo man has epistaxis 10 days
post-heart attack with weakness of the following polypectomy. What is the most
left side of the heart, or Wolff-Parkinson- likely dx?
White syndrome a. Nasal infection
 To assess the function of an implanted b. Coagulation disorder
pacemaker
c. Carcinoma
 To determine the effectiveness of therapy
for complex arrhythmias Ans. The key is A. Nasal infection. [Infection is one
of the most important cause of secondary
403. A woman dx with Ca Breast presents hemorrhage].
now with urinary freq. which part of the
brain is the metastasis spread to?
a. Brain stem
b. Pons 406. A woman had an MI. She was
c. Medulla breathless and is put on oxygen mask and
d. Diencephalon GTN, her chest pain has improved. Her
e. Cerebral cortex HR=40bpm. ECG shows ST elevation in
leads I, II, III.
Ans. The key is D. Diencephalon. [diencephalon is
What is your next step?
made up of four distinct components: i) the
a. LMWH c. Adenosine
b. Streptokinase d. Verapamil
c. Angiography e. Flecainide
d. Continue current management
e. None Ans. The key is B. Beta blockers [the probable
arrhythymia is AF secondary to thyrotoxicosis. So
Ans. The key is B. Streptokinase. to rapid control the symptoms of thyrotoxicosis
Beta blocker should be used which will improve
407. A 67yo male presents with polyuria the arrythmia].
and nocturia. His BMI=33, urine culture =
negative for nitrates. What is the next dx 410. A T2DM is undergoing a gastric
inv? surgery. What is the most appropriate
a. PSA pre-op management?
b. Urea, creat and electrolytes a. Start him in IV insulin and glucose and K+
c. MSU culture and sensitivity just before surgery
d. Acid fast urine test b. Stop his oral hypoglycemic on the day of
e. Blood sugar the procedure
c. Continue regular oral hypoglycemic
d. Stop oral hypoglycemic the prv night and
Ans. The key is E. Blood sugar. [Age at start IV insulin with glucose and K+ before
presentation and class1 obesity favours the surgery
diagnosis of type2 DM]. e. Change to short acting oral hypoglycemic
408. A pt from Africa comes with nodular Ans. The key is D. Stop oral hypoglycemic the prv
patch on the shin which is reddish brown. night and start IV insulin with glucose and K+
What is the most probable dx? before
a. Lupus vulgaris
b. Erythema nodosum Surgery.
c. Pyoderma gangrenosum
d. Erythema marginatum
e. Solar keratosis 411. A 19yo boy is brought by his mother
with complaint of lack of interest and no
Ans. The key is B. Erythema nodosum. [Causes of social interactions. He has no friends, he
erythema nodosum: MOST COMMON CAUSES- i) doesn’t talk much, his only interest is in
streptococcal infection ii) sarcoidosis. Other
collecting cars/vehicles having around
causes- tuberculosis, mycoplasma pneumonia,
2000 toy cars. What is the most
infectious mononucleosis, drugs- sulfa related
drug, OCP, oestrogen; Behcet’s disease, CD, UC;
appropriate dx?
a. Borderline personality disorder
lymphoma, leukemia and some others].
b. Depression
409. A 29yo lady came to the ED with c. Schizoaffective disorder
complaints of palpitations that have been d. Autistic spectrum disorder
there for the past 4 days and also feeling
warmer than usual. Exam: HR=154bpm,
irregular rhythm. What is the tx for her Ans. The key is D. Autistic spectrum disorder.
condition?
Autism spectrum disorders affect three different
a. Amiadarone
areas of a child's life:
b. Beta blockers
 Social interaction baby of his age. What is the single most
 Communication -- both verbal and recommended technique for cardiac
nonverbal compressions?
a. All fingers of both hands
 Behaviors and interests b. All fingers of one hand
In some children, a loss of language is the major c. Heel of one hand
impairment. In others, unusual behaviors (like d. Heel of both hand
spending hours lining up toys) seem to be the e. Index and middle fingertips of one hand
dominant factors.
Ans. The key is E. Index and middle fingertips of
one hand.

412. A 45yo man who is diabetic and HTN 414. A 70yo man had a right
but poorly compliant has chronic SOB, hemicolectomy for ceacal carcinoma
develops severe SOB and chest pain. Pain 6days ago. He now has abdominal
is sharp, increased by breathing and distension and recurrent vomiting. He has
relieved by sitting forward. What is the not opened his bowels since surgery.
single most appropriate dx? There are no bowel sounds. WBC=9,
a. MI Temp=37.3C.
b. Pericarditis What is the single most appropriate next
c. Lung cancer management?
d. Good pastures syndrome a. Antibiotic therapy IV
e. Progressive massive fibrosis b. Glycerine suppository
c. Laparotomy
d. NG tube suction and IV fluids
Ans. The key is B. Pericarditis. [Nature of pain i.e. e. TPN
sharp pain increased by breathing and relieved by
sitting forward is suggestive of pericarditis]. Ans. The key is D. NG tube suction and IV fluids.
[The patient has developed paralytic ileus which
Nature of pericardial pain: the most common should be treated conservatively].
symptom is sharp, stabbing chest pain behind the
sternum or in the left side of your chest. 415. A 60yo man with a 4y hx of thirst,
However, some people with acute pericarditis urinary freq and weight loss presents with
describe their chest pain as dull, achy or pressure- a deep painless ulcer on the heel. What is
like instead, and of varying intensity. the most appropriate inv?
The pain of acute pericarditis may radiate to your a. Ateriography
left shoulder and neck. It often intensifies when b. Venography
you cough, lie down or inhale deeply. Sitting up c. Blood sugar
and leaning forward can often ease the pain.  d. Biopsy for malignant melanoma
e. Biopsy for pyoderma
413. A 6m boy has been brought to ED
following an apneic episode at home. He Ans. The key is C. Blood sugar. [The patient
is now completely well but his parents are probably developed diabetic foot].
anxious as his cousin died of SIDS (Sudden 416. A 16yo boy presents with rash on his
Infant Death Syndrome) at a similar age. buttocks and extensor surface following a
The parents ask for guidance on BLS for a
sore throat. What is the most probable management for type A dissection is surgery and
dx? for type B is conservative].
a. Measles 419. A 54yo myopic develops flashes of
b. Bullous-pemphigoig light and then sudden loss of vision. That
c. Rubella is the single most appropriate tx?
d. ITP a. Pan retinal photo coagulation
e. HSP b. Peripheral iridectomy
c. Scleral buckling
Ans. The key is D. ITP. It’s probably a wrong key! d. Spectacles
The correct key should be E. HSP. [In HSP rash e. Surgical extraction of lens
typically found in buttocks, legs and feets and
may also appear on the arms, face and trunk. But Ans. The key is C. Scleral buckling. [It is a case of
in ITP it mostly occurs in lower legs. HSP usually retinal detachment with treatment option of
follow a sorethroat and ITP follow viral infection scleral buckling].
like flue or URTI. HSP is a vasculitis while ITP is
deficiency of platelets from more destruction in 420. A 40yo chronic alcoholic who lives
spleen which is immune mediated]. alone, brought in the ED having been
found confused at home after a fall. He
417. A 34yo man with a white patch on complains of a headache and gradually
the margin of the mid-third of the tongue. worsening confusion. What is the most
Which is the single most appropriate LN likely dx?
involved? a. Head injury
a. External iliac LN b. Hypoglycemia
b. Pre-aortic LN c. Extradural hematoma
c. Aortic LN
d. Subdural hematoma
d. Inguinal LN
e. Iliac LN e. Delirium
f. Submental LN
Ans. The key is D. Subdural hematoma. [subdural
g. Submandibular LN
h. Deep cervical LN hematoma may be acute or chronic. In chronic
symptoms may not be apparent for several days
Ans. The key is G. Submandibular LN. or weeks. Symptoms of subdural hematomas are:
fluctuating level of consciousness, ± insidious
418. A 50yo lady presents to ED with physical or intellectual slowing, sleepiness,
sudden severe chest pain radiating to both headache, personality change and unsteadiness.
shoulder and accompanying SOB. Exam: Tx. Irrigation/evacuation e.g. via barr twist drill
cold peripheries and paraparesis. and barr hole craniostomy 1st line. Craniotomy if
What is the single most appropriate dx? the clot organized 2nd line].
a. MI
421. A 54yo man with alcohol
b. Aortic dissection
dependence has tremor and sweating
c. Pulmonary embolism
3days into a hosp admission for a fx
d. Good pastures syndrome
femur. He is apprehensive and fearful.
e. Motor neuron disease
What is the single most appropriate tx?
Ans. The key is B. Aortic dissection. [Cold a. Acamprossate
peripheries due to reduced blood flow to dista b. Chlordiazepoxide
parts of dissection and reduced perfusion of c. Lorazepam
nerves resulted in paraparesis. Usual d. Lofexidine
e. Procyclidine muscles tightness on 3 occasions in the
last 6 wks. He doesn’t complain of
Ans. The key is B. Chlordiazepoxide. C. Lorazepam
is also correct key! [According to NICE 1 st line
headache and his BP is WNL.
treatment is oral lorazepam and if symptom What is the single most appropriate long-
persists or oral medication is declined, give term tx for him?
parenteral lorazepam, haloperidol or olanzapine. a. Diazepam
According to OHCM 1st line treatment is b. Olanzapine
chlordiazepoxide. c. Haloperidol
d. Fluoxetine
Chlordiazepoxide should only be used at the
e. Alprazolam
lowest possible dose and for a maximum of up to
four weeks. This will reduce the risks of Ans. The key is D. Fluoxetine. [Recommended
developing tolerance, dependence and treatment for panic disorder is i) CBT ii)
withdrawal]. Medication (SSRIs or TCA). NICE recommends a
total of seven to 14 hours of CBT to be completed
within a four month period. Treatment will
422. A 5yo child complains of sore throat usually involve having a weekly one to two hour
and earache. He is pyrexial. Exam: tonsils session. When drug is prescribed usually a SSRI is
enlarged and hyperemic, exudes pus when preferred. Antidepressants can take two to four
pressed upon. weeks before becoming effective].
What is the single most relevant dx?
a. IM
b. Acute follicular tonsillitis 424. A 28yo man presents with rapid
c. Scarlet fever pounding in the chest. He is completely
d. Agranulocytosis conscious throughout. The ECG was taken
e. Acute OM (SVT). What is the 1st med to be used to
manage this condition?
Ans. The key is B. Acute follicular tonsillitis. a. Amiodarone
[Tonsillitis is usually caused by a viral infection b. Adenosine
or, less commonly, a bacterial infection. The c. Lidocaine
given case is a bacterial tonsillitis (probably d. Verapamil
caused by group A streptococcus). There are four
e. Metoprolol
main signs that tonsillitis is caused by a bacterial
infection rather than a viral infection. They are: Ans. The key is B. Adenosine. [Management of
SVT: i) vagal manoeuvres (carotid sinus message,
 a high temperature
 white pus-filled spots on the tonsils valsalva manoeuvre) transiently increase AV-
 no cough block, and unmask the underlying atrial rhythm. If
 swollen and tender lymph nodes (glands). unsuccessful then the first medicine used in SVT is
adenosine, which causes transient AV block and
works by i) transiently slowing ventricles to show
the underlying atrial rhythm ii) cardioverting a
junctional tachycardia to sinus rhythm. OHCM].

423. A man with a fam hx of panic


disorder is brought to the hosp with 425. A 56yo woman who is depressed
palpitations, tremors, sweating and after her husband died of cancer 3m ago
was given amitryptaline. Her sleep has b. Long saphenous vein
improved and she now wants to stop c. Deep venous system
medication but she still speaks about her d. Popliteal veins
husband. How would you manage her? e. Saphano-femoral junction
a. CBT
Ans. The key is A. Short saphenous vein. [short
b. Continue amitryptaline
saphenous vein travels lateral aspect of ankle
c. Psychoanalysis
while great or long saphenous vein travels medial
d. Bereavement counselling aspect of ankle].
e. Antipsychotic

Ans. The key is B. Continue amitriptyline.


[depression is important feature of bereavement. 428. A 55yo man presents with hx of
Patient may pass sleepless nights. As this patients weight loss and tenesmus. He is dx with
sleep has improved it indicate she has good rectal carcinoma.
response to antidepressant and as she still speaks Which risk factors help to develop rectal
about her husband there is chance to carcinoma except following?
deterioration of her depression if antidepressant a. Smoking
is stopped. For depressive episodes b. Family hx
antidepressants should be continued for at least c. Polyp
6-9 months]. d. Prv carcinoma
e. High fat diet
f. High fibre diet
426. A 64yo man presents with a hx of left
sided hemiparesis and slurred speech. He Ans. The key is F. High fibre diet. [except high
was absolutely fine 6h after the episode. fiber diet all others are risk factors to develop
rectal carcinoma].
What is the most appropriate prophylactic
regimen?
a. Aspirin 300mg for 2 weeks followed by 429. A pt presents with a painful, sticky
aspirin 75mg red eye with a congested conjunctiva.
b. Aspirin 300mg for 2 weeks followed by What is the most suitable tx?
a. Antibiotic PO
aspirin 75mg and dipyridamole 200mg
b. Antihistamine PO
c. Clopidogrel 75mg
c. Antibiotic drops
d. Dipyridamole 200mg
d. Steroid drops
e. Aspirin 300mg for 2 weeks
e. IBS
Ans. The key is B. Aspirin 300mg for 2 weeks
followed by aspirin 75mg and dipyridamole Ans. The key is C. Antibiotic drops. [bacterial
200mg. conjunctivitis is treated with antibiotic drops].

427. A 63yo lady with a BMI=32 comes to 430. A 45yo woman complains of pain in
the ED with complaints of pigmentation her hands precipitated by exposure to the
on her legs. Exam: dilated veins could be cold weather. She has breathlessness on
seen on the lateral side of her ankle. walking. When she is eating, she can feel
Which of the following is involved? food suddenly sticking to the gullet. It
a. Short saphenous vein seems to be in the middle of the
esophagus but she can’t localize exactly Ans. The key is D. ITP. [As the patient is otherwise
where it sticks. It is usually relieved with a well acute leukemia, HIV and SLE is unlikely.
drink of water. Normal wbc count excludes aplastic anemia. So
likely diagnosis is ITP].
Choose the single most likely cause of
dysphagia from the options?
a. Esophageal carcinoma
b. Systemic sclerosis 433. A 30yo man complains of episodes of
c. SLE hearing music and sometimes threatening
d. Pharyngeal carcinoma voices within a couple of hours of heavy
e. Globus hystericus drinking. What is the most likely dx?
a. Delirium tremens
Ans. The key is B. Systemic sclerosis. [Raynods b. Wernicke’s encephalopathy
phenomena, pulmonary involvement, c. Korsakoff’s psychosis
oesophageal dysmotility are suggestive of d. Alcohol hallucinosis
systemic sclerosis]. e. Temporal lobe dysfunction

Ans. The key is D. Alcoholic hallucinosis. [Alcohol


hallucinosis can occur during acute intoxication or
withdrawal.  It involves auditory and visual
431. A 3yo child brought to the ED with a hallucinations, most commonly accusatory or
swelling over the left arm. XR shows threatening voices. Source: Wikipedia].
multiple callus formation in the ribs.
Exam: bruises on childs back. 434. A pt had TIA which he recovered
What is the most appropriate next step? from. He has a hx of stroke and exam
a. Check child protection register shows HR in sinus rhythm. He is already
b. Coagulation profile on aspirin 75mg and anti-HTN drugs.
c. Skeletal survey What other action should be taken?
d. Serum calcium a. Add clopidogrel only
e. DEXA scan b. Increase dose of aspirin to 300mg
c. Add warfarin
Ans. The key is C. Skeletal survey. [Skeletal survey d. Add clopidogrel and statin
is a series of x-ray which is usually used in NAI]. e. Add statin only
[after survey should think of childs protection].
Ans. The key is D. Add clopidogrel and statin. This
is wrong key! Correct key should be E. Add statin
432. A 35yo woman has had bruising and only. [He who is already on aspirin need no
change in aspirin dose. Clopidogrel or warfarine is
petechiae for a week. She has also had
not indicated. Correct option is add statin only].
recent menorrhagia but is otherwise well.
Blood: Hgb=11.1, WBC=6.3, Plt=14.
What is the single most likely dx?
a. Acute leukemia
435. A 40yo woman suddenly collapsed
b. Aplastic anemia
and died. At the post-mortem autopsy, it
c. HIV infection was found that there a bleed from a berry
d. ITP aneurysm from the circle of Willis.
e. SLE In which space did the bleeding occur?
a. Subarachnoid
b. Subdural
c. Extradural 439. A 72yo man is found to be not
d. Subparietal breathing in the CCU with the following
e. Brain ventricles rhythm. What is the most likely dx?
Ans. The key is A. Subarachnoid.

436. A schizophrenic pt hears people only


when he is about to fall asleep. What is
the most likely dx?
a. Hypnopompic hallucinations a. SVT
b. Hyponogogic hallucinations b. VT
c. Hippocampal hallucinations c. VF
d. Delirious hallucinations d. Atrial fib
e. Auditory hallucinations e. Atrial flutter

Ans. The key is B. Hypnogogic hallucinations.


Ans. The key is C. VF.

437. A pt who came from India presents


with cough, fever and enlarged cervical
LN. Exam: caseating granulomata found 440. A 65yo man with difficulty in
in LN. What is the most appropriate dx? swallowing presents with an aspiration
a. Lymphoma
pneumonia. He has a bovine cough and
b. TB adenitis
fasciculating tongue. Sometimes as he
c. Thyroid carcinoma
swallows food it comes back through his
d. Goiter
e. Thyroid cyst
nose. Choose the single most likely cause
of dysphagia from the given option?
Ans. The key is B. TB adenitis. [caseating a. Bulbar palsy
granulomata are diagnostic of TB]. b. Esophageal carcinoma
c. Pharyngeal pouch
d. Pseudobulbar palsy
438. A 44yo man comes with hx of early e. Systemic sclerosis
morning headaches and vomiting. CT
Ans. The key is A. Bulbar palsy. [Dysphagia, nasal
brain shows ring enhancing lesions. What
regurgitation, fasciculating tongue these are
is the single most appropriate option? features of bulbar palsi due to lower motor
a. CMV
neuronal lesion of IX-XII nerves].
b. Streptococcus
c. Toxoplasmosis
d. NHL
441. A 16yo teenager was brought to the
e. Pneumocystis jerovii
ED after being stabbed on the upper right
Ans. The key is C. Toxoplasmosis. side of his back. Erect CXR revealed
homogenous opacity on the lower right
lung, trachea was centrally placed. What
is the most probable explanation for the What is the single most appropriate tx?
XR findings? a. Pan retinal photo coagulation
a. Pneumothorax b. Pilocarpine eye drops
b. Hemothorax c. Corticosteroids
c. Pneumonia d. Peripheral iridectomy
d. Tension pneumothorax e. Surgical extraction of lens
e. Empyema
Ans. The key is C. Corticosteroid. [Sudden loss of
Ans. The key is B. Hemothorax. [In blunt trauma vision, remission and relapse of optic neuritis and
there may be hemo-pneumothorax but in sharp focal neurological symptoms and exaggerated
wound like stabbing there may occur only reflexes all points towards multiple sclerosis.
hemothorax]. Treatment option is corticosteroids].

444. A 15yo boy presents with a limp and


442. A 55yo woman complains of pain in the knee. Exam: leg is externally
retrosternal chest pain and dysphagia rotated and 2cm shorter. There is
which is intermittent and unpredictable. limitation of flexion, abduction and medial
The food suddenly sticks in the middle of rotation. As the hip is flexed external
the chest, but she can clear it with a drink rotation is increased. Choose the most
of water and then finish the meal without likely dx?
any further problem. A barium meal a. Juvenile rheumatoid arthritis
shows a ‘corkscrew esophagus’. b. Osgood-schlatter disease
What is the single most likely dysphagia? c. Reactive arthritis
a. Esophageal candidiasis d. Slipped femoral epiphysis
b. Esophageal carcinoma e. Transient synovitis of the hip
c. Esophageal spasm
d. Pharyngeal pouch Ans. The key is D. Slipped femoral epiphysis. [The
e. Plummer-vinson syndrome given presentation is classic for slipped femoral
epiphysis].
Ans. The key is C. Esophageal spasm. [chest pain,
unpredictable intermittent dysphagia and food
suddenly sticks in the middle of the chest which 445. A 64yo woman has difficulty moving
can be cleared with a drink of water indicates her right shoulder on recovering from
uncoordinated irregular esophageal peristalsis surgery of the posterior triangle of her
which is characteristic of esophageal spasm! Also neck. What is the single most appropriate
“cork-screw esophagus” seen in barium swallow option?
is diagnostic of esophageal spasm]. a. Accessory nerve
b. Glossopharyngeal nerve
c. Hypoglossal nerve
d. Vagus nerve
443. A 38yo female presents with sudden e. Vestibule-cochlear nerve
loss of vision but fundoscopy is normal.
She a similar episode about 1 y ago which
resolved completely within 3m. Exam: Ans. The key is A. Accessory nerve.
mild weakness of right upper limb and
exaggerated reflexes.
446. A 37yo man with an ulcer on the
medial malleolus. Which of the following 449. A young pt is complaining of vertigo
LN is involved? whenever she moves sideways on the bed
a. External iliac LN while lying supine. What would be the
b. Pre-aortic LN most appropriate next step?
c. Aortic LN a. Head roll test
d. Inguinal LN b. Reassure
e. Iliac LN
c. Advice on posture
f. Submental LN
g. Submandibular LN d. Carotid Doppler
h. Deep cervical LN e. CT

Ans. The key is D. Inguinal LN. Ans. The key is A. Head roll test. [this is a case of
“benign paroxysmal positional vertigo” for which
the diagnosis is made by head roll test].

447. A pt presents with weight loss of


5kgs despite good appetite. He also 450. A 32yo man has OCD. What is the
complains of palpitations, sweating and best tx?
diarrhea. He has a lump in front of his
a. CBT
neck which moves on swallowing. What is
b. SSRI
the most appropriate dx?
c. TCA
a. Lymphoma
d. MAO inhibitors
b. TB adenitis
e. Reassure
c. Thyroid Ca
d. Goiter Ans. The key is B. SSRI. [It is wrong key! There is a
e. Thyroid cyst GMC sample question with most appropriate
management being CBT. So the correct key is
Ans. The key is D. Goiter.
CBT].

448. A 76yo woman has become tired and 451. A 65yo woman says she died 3m ago
confused following an influenza like and is very distressed that nobody has
illness. She is also breathless with signs of buried her. When she is outdoors, she
consolidation of the left lung base. hears people say that she is evil and needs
What is the most likely dx? to be punished. What is the most likely
a. Drug toxicity
explanation for her symptoms?
b. Delirium tremens
a. Schizophrenia
c. Infection toxicity
b. Mania
d. Hypoglycemia
c. Psychotic depression
e. Electrolyte imbalance
d. Hysteria
A. The key is C. Infection toxicity. [Infection e. Toxic confusional state
toxicity or toxic shock syndrome is likely here as
Ans. The key is C. Psychotic depression. [Psychotic
preceding flue like illness points towards toxin
depression, also known as depressive psychosis,
(enterotoxin type B) from Staphylococcus
is a major depressive episode that is accompanied
aureus].
by psychotic symptoms (hallucinations,
delusions). In this patient nihilistic delusion Ans. The key is C. 4 x weight(kgs) x area of burn =
favours the diagnosis of psychotic depression. It ml of fluids.
can occur in the context of bipolar disorder or
majordepressive disorder].
454. A 65yo male presents with dyspnea
and palpitations. Exam: pulse=170bpm,
452. A 50yo woman presents following a BP=120/80mmHg. Carotid massage has
fall. She reports pain and weakness in her been done as first instance.
hands for several months, stiff legs, What is the next step of the
swallowing difficulties, and has bilateral management?
wasting of the small muscles of her hands. a. Adenosine
Reflexes in the upper limbs are absent. b. Amlodipine
Tongue fasciculations are present and c. DC cardioversion
both legs show increased tone, pyramidal d. Lidocaine
weakness and hyper-reflexia with e. Beta blocker
extensor plantars. Pain and temp
Ans. The key is A. Adenosine. [The likely diagnosis
sensation are impaired in the upper limbs.
is SVT. 1st vagal manoeuvres, if fails iv adenosine.
What is the most likely dx?
a. MS • Vagal manoeuvres (carotid sinus massage,
b. MND Valsalva manoeuvre) transiently increase
c. Syringobulbia
AV block, and may unmask an underlying atrial
d. Syringomyelia rhythm.
e. Myasthenia gravis
• If unsuccessful, give adenosine, which causes
Ans. The key is C. Syringobulbia. [In MS there are transient AV block; OHCM, 9th edition].
characteristic relapse and remission which is
absent here; In MND there is no sensory deficit;
Syringomyelia doesn’t cause cranial nerve lesion 455. A 48yo farmer presented with fever,
and in myasthenia there is muscular weakness
malaise, cough and SOB. Exam:
without atrophy. Here the features described well
tachypnea, coarse endinspiratory crackles
fits with syringobulbia].
and wheeze throughout, cyanosis. Also
complaint severe weight loss. His CXR
453. Which of the following formulas is shows fluffy nodular shadowing and there
is PMN leukocytosis. What is the single
used for calculating fluids for burn pts?
most appropriate dx?
a. 4 x weight(lbs) x area of burn = ml of a. Ankylosing spondylitis
fluids b. Churg-strauss syndrome
b. 4 x weight(kgs) x area of burn = L of fluids c. Cryptogenic organizing
c. 4 x weight(kgs) x area of burn = ml of d. Extrinsic allergic alveolitis
fluids e. Progressive massive fibrosis
d. 4 x weight(lbs) x area of burn = L of fluids
e. 4.5 x weight(kgs) x area of burn = dL of
fluids Ans. The key is D. Extrinsic allergic alveolitis.
456. A 35yo lady is admitted with pyrexia, Ans. The key is D. Vancomycin.
weight loss, diarrhea and her skin is lemon [Pseudomembranous colitis is treated with
yellow in color. CBC = high MCV. What is metronidazole or vancomycin].
the most probably dx?
a. Aplastic anemia
b. Pernicious anemia 459. A 56yo man has symptoms of sleep
c. Leukemia apnea and daytime headaches and
d. ITP somnolence. Spirometry shows a
e. Lymphoma decreased tidal volume and vital capacity.
What is the single most appropriate dx?
Ans. The key is B. Pernicious anemia. [It may be a. Ankylosing spondylitis
graves with pernicious anemia. Lemon yellow b. Churg-strauss syndrome
pallor occurs in pernicious anemia. c. Good pasture syndrome
Hyperthyroidism may cause persistently raised d. Motor neuron disease
body temperature. Both are autoimmune disease
e. Progressive massive fibrosis
which favours this association].
f. Spinal cord compression

Ans. The key is D. Motor neuron disease.


457. A 72yo woman who had a repair of [involvement of respiratory muscles in MND is
strangulated femoral hernia 2 days ago associated with poor respiration causing sleep
becomes noisy, aggressive and confused. apnoea].
She is febrile, CBC normal apart from
raised MCV. What is the most likely dx? 460. A 55yo man presents with mild
a. Electrolyte imbalance
headache. He has changed his spectacles
b. Delirium tremens
thrice in 1 yr. there is mild cupping present
c. Wernicke’s encephalopathy
in the disc and sickle shaped scotoma
d. Infection toxicity
present in both eyes. What is the single
e. Hypoglycemia
most appropriate tx?
Ans. The key is B. Delirium tremens. [Electrolyte a. Pan retinal photo coagulation
imbalance may cause confusion but not b. Pilocarpine eye drops
aggressiveness; infection toxicity will cause high c. Corticosteroids
fever, low BP, rash etc which are absent here d. Scleral buckling
(fever here is hyperthermia of delirium tremens). e. Analgesics alone
Abstinance from alcohol in the hospital caused
delirium tremens (chronic alcoholism is Ans. The key is B. Pilocarpine. [This is a case of
supported by high MCV) here]. open angle glaucoma, treated with pilocarpine].

458. An old lady had UTI and was treated 461. A 55yo woman was found collapsed
with antibiotics. She then developed at home, paramedics revived her but in
diarrhea. What is the single most likely tx? the ambulance she had a cardiac arrest
a. Co-amoxiclav and couldn’t be saved. The paramedic’s
b. Piperacillin + tazobactam report tells that the woman was immobile
c. Ceftriaxone lately due to hip pain and that they found
d. Vancomycin
ulcers on the medial side of ankle. She had
DM and was on anti-diabetics.
What is the cause of her death? 464. A 30yo female attends OPD with a
a. Acute MI fever and dry cough. She says that she
b. DKA had headache, myalgia and joint pain like
c. Pulmonary embolism one week ago. Exam: pulse=100bpm,
d. Acute pericarditis temp=37.5C. CXR: bilateral patchy
e. Cardiac tamponade consolidation.
Ans. The key is C. Pulmonary embolism.
What is the single most likely causative
[Immobilization due to hip pain may resulted in organism?
DVT and later pulmonary embolism]. a. Pneumococcal pneumonia
b. Legionella
c. Mycoplasma
d. Klebsiella
462. An 18yo previously well student is in
e. Chlamydia pneumonia
his 1st year at uni. He has been brought to
the ED in an agitated, deluded and Ans. The key is C. Mycoplasma. [mycoplasma give
disoriented state. What is the most flu-like symptoms and has a gradual onset plus on
probable reason for his condition? cxr shows bilateral consolidation].
a. Drug toxicity
b. Delirium tremens
c. Infection toxicity
465. A 46yo man is being investigated for
d. Electrolyte imbalance
indigestion. Jejunal biopsy shows
e. Head injury
deposition of
macrophages containing PAS (Periodic
Ans. The key is A. Drug toxicity. [Young age and 1 st acid-schiff) +ve granules. What is the most
yr in university is likely to point towards drug likely dx?
toxicity]. a. Bacterial overgrowth
b. Celiac disease
c. Tropical sprue
463. A young adult presents to the ED d. Whipple’s disease
after a motorcycle crash. The pt has e. Small bowel lymphoma
bruises around the left orbital area.
Ans. The key is D. Whipple’s disease. [periodic
GCS=13, examination notes alcoholic
acid-schiff +ve granules containing macrophages
breath. Shortly afterwards, his GCS drops
in jejuna biopsy is diagnostic of whipples disease].
to 7.
What is the single most important initial
assessment test? 466. A 32yo woman of 38wks gestation
a. MRI brain complains of feeling unwell with fever,
b. CT brain rigors and abdominal pains. The pain was
c. CXR initially located in the abdomen and was
d. CT angio brain a/w urinary freq and dysuria. The pain has
e. Head XR now become more generalized specifically
radiating to the right loin. She says that
Ans. The key is B. CT brain. [Likely cause is
she has felt occasional uterine tightening.
epidural hematoma].
CTG is reassuring. Select the most likely Ans. The key is A. Alcohol consumption.
dx? [abstinence from alcohol in the hospital lead to
a. Acute fatty liver of pregnancy delirium tremens].
b. Acute pyelonephritis
c. Roung ligament stretching
d. Cholecystitis 469. A 10yo girl presents with pallor and
e. UTI features of renal failure. She has
hematuria as well as proteinuria. The
Ans. The key is B. Acute pyelonephritis. [Fever, serum urea and creat are elevated. These
rigor, abdominal pain a/w frequency and dysurea
symptoms started after an episode of
and radiation to the rt loin suggests rt sided
bloody diarrhea 4days ago.
pyelonephritis].
What is the most probable dx?
a. TTP
467. A 32yo pt presents with cervical b. HUS
lymphadenopathy and splenomegaly. c. ITP
What is the single most appropriate d. HSP
option? e. ARF
a. Hemophilus
b. Streptococcus Ans. The key is B. HUS. [Most cases of hemolytic
uremic syndrome develop in children after two to
c. Toxoplasmosis
14 days of diarrhea often bloody, due to infection
d. NHL
with a certain strain of E. coli. Features may be i)
e. Pneumocystis jerovcii
abdominal pain, ii) pale skin, iii) hematuria and
Ans. The key is D. NHL. [ Here only two points are proteinuria, iv) features of renal failure like-
mentioned- cervical lymphadenopathy and nausea/vomiting, swelling of face, hand, feet or
splenomegaly! This combination makes NHL as entire body etc. v) elevated urea and creatinine
the most likely cause thouh splenomegaly is a etc.].
relatively uncommon feature of it!!! This
combination does not fit in other options!]
470. A 40yo woman has had intermittent
tension, dizziness and anxiety for
468. A 62yo man who was admitted for 4months. Each episode usually resolves
surgery 3days ago suddenly becomes after a few hours. She said she takes
confused. His attn span is reduced. He is alcohol to make her calm. She is in a
restless and physically aggressive and loving relationship and has no probs at
picks at his bed sheets. What single aspect work or home.
of the pt’s hx recovered in his notes is What is the next step in her
most likely to aid in making the dx? management?
a. Alcohol consumption a. Collateral info
b. Head trauma b. CT b0rain
c. Hx of anxiety c. CBC
d. Prescribed med d. LFT
e. Obvious cognitive impairment e. TFT

Ans. The key is A. Collateral info. [Likely diagnosis


is panic disorder. Collateral info from family,
friends and other peers should be asked to find d. Optic nerve
out the cause for her anxiety]. e. Trigeminal nerve

471. A 45yo IV drug abuser is brought into Ans. The key is B. Trochlear nerve. [oculomotor
the ED with complaint of fever, shivering, may cause palsy of inferior rectus, medial rectus
malaise, SOB and productive cough. and superior rectus causing double vision in
Exam: temp=39C, pulse=110bpm, multiple gaze! But trochlear involving superior
BP=100/70mmHg. Inv: CXR=bilateral oblique only causes diplopia in downgaze only. So
cavitating bronchopneumonia. the answer is Trochlear nerve].
What is the single most likely causative
organism?
a. Mycoplasma 474. L1 level, what is the most
b. Staphylococcus appropriate landmark?
c. Chlamydia pneumonia
a. Mcburney’s point
d. Pseudomonas
b. Stellate ganglion
e. PCP
c. Deep inguinal ring
Ans. The key is B. Staphylococcus. [Among the d. Termination of the spinal cord
given causes Staphylococcus and PCP are e. Transpyloric plane
recognized cause of cavitating pneumonia. This
Ans. The given key is D. Which is a wrong key. The
case is with productive cough which goes more
correct key is E. Transpyloric plane. [
with staphylococcus as PCP is not productive but
The termination of the spinal cord is between L1
rather associated with dry cough. Drug abuse can
and L2 (variable between people).  L1 landmark-
support both staphylococcus and PCP].
duodenum first part, superior mesenteric artery,
hila of both kidneys, upper border of pancreas,
splenic artery, pylorus and fundus of gall
472. A 71yo woman looks disheveled, bladder].
unkempt and sad with poor eye contact.
She has recently lost her husband. Which
of the following describes her condition? 475. A 32yo woman presents to the ED
a. Anxiety
with headache and vomiting. She was
b. Hallucination
decorating her ceiling that morning when
c. Mania
the headache began, felt mainly occipital
d. High mood
with neck pain. Some 2hs later she felt
e. Low mood
nauseated, vomited and was unable to
Ans. The key is E. Low mood. walk. She also noticed that her voice had
altered. She takes no reg meds and has no
significant PMH. Exam: acuity, field and
473. A 62yo male comes to the GP fundi are normal. She has
complaining of double vision while upbeat nystagmus in all directions of gaze
climbing downstairs. Which of the with normal facial muscles and tongue
following nerve is most likely involved? movements. Her uvulas deviated to the
a. Abducens nerve right and her speech is slurred. Limb
b. Trochlear nerve exam: left arm past-pointing and
c. Oculomotor nerve dysdiadochokinesis with reduced pin prick
sensation in her right arm and leg. 477. A 55yo man has a chronic cough and
Although power is normal, she can’t walk sputum, night sweats and weight loss.
as she feels too unsteady. What is the single most likely causative
Where is the most likely site of lesion? organism?
a. Right medial medulla a. Coagulase +ve cocci in sputum
b. Left medial pons b. Gram -ve diplococci in sputum
c. Left cerebellar hemisphere c. Gram +ve diplococci in sputum
d. Right lateral medulla d. Pneumocystis carinii in sputum
e. Left lateral medulla e. Sputum staining for mycobacterium
tuberculosis
Ans. The key is Left lateral medulla. [ There is a
loss of pain and temperature sensation on the Ans. The key is E. Sputum staining for
contralateral (opposite) side of the body mycobacterium tuberculosis. [Chronic cough and
and ipsilateral (same) side of the face. There is sputum, night sweats and weight loss are classic
associated cerebellar symptoms and other cranial features of tuberculosis].
nerve involvement. The condition is known as
Lateral Medullary Syndrome].
478. A 20yo pregnant 32wks by date
presents to the antenatal clinic with hx of
476. A 28yo female presents with 1 wk hx painless vaginal bleeding after
of jaundice and 2d hx of altered sleep intercourse. Exam: P/A – soft and relaxed,
pattern and moods. She was dx with uterus=dates, CTG=reactive. Choose the
hypothyroidism for which she is receiving single most likely dx?
thyroxine. TFT showed increased TSH. a. Abruption of placenta 2nd to pre-
PT=70s. What is the most probable dx? eclampsia
a. Acute on chronic liver failure b. Antepartum hemorrhage
b. Hyper-acute liver failure c. Placenta previa
c. Autoimmune hepatitis d. Preterm labor
d. Acute liver failure e. Placenta percreta
e. Drug induced hepatitis
Ans. The key is C. Placenta previa.
Ans. The key is C. Autoimmune hepatitis.
[Autoimmune hepatitis may present as acute
hepatitis, chronic hepatitis, or well-established 479. A 30yo man presents to the ED with
cirrhosis. Autoimmune hepatitis rarely presents
difficulty breathing. He has returned from
as fulminant hepatic failure. One third may
India. Exam: throat reveals grey
present as acute hepatitis marked by fever,
hepatic tenderness and jaundice. Non specific membranes on the tonsils and uvula. He
features are anorexia, weight loss and has mild pyrexia. What is the single most
behavioural change (here altered sleep pattern relevant dx?
and moods). There may be coagulopathy (here a. Diphtheria
PT=70s.) leading to epistaxis, gum bleeding etc. b. IM
Presence of other autoimmune disease like c. Acute follicular tonsillitis
hypothyroidism supports the diagnosis of d. Scarlet fever
autoimmune hepatitis]. e. Agranulocytosis
Ans. The key is A. Diphtheria. [history of travel to 482. A 25yo woman presents with urinary
india, grey membrane in tonsil and uvula, low freq, dysuria and fever. Urine microscopy
grade fever, and dyspnoea support the diagnosis shows 20-50 RBC and 10-20 WBC in each
of diphtheria].
field. What is the most probable dx?
a. Schistosmiasis
b. Kidney trauma
480. A 23yo man comes to the ED with a c. Ureteric calculus
hx of drug misuse. He recognizes that he d. Bladder calculi
has a prb and is willing to see a e. Cystitis
psychiatrist. Which of the following terms
best describes this situation? Ans. The key is E. Cystitis. [Hematuria and
a. Judgement significant WBC in urine (>10 per HPF) makes
b. Thought insertion cystitis the most likely diagnosis].
c. Thought block
d. Mood
483. A 65yo presents with dyspareunia
e. Insight
after sex. She in menopause. She
Ans. The key is E. Insight. complains of bleeding after sex. What is
[in psychiatry, the patient's awareness and under the most probably dx?
standing of the origins and meaning of his a. Cervical ca
attitudes, feelings, and behavior and of his disturb b. Endometrial ca
ing symptoms (self-understanding) is known as c. Ovarian ca
insight]. d. Breast ca
e. Vaginal ca

Ans. The key is B. Endometrial carcinoma. [Any


481. A pt with hodgkins lymphoma who is
post menopausal bleeding if associated with pain
under tx develops high fever. His blood
lower abdomen (may be even in the form of
results show WBC <2800 and has a chest dyspareunia) is endometrial carcinoma unless
infection. Choose the most likely tx? proven otherwise. (even painless vaginal bleeding
a. Co-amoxiclav in post menopausal women is highly suspicious!)].
b. Piperacillin+tazobactam
c. Erythromycin
d. Piperacillin+Co-amoxiclav
e. Penicillin+tazobactam
484. A 45yo man underwent an
Ans. The key is B. Piperacillin+tazobactam. [Here
emergency splenectomy following a fall
patients WBC is <2800, i.e. patient has leucopenia
from his bicycle. He smokes 5
(probable neutropenia). Piperacillin/Tazobactam
may be used in the management of neutropenic
cigarettes/day. Post-op, despite mobile,
patients with fever suspected to be due to a he develops swinging pyrexia and a
bacterial infection as in patient with swollen painful left calf. His CXR shows
postchemotherapy neutropenia. Even febrile lung atelectasis and abdominal U
neutropenia can be seen in patients with cancer demonstrates a small sub-phrenic
per-se!]. collection. What is the single most likely
risk factor for DVT in this pt?
a. Immobility
b. Intraperitoneal hemorrhage placenta but abrutio placenta 2nd to pre-
c. Smoking eclumpsia can’t be the option as no features are
d. Splenectomy suggestive of pre-eclumpsia here. So better
e. Sub-phrenic collection option is B. Antepartum hemorrhage which also
includes abruption placenta. Only bleeding
Ans. The key is D. Splenectomy. [Splenectomized cannot indicate whether it is concealed, mixed or
patients have an increased risk of developing revealed].
deep vein thrombosis and pulmonary embolism].

485. A 6m baby had LOC after which he


487. A 62yo lady presents with right sided
had jerky movement of hands and feet.
headache and loss of vision. What is the
What is the most probable dx?
a. Infantile spasm single most inv?
a. ESR
b. Absence
b. BUE
c. Partial simple seizure
c. CT head
d. Atonic seizure
d. XR orbit
e. Partial complex
e. IOP
Ans. The given key is A. Infantile spasm which is a
Ans. The key is A. ESR. [Elderly (age 62), rt sided
wrong key! Correct key is E. Partial complex. [In
headache and loss of vision are suggestive of
partial simple seizer there is no LOC (loss of
temporal arteritis where elevated ESR is highly
consciousness). Infantile spasm though may be
suggestive of this diagnosis].
associated with occasional LOC but its nature is
more generalized rather than the focal nature
described here. So the likely option is E. Partial
complex seizure]. 488. A 24yo man asks his GP for a sick
note from work. He says that feels down,
is lethargic and has stopped enjoying
486. A 24yo primigravida who is 30wk playing the piccolo (his main hobby). He
pregnant presents to the labor ward with was admitted to the psychiatry ward last
a hx of constant abdominal pain for the year following an episode of
last few hours. She also gives a hx of overspending, promiscuity and
having lost a cupful of fresh blood per distractibility. What is the most probable
vagina before the pain started. Abdominal dx?
exam: irritable uterus, CTG=reactive. a. Psychosis
Choose the single most likely dx? b. Cyclothymia
a. Abruption of placenta 2nd to pre- c. Bipolar affective disorder
eclampsia d. Seasonal affective disorder
b. Antepartum hemorrhage
Ans. The key is C. Bipolar affective disorder.
c. Placenta previa
[presently patient has depression and previous
d. Vasa previa
features of mania makes the diagnosis of bipolar
e. Revealed hemorrhage affective disorder likely].
Ans. The key is B. Antepartum hemorrhage.
[Though presentation indicates abruption
489. A 42yo female who is obese comes Ans. The key is E. IVIg. [If the pregnant woman is
with severe upper abdominal pain with a not immune to VZV and she has had a significant
temp=37.8C. She has 5 children. What is exposure, she should be offered varicella-zoster
immunoglobulin (VZIG) as soon as possible. VZIG
the most probable dx?
a. Ectopic pregnancy is effective when given up to 10 days after contact
(in the case of continuous exposures, this is
b. Ovarian torsion
defined as 10 days from the appearance of the
c. Hepatitis
rash in the index case). RCOG guideline].
d. Biliary colic
e. Cholecystitis

Ans. The key is cholecystitis. [Fat, female, fare, 492. A 40yo woman presents to the GP
forty and fertile are the pneumonic for with low mood. Of note, she has an
cholecystitis! Here the presentation of severe increased appetite and has gone up 2
upper abdominal pain with fever along with the dress sizes. She also complains that she
pneumonic features points towards the diagnosis can’t get out of bed until the afternoon.
of cholecystitis]. What is the most likely dx?
a. Pseudo depression
b. Moderate depression
490. A child has just recovered from c. Severe depression
meningitis. What inv will you do before d. Dysthymia
discharge? e. Atypical depression
a. CT scan
b. EEG Ans. The key is E. Atypical depression. [Atypical
c. Blood culture depression is a subtype of major
d. Repeat LP depression or dysthymic disorder that involves
e. Hearing test several specific symptoms, including increased
appetite or weight gain, sleepiness or
Ans. The key is E. Hearing test. [deafness is a excessive sleep, marked fatigue or weakness,
common complication of meningitis, so hearing moods that are strongly reactive to
test is suggested before discharge]. environmental circumstances, and feeling
extremely sensitive to rejection].

491. A primiparous woman with no prv


infection with herpes zoster is 18wk 493. An 8yo boy is clinically obese. As a
pregnant. She had recent contact with a baby he was floppy and difficult to feed.
young 21yo pt having widespread chicken He now has learning difficulties and is
pox. constantly eating despite measures by his
What is the most suitable management parents to hide food out of reach. What is
for the pregnant lady? the most probable dx?
a. Acyclovir PO a. Cushing’s syndrome
b. Acyclovir IV +IVIG b. Congenital hypothyroidism
c. Acyclovir IV c. Prader Willi syndrome
d. Reassure d. Lawrence moon biedel syndrome
e. IVIG e. Down’s syndrome
Ans. The key is C. Prader Willi syndrome. [Prader 496. A 45yo woman has been dx with GCA
Willi syndrome rare congenital disorder and is being treated with steroids.
characterized by learning difficulties, growth What is the other drug that can be added
abnormalities, and obsessive eating, caused
to this?
especially by the absence of certain genes a. ACEi
normally present on the copy of chromosome 15
b. Beta blockers
inherited from the father].
c. Aspirin
d. Interferons
e. IVIG
494. A 20yo lady is suffering from fever
and loss of appetite. She has been dx with Ans. The key is C. Aspirin [Low dose aspirin is
toxoplasmosis. What is the tx? increasingly being recommended for people with
a. Pyrimethamine a history of giant cell arteritis. It has been found
b. Pyrimethamine + sulfadiazine to be effective in preventing complications of
c. Clindamycin giant cell arteritis, such as heart attacks or stroke
d. Spiramycin (nhs.uk)].
e. Trimethoprim + sulfamethoxazole
497. A 17yo man has acute pain and
Ans. The key is B. Pyrimethamine + sulfadiazine. earache on the right side of his face.
[If the eye is involved, or if immunocompromized, Temp=38.4C and has extensive pre-
tx option is pyrimethamine + sulfadiazine OHCM auricular swelling on the right, tender on
9th edition, page 404]. palpation bilaterally. What is the single
most likely dx?
a. Acute mastoiditis
495. A 68yo woman has a sudden onset of b. Acute otitis externa
pain and loss of hearing in her left ear and c. Acute OM
unsteadiness when walking. There are d. Mumps
small lesions visible on her palate and left e. OM with effusion
external auditory meatus. What is the
Ans. The key is D. Mumps. [C/F: prodromal
single most likely dx? malaise, increased temperature, painful parotid
a. Acute mastoiditis
swelling, becoming bilateral in 70%. OHCS 9th
b. Cholesteatoma
edition, page 142].
c. Herpes zoster infection
d. Oropharyngeal malignancy
e. OM with infusion
498. An ECG of an elderly lady who
Ans. The key is C. Herpes zoster infection. [Herpes collapsed in the ED shows rapid
zoster oticus (Ramsay Hunt syndrome) occurs ventricular rate of 220bpm, QRS=140ms.
when latent varicalla zoster virus reactivates in What is the most probable dx?
the geniculate ganglion of the 7th cranial nerve. a. Atrial fibrillation
Symptoms: Painful vesicular rash on the auditory b. VT
canal ± on drum, pinna, tongue, palate or iris with c. SVT
ipsilateral facial palsy, loss of taste, vertigo, d. Mobitz type1 2nd degree heart block
tinnitus, deafness, dry mouth and eyes. OHCM 9 th e. Sinus tachycardia
edition, page 505].
Ans. The key is B. Ventricular tachycardia. [Dx: i)
history (if IHD/MI likelihood of a ventricular
arrhythmia is > 95%), ii) 12 lead ECG, and iii) lack
of response to IV adenosine). ECG findings in
favour of VT: 501. A 20yo boy is brought by his parents
suspecting that he has taken some drug.
 Positive QRS concordance in chest leads He is agitated, irritated and can’t sleep.
 Marked left axis deviation
Exam: perforated nasal septum.
 AV dissociation (occurs in 25%) or 2:1 or
3:1 AV block Which of the following is the most likely to
 Fusion beats or capture beats be responsible for his symptoms?
OHCM 9th edition, page 816]. [In the given case a. Heroine
collapse, ventricular rate of 220 and broad QRS of b. Cocaine
140ms points towards VT]. c. Ecstasy/MDMA/amphetamine
d. Alcohol
e. Opioids
499. A pt presents with purple papular
lesions on his face and upper trunk
measuring 1-2 cm across. They aren’t Ans. The key is B. Cocaine. [drug abuse with
painful or itchy. What is the single most perforated nasal septum indicates cocaine
likely dx? abuse].
a. Kaposi’s sarcoma
b. Hairy leukoplakia
c. Cryptosporidium 502. For a pt presenting with Parkinson’s
d. CMV infection disease which of the following drugs is
e. Cryptococcal infection most useful in the management of the
tremor?
Ans. The key is A. Kaposis sarcoma. [It is a spindle- a. Apomorphine
cell tumour derived from capillary endothelial b. Cabergoline
cells or from fibrous tissue, caused by human
c. Selegiline
herpes virus. It presents as purple papules (½ to 1
d. Amantadine
cm) or plaques on skin and mucosa (any organ). It
e. Benzhexol
metastasizes to nodes. OHCM 9th edition, page
716].

Ans. The key is E. Benzhexol. [Benzhexol (an


antiparkinsonian agent of anticholinergic class) is
500. A 6yo boy is clinically obese, his BMI
the drug of choice in parkinson’s disease induced
>95th centile. He has no other medical
tremor)].
prbs, examination is unremarkable. His
mother says that she has tried everything
to help him lose weight. What is the most
503. A 26yo woman has become aware of
probable dx? increasing right sided hearing deficiency
a. Cushing’s syndrome
since her recent pregnancy. Her eardrums
b. Congenital hypothyroidism
are normal. Her hearing tests show: BC-
c. Down’s syndrome
d. Lawrence moon biedel syndrome
normal. Weber test lateralizes to the right
e. Primary obesity
ear. What is the single most likely dx?
a. Encephalopathy
Ans. The key is E. Primary obesity. b. Functional hearing loss
c. Tympano-sclerosis
d. Otosclerosis conscious and answering questions now.
e. Sensorineural deafness His ECG shows irregular rhythm.
Your choice of inv:
a. CT
Ans. The key is D. Otosclerosis. [There are no b. MRI
features of encephalopathy. As Weber test is c. 24h ECG
lateralized it is unlikely to be functional hearing d. Echo
loss. In tympanosclerosis ear drum becomes
chalky white. So as the ear drum is normal it is Ans. The key is D. Echo. [Echo may show clot in
not tympanosclerosis. Weber test is lateralized to atrial appendage responsible for this attack of TIA
right and deafness is also on the right. So it not secondary to atrial fibrillation].
sensorineural deafness but conductive deafness
which makes otosclerosis as the most likely
diagnosis]. 506. A 10yo boy is clinically obese and the
shortest in his class. He had a renal
transplant last year and his mother is
worried that he is being bullied.
What is the most probable dx?
504. A 58yo T1DM on anti-HTN therapy
a. Cushing’s syndrome
for 13yrs developed central chest pain for b. Congenital hypothyroidism
45 mins while driving a/w cold sweating c. Pseudocushing’s syndrome
and dyspnea. d. Lawrence moon biedel syndrome
What is the single most appropriate dx? e. Down’s syndrome
a. MI
b. Pericarditis Ans. The key is A. Cushing’s syndrome. [Renal
c. Pulmonary embolism transplant--> immune suppression is needed->
d. Costochondritis exogenous steroid--> cushing syndrome. short
e. Pneumothorax stature--> if steroids are used in early age then
they cause premature fusion of growth
Ans. The key is A. MI. [In pericarditis pain is plate/calcification].
aggravated by inspiration or lying flat and
relieved by leaning forward. Pericardial rub may
present and there may be fever. In
pneumothorax pain is not central but pleuritic.
Pulmonary embolism=dyspnoea and pleuritic 507. A 45yo man had cancer of head of
chest pain. In costrochondritis localized pancreas which has been removed. He has
pain/tenderness at the costochondral junction a hx of longstanding heartburn. He now
enhanced by motion, coughing, or sneezing. The comes with rigid abdomen which is
given picture of central chest pain for 45 minutes
tender, temp 37.5C,BP=90/70mmHg,
(more than 30 minutes), sweating and dyspnoea
with major risk factor of DM and Htn suggest the
pulse=120bpm. What is the next step of
diagnosis of MI]. the inv?
a. CT abdomen
b. XR abdomen
c. MRI abdomen
505. A man was brought to the ED from a
d. US abdomen
shopping mall after collapsing there. He is
e. Endoscopy
The answer is B. X-ray abdomen. [X-ray abdomen 510. A 45yo manual worker presented
will help diagnosing perforation by showing gas with a 2h hx of chest pain radiating to his
under diaphragm. This is a case of perforated left arm. His ECG is normal. What is the
peptic ulcer with the features of shock,
single most appropriate inv?
abdominal rigidity and raised temperature. Stress a. Cardiac enzymes
from serious disease and operation causes the
b. CXR
body to produce higher amounts of acid, which
c. CT
can irritate preexisting ulcers leading to easy
d. ECG
perforation].
e. V/Q scan
508. A 50yo man presents to the ED with
Ans. A. Cardiac enzymes.
acute back pain radiating down to his
legs. Pain which is usually relieved by lying 511. A 26yo woman had bipolar disorder
down and exacerbated by long walks and for 10yrs and is on Lithium for it. She is
prolong sitting. What inv would be the symptom free for the past 4 years. She is
best option? now planning her pregnancy and wants to
a. MRI know whether she should continue taking
b. CT spine lithium.
c. XR spine
What is the single most appropriate
d. Dual energy XR abruptiometry
advice?
e. Serum paraprotein electrophoresis a. Continue lithium at the same dose and
stop when pregnancy is confirmed
Ans. The key is A. MRI. [Back pain radiating to leg,
pain releaved by lying down and exacerbated by b. Continue lithium during pregnancy and
long walk and prolonged sitting are characteristic stop when breast feeding
of lumber (intervertebral) disc disease]. c. Reduce lithium dosage but continue
throughout pregnancy
d. Reduce lithium gradually and stop when
509. What is the most appropriate pregnancy is confirmed
antibiotic to treat uncomplicated e. Switch to sodium valproate
chlamydial infection in a 21yo female who
isn’t pregnant?
a. Erythromycin
Ans. The key is D. Reduce lithium gradually and
b. Ciprofloxacin
stop when pregnancy is confirmed.
c. Metronidazole
d. Cefixime 512. A pt presents with dysphagia and
e. Doxycycline pain on swallowing. He has sore mouth
and soreness in the corners of the mouth.
Ans. The key is E. Doxycicline. [Doxycycline 100
What is the single most likely dx?
mg twice-daily for seven days or a single dose of 1
a. Kaposi’s sarcoma
g of azithromycin or Erythromyin 500 mg twice
b. Molluscum contagiosum
daily for 14 days or four times daily for seven days
c. CMV infection
or Ofloxacin 200 mg twice-daily or 400 mg once-
daily for 7 days. In pregnant Azithromycine 1g d. Candida infection
single dose is recommended then erythromycin e. Toxoplasma abscess
500 mg twice daily for fourteen days or four
Ans. The key is D. Candida infection. [Candida is
times daily for seven days. Then amoxicillin 500
more common than CMV].
mg three times daily for 7 days.].
513. A 30yo lady has epistaxis for 30mins. b. Paracetamol
Her Hgb is normal, MCV normal, WBC c. Ibuprofen
normal, d. Metronidazole
PT/APTT/Bleeding time are normal.
Ans. The key is A. Sulfasalazine. [Pain in RIF,
Where is the defect? diarrhea, granular transmural ulcers near the ileo-
a. Plts
cecal junction points towards the diagnosis of
b. Coagulation factor Crohn’s diseas (predominantly ileo-cecal type)].
c. Sepsis
d. Anatomical 517. A 60yo woman presents with acute
e. RBC onset of bone and back pain following a
rough journey in a car. Exam: tenderness
at mid-thoracic vertebra with spasm, she
Ans. The key is D. Anatomical. [bleeding time, feels better once she bends forward. What
coagulation profile, Hb%, cell count and is the single most probable dx?
parameters are normal. So the cause of bleeding a. Osteoporotic fx verterbra
here is anatomical defect]. b. Myofacial pain
c. Whiplash injury
514. Midpoint between the suprasternal
d. MI
notch and pubic symphysis. e. Pancreatitis
What is the single most appropriate
landmark? Ans. The key is B. Myofacial pain. [Myofascial pain
a. Fundus of the gallbladder syndrome is a chronic pain disorder. In myofascial
b. Mcburney’s point pain syndrome, pressure on sensitive points in
c. Stellate ganglion your muscles (trigger points) causes pain in
d. Deep inguinal ring seemingly unrelated parts of your body. This is
e. Transpyloric plane called referred pain. Myofascial pain
syndrome typically occurs after a muscle has
been contracted repetitively].

Ans. The key is E. Transpyloric plane.

515. Tip of the 9th costal cartilage. What 518. A 70yo woman presents with
is the single most appropriate landmark? recurrent episodes of parotid swelling. She
complains of difficulty in talking and
a. Fundus of the gallbladder
speaking and her eyes feel gritty on
b. Deep inguinal ring
waking in the morning. What is the single
c. Termination of the spinal cord
most likely dx?
d. Transpyloric plane
a. C1 esterase deficiency
e. Vena cava opening in the diaphragm
b. Crohns disease
Ans. The key is A. Fundus of the gallbladder. c. Mumps
d. Sarcoidosis
516. A child complains of RIF pain and e. Sjogrens syndrome
diarrhea. On colonoscopy, granular
transmural ulcers are seen near the ileo- Ans. The key is E. Sjogrens syndrome. [parotid
cecal junction. What should be the swelling, difficulty talking and speaking (due to
management? dryness or less salive), eyes feeling gritty on
a. Sulfasalazine
waking in the morning due to dryness of eye are 521 A 22yo man has rushed into the ED
suggestive of Sjogrens syndrome]. asking for help. He describes recurrent
519. A 39yo woman has not had her episodes of fearfulness, palpitations,
period for 10months. She feels well but is faintness, hyperventilation, dryness of the
anxious as her mother had an early mouth with peri-oral tingling and
menopause. Choose the single most cramping of the hands. His symptoms last
appropriate initial inv? 5-10 mins and have worsened since their
a. Serum estradiol conc. onset 3months ago. He is worried he may
b. Serum FSH/LH be having a heart attack. An ECG shows
c. Serum progesterone conc. sinus tachycardia.
d. None What is the single most appropriate
e. Transvaginal US immediate intervention?
a. High flow oxygen
Ans. The key is B. Serum FSH/LH [here serum b. IV sedation
oestrogen is also important as i) low oestrogen + c. Rebreathe into a paper bag
low FSH + low LH suggest hypothalamic d. Refer for anxiety management course
amenorrhoea and i) low oestrogen + high FSH + e. Refer for urgent cardiology opinion
high LH suggest premature ovarian failure! So the
main determinant is serum FSH/LH. Likely cause
here is premature ovarian failure].
Ans. The key is C. Rebreathing into paper bag.
520. A 50yo man with DM suddenly [Patient has anxiety disorder (panic) which causes
develops persistent crushing central chest hyperventilation and CO2 washout leading to
pain radiating to the neck. What is the respiratory alkalosis. Symptoms will improve by
single most appropriate dx? rebreathing into paper bag as it will cause gradual
a. Angina increase of CO2 in paper bag and decrease the
b. Costochondritis (tietz’s disease) severity of respiratory alkalosis].
c. Dissecting aneurysm 522. An 8yo boy has longstanding asthma.
d. MI He has admitted with a severe episode
e. Pulmonary embolism and is tired and drowsy. He has not
Ans. The key is C. Dissecting aortic aneurism. improved on oxygen, inhaled B2 agonist
Probably wrong key. Correct key should be D. MI. and IV hydrocortisone. CXR shows
[The features described is insufficient and can be bilateral hyperinflation. He is too
seen in both aortic dissection and MI. However breathless to use a peakflow meter and is
dissection pain is described as tearing and O2 sat <90%. What is the single most
crushing pain is often used for mi pain. Both appropriate inv?
dissection and mi can have pain radiation to neck. a. CBG
History of diabetes goes with mi as it is a b. CXR
recognized risk factor for mi. Some may argue in c. CT chest
DM mi will be painless! But it is not always the d. Pulse oximetry
case. MI is only painless when autonomic e. Spirometry
neuropathy becomes well established].
Ans. The key is A. CBG. [It will point towards
acidosis and indicate whether assisted ventilation
is needed or not].
523. A man was operated for colorectal Ans. The key is B. Evaluate patients capacity to
ca. His pain is relieved with morphine refuse treatment.
60mg bd PO but now he can’t swallow 527. A 23yo woman with painless vaginal
medications. What will be the next bleeding at 36wks pregnancy otherwise
regimen of analgesic administration? seems to be normal. What should be done
a. Oxycodone
next step?
b. Fentanyl patch
a. Vaginal US
c. Morphine 60mg IV/d
b. Abdominal US
d. Morphine 240mg IV/d
c. Vaginal exam
Ans. The key is B. Fentanyl patch. [Here S/C d. Reassurance
morphine 1/2 the dose of oral can be given (not
Ans. The key is B. Abdominal US. This is a wrong
present in option) or I/V morphine 1/3rd the oral
key. The correct key is A. Vaginal US.[Painless
dose can be given. Here I/V doses are not
vaginal bleeding at 36 weeks indicates the
appropriate so we should go for B. Fentanyl patch
diagnosis of placenta previa, which can be better
as required morphine dose is known].
evaluated by vaginal US].
524. Just above the mid-inguinal point.
What is the single most appropriate
landmark?
a. Femoral artery pulse felt
b. Mcburney’s point
c. Stellate ganglion
d. Deep inguinal ring
e. Transpyloric plane

Ans. The key is D. Deep inguinal ring. 528. A 29yo lady admitted with hx of
repeated UTI now developed hematuria
525. 5th ICS in the ant axillary line. What
with loin pain. What is the most probable
is the single most appropriate landmark?
a. Apex beat dx?
a. Acute pyelonephritis
b. Chest drain insertion
b. Chronic pyelonephritis
c. Stellate ganglion
c. UTI
d. Transpyloric plane
d. Bladder stone
e. Vena cava opening into the diaphragm
Ans. The key is A. Acute pyelonephritis. [In a
Ans. B. Chest drain insertion.
patient having hematuria and loin pain with
526. A 34yo man with MS has taken an history of repeated UTI suggest acute
OD of 100 tablets of paracetamol with pyelonephritis].
intent to end his life. He has been brought 529. A 45yo chronic smoker attends the
to the ED for tx but is refusing all OPD with complaints of persistent cough
intervention. and copious amount of purulent sputum.
a. Assessment
He had hx of measles in the past. Exam:
b. Evaluate pt’s capacity to refuse tx
finger clubbing and inspiratory
c. Establish if pt has a prv mental illness
crepitations on auscultation. What is the
single most likely dx?
a. Interstitial lung disease feeling tired and lethargic, sorethroat,
b. Bronchiectasis hepatosplenomegally all are well known features
c. Asthma of ALL].
d. COPD
532. A 65yo man presents with back pain.
e. Sarcoidosis
Exam: splenomegaly and anemia. Blood:
Ans. The key is B. Bronchiectasis. [Persistent WBC=22, Hgb=10.9, Plt=100, ESR=25. He
cough with copious purulent sputum and finger has been found to have Philadelphia
clubbing points towards the diagnosis of chromosome. What is the single most
bronchiectasis. Severe lung infections such as likely dx?
tuberculosis (TB), whooping cough, pneumonia or a. ALL
measles can damage the airways at the time of b. AML
infection. Bronchiectasis may then develop c. CML
(WHO)]. d. CLL
e. Lymphoma

530. A 68yo man has had malaise for 5 Ans. The key is C. CML. [anaemia, raised WBC
days and fever for 2 days. He has cough count, low platelet (platelet may be variable) are
and there is dullness to percussion at the known features of CML, splenomegaly
(particularly if massive) is very suggestive of CML
left lung base. What is the single most
and Philadelphia chromosome is characteristic of
appropriate inv?
CML].
a. Bronchoscopy
b. CXR 533. A 24yo woman has 8wk amenorrhea,
c. CT right sided pelvic pain and vaginal
d. MRI bleeding. She is apyrexial. Peritonism is
e. V/Q scan elicited in the RIF. Vaginal exam reveals
right sided cervical excitation. What is the
most probable dx?
Ans. The key is B. CXR. [Given presentation is a. Ectopic pregnancy
suggestive of pneumonia for which investigation b. Salpingitis
of choice is CXR]. c. Endometriosis
d. Ovarian torsion
e. Ovarian tumor
531. A 5yo child was admitted with hx of
feeling tired and lethargic all the time, Ans. The key is A. [Salpingitis, Endometriosis,
bleeding gums and sore throat since the overian torsions do not associated with
amenorrhoea. In ovarian tumour three main
last 3months. Exam: hepatosplenomegaly.
features are i) increased abdominal size and
What is the most probable dx?
persistent bloating (not bloating that comes and
a. ALL
goes) ii) persistent pelvic and abdominal pain iii)
b. AML
difficulty eating and feeling full quickly, or feeling
c. CML
nauseous. Patient with pelvic pain and vaginal
d. CLL bleeding, peritonism and cervical exitation
e. Lymphoma obviously points towards Ectopic pregnancy].
Ans. The key is A. ALL. [Commonest leukemia in
children is ALL. Bleeding gums (low platelet),
534. A 64 yo woman has been treated for a. CXR
breast cancer with tamoxifen. What other b. None
drug should be added to her tx regime? c. Peak flow rate diary
a. Bisphosphonates d. Pulse oximetry
b. Calcium
Ans. The key is C. Peak flow rate diary. [Peak flow
c. Vit D
rate diary shows diurnal variation. This diary
d. Calcitonin
shows when the bronchoconstriction remains
e. Phosphate binders
worse and guides to use bronchodilators prior to
that times].

Ans. The key is A. Bisphosphonates. 537. A woman presented with blurred


[ bisohosphonates reduce the risk of bone vision and intermittent clumsiness for 3m.
metastasis in cancers and is normally taken as Reflexes are brisk in her arm and optic disc
adjuvant therapy in many types of tumours is pale. What is the single most
including breast cancer. Plus it prevents bone appropriate test to confirm dx?
resorption]. a. CSF analysis
b. CT
535. A 26yo woman with regular menses
c. MRI
and her 28yo partner comes to the GP
d. EEG
surgery complaining of primary infertility
e. EMG
for 2yrs. What would be the single best
investigation to see whether she is Ans. The key is C. MRI. [Features are suggestive of
ovulating or not? multiple sclerosis. Investigation of choice is
a. Basal body temp estimation gadolinium enhanced MRI].
b. Cervical smear
c. Day2 LH and FSH
538. A 63yo man presents after having a
d. Day21 progesterone
seizure. Exam: alert, orientated,
e. Endometrial biopsy inattention on the left side and
hyperreflexia of the arm. What is the most
probable dx?
a. Cerebral tumor
Ans. The key is D. Day 21 progesterone. [Mid-
b. Pituitary adenoma
luteal progesterone level to assess ovulation: If
c. Cerebellar abscess
low, it may need repeating, as ovulation does not
occur every month. The blood test is taken seven d. Huntingtons chorea
days before the anticipated period, that is on day e. Parkinsonism
21 of a 28-day cycle. However, this day will need
Ans. The key is A. Cerebral tumour.
to be adjusted for different lengths of cycle. Ref:
patient.co.uk]. 539. A 40yo man with a 25y hx of smoking
536. A 10yo boy who takes regular high presents with progressive hoarseness of
dose inhaled steroids for his longstanding voice, difficulty swallowing and episodes
asthma has been advised to use of hemoptysis. He mentioned that he used
bronchodilators to control his acute to be a regular cannabis user. What is the
attacks. His parents are unsure when single most likely dx?
a. Nasopharyngeal cancer
should he use his bronchodilator.
b. Pharyngeal carcinoma
What is the single most appropriate inv?
c. Sinus squamous cell carcinoma sensation are lost due to spinothalamic tract
d. Squamous cell laryngeal cancer damage), wasted and weak hands with
e. Hypopharyngeal tumor diminished reflexes, weak spastic legs with
dissociated sensory loss are features suggestive
Ans. The key is D. Squamous cell laryngeal cancer. of Syringomyelia].
Common features are given below:
 Chronic hoarseness is the most common Sensory features: 1) loss of pain and temperature
early symptom. sensation 2) sensory loss is experienced over the
 Other symptoms of laryngeal cancer arms, shoulders and upper body 3) light touch,
include pain, dysphagia, a lump in the vibration and position senses in the feet are
neck, sore throat, earache or a persistent affected as the syrinx enlarges into the dorsal
cough. column.
 Patients may also describe Motor features: (when lower motor neurons of
breathlessness, aspiration, haemoptysis, the anterior horn cells are affected) 1) muscle
fatigue and weakness, or weight loss. wasting and weakness begins in the hands and
(Patient.co.uk) then affects the forearms and shoulders. 2)
tendon reflexes are lost. Autonomic involvement
540. A 30yo lady complains of intermittent like bladder and bowel can occur. [patient.co.uk]
diarrhea, chronic abdominal and pelvic
pain and tenesmus. Sometimes she 542. A 23yo woman is being followed up
notices blood in her stool. Select the most 6wks after a surgical procedure to
likely cause leading to her symptoms? evacuate the uterus following a
a. Inflammatory bowel disease miscarriage. The histology has shown
b. Diverticulosis changes consistent with a hydatidiform
c. Irritable bowel disease mole. What is the single most appropriate
d. Adenomyosis inv in this case?
e. UTI a. Abdominal US
b. Maternal karyotype
Ans. The key is A. Inflammatory bowel disease.
c. Paternal blood group
[Tenesmus excludes diverticulitis, occasional
d. Serum B-HCG
blood in stool excludes irritable bowel disease.
Features are not consistent with adenomyosis or e. Transvaginal US
UTI but suggestive of inflammatory bowel
Ans. The key is D. Serum β-HCG. [When you are
disease].
first diagnosed with a hydatidiform mole, your
541. A 50yo lady with weak limbs when hCG level will be raised. When the hydatidiform
examined was found to have burn marks mole is treated (removed), the hCG level will
usually return to a normal, non-pregnant amount
on finger tips, wasted and weak hands
and should remain so. If you develop GTN, the
with diminished felexes. She also has hCG level can remain elevated or continue to rise
weak spastic legs and dissociated sensory further. So, this blood test is a good way to check
loss. What is the dx? for the possible development of GTN (Gestational
a. MS trophoblastic neoplasia).
b. Syringomyelia
c. MND 543. A 67yo man with hx of weight loss
d. Guillian-barre complains of hoarseness of voice. CT
e. Freidriech’s ataxia reveals opacity in the right upper
mediastinum. He denied any hx of
Ans. The key is B. Syringomyelia. [weak limbs, difficulty breathing.
burn mark on fingertip (as pain and temperature
What is the single most appropriate inv? Ans. The key is E. Normal pressure
a. Laryngoscopy hydrocephalus. [hx of falls and broad based
b. Bronchoscopy clumsy gait (balance and gait disturbance),
c. LN biopsy strange behavior and disorientation to time and
d. Bronchoalevolar lavage place (due to dementia), episodes of enuresis
e. Barium swallow (urinary incontinence) points towards normal
pressure hydrocephalus. Classic triad of normal
Ans. The key is C. Lymph node biopsy. [There is pressure hydrocephalus: i) gait abnormality ii)
weight loss and there is an opacity in right upper urinary incontinence and iii) dementia].
mediastinum. May indicate enlarged lymph node
or lymphoma causing pressure on right recurrent
546. A 75yo nursing home resident
laryngeal nerve resulting in horseness. As CT complains of headache, confusion and
didn't reveal any bronchial lesion and no impaired vision for 4days. She has
breathing difficulty it is unlikely to be a bronchial multiple bruises on her head.
pathology. So CT guided lymph node biopsy can What is the most likely cause of confusion
reveal the diagnosis]. in this pt?
a. Alcohol intoxication
544. A 52yo man whose voice became b. Infection
hoarse following thyroid surgery 1 wk ago c. Subdural hematoma
shows no improvement. Which d. Hypoglycemia
anatomical site is most likely affected? e. Hyponatremia
a. Bilateral recurrent laryngeal nerve
b. Unilateral recurrent laryngeal nerve Ans. The key is C. Subdural hematoma. [elderly
c. Unilateral external laryngeal nerve patient with multiple bruises on her head suggest
d. Bilateral external laryngeal nerve head injury which can lead to subdural
e. Vocal cords hematoma. Headache, confusion and impaired
vision for 4 days indicate subacute subdural
hematoma (three pahges i) acute ii) subacute 3 –
7 days and iii) chronic 2 – 3 weeks].
Ans. The key is B. Unilateral recurrent laryngeal
nerve.

545. A 73yo male presents with a 12m hx


of falls. His relatives have also noticed
rather strange behavior of late and more
recently he has had episodes of enuresis. Symptoms of subdural hematoma:
Exam: disorientation to time and place,  a headache that keeps getting worse
broad-based, clumsy gait.  feeling and being sick
What is the most probable dx?  confusion
a. Dementia  personality changes, such as being
b. Pituitary adenoma unusually aggressive or having rapid
c. CVD mood swings
 feeling drowsy and finding it difficult to
d. Syringomyelia
keep your eyes open
e. Normal pressure hydrocephalus
 speech problems, such as slurred speech
 problems with your vision, such as double
vision
 paralysis (loss of movement) on one side 549. A 21yo woman has had several
of the body sudden onset episodes of palpitations,
 problems walking and frequent falls
sweating, nausea and overwhelming fear.
 seizures (fits)
 loss of consciousness
On one occasion she was woken from
[Reference: nhs.uk] sleep and feared she was going insane.
There is no prv psychiatric disorder. What
547. A 50yo woman returned by air to the is the most probable dx?
UK from Australia. 3days later she a. Pheochromocytoma
presented with sharp chest pain and b. Panic disorder
breathlessness. Her CXR and ECG are c. GAD
normal. What is the single most d. Phobia
appropriate inv? e. Acute stress disorder
a. Bronchoscopy
b. Cardiac enzymes Ans. The key is B. Panic disorder. [here closest d/d
c. CT to panic disorder is pheochromocytoma. But in
d. MRI pheochromocytoma the most important feature
is resistant hypertension and other important
e. Pulse oximetry
features are headache and abdominal pain which
f. V/Q scan
all are absent here. Moreover overwhelming fear
g. CTPA
is more suggestive of panic disorder. Hence here
the diagnosis is B. Panic disorder].

Ans. The key is G. CTPA. [Prolonged plane journey 550. A 55yo woman with a persistent
is a recognized risk factor for thromboembolism cough and hx of smoking develops left
and hence pulmonary embolism also! Sharp chest sided chest pain exacerbated by deep
pain and breathlessness after 3 days of plane breathing with fever and localized
journey is highly suggestive of pulmonary crackles.
embolism the investigation of choice for which is What is the single most appropriate dx?
CTPA]. a. Dissecting aneurysm
b. Pericarditis
548. A tall thin young man has sudden
c. Pneumonia
pain in the chest and becomes breathless
d. Pneumothorax
while crying. What is the single most
e. Pulmonary embolism
appropriate inv?
a. Cardiac enzymes Ans. The key is C. Pneumonia. [chest pain
b. CXR exacerbated with deep breathing, fever and
c. CT localized crackles are highly suggestive of
d. ECG pneumonia].
e. V/Q scan
551. A 40yo woman complains of
dysphagia for both solids and liquids. She
Ans. The key is B. CXR. [tall thin young men are sometimes suffers from severe
particularly prone to develop pneumothorax. retrosternal chest pain. Barium swallow
Sudden pain and breathlessness in this young reveals a dilated esophagus which tapers
man are highly suggestive of pneumothorax. So
to a fine distal end. What is the best
investigation of choice is CXR].
management strategy?
a. Reassurance
b. Antispasmodics d. Thalamus
c. Dilatation of the LES e. Brain stem
d. Endoscopic diverticulectomy
Ans. The key is C. internal capsule. [Hippocampal
e. Barium swallow
lesion causes mainly memory impairment.
Ans. The key is C. Dilatation of LES. [Dysphagia for Cerebellum has its diagnostic features that is not
both solids and liquids suggest neuromuscular present here. Thalamic lesion can lead to
dysphagia while dysphagia only for solid suggests impairment of arousal, orientation, learning and
mechanical obstruction. Here features are memory, facial paresis, language deficit,
consistent with achalasia for which lower hemispatial neglect, hemisensory loss,
oesophageal sphincter dilation (balloon hemiparesis, hemiataxia and visual field defect.
dilatation) is a treatment modality]. Brainstem stroke causes impaired consciousness,
disorder of blood pressure, and breathing
552. A 38yo female G4 at 32wks of abnormality. Given picture is typical of lesion in
pregnancy presented with thick white internal capsule].
marks on the inside of her mouth for
3wks. Her mouth including her tongue 554. A 72yo man brought to the ED with
appeared inflamed on examination. She onset of paraplegia following a trivial fall.
smokes 20 cigarettes/day despite advice He was treated for prostatic malignancy
to quit. She attends her ANC regularly. in the past. What is the single most
What is the most probable dx? probable dx?
a. Paget’s disease
a. Lichen planus
b. Osteoporotic fx of vertebre
b. Aphthous ulcer
c. Secondary
c. Smoking
d. Multiple myeloma
d. Candidiasis
e. Spondylosis
e. Leukoplakia

Ans. The key is D. Candidiasis. [lichen planus may


have lace like appearance and not thick white Ans. The key is C. Secondary. [In male
mark. Aphthous ulcer has yellowish floor and osteoporotic fracture is less common. As patient
surrounded by erythematous halo. Smoking may had prostatic malignancy pathological fracture
cause tongue coating but not like thick white from secondary metastasis to bone (here
mark on the inside of mouth. Leukoplakia is with vertebra leading to paraplegia) is more common].
raised edges/Bright white patches and sharply
defined and cannot be rubbed out like candida
patch; here also inflamed tongue points towards 555. A 14yo girl has developed an itchy,
infection. So candidiasis is the most probable
scaly patch on her scalp. She had a similar
option].
patch that cleared spontaneously 2yrs
553. A 69yo woman has had a stroke. Her ago. Her aunt has a similar undiagnosed
left upper and lower limbs are paralyzed rash on the extensor aspects of her elbows
and she is having difficulty in speaking. and knees. What is the single most likely
Which anatomical site is most likely dx?
affected? a. Eczema
a. Hippocampus b. Fungal infection
b. Cerebellum c. Impetigo
c. Internal capsule d. Lichen planus
e. Psoriasis 558. A 27yo woman had pre-eclampsia
and was delivered by C-section. She is now
complaining of RUQ pain different from
Ans. The key is E. Psoriasis. [itchy, scaly patch on wound pain. What inv will you do
scalp are classic presentation of scalp psoriasis. immediately?
Her aunts presentations (similar rash on extensor a. Coagulation profile
aspects of her elbow and knees) are suggestive of b. LFT
psoriasis. It is thought to be an immunological c. Liver US
disease]. d. MRCP
556. A pt after transurethral prostatic e. None
biopsy. What electrolyte imbalance can he Ans. The key is B. LFT. [In a post ceasarian patient
develop? having pre-eclampsia RUQ pain different from
a. Hyperkalemia wound pain points towards the diagnosis of
b. Hyponatremia HELLP syndrome. So immediate investigation to
c. Hypocalcemia be done is LFT]. [OHCS, 9th edition, page 26].
d. Hypernatremia
e. Hypercalcemia 559. A 10yo girl has been referred for
assessment of hearing as she is finding
Ans. The key is B. Hyponatremia. [Use of fluid for difficulty in hearing her teacher in the
bladder irrigation may lead to hyponatremia]. class. Her hearing tests show: BC normal,
557. A 28yo woman has been admitted at symmetrical AC threshold reduced
38wks gestation. Her BP=190/120mmHg bilaterally, weber test shows no
and proteinuria +++. Immediately lateralization.
following admission she has a grand-mal What is the single most likely dx?
a. Chronic perforation of tympanic
seizure. What is the single most
membrane
appropriate initial management?
a. Diazepam IV b. Chronic secretory OM with effusion
b. Fetal CTG c. Congenital sensorineural deficit
c. Hydralazine IV d. Otosclerosis
d. Immediate delivery e. Presbycusis
e. Magnesium sulphate IV

Ans. The key is E. Magnesium sulphate IV. Ans. The key is B. Chronic secretory OM with
[patient has established eclampsia. So she should effusion.
be treated with Magnesium sulphate as with 4g
magnesium sulfate in 100mL 0.9% saline IVI over
5min + maintenance IVI of 1g/h for 24h. Beware 560. A thin 18yo girl has bilateral parotid
depressed respiration. If recurrent seizure give 2g swelling with thickened calluses on the
IVI magnesium sulfate over 5 min. Check tendon dorsum of her hand. What is the single
reflexes and respiratory rate every 15min. Stop most likely dx?
magnesium sulfate IVI if respiratory rate <14/min a. Bulimia nervosa
or tendon reflex loss, or urine output <20mL/h]. b. C1 esterase deficiency
[OHCS, 9th edition, page 49]. c. Crohn’s disease
d. Mumps
e. Sarcoidosis
and his stools are pale. What is the most
Ans. The key is A. Bulimia nervosa. [18 year thin likely dx?
girl, bilateral parotid swelling and thickened a. Galactosemia
calluses on the knuckles from self induced b. Biliary atresia
vomiting are suggestive of Bulimia nervosa. c. G6PD deficiency
Bulimia often is associated with bilaterak parotid d. Rh incompatibility
swelling (parotid hypertrophy)]. e. Congenital viral infection

561. A 48yo presents with severe chest Ans. The key is B. Biliary atresia. [Deep jaundice at
pain since the last 40mins. In the ED he is 6th week with pale stools suggests obstructive
given oxygen, GTN, morphine. ECG=ST jaundice. So most likely diagnosis here is biliary
elevation. Bloods=increased troponin atresia].
levels. What is the next step of
management?
a. Beta blockers 564. A 45yo man with colon cancer now
b. Percutaneous angiography develops increased thirst, increased
c. Anticoagulant & heparin frequency in urination and weight loss. His
d. Clopidogrel fasting blood glucose=9mmol/L.
e. Aspirin what is the most appropriate
management?
a. Oral hypoglycemic
Ans. The key is B. Percutaneous angiography. b. Insulin long acting
c. Short acting insulin before meal
d. IV insulin
562. A 34yo female presents with a foul e. Subcutaneous insulin
smelling discharge. What set of organisms
are we looking for to be treated here?
a. Chlamydia, gonorrhea Ans. The key is A. Oral hypoglycemic. [Increased
b. Chlamydia, gardenella thirst and increased frequency in urination along
c. Chlamydia, gonorrhea, gardenella with weight loss is suggestive of DM supported by
fasting blood glucose of 9 mmol/L. At the age of
d. Gonorrhea, gardenella
45 most likely type of diabetes is NIDDM or type 2
e. Gardenella only
DM which is treated by oral hypoglycemic
agents].

Ans. The key is E. Gardenella only. [Here foul


smelling discharge is caused by gardenella. So
565. A 34yo man from Zimbabwe is
most acceptable answer is E. Gardenella only].
admitted with abdominal pain to the ED.
An AXR reveals bladder calcification. What
is the most likely cause?
a. Schistosoma mansoni
563. A 6wk formula fed baby boy is found b. Sarcoidosis
at the child health surveillance to be c. Leishmaniasis
deeply jaundiced. His weight gain is poor d. TB
e. Schistosoma hematobium Ans. The key is A. Topical permathrine. [The
intensely itchy rash, scratch marks and burrows
on wrist and elbow red papules on penis are
Ans. The key is E. Schistosoma hematobium. suggestive of scabies. Topical permethrine are
[Bladder involvement is caused by Schistosoma used to treat it].
hematobium while Schistosoma mansoni causes
intestinal disease].
568. A 4yo has earache and fever. Has
taken paracetamol several times. Now it’s
noticed that he increases the TV volume.
His preschool hearing test shows
symmetric loss of 40db. What is the most
566. A 6yo came with full thickness burn. likely dx?
He is crying continuously. What is the next a. OM with effusion
step of management? b. Otitis externa
a. Refer to burn unit c. Cholesteatoma
b. IV fluid stat d. CSOM
c. Antibiotic e. Tonsillitis
d. Analgesia
e. Dressing
Ans. The key is A. OM with effusion. This is a
wrong key. Correct key is B. Otitis externa. [The
Ans. The key is analgesia. This is a wrong key. childs preschool hearing loss and increasing the tv
Correct key should be B. IV fluid stat. [Here volume suggests that he has OM with effusion
already mentioned full thickness burn which is but present earache and fever points towards the
painless. Child often cry from anxiety for diagnosis of otitis externa].
hypoxaemia and hypovolaemia rather than pain.
The patient then responds better to oxygen or
increased fluid administration rather than to 569. A pt presents with gradual onset of
narcotic analgesics. Ref: patient.info]. headache, neck stiffness, photophobia
and fluctuating LOC. CSF shows
lymphocytosis but no organism on gram
567. A 78yo nursing home resident is stain. CT brain is normal. What is the
revived due to the development of an single most likely dx?
intensely itchy rash. Exam: white linear a. Hairy leukoplakia
lesions are seen on the wrists and elbows b. TB
and red papules are present on the c. CMV infection
penis. What is the most appropriate d. Candida infection
management? e. Cryptococcal infection
a. Topical permethrin
b. Referral to GUM clinic
c. Topical betnovate Ans. The key is B. TB. [Fungal meningitis can also
d. Topical ketoconazole present like this but it is much more rare.
e. Topical selenium sulphidehyosine Moreover negative gram stain excludes fungal
cause here. Hence TB meningitis is more acceptable. However Ampicillin + gentamycin is
acceptable answer]. the drug combination of choice].

570. An 18m boy has been brought to the 572. A pt presents with fever, dry cough
ED because he has been refusing to move and breathlessness. He is tachypneic but
his left arm and crying more than usual chest is clear. Oxygen saturation is normal
for the past 24h. He has recently been at rest but drops on exercise.
looked after by his mother’s new bf while What is the single most likely dx?
she attended college. Assessment shows a. CMV infection
multiple bruises and a fx of the left b. Candida infection
humerus which is put in plaster. What is c. Pneumocystis carinii infection
the single most appropriate next step? d. Cryptococcal infection
a. Admit under care of pediatrician e. Toxoplasma abscess
b. Discharge with painkillers
c. Follow up in fx clinic
d. Follow up in pediatric OPD Ans. The key is C. Pneumocystis carinii infection.
e. Follow up with GP [Fever, dry cough, breathlessness, tachypnoea
with clear chest is seen in pneumocystis carinii
pneumonia. Normal oxygen saturation which
Ans. The key is A. Admit under care of drops on exercise is characteristic of
pediatrician. [This is NAI. So the child cannot be pneumocystis carinii pneumonia].
handover to the risk again and should be
admitted to protect him from further injury done
by mothers boyfriend while serial x-rays and 573. A 14yo boy fell and hit his head in the
relevant investigations done and asked for child playground school. He didn’t lose
protection unit’s help]. consciousness. He has swelling and
tenderness of the right cheek with a
subconjuctival hemorrhage on his right
571. A 74yo female presents with eye.
headache and neck stiffness to the ED. What is the most appropriate initial inv?
Following a LP the pt was started on IV a. CT brain
ceftriaxone. CSF culture = listeria b. EEG
monocytogenes. c. MRI
What is the appropriate tx? d. Skull XR
a. Add IV amoxicillin e. Facial XR
b. Change to IV amoxicillin + gentamicin
c. Add IV ciprofloxacin
d. Add IV co-amoxiclav Ans. The key is E. Facial X-ray. This is a wrong key
e. Continue IV ceftriaxone as mono-therapy Correct key is A. CT brain. [With the risk of basal
fracture we should do CT scan to diagnose this. In
present case CT is better than MRI. Skull X-rays
Ans. The key is B. Change to IV amoxicillin + are no longer recommended as first line
gentamycin. [From the given option B is the most investigation].
574. A 15m child is due for his MMR d. Sarcoidosis
vaccine. There is a fam hx of egg allergy. e. Sjogren’s syndrome
He is febrile with acute OM. What is the
single most appropriate action?
a. Defer immunization for 2wks Ans. The key is C. HIV disease. [The lesion
b. Don’t give vaccine described is leukoplakia which is likely association
c. Give half dose of vaccine of HIV disease].
d. Give paracetamol with future doses of
the same vaccine
e. Proceed with standard immunization 577. A 3m baby was miserable and cried
schedule for 2h following his 1st routine
immunization with DTP, HiB and
meningitis. What is the single most
Ans. The key is A. Defer immunization for 2 wks. appropriate action?
a. Defer immunization for 2wks
b. Don’t give vaccine
c. Give half dose of vaccine
575. A 33yo lady with Hodgkin’s d. Give paracetamol with future doses of
lymphoma presents with temp=40C, left the same vaccine
sided abdominal pain and lymphadenitis. e. Proceed with standard immunization
Blood was taken for test. What will you do schedule
next?
a. Wait for blood test
b. Start broad spectrum IV antibiotics Ans. The key is E. Proceed with standard
c. Oral antibiotics immunization schedule.
d. CBC
e. Monitor pyrexia
578. A 65yo man with HTN develops
gingival hyperplasia. What is the single
Ans. The key is B. Start broad spectrum IV most likely dx?
antibiotics. [The patient is immunocompromized a. ACEi
with signs of infection (temp=40◦C, left sided b. Beta blockers
abdominal pain and lymphadenitis) broad c. Crohns disease
spectrum IV antibiotic should be started d. Nifedipine
empirically while waiting for blood reports]. e. Sarcoidosis

576. A 40yo man with marked weight loss Ans. The key is D. Nifedipine. [Gingival
over the preceding 6m has bilateral white, hyperplasia is a recognized side effect of calcium
vertically corrugated lesion on the lateral channel blockers].
surfaces of the tongue.
What is the single most likely dx?
a. C1 esterase deficiency 579. A 65yo woman is undergoing
b. Crohns disease coronary angiography. What measure will
c. HIV disease protect her kidneys from contrast?
a. Furosemide adrenal insufficiency (here vomiting, abdominal
b. Dextrose pain and sudden falls in the morning secondary to
c. 0.45% saline postural hypotension on getting up from bed
d. 0.9% saline points towards the diagnosis of adrenal
insufficiency)].

Ans. The key is D. 0.9% saline.


582. A 78yo woman presents with
unilateral headache and pain on chewing.
580. An 83yo woman who is a resident in ESR=70mm/hr. She is on oral steroids.
a nursing home is admitted to hospital What is the appropriate additional tx?
with a 4d hx of diarrhea. She has had no a. Bisphosphonates
weight loss or change in appetite. She has b. HRT
been on analgesics for 3wks for her back c. ACEi
pain. She is in obvious discomfort. On d. IFN
rectal exam: fecal impaction. What is the e. IV steroids
single most appropriate immediate
management?
a. Codeine phosphate for pain relief Ans. The key is A. Bisphosphonates. [The elderly
b. High fiber diet lady with giant cell arteritis is getting high dose
c. Oral laxative steroid which can lead to demineralization and
d. Phosphate enema osteopenia or osteoporosis. So to prevent this
e. Urinary catheterization bisphosphonates are given].

Ans. The key is D. Phosphate enema. [In feacal 583. A 30yo man is suffering from fever,
impaction oral laxative is not the choice but rash and photophobia. Doctors are
phosphate enema is the best option here]. suspecting he is suffering from meningitis.
Which is the best medication for this
condition?
581. A 26yo woman being treated for a a. Ampicilling
carcinoma of the bronchus with steroids b. Cefotaxime
presents with vomiting, abdominal pain c. Tetracycline
and sudden falls in the morning. What is d. Acyclovir
the most specific cause for her symptoms? e. Dexamethasone
a. Steroid side effects
b. Postural hypotension
c. Adrenal insufficiency Ans. The key is B. Cefotaxime. [The patient is
d. Conn’s disease getting probable meningococcal meningitis.
e. Cushing’s disease Before confirming the diagnosis suggested
treatment is, where

the organism is unknown:


Ans. The key is C. Adrenal insufficiency. [Patients
• <55yrs: cefotaxime 2g/6h slow IV.
on steroid develop suppression of hypothalamic
pituitary adrenal axis and frequently may lead to
• >55yrs: cefotaxime as above + ampicillin 2g 586. A 4yo boy who prv had normal
IV/4h (for Listeria). So in given case Cefotaxime is hearing, has a mild earache relieved by
the option. Ref: OHCM, 9th edition, page 832]. paracetamol. He has been noticed to turn
up the vol on the TV. He has bilateral dull
tympanic membranes. His
5084. A 15yo girl was admitted with preschoolhearing test shows symmetrical
anemia, chest infection and loss of 40dB.
thrombocytopenia. She was treated and What is the single most likely dx?
her symptoms had regressed. She was a. Acute otitis externa
brought again with fever and the same b. Acute OM
symptoms a few days later. She also c. Ear wax
seems to have features of meningitis. d. Foreign body
What is the most likely dx? e. OM with effusion
a. AML
b. ALL
c. Aplastic anemia Ans. The key is OM with effusion.
d. CML
e. CLL
587. An 18yo man presents to his GP with
thirst and polyuria. Some 6m ago he had a
Ans. The key is B. ALL. [The age supports the significant head injury as the result of a
diagnosis of ALL along with the given picture. RTA. He is referred to the local endocrine
Same picture can happen in aplastic anaemia but clinic. Which of the following results
there is not a single factor mentioned in favour of would be the most useful in confirming
it. So ALL can be taken as best option in the given the dx of diabetes insipidus after a water
scenario]. deprivation test (without additional
desmopressin)?
a. Plasma sodium of 126mmol/l
585. A pt was admitted to the ED after a b. Plasma sodium of 150mmol/l
head injury. When examined on arrival his c. Plasma osmolality of 335mosmol/kg and
GCS=15 and then at night his GCS urine osmolality of 700mosmol/kg
deteriorated to 12. d. Plasma osmolality of 280mosmol/kg and
What investigation should be done? urine osmolality of 700mosmol/kg
a. CT head e. Plasma osmolality of 335mosmol/kg and
b. XR skull urine osmolality of 200mosmol/kg
c. IV mannitol
d. Drill a burr hole
e. Shift to OR Ans. The key is E. Plasma osmolality of 335
mosmol/kg and urine osmolality of 200
mosmol/kg. [In DI plasma osmolality will be high
Ans. The key is A. CT head. [Initial GCS 15 due to excess fluid loss with urine and urine
followed by later GCS 12 are suggestive of osmolality will be low due to polyuria. Hence E. is
intracranial haemorrhage. So the best the best option here(normal plasma osmolality
investigation to be done is CT head]. 275-295 mosmol/kg and normal urine osmolality
is 300-900 mosmol/kg)].
thumb, index and middle fingers for a
while. She has been treated with local
588. A 75yo man has left-sided earache
steroids but it hasn’t helped her much and
and discomfort when he swallows. There
now she has planned to undergo a
is ulceration at the back of his tongue and
surgical procedure. Which of the following
he has a palpable non-tender cervical
structures will be incised?
mass. What is the single most likely dx?
a. Flexor digitorum profundus
a. Acute mastoiditis
b. Transverse carpal ligament
b. Dental abscess
c. Palmar aponeurosis
c. Herpes zoster infection
d. Extensor retinaculum
d. Oropharyngeal malignancy
e. Tonsillitis

Ans. The key is B. Transverse carpal ligament.


[This is a case of carpal tunnel syndrome. So
Ans. The key is D. Oropharyngeal malignancy.
transverse carpal ligament is the structure which
[Pain ear and discomfort during swallowing,
will be incised].
ulceration at the back of the tongue and palpable
non tender cervical lymphnode is suggestive of
oropharyngeal malignancy. Acute mastoiditis and
tonsillitis will not cause tongue ulcer. Similarly 591. A 58yo pt presents with altered
dental abscess will not cause tongue ulcer. In bowel habits and bleeding per rectum.
herpes zoster pain and vesicle will be in the Exam and sigmoidoscopy showed an
affected nerve distribution]. ulcer. What is the single most likely dx?
a. Colorectal carcinoma
b. Celiac disease
589. A 42yo man has been tired and c. Crohns disease
sleepy for the last few weeks in the d. UC
morning. His work has started getting e. IBS
affected as he feels sleepy in the
meetings. His BMI=36.
What is the single most likely dx? Ans. The key is A. Colorectal carcinoma. [Age,
a. Idiopathic hypersomnia altered bowel habits, bleeding per rectum and
b. Narcolepsy isolated ulcer on sigmoidoscopy suggest
c. Chest hyperventilation syndrome colorectal cancer].
d. OSAS
e. REM-related sleep disorder
592. A mother is concerned that her 18m
son has a vocabulary of ten words but
Ans. The key is D. OSAS. [Day time somnolence can’t form a sentence. What is the best
and obesity (high BMI of 36) points the diagnosis management strategy?
of OSAS]. a. Arrange hearing test
b. Assess developmental milestones
c. Reassurance
590. A 35yo pregnant woman has been d. Refer to speech therapist
having tingling and numbness of her e. MRI brain
clinical picture described well fits with pharyngeal
pouch].
Ans. The key is C. Reassurance. [Two words
joining can be done in 2yrs and inability to form a
sentence in 18 months is quite normal. So the
option is reassurance]. 595. A 9m child is brought to the ED with
an irreducible firm swelling which
descended into the left groin when the
593. A 55yo man has weight loss, dyspnea child has been crying. Exam: both testicles
and syncope. He smokes 20 are palpable in the scrotum. What is the
cigarettes/day. Inv confirms squamous cell most appropriate management strategy?
a. Reassurance
carcinoma in the left bronchus. What is
b. Emergency herniotomy
the single most likely biochemical
c. Elective herniotomy
abnormality to be a/w the condition?
a. Hypercalcemia d. Emergency herniotomy + orchidopexy
b. Hyperkalemia e. Elective herniotomy + orchidopexy
c. Hypernatremia
d. Hypocalcemia
e. Hypomagnesium Ans. The key is C. Elective herniotmy. [If there
was features of strangulation we would go for
emergency herniotomy. But as only irreducible
we shall proceed to elective herniotomy].
Ans. The key is A. Hypercalcemia. [Hypercalcemia
is common in squamous cell carcinoma].

596. A 37yo woman was admitted for


femur fx repair after a RTA. On the 4th
594. A 72yo man presents with
post-op day she became confused and
intermittent difficulty in swallowing with
starts picking on her bed sheets and
regurgitation of stale food materials.
complains of seeing spiders all over. What
Sometimes he wakes up at night with a
is the most likely dx?
feeling of suffocation. Choose the single
a. Delirium tremens
most likely cause of dysphagia?
b. Wernickes encephalopathy
a. Benign structure
c. Korsakoffs psychosis
b. Esophageal carcinoma
d. Psychotic depression
c. Esophageal spasm
d. Pharyngeal pouch
e. Systemic sclerosis
Ans. The key is A. Delerium tremens. [withdrawal
of alcohol due to hospital admission lead to
delirium tremens. Warnicke’s encephalopathy
Ans. The key is D. Pharyngeal pouch. [In benign
has triad of i) mental confusion ii) abnormal eye
stricture, esophageal carcinoma and systemic
movements & iii) unsteady gait. Korsakoff's
sclerosis there is persistent dysphagia not
syndrome cannot be diagnosed until the person
intermittent. In oesophageal spasm there may be
has stopped drinking alcohol for several weeks, to
intermittent dysphagea but there will be no
enable the immediate symptoms of alcohol
regurgitation of stale food material and no
intoxication and withdrawal to subside. Features
nocturnal regurgitation in recumbency. The
of Korsakoffs psychosis i) dementia, loss of short
term memory ii) difficulty in acquiring new a. Acute marginal branch
information or learning new skills iii) personality b. Left ant descending artery
change iv) confabulation]. c. Septal branches
d. Circumflex artery
e. Right coronary artery
597. A 36yo pt came with diarrhea,
bleeding, weight loss and fistula. What is
Ans. B. Left anterior descending artery.
the single most likely dx?
a. Celiac disease
b. Crohns disease
c. UC 601. A mother presents with her 12m
d. IBS daughter. The child has no meaningful
words, is unable to sit unaided and can’t
play with her toys. She doesn’t laugh and
Ans. The key is B. Crohns disease. has poor interaction with her siblings.
What is the best management strategy?
598. A 72yo woman who is taking loop a. Arrange hearing test
diuretics is suffering from palpitations and b. Assess developmental milestones
muscle weakness. What is the electrolyte c. Reassure
imbalance found? d. Refer to speech therapist
a. Na+ 130mmol/l, K+ 2.5mmol/l e. MRI brain
b. Na+ 130mmol/l, K+ 5.5mmol/l
c. Na+ 140mmol/l, K+ 4.5mmol/l
d. Na+ 150mmol/l, K+ 3.5mmol/l Ans. The key is B. Assess developmental
e. None milestones. [At 12 month one word should be
said clearly, in 8 months child can sit
independently, smiles at 2 months and plays with
Ans. The key is A. Na+ 130mmol/l, K+ 2.5mmol/l. toys since early infancy. So she needs to assess
[Loop diuretic causes hypokalemia and developmental milestones].
hyponatremia].

599. A 60yo diabetic pt on anti-diabetic 602. A pt presents with progressive visual


medication developed diarrhea. deterioration. Exam: large, multiple cotton
What is the most likely cause for his wool spots in both eyes. What is the single
diarrhea? most likely dx?
a. Autonomic neuropathy a. Kaposi’s sarcoma
b. Infective b. Cryptosporidium
c. Celiac disease c. CMV infection
d. Crohns disease d. Pneumocystis carinii infection
e. Cryptococcal infection

Ans. The key is A. Autonomic neuropathy.

600. Which artery runs in the anterior Ans. The key is C. CMV infection. [Large multiple
cotton wool spots are seen in early stage of CMV
inter-ventricular groove?
retinitis].
603. A 53yo had a dental extraction after
which he recently had a mitral valve Ans. The key is B. 3rd person auditory
prolapse, high temp of 39C, cardiac failure hallucinations. [Third person hallucinations are
and new cardiac murmur. auditory hallucinations in which patients hear
What is the single most likely dx? voices talking about themselves, referring to
a. Atheroma them in the third person, for example "he is an
b. Congenital evil person".].
c. Regeneration
d. Infection
e. Neoplastic 606. A 65yo woman had an excision of
colonic tumor 3yrs ago. Now she is losing
weight and feels lethargic. Exam: pale but
Ans. The key is D. Infection. [Infective no abdominal findings.
endocarditis]. What is the most appropriate inv?
a. CA 125
604. A 12yo boy with a hx of fall on an
b. CA 153
outstretched hand was brought to the ED c. CA 199
with swelling and pain around the elbow. d. CEA
His radial nerve was affected. e. AFP
What is the type of fx?
a. Angulated fx
b. Epiphyseal fx
Ans. The key is D. CEA. [CA 125 = ovarian cancer;
c. Compound fx
CA 153 = cancer breast; CA 199 = pancreatic
d. Spiral fx
cancer; CEA = colorectal carcinoma; AFP =
hepatocellular carcinoma].

Ans. The key is D. Spiral fracture. It is wrong key.


The correct option should be A. Angulated 607. A 46yo African-Caribbean man is
(supracondylar fracture). [Around 50%
found to have BP=160/90mmHg on 3
interosseous nerve lesions occur in supracondylar
separate occasions. What is the best
fracture whereas 25% shows radial nerve
damage. If the fracture is spiral fracture of lower initial tx?
a. ACEi
third of humerus it causes nerve damage in 18%
almost all of which are radial nerve lesion. b. Beta-blockers
However as the fracture is around the elbow (not c. ARBs
above) it is supracondylar fracture]. d. None
e. CCB
605. A 32yo lady complains that she hears
everyone saying that she is an evil person.
What type of hallucinations is she Ans. The key is E. CCB. [If age less than 55 years
suffering from? but Afro-Caribbean origin then CCB].
a. 2nd person auditory hallucinations
b. 3rd person auditory hallucinations
c. Echo de la pense 608. A 39yo woman will undergo tubal
d. Gedankenlautwerden
sterilization and she wants to know the
failure rate of this type of sterilization.
a. 1:50 611. A 68yo lady complains of falls to the
b. 1:200 ground without any warning, maintains
c. 1:500 consciousness and no confusion. She says
d. 1:1000 this has occurred at number of times.
e. 1:5000 What is the dx?
a. Stokes Adams attack
Ans. The key is B. 1:200.
b. Hypoglycemia
609. Which of the following reflexes and c. Vasovagal syncope
innervating spinal nerves are correctly d. Drop attacks
paired? e. Epilepsy

a. Anal reflex – S1
b. Ankle jerk – L5 Ans. The key is D. Drop attacks. [Drop attacks are
c. Biceps jerk – C7 & C8 sudden spontaneous falls while standing or
d. Knee jerk – L3 & L4 walking, with complete recovery in seconds or
e. Triceps jerk – T1 minutes. There is usually no recognized loss of
consciousness, and the event is remembered].

Ans. The key is D. Knee jerk – L3 & L4. [Anal reflex


– S2-4; Ankle jerk – S1-2; Biceps jerk – C5-6; Knee 612. A 50yo man complains of being
jerk – L3-4; Triceps jerk – C7]. pursued by the police for a crime he
denies committing. He has poor
concentration and impaired short-term
610. A 62yo man with rheumatoid memory. He admits to drinking large
arthritis struck his hand against a door. He amounts of alcohol for the last 20yrs.
subsequently found that although he What is the most probable dx?
could extend the interphalangeal joint of a. Dementia
his right thumb, the MCP joint of the b. Hallucination
thumb remained flex. What is the single c. Wernicke’s encephalopathy
most likely tendon to have been d. Schizophrenia
damaged? e. Korsakoff psychosis
a. Extensor carpi ulnaris
b. Extensor digitorum
c. Extensor indicis Ans. The key is E. Korsakoff psychosis. [Dementia,
d. Extensor pollicis brevis i.e. short term memory loss is seen in korsakoff
e. Extensor pollicis longus psychosis].

Ans. The key is D. Extensor pollicis brevis. [Action 613. A pt with prv hx of HTN, the
of extensor pollicis brevis = extension of thumb at membranes have ruptured and the cervix
metacarpophalangeal joint. Extensor pollicis is 3cm dilated. 4h later on examination
longus = extends the terminal phalanx of the showed that the cervix was still 3cm
thumb]. dilated.
What is the single most appropriate
management for her labor?
a. Repeat vaginal examination in 4h Ans. The key is B. Bendroflumethiazide. [High
b. CTG blood sugar is a well known side effect of
c. C-section bendroflumethiazide].
d. External rotation
e. IV syntocin drip
616. A 27yo waitress has pelvic pain,
dysmenorrhea and increasingly heavy
Ans. The key is E. IV syntocinon drip. [There is no periods. She also complains of
progress of labour in 4 hours. Hence syntocinon dyspareunia. There is generalized pelvic
drip should be given]. tenderness without peritonism. Pelvic US
is normal. What is the most likely dx?
a. Endometriosis
614. A 6yo girl has had 2 short episodes of b. Uterine fibroid
cough and wheeze over the last 12m. c. Pelvic congestion syndrome
These 2 acute episodes responded quickly d. PID
to bronchodilator, she has no symptoms e. Tubal pregnancy
or abnormal physical signs. She has slight
eczema and her mother has asthma.
What is the single most appropriate inv? Ans. The key is C. Pelvic congestion syndrome. [In
a. CXR pelvic congestion syndrome there develops
b. Peak flow rate diary varicose veins in the lower abdomen from
c. Pulse oximetry prolonged standing (as occurred here in a
d. Spirometry waitress who remains standing for long) with
e. Sweat test some pain syndromes like pelvic pain,
dysmenorrea, dyspareunia and generalized pelvic
tenderness without peritonism. Also there may
be associated menorrhagia].
Ans. The key is D. Spirometry. [spirometry is the
preferred initial test (if available) to assess the
presence and severity of airflow obstruction less
effort dependent and more repeatable though
617. A 14yo girl is clinically obese. She has
less applicable in acute severe asthma]. not started her periods yet and has severe
acne. Among her inv, a high insulin level is
found. What is the most probable dx?
a. Cushing’s syndrome
615. A 45yo man had recently started
b. Grave’s disease
taking anti-HTN therapy. 6m later his
c. Acquired hypothyroidism
RBS=14mmol/l. Which single drug is most
d. PCOS
likely to have caused this?
e. Addison’s disease
a. Amlodipine
b. Bendroflumethiazide
c. Doxazosin
d. Lorsartan Ans. The key is D. PCOS. [It is not cushing’s as
e. Ramipril insulin levels in cushing’s are not usually raised!
Here obesity, primary amenorrhea , acne and
particularly high level of insulin makes the likely
diagnosis to PCOS].
believes that the staff and other pts know
exactly what she is thinking all the time.
618. An 18yo girl with primary
What is the most likely symptom this pt is
amenorrhea complains of severe
suffering from?
abdominal pain every 4-8weeks which is
a. Thought insertion
now getting worse. Exam: lower
b. Thought withdrawal
abdominal mass is felt. c. Thought block
What is the most probable dx? d. Though broadcasting
a. Ectopic pregnancy
e. Hallucination
b. Ovarian carcinoma
c. Hematometrium
d. Biliary colic
Ans. The key is D. Thought broadcasting.
e. Renal carcinoma

621. A 60yo woman is admitted to the


Ans. The key is C. Hematometrium. [Primary
amenorrhea and periodic pain indicate
hospital after a fall. She is noted to have
hematometrium either secondary to poor eye contact. When asked how she is
imperforated hymen or vaginal septum]. feeling, she admits to feeling low in mood
and losing enjoyment in all her usual
hobbies. She has also found it difficult to
619. A 14yo boy with asthma suddenly concentrate, feels that she is not good at
developed chest pain and increasing anything, feels guilty over minor issues
breathlessness during a game of football. and feels very negative about the future.
When seen in the ED he was not What is the most likely dx?
cyanosed. He has reduced breath sounds a. Mild depression
on the right side. His oxygen saturation is b. Moderate depression
94% on air. c. Severe depression
d. Psychosis
What is the single most appropriate inv?
a. Capillary blood gases e. Seasonal depression
b. CXR
c. CT chest
d. Exercise challenge Ans. The key is A. Mild depression. [Mild
e. MRI chest depression: i)Low mood ii) Anhedonia iii) Guilt iv)
Hopelessness v) Worthlessness vi) Inability to
concentrate].

Ans. The key is B. CXR. [Asthma is a predisposing


factor for spontaneous pneumothorax. The
presentation indicates pneumothorax for which 622. A 70yo woman lives in a nursing
most appropriate investigation is CXR]. home following a stroke has developed
reddish scaly rash on her trunk. She has
many scratch marks on her limbs and
620. A 36yo woman was recently trunk with scaling lesions on her hands
admitted to a psychiatric ward. She and feet. What is the single most
appropriate initial tx?
a. Aqueous cream considerable time pressure exerted on the nerve
b. Chlorphenaramine may cause nerve palsi].
c. Coal tar
d. 1% hydrocortisone ointment
e. Permethrin 625. A 25yo woman is presenting with
diarrhea and abdominal bloating over the
last 4m. Exam: she has blistering rash over
Ans. The key is E. Permethrine. [This is a case of her elbows. Biochemistry: low serum
scabies and scaly rash denotes the infection of albumin, calcium and folate conc. On
most severe type the crusted or Norwegian jejunal biopsy, there is shortening of the
scabies. Should be treated with permethrine]. villi and lymphocytosis. What is the most
likely dx?
a. Celiac disease
623. A 16yo boy following a RTA was b. Whipple’s disease
brought to the ED with a swelling and c. Crohn’s disease
deformity in his right thigh. Exam: airway d. Tropical sprue
is patent and is found to have a pulseless e. Giardiasis
leg. f. Cystic fibrosis
Which structure is involved in this fx?
a. Femoral artery
b. Posterior tibial artery Ans. The key is A. Celiac disease. [diarrhea,
c. Common peroneal nerve abdominal bloating, blistering skin rash over
d. Dorsalis pedis elbow (Dermatitis herpetiformis), low serum
albumin, calcium and folate conc. Supported by
shortening of villi and lymphocytosis on jejuna
Ans. The key is A. Femoral artery. biopsy is classic presentation of celiac disease].

624. A man sat cross-legged for about 626. A 19yo man presents for the 1st time
30mins. After this he was unable to with a firm and unshakable belief that he
dorsiflex his left foot and had loss of is being followed by terrorists who are
sensation in the web space between the plotting against him.
big toe and the 2nd toe. He also has What is the single best term for this man’s
sensory loss on the same side of the foot condition?
a. Delusion of persecution
after 2h.
b. Delusion of grandeur
Which of the following was affected?
a. Femoral nerve c. Delusion of control
b. Sural nerve d. Delusion of reference
c. Peroneal nerve e. Delusion of nihilism
d. Sciatic nerve

Ans. The key is A. Delusion of persecution.


Ans. The key is C. Peroneal nerve. [Common [Delusions of persecution refer to false beliefs or
peroneal nerve winds round the fibular neck at perceptions in which a person believes that they
knee joint and when a man sits cross legged for a are being treated with malicious intent, hostility,
or harassment – despite significant evidence to 629. A teacher had a respiratory infection
suggest otherwise]. for which she was prescribed antibiotics.
After the antibiotic course when she
rejoined school, she lost her voice
627. A 19yo female is brought in by her completely.
parents. They are concerned about her What is the single most appropriate dx?
BMI which is 12. She is satisfied with it. a. Recurrent laryngeal nerve palsy
What is the next step? b. Angioedema
a. Psychiatric referral for admission c. Laryngeal obstruction by medication
b. Family counselling d. Laryngitis
c. Social service e. Functional dysphonia/vocal cords
d. Start antidepo
e. Medical admission
Ans. The key is E. Functional dysphonia/vocal
cords. [Functional dysphonia is poor voice quality
Ans. The key is E. Medical admission. [The without any obvious anatomical, neurological or
diagnosis is anorexia nervosa. At this critical low other organic difficulties affecting the larynx or
BMI medical admission is indicated to improve voice box. It is often secondary to viral infection].
her deficiency states and proper nutrition. ((BMI
<15kg/m2, rapid weight loss + evidence of system
failure) requires urgent referral to eating disorder 630. A 43yo lady is admitted with pyrexia,
unit (EDU), medical unit (MU) or paediatric arthropathy, breathlessness and syncope.
medical wards]. She was recently dx with pulmonary
emboli. There is an early diastolic sound
and a mid-diastolic rumble. Her JVP is
628. A lady who works at a nursing home elevated with prominent a-waves.
presents with itching. Exam: linear tracks What is the most likely cause?
on the wrist. She says that 2d ago she had a. Mitral regurgitation
come in contact with a nursing home b. Ventricular ectopics
inmate with similar symptoms. What is c. Pulmonary regurgitation
the mechanism of itching? d. Atrial myxoma
a. Infection e. Complete heart block
b. Destruction of keratinocytes
c. Allergic reaction
d. Immunosuppression Ans. The key is D. [Pyrexia, arthropathy,
e. None breathlessness, syncope and early diastolic sound
and a mid diastolic rumble are known features of
atrial myxoma].
Ans. The key is C. Allergic reaction. [The probable
diagnosis is scabies in which there is itching due
to allergic reaction to mites (Sarcoptes scabii) 631. A 28yo man presents with a
waste products]. maculopapular rash over his trunk and
palms. He also has numerous mouth
ulcers. He had a penile ulcer which healed
2wks ago.
What will you do to confirm the dx? gallstones, 2yrs ago. What is the most
a. PCR for treponemal and non-treponemal likely inv to confirm dx?
antibiodies a. US abdomen
b. Dark ground microscopy from mouth b. LFT
ulcer c. Serum lipase
c. Blood culture for treponema d. Angiography
d. Dengue fever e. CT abdomen

Ans. The key is C. Serum lipase. [The likely


Ans. The key is A. PCR for treponomal and non-
diagnosis is pancreatitis hence serum lipase].
treponemal antibodies. [Non treponemal
antibody test if positive indicate that there may 634. A 32yo female with axillary freckles
be syphilis and it is not confirmatory alone. That and café au lait spots wants to know the
is why treponemal antibody test should also be chances of her child also having similar
done to confirm it. On the other hand resolved condition.
disease may show negative treponemal test a. 1:2
which is confirmed by positive non-treponemal
b. 1:4
test].
c. No genetic link
d. 1:16
e. Depends on the genetic make up of the
632. A 34yo man complains of arthralgia, partner
abdominal pain and vomiting, a facial
rash that is worse in the summer and
hematuria. Urea and creatinine are Ans. The key is E. Depends on genetic make up of
slightly elevated with urinalysis the partner. [Depends on the make up of the
demonstrating red cell casts. PMH is partner, ideally it's 1:2 since it's autosomal
remarkable for childhood eczema. Which dominant with complete penetrance but if the
inv is most likely to lead to a dx? patients spouse also has neurofibromatosis, it
a. US KUB raises it to at least 75% with a possible 100%].
b. Joint aspiration
c. Auto antibodies
d. IVU 635. A 40yo man has pain, redness and
e. Renal biopsy swelling over the nasal end of his right
lower eyelid. The eye is watery with some
purulent discharge. The redness extends
Ans. The key is C. Auto antibodies. [Likely on to the nasal peri-orbital area and
diagnosis is SLE for which auto antibody (anti ds mucoid discharge can be expressed from
DNA antibody) should be done]. the lacrimal punctum. What is the single
most appropriate clinical dx?
a. Acute conjunctivitis
633. A 56yo woman has had severe b. Acute dacrocystitis
abdominal pain for 24h radiating to her c. Acut iritis
back and is accompanied by nausea and d. Retrobulbar neuritis
vomiting. She appears to be tachycardic e. Scleritis
and in shock. She was found to have
c. Panic disorder
Ans. The key is B. Acute dacrocystitis. d. GAD
e. Bereavement
636. A 60yo lady has severe chest pain.
ECG shows changes of inferior wall MI.
ECG also shows progressive prolongation
Ans. The key is A. Adjustment disorder. [When 2
of PR interval until a QRS complex is
months passed it is no more normal bereavement
dropped.
but major depression or adjustment disorder].
What is the most probable dx?
a. Atrial fibrillation
b. VT
639. A 32yo man on psychiatric
c. SVT
medications complains of inability to
d. Mobitz type I 2nd degree heart block
e. Mobitz type II 2nd degree heart block
ejaculate.
Which drug is most likely to cause these
symptoms?
a. Lithium
Ans. The key is D. Mobitz type I 2nd degree heart
b. Haloperidol
block. [Inferior MI is frequently associated with
c. Chlorpromazine
conduction defect].
d. Fluoxetine
e. Clozapine

337. A 52yo woman speaks rapidly


without any pause and ignores
Ans. The key is D. Fluoxetine. [SSRIs are
interruptions. She doesn’t even pause to
frequently associated with delayed ejaculation].
take enough breaths.
What term best describes this kind of
speech?
640. A 4yo boy is brought by his parents
a. Flight of ideas
b. Broca’s aphasia
with complains of wetting his bed at night
c. Wernicke’s aphasia
and whenever he gets excited. What
d. Pressure of speech would be the most appropriate
e. Verbal dysphasia management for this child?
a. Desmopressin
b. Oxybutanin
c. Behavioural therapy
Ans. The key is D. Pressure of speech.
d. Tamsulosin
e. Restrict fluid intake

338. A 30yo woman has been feeling low


and having difficulty in concentrating
Ans. The key is C. Behavioural therapy. Probably
since her mother passed away 2m ago.
this is wrong option! There is no correct option
She feels lethargic and tends to have for this question. [The question is inappropriate.
breathlessness and tremors from time to Child is of 4 yrs of age and before 5 yrs only
time. What is the most likely dx? reassurance, no treatment is indicated].
a. Adjustment disorder
b. PTSD
641. A 34yo DM pt is undergoing contrast
radiography. What measure should be Ans. The key is D. Carbamazepine. [It is a case of
taken to prevent renal damage with bipolar disorder. Mainstay of treatment is mood
contrast dye? stabilizers such as i) Lithium carbonate ii)
a. Reduce contrast dye Anticonvulsant medicines iii) Antipsychotic
b. Plenty of fluids medicines. So from the given options
c. NSAIDS Carbamazepine is the most appropriate drug].
d. ACEi
e. IV dextrose
644. A 25yo woman complains of
Ans. The key is B. Plenty of fluids.
dizziness, nausea, vomiting, visual
disturbances and anxiety which keep
642. A 75yo woman presents to the breast coming from time to time. Most of the
clinic having noticed that she has had a attacks are a/w sudden change in posture.
blood stained discharge from the left What is the most likely dx?
nipple, together with dry skin over the left a. Panic disorder
areola. Exam: blood stained discharge b. Carotid sinus syncope
with dry flaky skin noted on the left c. BPPV
areola. The nipple was noted to be d. Vertebrobasilar insufficiency
ulcerated. What is the most appropriate e. Postural hypotension
inv?
a. FNAC
b. MRI Ans. The key is C. Benign paroxysmal positional
c. Punch biopsy vertigo (BPPV). [Dizziness, nausea, vomiting and
d. Open biopsy nystagmus which keep coming from time to time
e. Stereotactic biopsy are common features of BPPV].

Ans. The key is C. Punch biopsy. [As the lesion is 645. A 56yo man was recently put on anti-
on the surface punch biopsy can be well
HTN meds and recent biochemistry on 2
obtained].
occasions showed: Na+=132, K+=7.6,
643. A 50yo man presents with low mood, Urea=11.3, Creat=112.
poor concentration, anhedonia and Which of the following drugs is
insomnia. He has had 2 episodes of responsible for this result?
increased activity, promiscuity and a. Amlodipine
aggressive behavior in the past. He was b. Bendroflumethiazide
arrest 8m ago for trying to rob a bank c. Doxazosin
claiming it as his own. d. Atenolol
Which drug is most likely to benefit him? e. Ramipril
a. Haloperidol
b. Citalopram
c. Desipramine Ans. The key is E. Ramipril. [ACEI and ARB are
d. Carbamazepine known to raise the serum potassium level].
e. Ethosuximide
646. A 46yo woman has offensive yellow 649. A 55yo woman who attends the clinic
discharge from one nipple. She had a hx of has recently been dx with a depressive
breast abscess 3yrs ago. What is the episode. She complains of unintentionally
possible dx? waking early in the morning, a recent
a. Duct papilloma disinterest in sex and a loss of appetite,
b. Duct ectasia losing 5kg weight in the last month. She
c. Duct fistula feels that her mood is worse at the
d. Breast cancer beginning of the day. What is the most
likely dx for this pt?
a. Mild depression
a. Ans. The key is C. Duct fistula. b. Moderate depression
c. Severe depression
d. Low mood
647. A 35yo woman undergoing tx for TB e. Pseudo depression
presents with malar rash, photosensitivity
and hematuria. What is the single most
likely positive antibody? Ans. The key is B. Moderate depression. [Sleep
a. Anti Ds DNA disturbance, disinterest in sex and loss of appetite
b. Anti Sm points towards the diagnosis of moderate
c. Anti Histone depression].
d. Anti La
e. Anti centromere
650. An employer sent his worker to the
ED after having hit his head on a machine.
Ans. The key is C. Antihistone antibody. [Anti O/E: normal. What is the single most likely
histone antibody is present in 95% cases of drug inv you would do?
induced lupus]. a. Skull XR
b. CT head
c. MRI head
648. A 6wk child with profuse projectile d. Reassure
vomiting. What is the first thing you will Ans. The key is A. Skull XR. This is wrong key!
do? Correct key should be B. CT head.
a. US
b. Check serum K+ level
c. ABG 651. A lady with fam hx of ovarian
d. NG tube carcinoma has a pelvis US that fails to
e. IV fluids reveal any abnormality. What is the single
most appropriate inv?
a. Pelvic CT
b. CA 125
Ans. The key is B. Check serum K+ level. This is a
c. CA 153
wrong key! Correct key is E. IV fluid. [We shall
d. Laparoscopy
resuscitate first with normal saline].
e. MRI
Ans. The key is A. CRAO. [Pale retina with cherry
red spot in macular region is seen in CRAO].
Ans. The key is B. CA 125. [CA 125 is tumour
marker for ovarian tumour].

654. A 48yo woman presents with left-


652. A 10yo boy is taken to his GP by his sided severe headache. She also has a red,
parents with behavioural prbs. He attends watering eye and complains of seeing
a special school due to inappropriate colored haloes in her vision.
behavior and during the interview with his What is the most appropriate next step?
parents the boy barks at infrequent a. Measure IOP
episodes and shouts expletives. b. Relieve pain with aspirin
What is the most likely dx? c. 100% oxygen
a. Asperger syndrome d. CT
b. Cotard syndrome e. Relieve pain with sumatriptan
c. Rett syndrome
d. Ekbom syndrome
Ans. The key is A. Measure IOP. [Probable case of
e. Tourette’s syndrome
angle closure glaucoma requiring measurement
of IOP to establish the diagnosis].
Ans. The key is E. Tourette’s syndrome. [Tourete’s
syndrome may have motor tics like blinking, facial
655. A 31yo woman presents with 7-
grimacing, shoulder shrugging. Other complex
motor tics may be sniffing, touching objects,
10days following childbirth, with loss of
hopping, jumping, bending or twisting. It has feeling for the child, loss of appetite, sleep
vocal tics like throat clearing, sniffing, grunting or disturbance and intrusive and unpleasant
barking and more complex like coprolalia thoughts of harming the baby. What is the
(uttering socially inappropriate words) or best tx for this pt?
echolalia (repeating the word or phrase of a. Fluoxetine
others)]. b. Haloperidol
c. CBT
d. Reassurance
653. A 52yo male presents with sudden
e. ECT
complete loss of vision from right eye. He
also had been complaining of right sided Ans. The key is A. Fluoxetine. [The diagnosis is
headaches which would come up more on post-partum depression. Treatment is fluoxetine.
chewing. On fundoscopy, the retina was The mode of treatment may be 1. Drugs like a)
pale and a cherry red spot could be seen Antidepressant b) Antypsychotic or c) Mood
in the macular region. What caused this stabilizers like lithium. If drug treatment fails then
vision loss? 2. ECT].
a. CRAO
656. A 56yo male pt presents with
b. CRVO
intermittent vertigo, tinnitus and hearing
c. Branch RAO
loss.
d. Branch RVO
What is the best drug tx for this pt?
e. Circumciliary vein occlusion a. Buccal prochlorperazine
b. Oral flupenphenazine
c. TCA
d. Gentamicin patch on the round window with an attack then rebreathing in a paperbag to
e. No med tx available subside her acute problems].

Ans. The key is A. Buccal prochlorperazine.


659. A 32yo woman P3 of 39wks gestation
[Probable case of Menieres disease. Treated with
prochlorperazine].
reports having spontaneous ROM 4days
ago. She didn’t attend the delivery suite as
she knew that would happen and had
657. An 82yo woman has developed already decided on a home birth. Today
painful rash on one side of her forehead she feels very hot and sweaty. She
and ant scalp. Lesions have also affected thought that she was starting to have
her cornea. labour pains but she describes the pain as
What is the single most appropriate more constant. Exam: uterus is tender
option? throughout. Blood tests show raised CRP
a. Accessory nerve and WBC. Select the most likely dx?
b. Facial nerve a. Round ligament stretching
c. Olfactory nerve b. Chorioamnionitis
d. Optic nerve c. Uterine rupture
e. Trigeminal nerve d. Labor
e. DIC

Ans. The key is E. Trigeminal nerve. [Probable


herpes zoster opthalmicus]. Ans. The key is B. Chorioamnionitis. [Prolonged
rupture of membrane can lead to
chorioamnionitis].
658. A 24yo woman presents with
episodes of peri-oral tingling and carpo
pedal spasms every time she has to give a 660. A 63yo man continues to experience
public talk. This also happens to her chest pain and has a temp of 37.8C 2 days
before interviews, exams and after after an acute MI. His ECG shows
arguments. What is the best management widespread ST elevation with upward
strategy for this pt? concavity.
a. Diazepam What is the single most likely explanation
b. Rebreathe in a paper bag for the abnormal inv?
c. Desensitization a. Acute pericarditis
d. Buspirone b. Cardiac tamponade
e. Propranolol c. Atrial thrombus
d. Left ventricular aneurysm
e. Dressler syndrome
Ans. The key is B. Rebreathe in a paper bag. This
is wrong key. More correct option is C.
Desensitization.[Desensitization is the treatment Ans. The key is A. Acute pericarditis. [Chest pain,
of choice in long run. For prevention proranalol raised temperature and ECG findings of
before expected exposure and if patient presents widespread ST elevation with upwards concavity
is diagnostic of acute pericarditis particularly after 664. A 35yo IVDA (Intra Venous Drug
MI]. Abuser) on penicillin and flucloxacillin for
cellulitis now presents with jaundice, pale
stools and dark urine. What is the single
661. A 55yo man presents with an ulcer of most likely dx?
the scrotum. Which of the following LN is a. Hep A
involved? b. Cholestatic jaundice
a. External iliac LN c. Chronic active hepatitis
b. Pre-aortic LN d. Primary biliary cirrhosis
c. Aortic LN
e. Hep B
d. Inguinal LN
e. Iliac LN
f. Submental LN
g. Submandibular LN Ans. The key is B. Cholestatic jaundice.
h. Deep cervical LN [Flucloxacillin can cause cholestatic jaundice].

Ans. The key is D. Inguinal LN.


665. A 79yo woman has been dx with
T2DM. Her BMI=22. RBS are 8 and
662. A 35yo woman has butterfly rash on
10mmol/l. Her
her face and she suffers symmetrical joint
BP=130/80mmHg. Her fasting
pains on knee and elbow, ESR is raised.
cholesterol=5.7mmol/l. She is currently
What is the most discriminative inv for dx?
symptom-free but has microalbuminuria.
a. Anti DNA antibodies
What is the single most appropriate drug
b. Anti Jo1 antibodies
c. Anti nuclear antibodies
management?
a. ACEi and glibenclamide
d. Anti centromere antibodies
b. ACEi and metformin
e. Anti la antibodies
c. Statin and ACEi
d. Statin and glibenclamide
e. Statin and metformin
Ans. The key is A. Anti DNA antibodies. [Anti DNA
antibodies to diagnose SLE].

Ans. The key is C. Statin and ACEi. [Diabetic


663. Pt had a fight following which he patients are advocated statin irrespective of
developed bleeding, ringing and hearing cholesterol levels and diabetic microalbuminuria
loss from one ear. What is the inv of is best treated by ACEI. As initially we shall give
choice? lifestyle advice and no medicine for diabetes even
a. CT then we shall start with statin and ACEI].
b. XR skull
c. Otoscopy
d. MRI vestibule 666. A 68yo woman is unable to extend
e. Coagulation study the IP joint of her right thumb 7wks
following a fx of the right radius. Other
finger and thumb movements are normal.
Ans. The key is A. CT. This is a wrong key! Correct
What is the single most likely tendon to be
key is Otoscopy.
damaged?
a. Abductor pollicis longus step is to check ABG to give guidance for next
b. Extensor pollicis brevis treatment plan].
c. Extensor pollicis longus
d. Flexor digitorum profundus
e. Flexor pollicis longus 669. A 66yo woman has been brought to
the hospital on a number of occasions
with a hx of loss of memory. Her PMH is
Ans. The key is C. Extensor policis longus. [Full significant for an MI 6yrs ago. It is noted
extension of right thumb is achieved by extensor that she has a step wise decline of her
pollicis longus]. cognitive functions. What is the most
likely dx?
a. Alzhemiers
667. A mother presents her 6m son who is b. Vascular dementia
vocalizing. She has noticed that he doesn’t c. Pick’s dementia
respond to loud noises. His motor d. Huntington’s disease
milestones are normal. e. Lewy body dementia
What is the best management strategy?
a. Arrange hearing test
b. Assess development milestones Ans. The key is B. Vascular dementia. [Age 65 yrs
c. Reassure (vascular dementia is rare at age before 65),
d. Refer to speech therapist history of MI and stepwise decline of cognitive
e. MRI brain function indicates vasculopathy and hence
vascular dementia].

Ans. The key is A. Arrange hearing test. [Normal


motor milestones indicate normal development 670. A 55yo man returns for routine
but unable to respond to loud noise at 6 months follow up 6wks after an MI. He gets
may indicate deafness]. breathless when walking uphill. His ECG
shows ST elevation in leads V1, V2, V3 and
V4.
668. A 39yo man presents to the ED with What is the single most likely explanation
persistent cough, sputum and dyspnea. He for the abnormal investigation?
gave a hx of smoking 20 cigarettes/d for a. Heart block
the last 10 years. Pt was given oxygen in b. Right ventricular strain
ambulance but he is not improving. What c. Atrial thrombus
is the next step? d. Left ventricular aneurysm
a. Prednisolone e. Dressler’s syndrome
b. Salbutamol
c. Check ABG
d. CXR Ans. The key is D. Left ventricular aneurism.
e. ECG [Features of heart failure and persistent ST
elevation suggests the dx of left ventricular
aneurysm].
Ans. The key is C. Check ABG. [The patient has
COPD and as no improvement with oxygen, next
671. A 4m girl has severe FTT (Failure To 674. A 42yo overweight smoker comes
Thrive) and increasing jaundice which was with heavy periods. A scan reveals a
1st noticed at 1wk of age. She has an normal uterus. She would like a long term
enlarged liver and scratches on her skin. tx with minimal side effects that would
Her parents have been unable to seek offer tx for the menorrhagia and provide
medical care. What is the most likely dx? contraception. She is unsure whether she
a. Biliary atresia would like more children. She is adamant
b. G6PD deficiency that she doesn’t want surgery as she is
c. Hep B terrified of the prospect. Select the best
d. Spherocytosis management for her menorrhagia?
a. COCP
b. GrH analogues
Ans. The key is A. Biliary atresia. [Increasing
c. IU/systemic progesterone
jaundice at this age with failure to thrive,
enlarged liver and scratches (itching) indicate d. NSAIDs
cholestatic jaundice likely from biliary atresia]. e. Copper containing IUCD

672. A 76yo man suddenly collapsed and


Ans. The key is C. IU/systemic progesterone. [As
died. At post mortem exam, a patient is smoker, COCP should be avoided. In the
retroperitoneal hematoma due to given case option C. i.e. mirena is most suitable].
ruptured aortic aneurysm was noted.
What is the most likely underlying cause
of the aortic aneurysm? 675. A 10yo male child was brought by his
a. Atheroma mother complaining that her child
b. Cystic medial necrosis watches TV at very high volumes, doesn’t
c. Dissecting aneurysm
like to play outside and instead has
d. Polyarteritis nodosa
become more sincere with reading. She
e. Syphilis
also says that her son doesn’t respond to
her.
Ans. The key is A. Atheroma. What do you expect to see on otoscopy?
a. Flamingo pink tympanic membrane
b. Attic perforation
673. A 33yo male came to the hospital c. Bluish grey tympanic membrane with air
with complaint of occasional left sided fluid levels
chest pain that lasts <30mins, following d. Inflamed tympanic membrane with cart
exercise, which relieves upon taking rest. wheel appearance of vessels
What is the most probable dx? e. Red and inflamed tympanic membrane
a. Unstable angina
b. Decubitus angina
c. Stable angina
Ans. The key is C. Bluish grey tympanic membrane
d. Coronary spasm
with air fluid levels. [In glue air there occur
e. MI
conductive deafness so the child watch tv with
high volume and does not respond to others for
Ans. The key is C. Stable angina. this deafness].
d. Trigeminal nerve
e. Glossopharyngeal nerve
676. A lady underwent debulking surgery
for ovarian carcinoma. Soon after the
surgery she presents with signs of
Ans. The key is A. Facial nerve. [Viral infection is a
intestinal obstruction. What is the single common cause of facial nerve palsy].
most appropriate inv?
a. Pelvic CT
b. CA 125
679. A 5yo girl has had an URTI for 3 days
c. Laparotomy
and has been treated with paracetamol by
d. Laparoscopy
her mother. For the last 12h she has been
e. Abdominal US
hot and irritable with severe pain in her
right ear. What is the most likely dx?
a. Herpes zoster infection
Ans. The key is C. Laparotomy. [Here it is
b. Impacted ear wax
diagnostic and therapeutic laparotomy].
c. Mumps
d. OM
e. Perforation of eardrum
677. A 45yo woman undergoing tx for RA
for the last 5yrs presents with dizziness,
easy fatigabiliy and lack of energy. A
Ans. The key is D. OM.
blood film shows MCV 106.
What could be the most probable reason
for her
680. A 35yo man has a temp=39C, cough
anemia?
with purulent sputum and right sided
a. Steroids
chest pain on inspiration. He has herpes
b. Chronic disease
labialis.
c. NSAIDs
d. Methotrexate
What is the single most likely causative
e. Sulfasalazine
organism?
a. Coagulase +ve cocci in sputum
b. Gram -ve diplococci in sputum
c. Gram +ve diplococci in sputum
Ans. The key is D. Methotrexate. [Methotrexate
d. Pneumocystis carinii in sputum
leads to folate deficiency anaemia].
e. Serology for legionella

678. A 62yo man who has recently had


Ans. The key is C. Gram +ve diplococcic in
flu-like illness has woken to find his speech
sputum. [High temperature, cough with purulent
altered. Movement of his eyelids and lips sputum, pleuritic chest pain and herpes labialis
are weak on the right side. Exam: normal. are recognized feature of pneumococcal
Which anatomical site is most likely to be pneumonia (pneumococcus=Gram +ve
affected? diplococci].
a. Facial nerve
b. Hypoglossal nerve
c. Oculomotor nerve
681. A 27yo female was brought to the ED 683. A child admitted with progressive
by her friend from a movie theatre. She muscle weakness and frequent falls. What
complains of sudden severe pain in the is the most probable dx?
eye followed by vomiting and also was a. Duchenne’s MD
seeing colored halos. She gives a past hx b. Becker’s MD
of recurrent headaches which used to c. Polymyositis
resolve spontaneously. Exam: fixed, d. Dermatomyositis
dilated ovoid pupil seen. What is the first e. Polymyalgia rheumatic
inv?
a. CT head
b. MRI orbits Ans. The key is A. Duchenne’s MD.
c. Blood culture and sensitivity
d. Toxicology screen
e. Applanation tonometry 684. A 56yo man presents to the ED with
chest pain. The following ECG was taken.
What is the most likely dx?
Ans. The key is E. Applanation tonometry.
[Darkness can cause dilatation of pupil (which
occurred in the darkness of theatre room) which
(halfway dilatation) most often precipitate acute
attack of angle closure glaucoma and the test to
diagnose this is applanation tonometry].

682. An 82yo male suddenly becomes


unconscious and fell down. He recovered
completely within
minutes. What is the best inv you to to dx
the case?
a. ECG a. Anterior MI
b. EEG b. Inferior MI
c. Lateral MI
c. Blood glucose level d. Posterior MI
d. CT e. NSTEMI

e. CXR
Ans. The key is A. Anterior MI. [Here ST elevation
in L1, aVL, v2,v3,v4 and v5. So the diagnosis is
Ans. The key is A. ECG. [Here sudden anterior MI (as L1 and aVL and v2-v5,
unconciuosness without any provocation, which anterolateral would be more appropriate
makes arrhythmia the most likely cause; hence description)].
we should do ECG].
685. A schizophrenic says: life is unfair. I
like fairs. Fairs have food. It must be good.
What term describes this pt’s speech?
a. Neologism
b. Flight of ideas
Ans. The key is B. Hemophilia. [This is a
c. Broc1a’s aphasia
controversial question with description of both
d. Wernicke’s aphasia factor deficiency and platelet disorder!! Purpuric
e. Clang association rash on buttock does not support hemophilia and
other features like hemarthrosis in knee, isolated
raised APTT support hemophilia. Hopefully in
Ans. The key is E. Clang association. [The rhymic
PLAB this mixed picture will not be seen in a
use of words as described is known as clanging or
single question. Better to be with given key i.e.
clang association often seen in schizophrenia].
assuming the better option is B. Hemophilia].

686. A man comes to the ED with hx of


pulsatile swelling in the abdomen, he has 688. A 45yo woman presents with
hx of HTN and exam: pulse=120bpm, discharge from the left nipple. The
BP=70/40mmHg. He is restless and in discharge is brownish-green and foul
shock. smelling. What is the most likely dx?
What emergency management should be a. Duct papilloma
done on priority basis? b. Intra-ductal papilloma
a. Urgent abdominal CT c. Duct ectasia
b. Urgent abdominal US d. Mammary duct fistula
c. IV fluids 0.9% NS crystalloids to bring SBP e. Breast abscess
to 90mmHg
d. IV fluids 0.9% NS crystalloids to bring SBP
to 120mmHg Ans. The key is C. Duct ectasia. [Women near
e. Dopamine inj menopause are more affected by duct ectasia
characterized by brown, green or cheesy
discharge. This condition is harmless and tends to
Ans. The key is C. IV fluids 0.9% NS crystalloids to clear up without treatment].
bring SBP to 90mmHg [Probable ruptured aortic
aneurism. Immediate IV normal saline to raise the
BP to 90 mmHg to keep the vital organs perfused 689. A 10yo boy presents with generalized
till definitive measures are taken]. swelling. This has been present for 4days
and included swollen ankles and puffiness
of the face. It started a few days after he
687. A 5yo boy has cough and swelling at had a mild cold with runny nose. His only
the knee after falling on the ground with PMH was eczema. Urine analysis:
rashes on the buttocks which are non- hematuria, proteinuria 10g/24h, creat
blanching. PT=13, APTT=71, Hgb=11, 60umol/l and albumin=15g/l.
WBC=8, Plt=200. Choose the most likely What is the single most likely dx?
dx? a. IgA nephropathy
a. NAI b. HSP
b. Hemophilia c. Minimal change nephropathy
c. HSP d. Wilson’s disease
d. Osler weber reindu syndrome e. Cardiac failure
e. Von-Willebrand disease
Ans. The key is A. IgA nephropathy. [10 yr old boy, 692. A child with increasing jaundice and
history of URTI and hematuria points towards the pale stools. Choose the appropriate test:
diagnosis of IgA nephropathy. It may be present
with proteinuria and generalized swelling. The a. Endomyseal antibodies
important differentiating point from rapidly b. Sweat test
progressive GN is duration. IgA nephropathy <1o c. TFT
days (usually 4/5 days history of infection but in d. LFT
rapidly progressive GN history of infection for >10 e. US
days].

Ans. The key is E. US. [This is obstructive jaundice


690. A 28yo man complains of vertigo, nausea where US may show dilatation of common bile
and vomiting for more than 30 mins and tinnitus,
duct or stones].
hearing loss in the left ear. What is the tx for this
pt?
a. Buccal prochlorperazine (2nd line)
b. Metachlorpromide 693. A 73yo woman with RA is unable to
c. Cyclazine (1st line) extend the fingers of her right hand at the
d. Cotrimazole MCP joint and IP joints following a fall.
e. Ondansetron What is the single most likely tendon to
have been damaged?
a. Extensor carpi radialis
Ans. The key is C. Cyclizine. More appropriate is b. Extensor carpi ulnaris
A. Buccal prochlorperazine (patient.info). [Both c. Extensor digitorum
prochlorperazine and cyclizine can be used in d. Extensor indicis
Meniere’s disease]. e. Flexor digitorum profundum

Ans. The key is C. Extensor digitorum. [The


extensor digitorum extends the phalanges, then
691. A 16yo girl has been unwell for 5days the wrist, and finally the elbow. It acts principally
with malaise, headache and dry cough. on the proximal phalanges].
She has a few crackles in her chest. Her
CXR shows patchy consolidation in the
lower lobes. What is the single most likely 694. You are called to see a 20yo woman
causative organism? 2h post-LSCS. She has not passed urine
a. Cold agglutinins
since her operation. She denied any
b. Gram –ve diplococci in sputum
urinary symptoms preoperatively. Exam:
c. Gram +ve diplococcic in sputum
appears unwell, temp=37.5C,
d. Serology for legionella
BP=94/73mmHg, pulse=116bpm,
e. Sputum staining for mycobacterium TB
sat=97%. Her abdomen is distended with
tenderness in the left flank and suprapubic
region. Bowel sounds are not audible.
Ans. The key is A. Cold agglutinins. [Mycoplasma
Choose the most appropriate post C-
pneumonia, a form of atypical
bacterial pneumonia related to cold agglutinin section complication for this lady?
a. UTI
disease].
b. Urinary tract injury
c. Pleurisy
d. Acute pyelonephritis
e. Paralytic ileus

Ans. The key is B. Urinary tract injury.

695. A 58yo man has a headache and


confusion of 3 days after slipping and
hitting his head in the garden. What is the
most appropriate initial inv?
a. XR skull
b. XR face a. Anterior MI
c. CT brain b. Inferior MI
d. MRI brain c. Lateral MI
e. EEG d. Posterior MI
e. NSTEMI

Ans. The key is C. CT brain. [The likely diagnosis is


Ans. The key is B. Inferior MI. [There is
subdural haematoma for which appropriate initial
pathological Q wave and mild ST elevation in
investigation is CT scan of brain].
leads II, III and aVF].

696. A 4yo boy has a cough and arthritis 698. A young man has diarrhea, loss of
followed by rash on legs which are non- weight and flatulence for 3 days.
blanching on glass test. No hx of fever. What is the most imp tx?
PT=13, APTT=31, Hgb=12, WBC=6.5, a. Metronidazole
Plt=300. What’s the most likely dx? b. Fluconazole
a. Meningitis septicemia c. Vancomycin
b. Hemophilia d. Amoxicillin
c. HSP
Ans. The key is A. Metronidazole. [An incomplete
d. ITP
question!!]
e. TTP

699. A 6yo child presented with drooling


Ans. The key is C. HSP. [Usually occurs below 10 of saliva and severe stridor. He is febrile
yrs of age. The characteristic rash and lab findings and sick looking. XR Neck in extension
matches with HSP]. shows a thumb sign. Choose the single
most likely dx.
a. Croup
697. A 72yo man presents to the ED with b. URTI
chest pain. The following ECG was taken c. Diphtheria
What is the most likely dx? d. Acute epiglottitis
c. Carcinoma right bronchus
Ans. The key is D. Acute epiglottitis. [Drooling of d. Mesothelioma
saliva and stridor along with thumb sign in neck e. Pancoast tumor
X-ray are features of acute epiglottitis].

Ans. The key is E. Pancoast tumour. [Hoarseness


700. A mother presents with her 3yo son of voice is due to compression of the recurrent
who has indistinct nasal speech. He snores laryngeal nerve, ptosis due to compression of the
at night and has restless sleep. He is tired sympathetic ganglion, palpable mass in right
by day. What is the best management supraclavicular fossa due to involvement of the
strategy? supraclavicular lymph node. History of smoking
a. Arrange hearing test and given picture indicates the diagnosis of
b. Assess development milestones Pancoast tumour].
c. Refer to ENT surgeon
d. Refer to speech therapist 703. An 84yo man got surgical pain which
e. MRI brain
is well controlled by oral morphine 60mg
BD. However, now this pt is unable to
Ans. The key is C. Refer to ENT surgeon. [Probable swallow. What is the most appropriate
enlarged adenoid]. next step?
a. Morphine 60mg state
b. Morphine 60mg TDS
701. A 17yo boy while playing football got
c. Oxycodone 10mg OD
a kick and now he is complaining of severe d. Morphine 60mg IV
pain and swelling of the left side of his e. Fentanyl patches
scrotum. What inv is the most appropriate
to dx?
a. Needle aspiration of scrotum
Ans. The key is E. Fentanyl patches. [When dose
b. US scrotum
of oral morphine is known and rout should be
c. MSU
changed Phentanyl patch is adviced as the fixed
d. Surgical exploration of scrotum dose is known and patch can release the required
e. Urine test for hematuria dose for a given period (when we use patch we
can not change the dose). But if the pain control
is not optimal we should follow the next step (i.e.
Ans. The key is D. Surgcal exploration of scrotum.
parenteral morphine) as per pain ladder. But as
[This is a case of testicular torsion which needs
no correct dose of parenteral morphine is in the
urgent diagnostic and therapeutic surgical
options (iv morphine dose is one-third of oral
exploration of the scrotum].
morphinre) we have to go for fentanyl patches].

702. A 50yo man has had hoarseness of


704. A 19yo man has exercise induced
voice and drooping eyelid for 2m. a mass
asthma. This has prv been controlled
is palpable in the right supraclavicular
using a salbutamol inhaler as req, but he
fossa. He smokes 20 cigarettes/day for the
now gets attacks with exercise.
last 30yrs. What is the most likely dx?
a. Carcinoma larynx What is the single most appropriate tx?
a. Regular salbutamol
b. Carcinoma thyroid
b. Regular salbutamol and budesonide 707. A 57yo male presents with sudden
c. Sodium cromoglycate onset severe abdominal pain and rigidity
d. Oral steroid against a 4d background of LIF pain and
e. Inhaled steroid pyrexia. He has no PM/SHx of note and
isn’t on any meds. What is the most likely
Ans. The key is C. Sodium chromoglycate. This is dx?
wrong key! Correct key should be E. Inhaled a. Intussusception
steroid. [Chrommoglycate should be used in b. Ischemic colon
exercise induced asthma if inhaled steroid fails]. c. Sigmoid volvulus
d. Perforated diverticulum
e. Perforated Meckel’s diverticulum
705. A 3yo boy has a sudden onset of
fever, vomiting and bilateral face swelling.
Few days earlier the GP saw him for Ans. The key is D. Perforated diverticulum.
bilateral parotid pain and gave analgesics. [Sudden onset, severe abdominal pain, rigidity,
What is the most appropriate next step? left iliac fossa pain and fever are in favour of
a. Analgesic perforated diverticulum].
b. Antibiotic
c. Biopsy
d. Immediate surgery 708. A 46yo woman has weight gain,
e. Reassurance sensitivity to cold, pulse=50bpm, heart is
enlarged with murmur. What is the single
most likely dx?
a. Hypothyroidism
Ans. The key is E. Reassurance. [A case of mumps.
Self limiting condition]. b. Hyperthyroidism
c. Cushing’s syndrome
d. Addison’s disease11
e. Pheochromocytoma
706. A 75yo man with adenocarcinoma of
the prostate which has spread outside the
capsule of the gland has ARF. What is the
most appropriate next inv? Ans. The key is A. Hypothyroidism. [The given
a. MRI spine symptoms are classic presentation of
hypothyroidism].
b. Radionuclide bone scan
c. Trans rectal US
d. US pelvis
e. US KUB 709. An alcoholic who has completely
given up drinking hears voices. What is
the most appropriate tx?
a. Olanzapine
Ans. The key is E. US KUB. [extension beyond
b. Diazepam
capsule may cause obstruction of ureters, causing
loin pain, anuria, symptoms of acute kidney injury c. Acamprosate
or chronic kidney disease (here ARF)] ref: d. Disulfiram
patient.info e. Haloperidol
Ans. The key is A. Olanzapine. This is wrong key. apyrexial. What is the single most
Correct key is B. Diazepam. [A case of delirium appropriate diagnostic inv?
tremens. Treated with benzodiazepines like a. Blood culture
chlordiazepoxide or diazepam]. b. CRP
c. D-dimer
710. A 6yo boy has completed an d. XR knee
induction course of chemo for ALL. He has e. Serum uric acid
an enlarged left scrotum. What is the
most appropriate next step?
a. Herniotomy Ans. The key is E. Serum uric acid. [Thiazide
b. CT abdomen diuretics causes hyperuicemia which can
c. Biopsy precipitate acute attack of gout].
d. Immediate surgery
e. Reassurance
713. A 27yo woman with anxiety and
weight loss has tachycardia, tremor and
Ans. The key is B. CT abdomen.
mild proptosis. What single mechanism
[Secondaries/relapse may involve directly testis
accounts for her weight loss?
and other abdominal viscera. So CT abdomen is to
a. Deficiency in thyroid hormone
be done here].
b. Increased level of calcitonin
c. Increased metabolic rate
711. A 32yo miner is rescued after being d. Insulin resistance
trapped under a fallwn rock for 4h. After e. Reduced caloric intake
applying a bladder catether, 15-20ml of
reddish brown urine was obtained.
HR=120bpm, SBP=100mmHg. What would Ans. The key is C. Increased metabolic rate. [The
be the next appropriate step? given features are of thyrotoxicosis in which
a. Dopamine IV increased metabolism causes loss of patients
b. Fluid challenge weight].
c. Furosemide IV
d. 20% Mannitol IV
e. Antibiotics 714. A man with carcinoma and multiple
metastasis presents with intractable
nausea and vomiting. He has become
Ans. The key is B. Fluid challenge. [The diagnosis drowsy and confused.
is rhabdomyolysis. So IV fluid is the next What is the most appropriate
appropriate step]. management?
a. Dexamethasone IM
b. Dexamethasone PO
712. A 60yo man has had spontaneous c. Ondansetron IM
painful swelling of his right knee for d. Ondansetron PO
3days. 5days prv he had an inguinal e. Morphine oral
hernia repaired as a day case. He takes
bendroflumethiazide 2.5mg daily. He is
Ans. The key is C. Ondensatron IM. [For cancer or 717. In perforation of a post gastric ulcer,
chemotherapy induced vomiting ondensatron is where will the fluid accumulate in the
the drug of choice. As here vomiting is intractable peritoneal cavity?
IM ondensatron should be given not oral]. a. Left paracolic gutter
b. Pelvic cavity
c. Lesser sac
715. A 19yo man presents with weight d. Under the diaphragm
loss, increasing thirst and increasing e. Right paracolic gutter
frequency of going to the washroom. His
father, grandfather and 2 sisters have
been dx with DM. What is the most likely Ans. The key is C. Lesser sac.
type of DM this pt suffers from?
a. IDDM
b. NIDDM 718. A 62yo male is brought to the ED by
c. LADA
his daughter because of his persistent
d. MODY
lying. He is a known alcoholic and has
e. DKA
been admitted recently with delirium
Ans. The key is D. MODY. [Key features of MODY tremens. On questioning, he denies any
are: Being diagnosed with diabetes under the age problem with memory. He knows his
of 25 and having a parent with diabetes, with name and address and states that was at
diabetes in two or more generations]. the betting shop this morning, but his
daughter interjects calling him a liar
explaining that he was at her home.
716. A 42yo woman with a PMH of severe What is the most likely dx?
headache treated in the ED presents with a. Ganser syndrome
signs and symptoms of renal failure. She b. Cotard syndrome
has been seen by her GP for HTN and c. Wernicke’s encephalopathy
abdominal pain with OP inv pending. d. Korsakoff psychosis
Which inv is most likely to lead to a dx? e. Alcohol withdrawal
a. US KUB
b. CT brain
c. IVU Ans. The key is D. Korsakoff psychosis. [In
d. Renal artery Doppler Korsakoff psychosis there is confabulation (a
e. Renal biopsy memory disturbance, defined as the production
of fabricated, distorted or misinterpreted
memories about oneself or the world, without
Ans. The key is A. US KUB. [Hypertension, the conscious intention to deceive) which may
abdominal pain and features of renal failure present like this].
indicates the diagnosis of ADPKD for which the
diagnosis is best made by US KUB].
719. A 70yo man presented with muscle
weakness and inability to climb the stairs.
Inv: CPK raised, ESR 15. What is the most
likely dx?
a. Polymyositis 722. A 72yo man has been on warfarin for
b. Polymyalgia rheumatic 2yrs because of past TIAs and strokes.
c. Reactive arthritis What is the most imp complication the pt
d. RS should be careful with?
e. Duchenne’s MD a. Headache
b. Osteoporosis
c. Ear infection
Ans. The key is A. Polymyositis. [Proximal muscle
weakness (inability to climb the stairs) and raised d. Limb ischemia
CPK points towards polymyositis]. e. Diarrhea

720. A 65yo known alcoholic is brought Ans. The key is B. Osteoporosis. It is a wrong key.
into the hospital with confusion, The correct option is A. Headache. [Headache
aggressiveness and ophthalmoplegia. He from intracranial hemorrhage is more important
is treated with diazepoxide. complication about which patient should be
What other drug would you like to careful with (it is more important than
prescribe? osteoporosis)].
a. Antibiotics
b. Glucose
723. A 24yo woman is afraid to leave her
c. IV fluids
house as whenever she goes out, she
d. Disulfiram
tends to have SOB and sweating. She has
e. Vit B complex
stopped going out except with her
husband.
What is the most likely dx?
Ans. The key is E. Vit B complex. [This is a case of
a. Social phobia
Wernicke’s encephalopathy which is treated with
b. Claustrophobia
Vit B complex].
c. Depression
d. Panic disorder
721. A pt suffering from schizophrenia e. Agoraphobia
laughs while talking about his father’s
death. Which term best describes his
Ans. The key is E. Agoraphobia. [Agoraphobia:
condition?
Extreme or irrational fear of open or public
a. Depression
places].
b. Flat affect
c. Emotional liability
d. Incongruent affect 724. A pt on HTN drugs develops
e. Clang association hyperkalemia. Which anti-HTN is likely to
cause it?
a. Ramipril
Ans. The key is D. Incongruent affect. b. Lorsartan
[Incongruent affect means inappropriate c. Thiazide
emotional response like loughing hearing a sad d. Nifedipine
news or crying hearing a good news]. e. Furosemide
Ans. The key is A. Ramipril. [Both ramipril and Ans. The key is A. Stop smoking.
losartan can cause hyperkalemia].

725. A young man develops itching worse 728. A 64yo male was admitted to the
at night and following bathing. Exam: medical ward with complaint of diarrhea,
greysish white linear rash can be seen on abdominal pain and weight loss for few
the wrist and periumbilical area. What is months. Exam: clubbing, perianal skin
the dx? tags and abdominal tenderness.
a. Scabies Colonscopy reveals transmural
b. Polycythemia granulomatous inflammation involving
c. Urticarial ileocecal junction. He was dx with what?
d. Atopic eczema a. CD
e. Lichen planus b. UC
c. Bowel cancer
d. Gastric cancer
Ans. The key is A. Scabies. e. IBD

726. A 40yo lady who has been a smoker


since she was a teenager has the Ans. The key is A. CD. [The given picture is typical
following blood result: Hgb=19. What of Crohn’s disease].
hormone should you check?
a. Aldosterone
b. Cortisol 729. A pt presents with hemoptysis 7d
c. Erythropoietin after tonsillectomy. What is the next step?
d. T4 a. Packing
e. TSH b. Oral antibiotics and discharge
c. Admit and IV antibiotics
d. Return to OT and explore
Ans. The key is C. Erythropoetin. [Smoking causes e. Ice cream and cold fluids
raised carboxyhemoglobin level causing
hypoxemia and raised erythropoeitin which leads
to raised hemoglobin level (secondary Ans. The key is C. Admit and IV antibiotics.
polycythemia)]. [Secondary hemorrhage can occur from sloughing
of tissue from surgical wound following infection].

727. A 25yo man presents with


hoarseness of voice. He has swollen vocal
cords. His BMI=32 and he smokes 20-25
cigarettes/day. What would you advise
730. A 55yo man presents with HTN. He
him?
complains of headache and visual
a. Stop smoking
b. Lose weight
disturbances. He also reports itching after
a hot bath and burning sensation in finger
and toes. His face is flushed red. PE: mild
splenomegaly. Inv: Hgb=20g/dl, WBC=20,
plt=500, EPO normal. What is the likely 733. A woman is sad, fatigues and she is
dx? eating more and also has sleeping
a. Myelofibrosis disturbance and hears the
b. Polycythemia rubra vera voice of her husband who died 3yrs ago.
c. Essential thrombocythemia What is the dx?
d. CML a. OCD
e. CLL b. Psychotic depression
c. Grieving
d. Severe depression
Ans. The key is B. Polycythemia rubra vera.
[Raised hemglobin, raised cell counts and normal
erythropoeitine along with symptoms of Ans. The key is B. Psychotic depression. [Here
hyperviscosity like headache and associated features of atypical depression along with
hypertension are diagnostic of polycythemia hallucination makes the likely diagnosis to be
rubra vera]. psychotic depression].

731. An old man having T2DM with 734. A 40yo teetotaler woman is
increased skin tanning, heart failure and recovering from a hysterectomy 2days
having high ferritin (hemochromatosis) ago. At night she becomes agitated and
level is refusing tx. Where is the first site complains of seeing animals and children
of cancer? walking around the ward. What is the
a. Testes most likely dx?
b. Adrenal a. Delirium tremens
c. Liver b. Toxic confusional state
d. Pancreas c. Hysteria
d. Mania
e. Drug induced personality disorder
Ans. The key is C. Liver.

Ans. The key is B. Toxic confusional state. [This is


732. A 60yo DM lady presents with severe not delirium tremens as the womean is
peri-anal pain and swelling. What’s the teetotaler].
cause?
a. Anal carcinoma
b. Anal fissure 735. A woman with a hx of drug abuse
c. Hemorrhoid and increased alcohol intake, now comes
d. Anal abscess for help and she is concerned about her
problem.
What is the most appropriate
Ans. The key is D. Anal abscess. management option?
a. Voluntary admission
b. Psychiatry team
c. Mental health team
d. Psychiatry voluntary admission
Ans. The key is A. Cataract. [glaucoma may have
dilated oval pupil].
Ans. The key is B. Psychiatry team. This is
probably a wrong key. Correct option should be C.
Mental health team. [Drug abuse and alcohol
intake has psychological issue also and mental 738. A 53yo man was admitted to the
health team can give broader aspect of support hospital for inv of hemoptysis. 3 days after
than a psychiatry team. Mental Health Team: admission he developed alternating state
"These can include psychiatrists, psychologists, of consciousness, ataxic gait and some
community psychiatric nurses, social workers, visual problems. What is the most
and occupational therapists. They work with you appropriate management of this pt?
to treat your mental health. You might get help a. Acamprosate
from the team as a whole or from just one or two b. Chlordiazepoxide
professionals. Your GP or primary care service has c. Diazepam
to refer you to the MHT]." d. High potent vitamins
e. Disulfiram

736. A 28yo woman who is 32 wks


pregnant in her 3rd pregnancy is Ans. The key is D. High potent vitamins. [This is a
diagnosed as a case of placental case of Wernicke’s encephalopathy (as hospital
abruption. After all the effective admission prevented him from consuming
measures, she is still bleeding. alcohol) and the treatment for this is high potent
What is the underlying pathology? vitamins].
a. Clotting factor problem
b. Clauser’s syndrome
c. Platelet problem 739. A pt underwent hip surgery. Later he
d. Succiturate lobe presents with SOB and chest pain. What is
e. Villamentous insertion of placenta the dx?
a. Pulmonary embolism
b. MI
Ans. The key is A. Clotting factor problem. c. Tension pneumothorax
d. Fat embolism
e. None
737. An old woman having decreased
vision can’t see properly at night. She has
changed her glasses quite a few times but Ans. The key is A. Pulmonary embolism.
with no effect. She has normal pupils and
cornea.
What is the most likely dx? 740. A 25yo man presents with hx of
a. Cataract breathlessness. A transthoracic echo
b. Glaucoma reveals a patent foramen ovale. What
c. Retinal detachment diagnostic inv would you do for a patent
d. Iritis foramen ovale?
e. GCA a. Transesophageal echo
b. Bubble echo
c. Transthoracic echo
d. ECG
743. An 8yo boy dx with asthma is on
salbutamol and beclomethasone.
Ans. The key is B. Bubble echo. [Bubble echo is
actually extension of transoesophageal echo in
However, he wakes up at night due to his
that here additional bubbles are added during symptoms. What is the next appropriate
transoesophageal echo to get better visualization management?
of foramen ovale]. a. LABA
b. High dose steroid
c. Aminophylline
741. A 25yo woman with a hx of several d. Oral prednisolone
episodes of depression is brought to the e. Sodium cromoglycate
ED after she was found with several empty
bottles of her meds. She complains of
coarse tremor, nausea and vomiting. Ans. The key is A. LABA. [Patient is in step2 with
Which of the following drugs is likely to poor control. So next step is to add LABA and if
have caused her symptoms? still not controlled give high dose inhaled
a. Fluoxetine corticosteroids].
b. Amitryptilline
c. Lithium
d. Phenelzine 744. A woman presents with a hx of
e. Olanzapine poisoning 10x with different substances.
There are no obvious signs of depression
Ans. The key is C. Lithium. [Side effect of lithium is or suicidal behavior. What is the best
fine tremor (in therapeutic dose) but lithium preventive step?
toxicity or lithium poisoning causes coarse a. Open access to ED
tremor. Coarse tremor, nausea and vomiting are b. 24h help line
well known feature of lithium overdose]. c. CBT
d. Anti-depressants
e. Insight into problem
742. A 23yo man feels anxious and
agitated when faced with stress. He has
an interview in 3days and would like some
Ans. The key is E. Insight into problem. [Patient is
help in relieving his symptoms.
not depressed and there is no suicidal behaviour.
What is the most appropriate Despite repeated poisoning may indicate she is
management? facing some stress and so insight into her
a. SSRI problem should be sought for].
b. CBT
c. Propranolol
d. Diazepam 745. A boy was rushed to the ED
unconscious after he had taken
Ans. The key is C. Propranolol. [Inappropriate methadone belonging to the sister. He
anxiety during interview is performance phobia was given naloxone and he regained
which is better helped by propranolol when help consciousness. After a while he started
needed for short term like here as 3 days]. getting drowsy again. What is responsible
for his present drop in level of 747. A 32yo lady has recently become
consciousness? more active, sleeps less and bought a
a. Naloxone is absorbed faster than house and 2 new cars. What is the most
methadone likely dx?
b. Methadone is absorbed faster than a. Bipolar disorder
naloxone
b. Mania
c. He has also taken another substance
apart from methadone c. Hypomania
d. The methadone had already caused some d. Schizophrenia
brain damage
e. Naloxone is eliminated faster than
methadone Ans. The key is C. Hypomania. [Individuals in a
hypomanic state have a decreased need for sleep,
Ans. The key is E. Naloxone is eliminated faster are extremely outgoing and competitive, have a
than methadone. great deal of energy and are otherwise often fully
functioning (unlike full mania where unlike fully
functioning there is a. obvious difficulties at work
746. A 24yo male on remand in prison for or in social relationships and activities b. requires
admission to hospital to protect the person or
murder is referred by the prison doctor.
others, or c. the person is suffering psychosis)].
He is noted to be behaving oddly whilst in
prison and complains of seeing things. He
has a prv hx of IV drug abuse. On
748. The body of a 65yo man who was
questioning he provides inappropriate but
treated for TB and bronchitis was seen at
approximate answers to all questions
autopsy. His legs were swollen and his
stating that Bill Clinton is the prime
liver showed signs of a transudate fluid.
minister of England.
What was the cause of the transudate?
What is the prisoner suffering from? a. Liver cirrhosis
a. Capgras syndrome
b. Alcoholic liver disease
b. Cotard syndrome
c. Cardiac failure
c. Ganser syndrome
d. Budd-chiari syndrome
d. Ekbom syndrome
e. TB
e. Tourette’s syndrome

Ans. The key is C. Cardiac failure.

Ans. The key is C. Ganser syndrome. [Ganser


syndrome is characterized by nonsensical or 749. A 60yo woman has tiredness. She has
wrong answers to questions or doing things noticed that her skin looks permenantly
incorrectly often with visual pseudohallucinations tanned and she describes dizziness on
and a decreased state of consciousness. It is also standing up. What is the single most likely
sometimes called prison psychosis, because the
electrolyte pattern to be found?
syndrome occurs most frequently in prison
a. Na+=120mmol/L, K+=5.9mmol/L
inmates, where it may represent an attempt to
b. Na+=125mmol/L, K+=2.9mmol/L
gain leniency from prison or court officials].
c. Na+=140mmol/L, K+=4.5mmol/L
d. Na+=150mmol/L, K+=3.5mmol/L
e. Na+=150mmol/L, K+=5.9mmol/L 752. A 17yo lady presents with a worm in
her ear. She is very agitated and anxious.
What is the next step?
Ans. The key is A. Na+=120mmol/L, a. Remove under GA
K+=5.9mmol/L. [Tanned skin (hyperpigmentation) b. Suction
and postural hypotension in a tired women points c. Alcohol drops
towards Addison’s disease where hyponatremia d. Forceps
and hyperkalemia is seen].

Ans. The key is C. Alcohol drops. [It is a living


750. A 20yo girl with amenorrhea and BMI object. So it should be killed first and so alcohol
of 14 still thinks she has to lose weight. drop is the correct option].
What is the most likely dx?
a. Anorexia nervosa
b. Bulimia nervosa 753. A 20yo male smoker is noted to have
c. OCD intense rubor of the feet and absent foot
d. Depression pulse. Exam: amputated right 2nd toe.
e. Body dysmorphic disorder What is the most probable dx?
a. Intermittent claudication
b. Cardiovascular syphilis
Ans. The key is A. Anorexia nervosa. c. Buerger’s disease
d. Embolism
e. Acute limb ischemia
***751. A guy who has several convictions
and has been imprisoned several times,
breaks up with his family and doesn’t Ans. The key is C. Buerger’s disease. [ The
contact his children. traditional diagnosis of Buerger's disease is based
What type of personality disorder is this? on 5 criteria (smoking history, onset before the
a. Borderline age of 50 years, infrapopliteal arterial occlusive
b. Antisocial disease, either upper limb involvement or
c. Schizotypal phlebitis migrans, and absence of atherosclerotic
d. Schizoid risk factors other than smoking). As there is no
e. Criminal specific diagnostic test and an absence of positive
serologic markers, confident clinical diagnosis
should be made only when all these 5 criteria
have been fulfilled].
Ans. The key is B. Antisocial. [Antisocial
personality disorder is characterized by a
pervasive pattern of disregard for, or violation of, 754. A young lady after a heavy bout of
the rights of others. There may be an drinking last night comes to the ED with
impoverished moral sense or conscience and a
dizziness, abdominal pain, vomiting blood
history of crime, legal problems, and impulsive
with cool peripheries. After initial
and aggressive behavior].
resuscitation, oxygen and fluids, she still
continues to bleed with pulse=130bpm
and BP=85/58mmHg. What would be your Ans. The key is B. 24h BP monitoring. [All these
next best management? drugs (particularly prolonged use of thiazide and
a. Clotting screen amlodipine) are well established cause of postural
b. US hypotension. So 24 hour BP monitoring should be
c. CT done].
d. Endoscopy 757. A 43yo woman has been feeling
e. Omeprazole lethargic and tired. Her BP=160/90mmHg.
Bloods:
Na+=140mmol/L, K+=3.1mmol/L. What is
Ans. The key is D. Endoscopy. [In young it is the most likely dx?
unlikely to get oesophageal varices rather than a. Cushing’s syndrome
the bleeding is from probable gastric erosion and b. Conn’s syndrome
endoscopy should be done (diagnostic and
c. Hyperparathyroidism
therapeutic- like sclerotherapy].
d. Renal disease
e. Pheochromocytoma
755. A 12yo boy complains of acute
Ans. The key is B. Conn’s syndrome. [It can also
development of purpura on the dependent be cushing’s syndrome! So if we like to be sure
areas of his body 2wks after a URTI. The we have to know aldosterone and rennin level. i)
purpura doesn’t blanch on pressure, tests Renin low, aldosterone high- Conn’s syndrome
reveal: Hgb=12, plts=50, WBC=5. Bleeding (primary hyperaldosteronism) ii) Renin low,
time=10mins, APTT=40s, PT=1.02. aldosterone low- Cushing’s syndrome iii) Renin
What is the most likely dx? high, aldosterone high- Renovascular disease
a. ITP (secondary hyperaldosteronism)].
b. TTP
c. Von Willebrand’s disease
d. Hemophilia A 758. A 2yo child aspirated a foreign object
e. Hemophilia B which was removed at the hosp. the
parents are now asking how to remove it
if that ever happens at home. What do
Ans. The key is A. ITP. [Isolated thrombocytopenia you advise?
and H/O prior URTI with development of purpura a. Hemlich maneuver
on the dependent areas of the body favours the b. Bring to the hospital
diagnosis of ITP. (In ITP BT is prolonged which is c. Turn the child on his back and give
present here.)]. thumps
d. CPR
756. A woman presents with hx of falls,
e. Remove manually by fingers
becomes pale and clumsy. She is
hypertensive and takes atenolol,
bendroflumethiazide and amlodipine.
Ans. The key is C. Turn the child on his back and
What inv is needed?
give thumps.
a. 24h ECG
b. 24h BP monitoring
c. ECG
d. Echo 759. A 28yo pregnant lady presents with
e. CT head severe lower abdominal pain with
excessive per vaginal bleeding at 34wks in therapeutic range is fine tremor. Sodium
gestation. What should be the initial inv of valproate (a well known mood stabilizer) can
choice? cause coarse tremor in therapeutic range].
a. Coagulation profile
b. US abdomenu
c. CT pelvis 762. A 38yo man has had a liver biopsy as
d. D-dimer part of inv for abnormal LFTs. The
e. Kleiuber test pathologist report states: special stains
demonstrate the presence of a very large
amount of iron pigment within the
Ans. The key is B. US abdomen. [Likely case of hepatocytes. What single condition is
abruption placenta (as excessive bleeding identified by the pathologists report?
probably revealed type) for which initial a. Alpha 1 antitrypsin deficiency
investigation of choice is US abdomen. Other b. Hemangioma
features that will favour abruptio is hard uterus c. Hemochromatosis
and evidence of fetal distress. It will be difficult to d. Hemosiderosis
feel the fetal parts]. e. Wilson’s disease
760. A 3yo child with severe diarrhea and
vomiting, looks lethargic, has sunken eyes
Ans. The key is D. Hemosiderosis. It is a wrong
and a feeble cry. What is the choice of
key! The correct key should be C.
fluids?
Hemochromatosis. [In hemochromatosis
a. 0.9%NS
distribution pattern of iron in liver biopsy by
b. 0.9%NS + 5%Dextrose
pearl’s stain is more prominent iron granules in
c. 0.45%NS + 5%Dextrose
periportal hepatocyte and relative sparing of
d. 0.45%NS kupffer cells and decreased intensity near the
central vein. By contrast iron overload in
Ans. The key is A. 0.9% NS. [Patient has features hemosiderosis causes to accumulation of iron
of severe dehydration and should be resuscitated granules predominantly in kupffer cells and more
first and for resuscitation fluid of choice is 0.9% in central area rather than peripheral hepatocyte.
NS]. In given case there are large amount of iron
pigment in hepatocyte which is diagnostic of
hemochromatosis].
761. A lady with depression has a bag full
of meds. She now presents with coarse
763. A 27yo man presents to the ED after
tremors. Which drug caused her
an RTA where his foot was stuck under a
symptoms?
a. Lithium truck for several hours. He now has
b. Thyroxine swelling on the foot. Exam: foot
c. Amitriptyline tenderness, loss of sense in the space
d. Sodium valproate between the 3rd metatarsal and big toe
e. Tetrabenazine and his dorsalis pedis is not felt. What is
the most likely dx?
a. Compartment syndrome
b. Arterial rupture
Ans. The key is A. Lithium. It is a wrong key! Right
c. Arterial embolus
key is D. Sodium valproate. [Side effect of lithium
d. DVT d. Take antiacids
e. Fibular fx e. Take antiemetic

Ans. The key is A. Compartment syndrome. Ans. The key is A. Take it after meal.
[Acute compartment syndrome occurs after a
traumatic injury such as a car crash. The trauma
causes a severe high pressure in
766. A 52yo man with hx of ant MI 3 wks
the compartment which results in ago developed sudden onset of dyspnea.
insufficient blood supply to muscles and nerves. Exam:
Acute compartment syndrome is a medical BP=100/60mmHg, pulse=100bpm,
emergency that requires surgery to correct. If SaO2=88%, chest=bilateral crackles.
untreated, the lack of blood supply leads to Which inv would you do to find the
permanent muscle and nerve damage and can underlying cause?
result in the loss of function of the limb]. a. CXR
b. Echo
c. D-dimer
764. You are a FY doctor in the ED when a d. Radionucleotide lung scan
mother brings her 2yo son to you with a e. Troponin
1h hx of noisy breathing. She state that
although he had mild coryza over the last
week, he was improving and so they had Ans. The key is A. CXR. It is wrong key. Correct key
gone to a children’s picnic with nursery should be B. Echo. [The question does not want
friends. Another parent had found him to know the features of pulmonary oedema
coughing and spluttering, and ever since rather it is clearly mentioned “which investigation
his breathing has remained noisy. Though would you do to find out the UNDERLYING
he appears well in the ED, his current CAUSE”. As in this patient sudden papillary
muscle rupture or VSD may be the likely cause
observations demonstrate a raised RR and
(ventricular aneurism may take a more slower
sat=91% on air. What is the most likely course) so echo seems to be more logical
dx? approach].
a. Anaplyaxis
b. Croup
c. Foreign body aspiration
767. A 49yo man lost his job and now is
d. Epiglottitis
homeless. He was found wandering in the
park. He is muttering that some people
are after him. Alcohol was tested and it
Ans. The key is C. Foreign body aspiration.
was negative. What will your next step
be?
a. Thiamine
765. A pt taking doxycycline complains of b. Neuropsycho analysis
nausea, indigestion, abdominal pain and c. Mini mental state
vomiting. What will you advise? d. CT head
a. Take it after meals
e. MRI head
b. Take it before meals
c. Stop the drug
Ans. The key is B. Neuropsychoanalysis. [As the 770. A 45yo woman presents with
patient has persecutory delusion he is likely rotational vertigo, nausea and vomiting,
suffering from psychosis for which especially on moving her head. She also
neuropsychoanalysis is the logical approach].
had a similar episode 2yrs back. These
episodes typically follow an event of runny
nose, cold, cough and fever. What is the
768. A pt with SNHL and loss of corneal
most probable dx?
reflex on the left side. What is the most a. Acoustic neuroma
definitive inv? b. Meniere’s disease
c. Labyrinthitis
a. CT of internal auditory meatus
d. BPPV
b. Nuclear imaging of brain
e. Vestibular neuronitis
c. MRI of internal auditory meatus
d. Radio isotope scan
e. XR skull
Ans. The key is E. Vestibular neuronitis. [Here no
sensoryneural hearing loss (SNHL present in
acoustic neuroma, Meniere’s disease and
Ans. The key is C. MRI of internal auditory
labyrinthitis). Runny nose, cold, cough and fever
meatus. [The likely cause is acaustic neuroma for
are recognized trigger of vestibular neuronitis but
which appropriate investigation is MRI internal
not BPPV].
acoustic meatus].

771. A 65yo man with cancer of middle


769. A middle aged man complains of a
1/3 of the esophagus presents with
node which has been growing on his nose
dysphagia. What is the most immediate
for several months. Now it’s firm with
management?
central depression. It is 0.6cm in size. a. Chemotherapy
What is the single most likely dx? b. Radiotherapy
a. Basal cell carcinoma
c. Stenting
b. Squamous cell carcinoma
d. Gastrostomy
c. LN
e. TPN
d. Melanoma
e. Kaposi’s sarcoma
f. Keratoacanthoma
Ans. The key is C. Stenting.
g. Molluscum contagiosum

772. A 1yo child is brought to ED. He woke


Ans. The key is A. Basal cell carcinoma. [A pearly
white umbilicated ulcer (with central depression)
up in the middle of the night crying
any where in the face with rolled edges / inverted severely. What initial measure should be
edge is basal cell carcinoma. Any ulcer which is taken for this child?
located above the neck is always basal cell a. Refer to surgeon
carcinoma until proven otherwise. Derma Samson b. Discharge with advice
note]. c. Analgesia
d. Antibiotic
b. Calcium resonium
c. Calcium with vit D supplement
Ans. The key is C. Analgesia.
d. Vit D
e. Calcium

773. A 30yo lady was playing volleyball


when her hand got injured with the ball. Ans. A. Calcium gluconate. [ECG change of
The right hand is not swollen and there is prolonged QRS complex and tented T wave are
tenderness under the root of the thumb. suggestive of hyperkalaemia which is further
XR is normal. What is the most supported by K+ level of 6.2 mmol/l. So to protect
appropriate next management? the heart from arrhythmia calcium gluconate
a. Arm sling for 1 wk should be given].
b. Raise hand for 2d
c. Repeat XR 776. A 68yo man with DM and HTN was
d. Full arm cast noted to have cholesterol level of
3.4mmol/l. he was also noted to have
microalbuminuria. What is the best drug
Ans. The key is C. Repeat XR. [Likely diagnosis is to add to his regimen?
scaphoid fracture where X-ray may not show the a. ACEi
fracture right away. In such case put her hand in a b. Statin
below elbow cast and repeat X-ray in a week or c. Amlodipine
two to see if the fracture become visible. Usually d. Biguanides
it becomes visible as a healing fracture line].

Ans. The key is A. ACEI. [ACEI is renoprotective


774. A 33yo female presents with sudden and prevents progress in albuminuria].
severe colicky abdominal pain in her RIF. A
mobile mass is felt on examination. What
is the most likely dx? 777. A child playing with toys suddenly
a. PID develops breathlessness and stridor.
b. Appendicitis Which inv will lead to the dx?
c. Ovarian torsion a. Laryngoscopy
d. Constipation b. CXR
c. Peak flow meter
d. ABG
Ans. The key is C. Ovarian torsion.

Ans. The key is A. Laryngoscopy. [Breathlessness


775. A middle aged male is feeling unwell and stridor in a child playing with toy is most
likely due to aspiration of foreign body (toy or
after a recent MI. The recent ECG shows
part of toy) for which laryngoscopy should be
prolonged QRS complex and tented T
done].
wave. Na+=136mmol/l, K+=6.2mmol/l,
urea=5mmol/l. What is the most
appropriate management?
a. Calcium gluconate
778. Anxious parents ask you for resus Ans. The key is B. Asthma. [Here post
technique for their 3yo. What do you tell bronchodilator value is >80% which excludes
COPD and reversibility is >23% which is diagnostic
them?
of asthma].
a. 5 compression: 1 breath
b. 5 compression: 2 breaths
c. 15 compression: 2 breaths with nose 781. A 2yo male pt was brought by his
pinched mother with a swelling in the right side of
d. 15 compression: 2 breaths without nose his neck extending from the angle of the
pinched mouth to the middle 1/3 of the
e. 30 breaths: 2 compressions sternocleidomastoid muscle. The swelling
was on the anterolateral side of the
sternocleidomastoid and was brilliantly
Ans. The key is C. 15 compression:2 breaths with transilluminant. What is the likely dx?
nose pinched. a. Lymphangioma
b. Branchial cyst
c. Thyroglossal cyst
779. A 56yo man complains of increased d. Ranula
vol of sputum with specks of blood and e. Thyroid swelling
chest pain. He has a hx of DVT. Exam:
clubbing. What is the cause of blood in his
sputum? Ans. The key is B. Branchial cyst. It is a wrong key.
a. Pulmonary thrombosis Correct key is A. Lymphangioma. [Both
b. Bronchial carcinoma lymphangioma and brancheal cyst are lateral
c. Bronchiectasis neck mass. 90% of lymphangioma occur in
d. Pulmonary TB children less than 2 yrs. Brancheal cyst usually
does not transilluminate whereas lymphangioma
usually transilluminate brilliantly].
Ans. The key is C. Bronhiectasis. [Increased
volume of blood streaked sputum and clubbing
points towards bronchiectasis].
782. A 50yo newly dx with HTN complains
of urinary freq and dysuria. The urinalysis
780. A 32yo female has a hx of SOB and reveals presence of white cells and
fever. Pre-broncho-dilation test was done protein. Choose the single most
and it was 2/3.5 and post-bronchodilator appropriate tx?
was 3/3.7. The pt was dx of eczema and a. Imipramine
TB. b. Adjust diuretics
What is the possible dx? c. Vaginal estrogen
a. COPD d. Trimethoprim
b. Asthma
c. Pneumonia
d. Bronchiectasis Ans. The key is D. Trimethoprim. [UTI. Treatment
is with triethoprim].
783. A boy injured his ear during a rugby a. Fam hx
match. He reported it being painful. Exam: b. Tumor grading
red and tender pinna. Tympanic c. LN involvement
membrane was normal. What would be d. Her age
the next appropriate step?
a. Topical gentamicin
Ans. The key is D. Her age.
b. Oral flucloxacillin
c. IV flucloxacillin
d. Refer to ENT specialist 786. A 45yo man presents with hearing
e. No further intervention needed. loss and tinnitus in the right ear. Exam:
weber test lateralizes to the left.
Ans. The key is E. No further intervention needed. Audiometry: AC > BC in both ears.
[As tympanic membrane is normal so this What is the next best inv?
transient inflammation of pinna from injury a. CT
during rugby match is self limiting which needs no
b. MRI brain
further intervention].
c. Angiogram
d. Otoscopy
784. A 2yo girl prv well presents with a hx
of vomiting and diarrhea for 4hrs. What is
Ans. The key is B. MRI brain. [AC>BC indicate
the most suitable indication for IV fluid Rinne positive; i.e. The deafness is not
administration? conductive. Again hearing loss is on right side.
a. Capillary refill time >4s Weber lateralized to left. We know weber
b. HR >90bpm lateralized to same side if conductive deafness
c. Increased RR and to opposite if there is sensoryneural
d. Stool >10x/d deafness. So it is now obvious that the deafness
e. Weight of child = 10kgs of right ear is sensoryneural deafness for which
MRI brain is the next best investigation].

Ans. The key is A. Capillary refill time >4s.


787. A 74yo lady called an ambulance for
[Capillary refill time >4s is sign of severe
an acute chest pain. She has a hx of DM
dehydration indicating need for IV fluid
administration. Features of severe dehydration: i)
and HTN, and is a heavy smoker.
abnormally sleepy or lethargic ii) sunken eyes iii) Paramedics mentioned that she was
drinking poorly or not at all iv) reduced skin turgr overweight and recently immobile
> 2 seconds v) dry tongue and mucous membrane because of a hip pain. She collapsed and
vi) absent tears vii) oliguria viii) hypotension ix) died in the ambulance.
tachycardia x) prolonged capillary refill time xi) What is the most likely cause of death?
depressed anterior frontanelle]. a. Pulmonary embolism
b. MI
c. Stroke
*785. A 44yo woman with breast cancer
d. Cardiac arrhythmia
had an extensive removal and LN
e. Cardiac failure
clearance. She needs an adjunctive tx. Her
mother had cancer when she was 65.
Which of the following factors will be
against the tx?
Ans. The key is A. Pulmonary embolism. a. MSU
[Immobilisation for hip pain is the clincher of b. US abdomen
pulmonary embolism here]. c. US KUB
d. Serum calcium
788. A 77yo woman suffered diarrhea 4d
ago. What would be her blood gas result?
Ans. The key is B. US abdomen. [C. US KUB. Is also
a. pH =7.2, PaCO2 =8 correct! This is a case of POUR (post operative
b. pH =7.4, PaCO2 =1.5 urinary retention). We should do US to know the
c. pH =7.4, PaCO2 =2.6 urinary volume (urine volume in bladder). If
d. pH =7.4, PaCO2 =2.8 <400ml we shall observe. When volume is >600ml
we should do catheterization].

Ans. The key is B. pH = 7.4, PaCO2 = 1.5 792. A female pt with HTN and serum
K+=2.7mmol/l. which hormone would you
789. A man presents with abdominal pain, be looking for?
vomiting, pulse=120bpm, BP=90/60mmHg
a. Aldosterone
and a rigid abdomen. His chest is clear. b. Cortisol
What is the immediate management? c. Thyrotoxin
a. Call radiology
d. Renin
b. Admit to medical ward
e. Testosterone
c. Urgent admission to ITU (Intensive
Therapy Unit)

Ans. The key is A. Aldosterone. [Hpt and


hypokalemia is seen in hyperaldosteronism. So
Ans. The key is C. Urgent admission to ITU.
we should look for aldosterone].

790. A 60yo man presents with weight 793. A 25yo male with a hx of frequent
loss and Hgb=6. Hx reveals that he has binge drinking presents 4h after having
abdominal pain and diarrhea for the past had a take away meal following a nights
6m. What is the most appropriate inv? heavy drinking. He complains of nausea
a. Barium enema
and has vomited on several occasions.
b. Colonoscopy
After the last vomiting episode, he
c. Sigmoidoscopy
vomited approximately a cupful of blood.
On admission, he smells of alcohol,
Ans. The key is B. Colonoscopy. [Likely diagnosis is pulse=100bpm, BP=140/80mmHg. He has
IBD. So Colonoscopy should be done]. some tenderness in the epigastrium.
What is your dx?
a. Gastric carcinoma
791. A 35yo primigravida post C-section
b. Mallory-weiss tear
complains of inability to void. She denies
c. Esophageal carcinoma
dysuria but complains of fullness. She was
d. Esophageal varices
treated with an epidural for analgesia.
e. Esophageal varices
What is the single most appropriate inv?
f. Peptic ulceration 48. What is the most appropriate
management for this pt?
a. IV fluid
Ans. The key is Mallory-weiss tear. [Repeated b. Needle thoracocentesis
retching and vomiting is a recognized cause of c. 100% oxygen
Mallory-weiss tear which is supported here by d. Portable XR
vomiting a cupful of blood].

Ans. The key is 100% oxygen.


794. A young boy presented with bilateral
periorbital edema, ankle swelling and
increase in body weight. What is the most 797. A 44yo lady who has PCKD is
likely dx? concerned because her 38yo brother has
a. Chronic heart failure just died of an intracranial insult. She
b. Nephrotic syndrome knows he was not hypertensive.
c. Renal failure What was the most likely cause of her
d. Acute heart failure brother’s death?
e. Glomerulonephritis a. Subdural hematoma
b. Subarachnoid hemorrhage
c. Cerebral infarct
Ans. The key is B. Nephrotic syndrome. d. Epidural hematoma

795. A 53yo man with prv hx of COPD Ans. The key is B. Subarachnoid hemorrhage.
presents with breathlessness and purulent [Cerebral aneurisms are recognized association of
sputum. O2 stat=85% on air. ABG: PCKD which may lead to subarachnoid
PaO2=7.6, PaCOS=7. What is the hemorrhage].
appropriate management for his
condition? 798. A 36yo male dx with glioblastoma
a. 24% oxygen since last 5m has cerebral edema and is
b. Mechanical ventilation
on dexamethasone. He has diarrhea and
c. 100% oxygen
vomiting for the last 3days. He has been
d. Nebulized salbutamol
suffering from repeated falls today.
What could be the possible cause for his
falls?
Ans. The key is A. 24% oxygen. [Patient has a. Adrenal insufficiency
hypoxemia and hypercapnea (type2 respiratory b. Dehydration
failure). Hence he should be put on 24% oxygen.
c. Dexamethasone therapy
100% oxygen will abolish hypoxic drive and
d. Raised ICP
worsen hypercapnea].

796. A 34yo man was involved in a RTA Ans. The key is A. Adrenal insufficiency.
and whilst in the ambulance his GCS [Prolonged dexamethason use suppresses
deteriorated and RR increased from 30- adrenal axis and intercurrent illness can lead to
adrenal insufficiency (addisonian crisis) in such a. Aortic aneurysm
cases]. b. Mesenteric cyst
c. Umbilical hernia

799. A 2yo child is brought by his mother. Ans. The key is A. Aortic aneurism. [There is
features of shock with pulse 120bpm and bp
The mother had hearing impairment in
70/40mmHg and sudden collapse of the patient
her early childhood and is now concerned
with pulsatile mass in abdomen makes the likely
about the child. What inv would you do? diagnosis of ruptured aortic aneurism].
a. Audiometry
b. Distraction testing 802. A woman prv in good health presents
c. Scratch test with sudden onset of severe occipital
d. Tuning fork headache and vomiting. Her only physical
sign on exam is a stiff neck.
Choose the most likely dx.
Ans. The key is A. Audiometry. [Conditioned a. Subarachnoid hemorrhage
response audiometry 2-4 years, pure tone audio b. Subdural hematoma
gram for greater than 5 years, less than 6 months c. Cerebellar hemorrhage
otoacoustic emission or brainstem evoked d. Migraine
response, distraction test for 6-18 months, OHCS e. Cerebral embolus
ENT].
Ans. The key is A. Subarachnoid hemorrhage.
[Sudden onset severe occipital headache and
800. An 8yo child presents with recurrent vomiting with stiffness makes the likely diagnosis
to be subarachnoid hemorrhage].
abdominal pain, occasional headaches but
maintains a good appetite. Exam: normal. 803. A 34yo housemaid presents with
CBC, BUE, etc are normal. headaches in the back of her head for
What would you do for her next? several days and pain on flexing her neck.
a. US abdomen What is the most likely cause?
b. CT head a. Subdural hemorrhage
c. Reassure b. Cervical spondylosis
d. Analgesics c. Subarachnoid hemorrhage
d. Meningitis
e. Cluster headache
Ans. The key is C. Reassure. [Recurrent abdominal
pain and headaches with no abnormal finding on Ans. The key is B. Cervical spondylosis. [Headache
examination and investigation points to in the back of head and pain on flexing neck is
Abdominal Migraine. Management is early feature of cervical spondylosis which
reassurance]. gradually progress to later symptoms like
radiculopathies due to root compression in arms
and hands].
801. A 78yo gentleman suddenly 804. A 40yo man complains of thirst and lethargy.
collapsed. His HR=120bpm, His BP=140/90mmHg, corrected Ca2+=3.7mmol/l.
BP=70/40mmHg. Exam: pulsatile mass in What is the most appropriate management at this
abdomen. What is the most appropriate stage?
a. IV fluids
dx?
b. Prednisolone
c. IV hydrocortisone 808. A 55yo lady with swelling on the
d. Calcium prep abdomen below the umbilicus on the right
side. What is the single most appropriate
Ans. The key is A. IV fluids. [Hypercalcemia is
treated by IV fluids].
lymph node?
a. External iliac LN
805. A 75yo man on digoxin develops weakness in b. Pre-aortic LN
the right upper and lower limbs which resolves c. Aortic LN
within a few hours. What is the most definitive d. Inguinal LN
inv for this condition? e. Iliac LN
a. Carotid Doppler f. Submental LN
b. Angiography g. Submandibular LN
c. CT head h. Deep cervical LN
d. Digoxin level
Ans. The key is D. Inguinal LN.
Ans. The key is B. Angiography. [Most definitive 809. A 66yo male presents with painful
angiography; initial or most appropriate dopplar].
swallowing. What is the most likely dx?
806. A 35yo man presents with balance
a. Nesseria meningitides
problems, headache, SNHL and loss of b. Cryptococcus neoformans
coreneal reflex on the left side. What is c. Candida albicans
the most definitive inv? d. Isospora belli
a. CT scan of internal auditory meatus
e. Mycobacterium avium
b. Nuclear imagine of the brain
c. MRI of internal auditory meatus Ans. The key is C. Candida albicans. [Oesophageal
d. MRI brain candidiasis].

Ans. The key is D. MRI brain. Wrong key! “Most 810. A 45yo lady complains of expressable
definitive” is MRI internal auditory meatus. galactorrhea, decreased libido and
[Probable diagnosis is left sided acaustic amenorrhea, weight gain and easy
neuroma. For this most definitive investigation is fatigue. Her serum prolactin levels are
MRI internal auditory meatus]. 1100um/l. what is the likely cause of
807. A 52yo man has a painful, red, hyperprolactinemia?
a. Hypothyroidism
photophobic right eye with slightly blurred
b. Stress
vision and watering for 3days. He has had
c. Pregnancy
no such episodes in the past. On slit lamp
d. Prolactin secreting pituitary tumor
examination there are cells and in the ant
e. PCOS
chamber and pupil is sluggish to react.
What is the single most appropriate
clinical dx?
a. Acute close-angle glaucoma Ans. The key is D. Wrong key. Correct key should
be A. Hypothyroidism. [ It is likely to be
b. Acute conjunctivitis
hypothyroidism because prolactin level in
c. Acute dacrocystitis
hypothyroidism may be 30-200ng/ml which
d. Acute iritis
means 630-4200 mIU/L. Though weight gain can
e. Corneal foreign body
occur but fatigue is not a feature of prolactinoma
Ans. The key is D. acute iritis. unless it is a macroadenoma causing suppression
of other pituitary hormone! given prolactin level
of 1100 IU/L is definitely from a microadenoma chest x-ray should be done to evaluate
(in macroadenoma prolactin level is > 10,000 respiratory problem first].
MU/L) which does not cause fatigue. So the case
most likely seems to be hypothyroidism]. 814. A 68yo man has had increasing
dysphagia for solid food for 3m and has
***811. A 42yo female had sudden severe lost 5kgs in weight. What single inv is
headache and vomiting. She took most likely to lead to a def dx?
paracetamol and then collapsed. What is a. Barium swallow
the most likely dx? b. CXR
a. SAH c. CT chest
b. Viral encephalitis d. Endoscopy and biopsy
c. Meningitis e. Video-fluoroscopy
d. Anaphylaxis
Ans. The key is D. Endoscopy and biopsy. [Likely
cause is malignant stricture for which endoscopic
biopsy is the definitive investigation].
Ans. The key is A. SAH. [Sudden headache,
vomiting, collapse are more suggestive of SAH]. 815. A 24yo male was trying to move his
812. Parents of a 3m baby are worried wardrobe but it fell on his thigh. It was
about cot death. What advice would you there for a very long time before someone
give? was able to remove it. When he was seen
a. Lay on the back with feet towards head in ED he had casts in his urine but no
end RBCs. Other inv showed hypocalcemia and
b. Lay on the back with feet towards feet high serum creatinine. What is the cause
end for his renal failure?
c. Lay on side a. Acetylcholine
d. Lay on stomach b. Myoglobin
c. Myotroponin
Ans. The key is B. Lay on the back with feet d. Acetyl acetate
towards feet end.
Ans. The key is B. Myoglobin. [The likely diagnosis
813. A child born at 36wks developed is Rhabdomyolysis where renal failure occurs due
difficulty in breathing with intercoastal to myoglobin (myoglobins are released after
recession and nasal flaring. His temp is breakdown of skeletal muscles)].
normal but his mother had PROM 48h
816. Anatomical structure to be pierced
ago.
during surgery midline port during
What is the most likely inv that will lead to
gallstone removal.
tx? a. External iliac muscle
a. Blood culture
b. Cricoid cartilage
b. CXR c. Linea alba
c. Stool culture d. Rectus sheath muscle
d. Sputum culture e. Duramater
f. 1st tracheal cartilage
Ans. B. CXR. [Probable pneumonia as there was g. Conjoined tendon
prom! In prematurity there may also occur h. Intercostal muscles
respiratory distress syndrome. So at this stage Ans. The key is C. Linea alba.
817. A 48yo man has continuous ant chest start N-acetylcysteine, stopping it if level below
pain which is worse on inspiration and has treatment line and INR/ALT normal. OHCM, 9 th
edition, page 856].
temp of 37.5C since 4wks after a MI. His
819. A pt dx with DVT is taking warfarin.
ESR=45mm/h.
What is the single most likely explanation What is his cut off INR limit?
for the abnormal inv? a. <1
a. Acute pericarditis b. 1-2
b. Cardiac tamponade c. 2-3
c. Atrial thrombus d. 3-4
d. Left ventricular aneurysm
e. Dressler syndrome Ans. The key is C. 2-3. [This range is appropriate
for the prophylaxis or treatment of venous
The key is E. Dressler syndrome. [Dressler thromboembolism and reduction of the risk of
syndrome usually occurring two to five weeks systemic embolism for people with atrial
after the initial event but it can be delayed for as fibrillation and valvular heart disease].
long as three months.  It is characterised by
pleuritic chest pain, low-grade fever and 820. Inserting a drain in the mid-axillary
pericarditis]. line. What is the single most appropriate
anatomical structure?
818.  An 8yo child swallowed 12 tablets of a. External iliac muscle
paracetamol 4h ago. Serum paracetamol b. Linea alba
levels when tested were at critical level. c. Rectus sheath muscle
What would you do next? d. Conjoined tendon
a. Activated charcoal e. Intercostal muscles
b. IV N-acetylcysteine
c. Gastric lavage Ans. The key is E. Intercostal muscles.
d. Observation only
821. A 29yo man presents with hx of
Ans. The key is B. IV N-acetylcystine. [As 4 hour cough, weight loss and night sweats.
paracetol level is at critical level start IV N- Exam: pansystolic murmur. What is the
acetyltine at once]. most appropriate dx of underlying cause?
a. Malaria
Manaement of paracetamol poisoning:
b. HSP
GI decontamination is recommended in those c. HIV
presenting <4h after OD. Activated charcoal 1g/kg d. Dengue fever
(max 50g) is the treatment of choice, reducing
serum levels more than gastric lavage and limiting Ans. The key is C. HIV. [IV drug abusers may have
liver injury. tricuspid regurgitation causing pansystolic
murmur and HIV also very common in this group]
• Glucose, U&E, LFT, INR, ABG, FBC, HCO3
–; blood paracetamol level at 4h post-ingestion. 822. A 15yo boy presents with recurrent
• If <10–12h since overdose, not vomiting, and breathlessness and wheeze especially
plasma paracetamol is above the line after exercise. What is the most diagnostic
on the graph, start N-acetylcysteine.
• If >8–24h and suspicion of large overdose
inv?
a. CXR
(>7.5g) err on the side of caution and
b. Lung function test
c. PEFR
d. CT scan dysfunction) and there may also cerebellar sign
like ataxia be present].
Ans. The key is B. Lung function test. [Assessment
of severity of airflow obstruction can diagnose
asthma and COPD]. ***825. A 67yo man being managed for a
malignancy develops neutropenic fever.
He has been commenced on Ticarcillin,
823. A 34yo man was walking along the
Tazobactam and Gentamicin. He has also
country side when an insect bit him. After
recently commenced on Meropenem but
which he started to complain of an
on the 3rd day his temp still remains >39C.
annular rash spreading upwards.
a. Penicillin PO
2 blood tests and urine cultures show no
b. Doxycycline PO organism. Inv: Hgb=104g/dl, WBC=<0.5,
c. Flucloxacillin PO Plt=15.
d. Gentamicin PO What will you do next?
e. Ciprofloxacin PO a. Continue IV antibiotics and add oral
f. Antihistamine PO antifungals
g. Antihistamine IV b. Continue antibiotics and add IV
h. Corticosteroid IV
antifungals
i. Corticosteroid IM
j. Adrenaline IM c. Stop antibiotics
k. Adrenaline IV d. Continue only present antibiotics
l. Atropine IV
m. Reassurance Ans. The key is B. Continue antibiotics and add IV
antifungals.

Ans. The key is B. Doxycycline PO. [Described 826. A young girl who is known to have
rash is erythema migrans which is characteristic T1DM presented with drowsiness and
skin lesion seen in lyme disease. Lyme disease is deep breathing. Her sugar level=20. Her
treated with Doxycycline]. BP=120/80mmHg and her mucous
membranes are dry. What would be the
824. A 60yo man was brought in by his next appropriate step?
a. Serum urea
wife complaining of ataxia, urinary
b. Blood culture
incontinence and erectile dysfunction. He
c. CT
also complains of rigidity and slowing of
d. HbA1c
movement with a pill rolling tremor of the
e. ABG
hands. What is the likely dx?
a. Parkinson’s disease Ans. The key is E ABG. [Likely iagnosis is DKA
b. Idiopathic parkinson’s disease where ABG is done to demonstrate acidosis].
c. Shy-drager syndrome
d. Huntington’s disease 827. Removal of a glioma, which single
most appropriate anatomical structure
Ans. The key is C. Shy-drager syndrome. [Shy- will be pierced?
drager syndrome is characterized by a. Cricoid cartilage
parkinsonism (rigidity, slowing of movement with b. Rectus sheath muscle
a pill rolling tremor of the hands) plus a more c. Duramater
pronounced failure of the autonomic nervous d. Conjoined tendon
system (urinary incontinence and erectile e. Intercostal muscles
Ans. The key is C. Duramater. syndrome) or primary HIV infection there is
generalized lymphadenopathy].
828. A child is not breathing and
intubation failed. At what anatomical site
should the incision be made? 831. A 6yo child presents with edema and
a. External iliac muscle
b. Cricoid cartilage
mild proteinuria. No hematuria. What is
c. Linea alba the most likely dx?
d. Rectus sheath muscle a. PSGN
e. Duramater b. Membranous GN
f. 1st tracheal cartilage c. Minimal change GN
g. Conjoined tendon d. RPGN
h. Intercostal muscles
Ans. The key is C. Minimal change GN. [Minimal
Ans. The key is B. Cricoid cartilage. [Incision for change GN is usually seen in young children. It
tracheostomy is done which goes down from presents as edema and proteinuria].
cricoids cartilage].
832. An 80yo woman suffering from RA
829. A 10yo child who presented with fx of presents with severe epigastric pain and
the radius which was treated with a vomiting. She also complains of shoulder
plaster cast, complains of pain. Exam: tip pain. What is the single most
limb is swollen but warm and well discriminatory inv?
perfused, pulses are present. What should a. US Abdomen
you do next? b. Sigmoidoscopy
a. Send for repeat XR c. Colonscopy
b. Remove cast d. Barium meal
c. Replace cast with more padding e. Upper GI endoscopy
f. Erect CXR
d. Give analgesic

Ans. The key is F. Erect CXR. [Elderly patient with


Ans. D. Give analgesic. RA is usually on NSAIDs which may lead to
perforated peptic ulcer which presents as severe
epigastric pain, vomiting and shoulder tip pain.
Diagnosis is made by seeing free gas shadow
830. A 32yo man has been to Thailand under the diaphragm on erect CXR].
and returned with cervical
lymphadenopathy and fever. What is he 833. A 44yo man went on holiday to
most likely suffering from? Sudan 5wks ago. He now presents with
a. HIV red urine and fever. Exam: hepatomegaly.
b. EBV What is the most likely dx?
c. Typhoid a. Malaria
d. Measles b. Brucellosis
c. Leptospirosis
d. Schistosomiasis
Ans. The key is B. EBV. [Cervical
Ans. The key is D. Schistosomiasis. [Holyday in
lymphadenopathy mentioned in question which
Sudan, hematuria, fever and hepatomegaly goes
occurs in EBV infection. [In ARS (acute retroviral
most with Schistosomiasis amongst the given occurs in a normal heart though symptomatic it is
option]. benign in nature but if it is secondary to heart
disease like MI it may precipitate to more life
834. A 32yo homosexual comes with hx of threatening arrhythmia like ventricular
weight loss. Fundoscopy reveals retinal fibrillation].
hemorrhages. What is the single most
appropriate option?
a. Mycobacterium avium
b. CMV 837. A 17yo has acute pain around his
c. Hemophilus influenze right eye, pain on one side of his face and
d. NHL ear ache too. What is the single most dx?
e. Pneumocystic jerovici a. Ear wax
b. Ear foreign body
Ans. The key is B. CMV. [Weight loss in a c. Dental abscess
homosexual is likely to be due to AIDS and CMV d. Cellulitis
retinopathy with retinal hemorrhage is a e. Herpes zoster
recognized association].
Ans. The key is E. Herpes zoster. [It seems to be
835. A 30yo man comes with hx of early herpes zoster of the trigeminal nerve. Onset is
morning back pain and stiffness. Exam: acute, unilateral distribution favours herpes
red eyes. What is the single most zoster].
appropriate option?
a. Iris 838. A 12yo boy presented with itching in
b. Ciliary body his hands. Exam: skin is dry and red. His
c. Cornea mother is asthmatic and older brother has
d. Conjunctivitis hay fever.
e. Sclera What is the single most likely causative
factor?
Ans. The key is A. Iris. [Young man with early a. Dermatitis herpitiformis
morning back pain and stiffness is suggestive of b. Scabies
seronegative arthritis likely ankylosing spondylitis
c. Eczema
where iritis is a common association].
d. Uremia
836. A 70yo woman with longstanding e. Drug induced
anxiety is seen in the OPD. She complains
Ans. The key is C. Eczema. [Itchy lesion and family
of her heart skipping a beat quite often. history of asthma and hay fever in 1st degree
This particularly occurs when she is trying relatives favours the diagnosis of eczema].
to get to sleep. The palpitations are never
sustained. What is the most likely rhythm 839. A 45yo man presented with pruritic
disturbance? purple papules on the flexor surface of his
a. SVT wrist and some white lacy markings on his
b. VF buccal mucosa.
c. VT What is the single most likely causative
d. V-ectopics factor?
e. A-fib a. ALL
b. Lymphoma
Ans. The key is D. V-ectopics. [From the given c. Polycythemia
options the most likely answer is V-ectopics. If it
d. IDA Ans. The key is D. conjunctivitis. [Bacterial
e. Lichen planus conjunctivitis can cause sticky greenish
discharge].
Ans. The key is E. Lichen planus. [White lacy
pattern of lesion is characteristic of lichen
planus]. 843. A pt was admitted with erectile
dysfunction, reduced facial hair and
840. A known DM was admitted with
galactorrhea. What is the most probable
sudden LOC. What is the initial inv? dx?
a. CT scan a. Hyperprolactinemia
b. RBS b. Cushing’s syndrome
c. MRI c. Pheochromocytoma
d. ECG d. Hyperthyroidism
e. ABG e. Hypoparathyroidism

Ans. The key is B. RBS. [In unconsciousness in


diabetics, first hypoglycemia should be excluded Ans. The key is A. Hyperprolactinemia.
by doing RBS].

844. A 32yo man has been repeatedly


admitted to hospital for what was
described as anxiety or panic attacks and
841. A 36yo lady comes with hx of early palpitations. On occasions he is found to
morning stiffness of her small joints and be tremulous and hypertensive. A
with red and painful eye. What is the persistent weight loss is noted. What is
single most appropriate option? the most probable dx?
a. Iris a. Hyperthyroidism
b. Ciliary body b. Panic attacks
c. Cornea c. Pheochromocytoma
d. Conjunctivitis d. Cushing’s disease
e. Sclera e. GAD
f. Lichen planus
Ans. The key is C. Pheochromocytoma.
Ans. The key is E. Sclera. [Ocular manifestation of
rheumatoid arthritis is scleritis (red, painfull eye)]. 845. A 35yo man with T1DM is
dehydrated with BP of 90/50mmHg. What
is the single most appropriate initial inv?
842. A 23yo man comes with 2d hx of a. ABG
sticky greenish discharge from the eyes b. CBC
with redness. What is the single most c. HbA1c
appropriate option? d. LFT
a. Uveitis
e. Serum Urea
b. Ciliary body inflammation
c. keratitis Ans. The key is A. ABG. [Dehydration with low BP
d. Conjunctivitis points towards probable DKA. So from the given
e. Scleritis options ABG is the best response].
846. In OGTT what is the glucose venous 849. A 28yo man is inv for recurrent lower
plasma level 2h after glucose intake which back pain. A dx of AS is suspected. Which
indicates impaired glucose tolerance? of the following inv is most useful?
a. >11.1mmol/l a. ESR
b. Between 7.8-11.0mmol/l b. XR sacro-iliac joints
c. Between 8.0-10.9mmol/l c. HLA B27
d. Between 10.0-11.0mmol/l d. XR thoracic spine
e. Between 7.1-11.0mmol/l e. CT lumbar spine

Ans. The key is B. Between 7.8-11.0mmol/l. Ans. The key is B. XR sacro-iliac joints. [x-ray
sacroiliac joint shows fusion of both SI joints and
847. A young man who has no PMH thin, symmetrical syndesmophytes bridging the
presented with jaundice, low Hgb, retics intervertebral disc spaces].
8% and other indices WNL but occasional
spherocytes were seen on blood film.
What is the single most appropriate inv? ***850. A 4yo girl is taken by her mother
a. G6PD enzyme assay to the ED and complains of feeling unwell,
b. Direct coombs test urinary urgency and temp=39C. What is
c. Repeat blood film the single next best inv?
d. Indirect coombs test a. Catheter catch of urine
e. BMA b. Clean catch of urine
c. US
Ans. The key is B. Direct Coombs test. [The direct d. IVU
Coombs test, is used to determine whether the e. Suprapubic catch of urine
cause of hemolytic anemia, is due
to antibodies attached to RBCs which are seen in Ans. The key is B. Clean catch of urine. [The
autoimmune-related hemolytic anemia]. clinical features described are consistent with
urinary tract infection for which clean catch of
848. A 22yo man came to the hosp after urine is the next best investigation. Routine
an injury in his hand while playing investigations that are done in UTI are:
basketball. Exam: avulsion of extensor
 Dipstick analysis of urine - may treat as
tendon from the distal phalanx.
bacterial if there are positive results for
What is the single most probable nitrite and/or leukocytes.
deformity?  Urine microscopy - leukocytes indicate
a. Dinner fork deformity presence of infection.
b. Game keeper thumb  Urine culture - to exclude the diagnosis or
c. Mallet finger if high-risk (eg, pregnant,
d. Gun-stock deformity immunosuppressed, renal tract anomaly
e. Garden spade deformi or if failed to respond to earlier empirical
treatment).
Ans. The key is C. Mallet finger. [A finger that  An ultrasound evaluation of the upper
bends down at the end joint and cannot be urinary tract is recommended to rule out
straightened is called a mallet finger. It is caused urinary obstruction or renal stone disease
in acute uncomplicated pyelonephritis].
by an injury to the extensor tendon that
straightens (extends) the finger. A splint worn day
and night for 6-8 weeks will cure the problem in
851. A 2yo girl presents with a 4d hx of
most cases]. fever that started with a cough. Her
RR=45bpm, sat=94%, temp=38.9C, characteristic hyperintensity in fronto-temporal
capillary refill time=1s. There are region.
In the given case there are features of raised
crepitations at the left base on
intracranial pressure like drowsiness and seizer
auscultation. Urine shows negative and so we cannot proceed for LP unless guided by
dipstick. CT (or MRI). On the other hand there is no other
What is the single inv most likely to lead option that can be diagnostic of the given
to dx? condition. So we have to happy with CSF analysis
a. Blood for C&S as the key though at this moment we have to
b. ESR withheld this procedure].
c. CXR
853. A 6m boy is admitted with persistent
d. Urine for C&S
irritability. He is lethargic and is not
e. CSF analysis
feeding as well as usual. His RR=30bpm,
Ans. The key is C. CXR. [The features are sat=97%, temp=38.0C, capillary refill
consistent with RTI (probable pneumonia) for time=2s. Urine reveals leucocytes on
which CXR is the investigation of choice]. dipstick. What is the single inv most likely
to lead to dx?
a. Blood for C&S
852. A 3yo girl presents with fever for 2d.
b. ESR
She is drowsy and had a seizure causing c. CXR
twitching of the right side of the body for d. Urine for C&S
4mins. Her RR=30bpm, sat=90%, e. CSF analysis
temp=38.9C, capillary refill time=2s. Urine
negative on dipstick. Ans. The key is D. Urine for culture and sensitivity.
What is the single inv most likely to lead [Dipstick test if show leucocytes or nitrites is
to dx? suggestive of UTI. So to confirm it we should do
a. Blood for C&S urine C&S].
b. ESR
854. A 3yo boy presents with a 1d hx of
c. CXR
being unwell. He appears shocked and has
d. Urine for C&S
3h old rash made up of urticarial and
e. CSF analysis
purpural spots. His RR=30bpm, sat=94%,
Ans. The key is E. CSF analysis. [It is really very temp=39C, capillary refill time=1s. Urine is
difficult to differentiate between encephalitis and clean on dipstick.
meningitis. Encephalitis is mostly viral and in UK What is the single inv most likely to lead
herpes simplex virus is the main cause. Meningitis to dx?
also has a viral predominance though less than
a. Blood for C&S
encephalitis. 
b. ESR
Because encephalitis involves infection of the
brain itself, symptoms of altered brain function-- c. CXR
like confusion or decreased alertness--are usually d. Urine for C&S
present, while in cases of meningitis the patient is e. CSF analysis
initially alert and, though understandably
distracted by pain and misery, still in command of Ans. The key is A. Blood for C&S. [Likely diagnosis
their mental processes. is septicemia for which Blood culture is the
As CSF can not differentia between meningitis investigation of choice].
and encephalitis we have to take help of imaging
like MRI. In herpes simplex encephalitis there is
855. A child is dx with VUR. What would What is the single most appropriate next
you tell his parents? action?
a. Local exam
a. Requires antibiotic prophylaxis b. Exam under GA
b. Most will require surgery c. Continue regular child care
c. Most will have kidney scarring by 5yo d. Inform child protection services
d. Nothing can be done e. Coag profile
e. Reassure

Ans. The key is B. Exam under GA.


Ans. The key is B. Most will require surgery. [For 859. A 7d baby whose birth weight was
recurrent UTI = prophylactic antibiotic, for VUR =
3.5kg and now is 3kg. What is the most
surgery].
appropriate next step?
856. A 2yo child presents with severe a. Check child protection register
vomiting. Exam: mass felt in abdomen. b. Nutritional assessment
What inv is most appropriate? c. Skeletal survey
a. US d. Continue regular child care
b. XR e. Inform police
c. CT
d. CBC
Ans. The key is D. Continue regular child care. [In
1st week the baby loose some weight then start
Ans. The key is A. US. [The question is incomplete gain weight again. It is normal. So the option is
and is not suitable for further discussion with continue regular child care].
such little info].

857. A 13yo girl complains of a 2d hx of 860. A 6yo child fell on his nose 2d ago.
hoarseness of voice a/w dry cough. She His parents have now brought him with
feels feverish. On direct laryngoscopy, her difficulty in breathing. Exam: fever, nasal
vocal cords are grossly edematous. bones are straight. What is the single
What is the single most appropriate inv? most likely dx?
a. None req a. Nasal polyp
b. Sputum for AFB b. Septal hematoma
c. Laryngoscopy c. Septal abscess
d. Bronchoscopy d. Deviated nasal septum
e. XR cervical spine e. Fx nose

Ans. The key is A. None required. Ans. The key is C. Septal abscess.

858.  A 7yo girl is brought by her mother


with bright red staining of her underpants. 861. A 12yo pt came to the OPD with
She also gives a hx that her daughter complains of fever, malaise, weight loss,
recently started taking horse riding anorexia and productive cough. Exam:
lessons. temp=39C, pulse=100bpm. His mother
says that he has a hx of recurrent chest c. Croup
infections and he is not thriving well. d. Epiglottitis
What is the single most likely causative e. Tonsillitis
organism?
a. Pneumococcal pneumonia
b. Staphylococcus Ans. The key is D. Epiglottitis.
c. Mycobacterium TB
d. Pseudomonas
e. PCP 864. A pt with terminal cancer is being
treated with chemo develops tingling and
numbness of the fingertips of both arms.
Ans. The key is D. Pseudomonas. [Here the likely
diagnosis is cystic fibrosis in which RTI due to What is the single most likely cause of the
pseudomonas is a very common association]. symptoms?
a. Bone mets to cervical vertebrae
b. Post-chemo neuropathy
862. A 3yo child brought by his mother. c. Hyponatremia
Exam: bruises on the buttocks. Mother d. Hypocalcemia
also gives hx of runny nose 2wks ago.
What is the single most appropriate next
action? Ans. The key is B. Post-chemo neuropathy.
a. Check child protection register
b. Coag profile
c. Skeletal survey 865. An 80yo man has a permanent
d. Continue regular child care catheter. Catheter specimen urine found
e. Inform police
lots of e-coli. What is the single most
appropriate management as he wants to
Ans. The key is B. Coagulation profile. [Likely attend his daughter’s wedding next week?
diagnosis is HSP in which coagulation profile will a. Change the catheter
show significant increase in D-dimer b. Prolonged antibiotics
concentration. An activation of coagulation c. Bladder wash
including hyperfibrinolysis secondary to the d. Repeat MSU after wedding
endothelial damage is a typical feature of the e. Reassure
common types of HSP].

Ans. The key is A. Change the catheter.


863. A 4yo is brought to the ED by
ambulance. His mother reports that he
has been unwell with a sore throat for 8h. 866. A 35yo male typist who suffered a
He is sitting on his mother’s knee and is scaphoid fx was treated with a scaphoid
tolerating an oxygen mask but looks cast. After 2wks when the cast was
unwell. He has constant noisy breathing removed for a review XR, it was found
and he is drooling saliva. His temp=39C. that he had problems in moving the
What is the most imp dx? thumb, index and middle fingers. What
a. Acute asthma would you suggest as the management
b. Bronchiolitis for the recent prb?
a. Release of flexor retinaculum e. Ankylosing spondylitis
b. Release of common flexor sheath
c. Release of palmar sheath Ans. The key is D. Pseudogout. [Hypothyroidism
has well recognized association with
d. Ulnar nerve release
pseudogout].
e. Fasciotomy
870. A 45yo woman had her visual acuity
checked at her local optician. 12h later
Ans. The key is A. Release of flexor retinaculum. she presents to the ED with severe pain
and redness in her eye. What is the single
867. A pt on insulin is booked in for a
most appropriate option?
hernia operation. What is the most a. Iris
appropriate management of insulin? b. Ciliary body
a. Give insulin and saline pre-op
c. Ant chamber
b. Stop insulin for the duration of the op
d. Post chamber
c. Give IV insulin + dextrose + saline pre-op
e. Cornea
d. Give insulin as usual pre-op
e. None

Ans. The key is C. [In acute angle closure


glaucoma half-dilated pupil is the most likely
Ans. The key is C. Give IV insulin + dextrose + position which precipitates an acute attack as the
saline pre-op. trabecular meshworks are mostly closed by
868. A 35yo male who recently had an peripheral anterior synechia of peripheral iris in
this position. And mild illuminated darkened
appendicectomy has got severe pain in his
room like of an optician (also use of mydriatics
right big toe. Joint is red and swollen. He
accelerates this) or opera (cinema hall) are
consumes 30 units of alcohol/week. culprit to make this! As the block occurs in
What is the most probable dx? anterior chamber it is the likely option here].
a. Rhabdomyosarcoma
b. Osteoarthritis 871. A 75yo man who has DM and HTN
c. Gout experiences acute monocular blindness
d. Pseudogout which resolves after 1h. What is the most
e. Arthritis likely dx?
a. GCA
Ans. The key is C. Gout. [Alcoholic drinks can raise b. Optic neuritis
the level of uric acid in the blood].
c. Lacunar infarct
869. A 25yo male who recently noticed d. Pontine hemorrhage
change in his shoe size, he is also e. Amaurosis fugax
constipated, has a preference to hot Ans. The key is E. Amourosis fugax.
weather, his skin is dry, has severe pain in
wrist joint. Joint is red and swollen. What 872. A 26yo presents with prolonged
is the most probable dx? constipation, blood on side of stool and
a. Chondro-sarcoma very painful defecation. PR exam: very
b. Lipo-sarcoma painful. What is the single most likely dx?
c. Gout a. Ca Colon
d. Pseudogout b. UC
c. CD 876. A 60yo is on tx for IHD, HTN and
d. Anal fissure hyperlipidemia. During the night he
e. Constipation complains of wheeze and SOB. Which of
the following meds is responsible for that?
Ans. The key is D. Anal fissure.
a. Amlodipine
873. A 35yo man with painless left b. Atenolol
testicular enlargement for the past 6m c. Ramipril
which is increasing in size and 3x larger d. Simvastatin
than the right side. There is no tenderness e. Bendroflumethiazide
or redness. What is the most likely dx?
a. Testicular tumor
b. Hydrocele Ans. The key is B. Atenolol. [Asthma can be
c. Epididymal cyst precipitated by beta blockers].
d. Epididymo-orchitis
877. A 15yo boy who complains of pain in
e. Reassure
his leg which has settled with aspirin.
Ans. The key is A. Testicular tumour. What is the most probable dx?
a. Leomyosarcoma
874. A middle aged man who has had a hx b. Liposarcoma
of chronic sinusitis, nasal obstruction and c. Painful hip
blood stained nasal discharge. He now d. Exostosis
presents with cheek swelling, epiphora, e. Osteod osteoma
ptosis, diplopia, maxillary pain. What is
the single most likely dx?
a. Nasopharyngeal ca Ans. The key is E. Osteoid osteoma. [Younger age
b. Pharyngeal ca of onset and relieved with aspirin favours the
c. Sinus squamous cell ca diagnosis of osteoid osteoma].
d. Squamous cell laryngeal ca
e. Hypopharyngeal tumor 878. A 20yo fit man suddenly developed
severe lower back pain as getting up from
bed. What is the single most probable dx?
a. Paget’s disease
Ans. The key is C. Sinus squamous cell ca.
b. Multiple myeloma
875. A 60yo man with a long hx of c. PID
smoking and alcohol presents with nasal d. AS
obstruction, epistaxis, diplopia, otalgia e. Spondylosis
and conductive deafness. What is the
single most likely dx?
a. Nasopharyngeal ca Ans. The key is C. PID [Prolapsed Intervertebral
b. Pharyngeal ca Disc]. [Sudden onset of lower back pain and felt
c. Sinus squamous cell ca more during forward bending or similar
d. Squamous cell laryngeal ca movement like getting up from bed favours the
e. Hypopharyngeal tumor diagnosis of PID].

879. A 60yo man brought to the ED with


fx hip, he is deaf and has bilateral pedal
Ans. The key is A. Nasopharyngeal ca.
edema. What is the single most probable e. Allergic rhinitis
dx?
a. Paget’s disease
b. Osteoporotic fx vertebra Ans. The key is A. Maxillary sinus.
c. Secondary
d. Multiple myeloma
883. A 29yo man with hx of asthma comes
e. Spondylosis with post nasal discharge and bilateral
painless nasal blockage. What is the single
most likely dx?
a. Nasal polyp
Ans. The key is A. Paget’s disease. [H/O deafness
b. Septal hematoma
and evidence of heartfailure like pedal edema
c. Septal abscess
favours the diagnosis of Paget’s disease].
d. Atopic rhinitis
880. An 80yo lady presents wih pain on e. Allergic rhinitis
left 6th rib for a week. It is non-tender on
examination. What is your most likely dx?
a. Herpes zoster Ans. The key is A. Nasal polyp.
b. Costochondritis
c. Bone degeneration 884. A 24yo man has been found
d. Thoracic vertebra compression unconscious in an alleyway with a
RR=6bpm and HR=60bpm. His pupils are
Ans. The key is C. Bone degeneration. constricted. What is the best tx?
a. Methadone
b. Naloxone
881. A 68yo DM, HTN with a 45pack/year
c. Naltrexone
smoking hx, has left sided chest pain
d. Thiamine
increased with breathing. Exam: myosis
e. Glucose
on left side and wasting of small muscles
of left hand. What is the single most
appropriate dx?
Ans. The key is B. Naloxone. [Respiratory
a. Costochondritis
depression and miosis points towards opiates
b. Lung cancer
poisoning for which antidote naloxone is given].
c. Good pasture’s syndrome
d. MND 885. A 23yo female presents with back
e. Progressive massive fibrosis pain and early morning stiffness, also
complaining of eye problem and her sister
Ans. The key is B. Lung cancer. [Smoking history,
has a similar condition. What is the single
chest sign and Horner’s syndrome points towards
lung cancer (pancoast tumour)].
most probable dx?
a. Paget’s disease
882. A 34yo man had a cold 2d back. He b. PID
now presents with right sided facial pain. c. Myofacial pain
What is the single most likely dx? d. AS
a. Maxillary sinus e. Spondylosis
b. Ethmoid sinus
c. Septal hematoma
d. Septal abscess
Ans. The key is D. AS [Back pain with early Ans. The key is D. Thiamine. [Dx is delirium
morning stiffness and uveitis points towards the tremens. So following chlordiazepoxide thiamine
diagnosis of ankylosing spondylitis]. is the appropriate next step].

886. A 63yo female with a hx of 889. A woman with a prv hx of pain at the
osteoporosis suddenly falls on her left wrist following a fall 4m ago for which
outstretched hand while shopping. XR she didn’t seek any tx now presented with
shows fx at distal radius with backward pain in the same wrist below the thumb
shift of the distal fragment. What is the and the pain is aggravated whenever she
single most probable deformity? holds her baby. What is the cause?
a. Dinner fork deformity a. Fx radial head
b. Coxavara b. Scaphoid fx
c. Mallet finger c. Carpal tunnel syndrome
d. Cubitus valgus d. Colles fx
e. Garden spade deformity e. Ulnar fx

Ans. The key is A. Dinner fork deformity.

887. A 60yo man presents with severe Ans. The key is B. Scaphoid frature.
colicky pain from his right flank radiating
to his groin. His urinalysis reveals trace
blood cells. What is the single most 890. A 29yo man was involved in an RTA.
discrimatory inv? He presents with distended neck veins,
a. US abdomen clear breath sounds and a trachea which
b. XR KUB is in the midline. His RR=34bpm,
c. Colonoscopy BP=60/0mmHg. What is the most likely
d. Upper GI endoscopy dx?
e. Laproscopy a. Simple pneumothorax
b. Tension pneumothorax
Ans. The key is A. US abdomen. [Features are of c. Cardiac tamponade
ureteric colic. X-ray KUB may miss radiolucent d. Pericarditis
stones so US abdomen is the discriminatory inv].

888. A 45yo man has been admitted for an Ans. The key is C. Cardiac tamponade. [Distended
elective hernia surgery. 3d later he neck vein, clear breath sound and no tracheal
presents with agitation, sweating, shift and BP of 60/0 points towards cardiac
aggressiveness, and complains of seeing tamponade].
snakes on the hosp wall. Chlordiazepoxide
has been started for this pt. What is the
891. An elderly woman is found anemia.
most appropriate next step?
a. Diazepam
As part of her exam, she had a barium
b. Acamprosate
enema which reveals a mass lesion in the
c. Disulfiram
ascending colon. What is the single most
d. Thiamine appropriate dx?
a. Sigmoid volvulus
b. Anal fissure
c. Sigmoid carcinoma
d. Cecal carcinoma digoxin and isosorbide mononitrate. What drug is
e. Diverticular disease going to help him?
a. Ramipril
b. Bendroflumethiazide
c. Atenolol
Ans. The key is D. Cecal carcinoma. [Mass in
assending colon and anaemia makes cecal d. Amlodipine
carcinoma the likely diagnosis from the given e. Diltiazem
options].

892. A 55yo male after gastrectomy Ans. The key is B. Bendroflumethiazide. [As
developed anemia. His MCV=106fl. Exam: patient is still in heart failure and pulmonary
loss of proprioception and vibration sense. edema with the usage of loop diuretic...a thiazide
What is the most likely dx? diuretic could be added with loop when response
a. IDA is inadequate( Refernce;patient.info) this will
b. Folate def reduce the volume overload and improve
c. Vit B12 def breathing].
d. Anemia of chronic disease

Ans. The key is C. Vit B12 def. [gastrectomy –>


895. A 31yo man underwent an operation
deficiency of intrinsic factor –> Vitamin B12 def.
where his hand was hanging outside the
leading to macrocytic anemia and resulting
subacute combined degeneration of cord causing
table. After theoperation he had wrist
loss of proprioception and loss of vibration drop and sensory loss over the dorsum of
sense]. his hand. Which nerve was injured?
a. Radial
b. Ulnar
893. A 26yo male has been operated for c. Median
d. Axillary
abdominal trauma and splenectomy was
e. Brachial
done. On the 3rd post-op day the pt
developed acute abdominal pain and
distention in the upper abdominal area
Ans. The key is A. Radial. [As the man’s hand was
with hypotension. On insertion of ryles
hanging outside the ot table there was pressure
tubes, 2L of coffee ground fluid was
on radial nerve at arm level which is similar lesion
aspirated. What is the most probable dx? of Saturday night palsy].
a. Acute gastric dilatation
b. Reactionary hemorrhage
c. Subphrenic abscess
d. DVT 896. What is the mode of spread of
e. Left lower lobe atelectasis chicken pox?
a. Airborne
b. Close contact
Ans. The key is A. Acute gastric dilatation. c. Fecal-oral
d. Blood
894. A 50yo man presented with increased e. Vector
breathlessness at rest. He is currently on
furosemide,
Ans. The key is A. Airborne. d. Send bloods for inv
e. US

Ans. The key is C. Put leg splint. [Putting leg splint


897. A 64yo man presents with ipsilateral will reduce ongoing blood loss and we should
vertigo, tinnitus and left side hearing loss. then go for replacing loss by IV fluids as initial
Exam: Renne test +ve and Weber’s blood loss in femur fracture is alarming which
lateralizes to the right ear. could not be replaced adequately by iv fluid
unless it is reduced].
What is the most appropriate inv?
a. CT 900. A 70yo pt presents with cough and
b. MRI brain SOB. He stopped smoking cigarettes 2yrs
c. XR ago but has a 50yr smoking hx before
d. Audiometry
quitting. CXR=consolidation and bilateral
e. None
bihilar lymphadenopathy. What is the best
f. Caloric testing
inv for this pt?
Ans. The key is B. MRI brain. [Rinne positive, so it a. LN biopsy
is not conductive deafness. Again we know if b. Pleural fluid cytology
weber lateralize to deaf ear it is conductive and c. CT
lateralized away from deaf ear then it is d. MRI
sensoryneural. So here there is sensoryneural e. US
hearing loss in left ear. So the most appropriate
investigation is MRI brain]. Ans. The key is A. LN biopsy. [likely diagnosis is
lung cancer, so best investigation for this is LN
biopsy].
898. A 67yo man presents to the ED with
pain in his left groin. He suddenly 901. A 27yo pt met with a RTA. While the
collapses and his is not able to move or lift NGT is passing, bowel sounds are heard in
his leg. He is on alendronate. What is the the chest. CXR shows NGT curled. What is
dx? the dx?
a. Fx of neck of femur a. Diaphragm rupture
b. Post hip dislocation b. Aortic rupture
c. Fx of shaft of femur c. Splenic rupture
d. Pelvic base fx d. Bowel rupture
e. Peripheral vascular disease e. Liver rupture

Ans. The key is A. Fx neck of femur. [Use of Ans. The key is A. Diaphramatic rupture.
alendronate indicates osteoporosis where 902. A 62yo man dx with T2DM with
fracture neck of femur is more common].
BMI=33. Lifestyle modifications have
899. A young male met with a RTA and is failed to control blood sugar. Labs:
suspected to have a femur fx. His BP is urea=3.6mmol/l, creatinine=89mmol/l.
90/60mmHg. What is the next immediate what is the next appropriate
action? management?
a. XR a. Biguanide
b. IV fluids b. Sulfonylurea
c. Put leg splint c. Insulin
d. Glitazone
e. Sulfonylurea receptor binder tenderness. Exam: tympanic membance
normal. Aural toilet has been done. What
Ans. The key is A. Biguanide. [Patient is obese
type 2 diabetic. So biguanide is the tx of choice].
is the next appropriate med?
a. Antibiotic PO
903. A pt presents with progressive b. Antibiotic IV
dyspnea. He complains of cough, wheeze c. Steroid PO
and a table spoonful of mucopurulent d. Steroid drop
sputum for the last 18m. Spirometry has e. Antibiotic drop with steroid
been done. FEV1/FVC=2.3/3.6. After
Ans. The key is E. Antibiotic drop with steroid.
taking salbutamol, the ratio=2.4/3.7. [Discharge from ear and tragal tenderness are
What is the most likely dx? features of otitis externa. Key treatment is aural
a. Chronic bronchitis toileting. Drop advised is Sofradex (Framycetin +
b. Asthma dexamethasone) OHCS, 9th edition, page 542].
c. Bronchiectasis
d. Lung fibrosis 906. A 23yo man sprained his right ankle
e. Sarcoidosis 6wks ago while playing football. He was tx
with a below knew walking cast. On
Ans. The key is A. Chronic bronchitis. [Progressive removal of the cast, the pt noted to have
dyspnoea, wheeze productive cough and the right foot drop. He has weakness of
result of spirometry points towards the diagnosis
extensors of the ankle and toes and
of chronic bronchitis].
diminished pin prick sensation over the
904. A 62yo man presents with cough, dorsum of the foot. The ankle jerk is
breathlessness and wheeze. 24% O2, present and plantar reflex is flexor.
salbutamol and hydrocortisone were What is the most likely cause of the foot
given. The symptoms haven’t improved drop?
and so nebulized bronchodilator was a. Compression of common peroneal nerve
repeated and IV aminophylline was given. b. Compression of the tibial nerve
ABG: pH=7.31, RR=32. What is the next c. Compression of the S1 nerve root
appropriate management? d. Rupture of Achilles tendom
a. Nasal IPPV e. Tx of the medial collateral lig of the ankle
b. Intubation and ventilation
Ans. The key is A. Compression of common
c. LABA
peronial nerve. [Foot drop, weakness of extensors
d. Toxapram
of the ankle and toes and diminished pin prick
e. Amoxicillin PO sensation over the dorsum is sugestive of
compression of common peroneal nerve].
Ans. The key is A. Nasal IPPV. [Here given case is
COPD has following indications of Nasal IPPV i) 907. A young man was knocked down
Tachypnea (>24 breaths/min) and ii) during a fight in the waiting room of the
Hypercapnic respiratory acidosis (pH range 7.10-
ED. He is now unconscious and
7.35)].
unresponsive. What is the 1st thing you
would do?
a. Turn pt and put in recovery position
905. A young girl returns from holidays in b. Put airway
Spain. She complains of discharge from c. Endotracheal intubation
her ear and complains of tragal d. Assess GCS
e. Start CPR Ans. No key is given!! Correct answer is D. Large
cell ca. [Large cell carcinoma is, by definition, a
Ans. The key is B. Put airway. [ABC protocol]. poorly differentiated malignant epithelial tumor.
It consists of sheets or nests of large polygonal or
908. A 52yo man underwent a
giant multinuclear cells and probably represents
hemicolectomy. After a few days he
SCC ."]
complains of left ventricular pain and
fever. ECHO has been done and shows a 911. A 27yo man presents with chest pain
systolic murmur. and respiratory distress. Exam:
What is the next appropriate inv? tachycardia, hypotension and neck vein
a. CT distension. Trachea is deviated to the left
b. US side, breathing sounds on right side are
c. CXR absent and diminished on left side.
d. Blood culture What is the next appropriate
e. LFT management?
a. CXR
Ans. The key is D. Blood culture. [Dx a case of
b. Right side aspiration (16G)
infective endocarditis. Therefore the next
c. Left side aspiration (16G)
appropriate investigation is blood culture].
d. Right side drain with a small tube (12F)
909. A 19yo man has exercised induced e. Left side drain with a small tube (12F)
asthma and is using a salbutamol inhaler
as req and beclamethasone 400ug BD. He
Ans. The key is B. Right side aspiration (16G). [The
complains that he has to wake up at night features described is diagnostic of right sided
for his inhaler. What is the single most tension pneumothorax. Next appropriate
appropriate tx? management is To remove the air, insert a large-
a. Beclo bore (14–16G) needle with a syringe, partially
b. Regular salbutamol and budesonide filled with 0.9% saline, into the 2nd intercostal
c. Sodium cromoglycate interspace in the midclavicular line on the side of
the suspected pneumothorax. Remove plunger to
d. Oral steroid
allow the trapped air to bubble through the
e. Inhaled steroid syringe (with saline as a water seal) until a chest
tube can be placed. Alternatively, insert a large-
bore Venfl on in the same location OHCM, 9th
Ans. The key is C. Sodium cromoglycate. edition, page 824].

910. Pt with a long hx of smoking is now


suffering from bronchial ca. histology 912. A 16wk pregnant pt who was
reveals there are sheets of large polygonal exposed to a child with chicken pox came
or giant MNC. What is the most likely dx? to GP for help. She was tested –ve for
a. Squamous cell ca varicella antibody.
b. Small cell ca What is the next most imp step in
c. Adenocarcinoma management?
d. Large cell ca a. Reassurance
e. Oat cell ca b. Ig
c. Ig + vaccine
d. Vaccine only
e. Acyclovir
Ans. The key is B. Ig. [If you are pregnant, come b. Oral acyclovir
contact with chicken pox case, found antibody c. Oral antibiotics
negative on blood test you have to take injection d. Topical steroids
Ig. Ref: patient.info]. e. None

913. A 68yo woman dx with T2DM and Ans. The key is E. None. [A case of chickenpox.
BMI=33. Lab: GFR=29, urea=13, None of the given treatment is used in
creatinine=390mmol/L. what is the next chickenpox. Symptomatic treatment like,
appropriate management? acetaminophen if fever, antihistamine and
a. Biguanide calamine lotion is given].
b. Sulfonylurea
c. Insulin
915. A woman came with the complaint of
d. Glitazone
pain in her right arm when she abducts it.
e. Sulfonylurea receptor binder She has recently moved to a new house.
There is no history of trauma.
What is the likely cause of her pain?
a. Rupture of the long head of biceps
Ans. The key is C. insulin. [Insulin is devoid of
b. Sprain of the acromio-clavicular ligament
significant side effect than Glitazones (like fluid
retention). In renal failure there is reduced GFR c. Tendinitis of the abductor sheat
and some fluid retention. It is not desirable that d. Supraspinatus tendinitis
glitazone to cause more fluid retention by causing e. Shoulder dislocation
oedema. The oral agents that are thought to be
relatively safe in patients with nondialysis CKD
include short-acting sulfonylureas (eg, glipizide)
Ans. The key is D. Supraspinatus tendinitis.
and repaglinide. If an oral agent is used, the
short-acting sulfonylurea, glipizide, is the [Tendinitis and partial tears in the supraspinatus
preferred agent among nondialysis CKD patients tendon causes a ‘painful arc’ since as the person
who have an estimated glomerular filtration rate elevates his arm sideways, the tendon begins to
(eGFR) <30 mL/min/1.73. The dose for glipizide is impinge under the acromion throught the middle
2.5 to 10 mg/day. Glyburide and other long-acting part of the arc, and this is usually relieved as the
sulfonylureas are generally not recommended in arm reaches 180 degrees (vertical)].
any CKD patient with type 2 diabetes, because of
the risk of hypoglycemia. Some clinicians
recommend the use of the meglitinide
repaglinide (starting with a dose of 0.5 mg) for
916. An 83yo man with longstanding
nondialysis CKD patients since these agents are COPD has become progressively
not renally cleared. Nondialysis CKD patients with breathless over the last 2yrs. He is on
type 2 diabetes may be treated with an oral salbutamol, ipratropium, salmetarol,
agent, although many patients end up on insulin beclomethasone and theophylline. His
therapy because it is more effective. So it may be
FEV1<30%.
that we can go for insulin as the answer!!].
What is the next appropriate
management?
914. A 5yo boy was brought to GP with a. Lung transplant
high temp and many vesicles on his back. b. Trial of CPAP
What is the most appropriate c. Trial of non-invasive ventilation
management? d. Assessment for long term O2 therapy
a. Topic acyclovir e. Short course of O2 therapy
Ans. The key is D. Assessment for long term O2 918. A man had a soft mass on his
therapy. [Patient is progressively breathless with mandible. Mass is freely mobile and has
present FEV1 of <30%. So his respiratory
started growing progressively over the
deterioration indicates progressive respiratory
failure for which he should be assessed for long past 6m. The mass still moves freely.
term O2 therapy.  Long-term oxygen therapy What is the best inv for this pt?
(LTOT) for more than 15 h/day improved a. FNAC
mortality and morbidity in a well-defined group of b. CT
patients with chronic obstructive pulmonary c. XR
disease. Requirement of condition to proceed to d. MRI
LTOT is patient should be stable and on
e. ESR
appropriate optimum therapy (as in given case)
and having stopped smoking tobacco. Patient
should be shown to have a PaO2 less than 7.3 kPa Ans. The key is A. FNAC.
and/or a PaCO2 greater than 6 kPa on two
occasions at least 3 weeks apart. FEV1 should be
less than 1.5 litres, and there should be a less 919. A 63yo man has been brought to the
than 15% improvement in FEV1 after
bronchodilators. Patients with a PaO2 between
hosp after collapsing during a wedding.
7.3 and 8 kPa who have polycythaemia, right His ECG is below. What is the most likely
heart failure or pulmonary hypertension may gain dx?
benefit from LTOT].

917. A 49yo man complains of fullness in


his left ear, recurrent vomiting and
tinnitus. What is the most appropriate
med?
a. Buccal prochlorperazine
b. Oral chlorpheniramine
c. Oral flupenphenazine
d. Buccal midazolam
e. IV rantidine

Ans. The key is A. Buccal prochlorperazine.


[Meniere’s disease Dilatation of the
endolymphatic spaces of the membranous
labyrinth causes vertigo for ~12h with
prostration, nausea/vomiting, a feeling of fullness
a. VT
in the ear; uni- or bilateral tinnitus, sensorineural
deafness (eg fluctuating). Attacks occur in clusters b. A-fib
(<20/month). Cause: A mystery! c. VF
Electrocochleography; endolymphatic space MRI. d. A-flutter
Prochlorperazine as BuccastemR 3mg/8h PO (1st- e. SVT
line if vomiting) or betahistine 16mg/8h PO or
chlorthalidone may help. Ref: OHCS, 9 th edition,
page 554].
Ans. The key is C. VF. [QRS complex is chaotic,
wide, bizarre and irregular which is characteristic
of ventricular fivrillation]. 922. A 71yo man with a hx of 50yrs of
smoking presents with cough, hemoptysis,
dyspnea and chest pain. He also has
*920. A 75yo war veteran complains of anorexia and weight loss. The dx of lung
loss of appetite and says he has lost cancer has been stabilized. Which
weight over the past few months. He says electrolyte abnormality can be seen?
that he has passed some blood in his a. Hyperkalemia
urine, however, he had no pain. A recent b. Hypocalcemia
report shows that PSA >5.5ng/ml. how c. Hyponatremia
will you manage this pt? d. Hypernatremia
a. Radical prostatectomy e. Hypomagnesemia
b. TURP
c. Cryosurgery
d. Brachytherapy Ans. The key is C. Hyponatremia. [Likely diagnosis
e. Irradiation is squamous cell lung cancer (as the patient is
smoker) causing SIADH and resulting
hyponatremia].
Ans. The key is A. Radical prostatectomy. This
answer is controversial. [Active Surveillance is 923. A 56yo man who is hypertensive
offered first for Low Gleason score. esp at age of recently underwent a change in meds.
75. and any other treatment depending on
2days later he developed wheezing. Which
extension of tumor will be offered after excluding
mets].
drug can cause this?
a. Atenolol
b. Ramipril
921. A 19yo boy comes to the ED with c. Bendroflumethiazide
pain, swelling and tenderness 2cm distal d. Verapamil
to Lister’s tubercle of radius. Exam: e. Furosemide
proximal pressure on the extended thumb
and index finger is painful. XR: no fx. What
is the next appropriate management for Ans. The key is A. Atenolol. [Beta blockers can
the pt? precipitate asthma attacks].
a. Immobilization with cast
b. Repeat XR
c. MRI 924. A 33yo man has a temp=38.5C,
d. Surgery cough and chest pain on the right side on
e. None inspiration. He also has purulent sputum.
What is the most likely organism to cause
pneumonia in this pt?
Ans. The key is A. Immobilization with cast. [Likely a. Gram +ve diplococcic
diagnosis is scaphoid fracture. Here next b. Coagulase +ve cocci
appropriate management is immobilization with c. PCP cold agglutinins
cast and repeat X-ray in 1 to 2 weeks later when d. AFB
fracture line will become visible mostly as a e. Gram –ve diplococcic
healing line].
c. Meningococcal meningitis
Ans. The key is C. PCP cold agglutinins. It is wrong d. SAH
key! Correct key should be A. Gram +ve e. Cerebral tumor
diplococci. [It is a case of community acquired f. Pneumonia
pneumonia caused by streptococcus
pneumoniae].

Ans. The key is B. Cerebral malaria. [Incubation


925. A young man’s arm was caught in a period of malaria is 7 – 30 days. Malaria
machine. XR showed no fx but arm is very prophylaxis cannot give confirmed protection and
swollen. What is the best tx? there is often failure of prophylaxis. Fever,
a. Plaster cast vomiting, neck stiffness are consistent with
b. Wide splint with upward position cerebral malaria].
c. Analgesics
d. Antibiotics
e. Tetanus prophylaxis 928. A HTN pt on bendroflumethiazide
2.5mg/d has come for his routine checkup.
Exam:
Ans. The key is B. Wide splint with upward
BP=145/85mmHg. Lab: K+=5.9, Na+=137.
position. [If the limb is kept hanging it will
What is the most appropriate
aggravate the swelling as fluid shifts to
dependant part. Here wide splint with upward
management for this pt?
a. Stop meds
position will help the accumulated blood or fluid
b. Continue same dose
to move down and will improve the swelling].
c. Increase the dose
d. Decrease the dose
926. A child was brought in to ED by his e. Repeat the blood test
parents for taking his grand-dad’s meds.
There is an extra systole in the ECG. Which
drug was taken? Ans. The key is E. Repeat the blood test.
a. Digoxin [Bendroflumethiazide causes hyponatremia and
b. Amitryptiline hypokalemia. But the findings are opposite which
c. Atenolol is probably error of test. Hence blood test should
d. Ramipril be repeated to confirm the level of potassium
e. Bendroflumethiazide and sodium].

Ans. The key is A. Digoxin. [Both digoxin and


929. A 65yo man presents with significant
amitryptiline can cause extrasystole].
weight loss and complains of cough, SOB
and chest pain. Exam: left pupil
927. A 5yo child came from Ghana 6wks constricted, drooping of left eyelid. What
ago. 2d ago he developed fever, vomiting is the most likely dx?
and neck stiffness. He had taken malaria a. Pancoast tumor
prophylaxis and had no rash. What is the b. Thoracic outlet syndrome
dx? c. Cervical rib
a. Cerebral abscess d. Pneumonia
b. Cerebral malaria e. Bronchogenic ca
 History of fever
 Tonsillar exudates
Ans. The key is A. Pancoast tumor. [Pancoast  No cough
tumour is the apical lung cancer that is associated  Tender anterior cervical
with destructive lesions of the thoracic inlet and lymphadenopathy. [Source: patient.info].
involvement of the brachial plexus and cervical Indications for tonsillectomy:
sympathetic nerves (the stellate ganglion) leading
to horner’s syndrome].  The child has five or more episodes of
acute sore throat per year, documented
930. A 4yo boy presents with fever, sore by the parent or clinician.
throat and lymphadenopathy. The dx of  Symptoms have been occurring for at
least a year.
tonsillitis has been made. He had 3  The episodes of sore throat have been
episodes last yr. What is the most severe enough to disrupt the child's
appropriate management for this pt? normal behaviour or day-to-day
a. Tonsillectomy functioning.
b. Paracetamol/ibuprofen 931. A pt had passed a 4mm stone in his
c. Oral penicillin V urine. He has a 3mm stone in the renal
d. IV penicillin pelvis found on US. What is the
e. None management?
a. ESWL
b. None
Ans. The key is B. Paracetamol/ibuprofen. c. Dormier basket
d. Surgery
[Explanation: Drugs:
e. PCNL
 Antipyretic analgesics such as
paracetamol and ibuprofen are of value.

 For most patients, antibiotics have little Ans. The key is B. None.
effect on the duration of the condition or  [Stones less than 5 mm in diameter pass
the severity of symptoms. The National spontaneously in up to 80% of people.
Institute for Health and Care Excellence  Stones between 5 mm and 10 mm in
(NICE) suggests that indications for diameter pass spontaneously in about
antibiotics include:[1] 50% of people.
 Stones larger than 1 cm in diameter
 Features of marked systemic usually require intervention (urgent
upset secondary to the acute sore intervention is required if complete
throat. obstruction or infection is present).
 Two thirds of stones that pass
 Unilateral peritonsillitis.
spontaneously will do so within four
 A history of rheumatic fever. weeks of onset of symptoms]. Ref:
patient.info
 An increased risk from acute 932. A 4yo boy presents with fever, severe
infection (such as a child with ear ache, vomiting and anorexia. He also
diabetes mellitus or
has mod tonsillitis. Exam: tympanic
immunodeficiency).
membrane bulging. He came to the GP a
 Acute tonsillitis with three or few days ago and was dx with URTI. What
more of the following Centor is the most appropriate dx?
criteria present: a. OE
b. Acute OM loss. CXR shows bilateral fibrosis and left
c. Serous otitis side pleural effusion.
d. Chronic suppurative OM What is the best inv that will lead to dx?
e. Mastoiditis a. CXR
b. Pleural fluid aspiration of cytology
c. MRI
Ans. The key is B. Acute OM. [High fever, severe d. Pleural biopsy
earache, vomiting, bulging tympanic membrane e. CT
and H/O associated URTI is highly suggestive of
acute OM].
Ans. The key is D. Pleural biopsy. [There is
asbestos exposure in builders and associated
smoking greatly increases the possibility of
933. A 3yo girl presents with complains of
developing mesothelioma and the given
sudden right facial weakness and presentation is typical of mesothelioma. Best
numbness and pain around her ear. There investigation is pleural biopsy].
are no symptoms. What is the most
appropriate dx?
a. SAH 936. During a basketball match, one of
b. Bell’s palsy the players suddenly collapsed to the
c. Stroke ground with coughing and SOB. What is
d. TIA the inv of choice?
e. Subdural hemorrhage a. CXR
b. CT
c. MRI
Ans. The key is B. Bell’s palsy. d. V/Q scan
e. CTPA

934. A 6yo boy fell in the playground and Ans. The key is A. CXR. [Likely diagnosis is
has been holding his forearm complaining pneumothorax. So investigation of choice is CXR].
of pain. Exam: no sign of deformity or
swelling. However, there is minimal
937. A 57yo man having HTN on oral anti-
tenderness on exam. What is the dx?
HTN. However, he is finding it difficult to
a. Fx mid radius
mobilize as he feels dizzy whenever he
b. Fx mid ulnar
c. Fx neck of humerus
tries to get up. What is the most
d. Fx shaft of humerus
appropriate inv for him?
a. Ambulatory BP
e. Green stick fx of distal radius
b. ECG
c. MRI
d. CXR
Ans. The key is E. Green stick fx of distal radius.
e. CT
935. A 62yo man has been smoking about
15 cigarettes/day for 45yrs, and has been Ans. The key is A. Ambulatory BP. [Ambulatory BP
working as a builder since he was 24yo. to document low BP as cause of presenting
He presents with chest pain, SOB, weight symptom. The case seems to be of postural
hypotension and low BP as a result of given anti c. Lateral collateral
hypertensive]. d. Post cruciate
e. Meniscus
938. A 33yo female complains of diplopia
on upright gaze. Exam: ptosis can be seen.
There are no other complains or any Ans. The key is B. Medial collateral. [The valgus
stress test involves placing the leg into extension,
significant PMH. What is the most
with one hand placed as a pivot on the knee.
appropriate inv for him?
With the other hand placed upon the foot
a. Ophthalmoscopy
applying an abducting force, an attempt is then
b. Visual field test
made to force the leg at the knee into valgus. If
c. TFT the knee is seen to open up on the medial side,
d. CT this is indicative of medial collateral ligament
e. Checking red reflex damage].

Ans. The key is E. Checking red reflex. This is a


wrong key. Right key should be D. CT. [A case of 941. A 75yo man comes in complaining of
3rd nerve palsy. So to delineate the cause CT difficulty in passing urine, poor stream
should be done. Red reflex is seen to diagnose and dribbling at the end of voiding and
cataract]. anorexia. US shows bilateral
hydronephrosis.
What is the cause of these findings?
939. A tall rugby player was hit in the a. BPH
chest by a player of the opponent team. b. Renal stones
He developed breathlessness and his face c. Bladder stones
went blue and purple. You have been d. Prostatic ca
called to look at him, how will you e. UTI
manage him?
a. Insert a needle in the 2nd ICS in the mid-
clavicular line Ans. The key is D. Prostatic ca. [Elderly patient of
b. Insert a needle in the 5th ICS in the mid- 75yrs with obstructive symptoms of lower urinary
axillary line tract, bilateral hydronephrosis all can occur in
c. Intubate the pt BPH or ca prostate. Anorexia is clincher here. It is
d. Start CPR a feature of carcinoma rather than BPH].
e. Give oxygen

942. 2h after an appendectomy, a pt


Ans. The key is E. give oxygen. complains of a rapid HR and fever. He says
there is also abdominal pain and pain in
the shoulder area. What is happening to
940. A young woman fell and hit her knee. this pt?
Exam: valgus test +ve. What ligament was a. Intra-abdominal bleeding
most probably injured? b. Anastomotic leak
a. Ant cruciate c. Sepsis
b. Medial collateral d. Intestinal obstruction
945. DM man feels hot, painful lump near
Ans. The key is A. Intra-abdominal bleeding. the anal region. What is the most
probable dx?

943. A 50yo man presents with the a. Anal fissure


complaints of recurrent UTI and b. Abscess
occasional blood in the urine. Some c. Hematoma
unusual cells have been seen in urine on d. Wart
routine exam. e. External hemorrhoids
Which os the following inv would you like
to carry out now?
a. Cystoscopy Ans. The key is B. Abscess. [DM patients are much
b. Urine C&S prone to infection].
c. XR KUB
d. US 946. A 65yo lady with T1DM for the last
e. CBC 20y comes with a tender lump near the
anal opening. She says she also has a
fever. What tx should she get?
Ans. The key is D. US. It is wrong key! Correct key a. I&D + antibiotics
is A. Cystoscopy. [Likely diagnosis is bladder b. IV antibiotics
cancer. Occasional blood in urine and unusual c. C&S of aspirate from swelling
cells (suggestive of malignant cells) suggests the d. Painkillers
diagnosis. It is also true that in bladder cancer e. Cautery of swelling
there occurs frequent uti like symptoms].

Ans. The key is A. I&D + antibiotics.


944. A 28yo drug user presents to ED
collapsed and anuria. His serum
K+=7.5mmol/l. CXR shows early 947. An 80yo DM lady presents with
pulmonary edema. redness and swelling over her right foot. It
What is the next appropriate is tender to touch, warm and glossy. What
management for this pt? are the complications this pt might
a. Urgent hemodialysis develop?
b. IV calcium gluconate a. Meningitis
c. IV insulin + dextrose b. Sepsis
d. Furosemide c. Ulcer
e. IV 0.9% NS d. Gangrene

Ans. The key is B. IV calcium gluconate. [In severe Ans. The key is D. Gangrene.
hyperkalemia IV calcium gluconate is given to
protect the heart from cardiac arrest or life-
threatening arrythmias]. 948. After surgery a pt’s left leg has
become swollen and tender. The diameter
of the calf has increased and passive
movements cause pain. What is the most Ans. The key is A. Paracetamol. [Pain on
probable dx? movement, pain worse at end of day and
a. DVT decreased joint space are characteristic of
b. Lymphedema osteoarthritis supported by no significant raise in
c. Peripheral vascular disease inflammatory marker (CRP=12). 1st line medicine
is paracetamol].
d. Hematoma
e. Superficial thrombophlebitis 951. A 68yo man presents with muscle
weakness. He is not able to climb stairs.
He also complains of mild breathlessness.
Ans. The key is A. DVT. [Swelling, tenderness and He says that he sometimes feels difficulty
enlarged calf diameter are features of DVT in swallowing food. Labs: ALP=216,
supported by positive Homan’s test (pain on
AST=49, ALT=43, CK=417, ESR=16.
passive movement)].
What is the most likely dx?
949. 2h after an appendectomy, a pt a. Polymyositis
complains of a rapid HR and fever. He says b. Polymyalgia rheumatic
there is also abdominal pain and pain in c. Muscular dystrophy
the shoulder area. What is the first step in d. Esophageal carcinoma
the management? e. Osteoarthritis
a. Maintain IV access and give IV fluids
b. Start IV antibiotics
c. Insert NGT for intestinal decompression Ans. The key is A. Polymyositis. [Proxymal
d. Cross match blood myopathy (not able to climb stairs), mild
e. Emergency exploratory laparotomy breathlessness (involvement of thoracic muscles
or interstitial lung disease), dysphagia due to
involvement of the oropharyngeal striated
muscles and upper oesophagus)and raised CK
Ans. The key is A. Maintain IV access and give IV
suggests the diagnosis of Polymyositis].
fluids. [Features given are of internal bleeding! So
maintain iv access and giving iv fluid is the first
step in management of this patient].
952. A 67yo builder presents with a
persistent nodular lesion on upper part of
950. A pregnant woman presents with pinna with some telangiectasia around
knee pain on movements. The pain the lesion. What is the dx?
a. Basal cell
becomes worse at the end of the day.
b. Squamous cell
Radiology shows decreased joint space.
c. Keratocanthoma
Labs: CRP=12.
d. Actinic keratosis
What is the 1st line med?
e. Bowens disease
a. Paracetamol
b. NSAIDs
c. Oral steroid
Ans. The key is A. Basal cell ca. [Any ulcer which is
d. Intra articular steroid
located above the neck is always basal cell
e. DMARDs
carcinoma until proven otherwise. Source:
SAMSON notes].
953. A 68yo pt wakes up with slurred 355. A 24yo woman known to be suffering
speech and right sided weakness. CT from panic disorder presents to the
shows cerebral infarct. What is the most hospital with tingling and numbness in her
appropriate tx? fingers. ABG: pH=7.52, PCO2=2.2kPa,
a. Aspirin PO2=11kPa, Bicarb=20. What is the most
b. Alteplase likely condition?
c. Warfarin a. Acute metabolic alkalosis
d. Clopidogrel b. Acute resp alkalosis
e. Dipyridamole c. Compensated resp alkalosis
d. Compensated metabolic acidosis
e. Acute metabolic acidosis
Ans. The key is B. Alteplase. It is a wrong key! The
correct key is A. Aspirin. [The window period to
administer alteplase is 4.5 hours. If we cannot Ans. The key is B. Acute respiratory alkalosis. [In
certain this period we cannot proceed for panic attack there occurs hyperventilation which
alteplase]. causes washout of CO2 and leads to acute
respiratory alkalosis resulting in raised pH >7.45
(here 7.52), low PCO2 (here 2.2 kPa) with
954. A 73yo man who is recovering from compensatory decrease in HCO3 (here 20meq/l)].
surgery on the left carotid artery in his
neck. He has slurred speech. On protusion
of his tongue, the tip deviated to the left. 956. A 65yo man on dexamethasone
What is the single most appropriate underwent surgery. During and after the
option? surgery, his blood glucose was around 17-
a. Accessory nerve 19mmol/l. What will you give the pt?
b. Facial nerve a. Insulin
c. Glossopharyngeal nerve b. Oral hypoglycemic
d. Hypoglossal nerve c. Remove dexamethasone
e. Vagus nerve d. IV Saline
e. IX dextrose

Ans. The key is D. Hypoglossal nerve. [Testing


function of the nerve is performed by asked the Ans. The key is D. IV saline. [It is estimated that
subject to stick their tongue straight out. If there adults secrete 75-150mg of cortisol in response to
is a loss of innervation to one side, the tongue will major surgery and 50mg a day for minor surgery,
curve toward the affected side, due to unopposed and secretion parallels duration and extent of
action of the opposite genioglossus muscle. If this surgery
is the result of alower motor neuron lesion, the
tongue will be curved toward the damaged side, Patients undergoing a surgical procedure or
combined with the presence of fasciculations or responding to stress, trauma, or an acute illness
atrophy. However, if the deficit is caused by will exhibit an increase in adrenal cortisol
an upper motor neuron lesion, the tongue will be production up to 6-fold normal levels. However,
curved away from the side of the cortical damage, in patients on chronic exogenous steroid therapy,
without the presence of fasciculations or
atrophy of the hypothalamicpituitary-adrenal
atrophy].
(HPA) axis may occur through feedback inhibition,
leading to an inability to respond to stress. It has Surgical patients commonly develop
historically been believed that patients receiving hyperglycemia related to the hypermetabolic
long-term corticosteroids require supplemental, stress response, which increases glucose
perioperative doses, and that failure to provide production and causes insulin resistance.
such coverage in secondary adrenal insufficiency Although hyperglycemia is associated with worse
may lead to an adrenal crisis characterized by outcomes, the treatment of hyperglycemia with
hypotension and cardiovascular collapse. insulin infusions has not provided consistent
benefits. Despite early results, which suggested
*** So who are telling remove dexamethason are decreased mortality and other advantages of
wrong!!!*** “tight” glucose control, later investigations found
patients receiving “physiologic replacement” no benefit or increased mortality when
doses (<10 mg/ day) do not need additional hyperglycemia was aggressively treated with
steroids perioperatively beyond their standard insulin. Because of these conflicting data, the
regimen. Patients receiving doses exceeding the optimal glucose concentration to improve
normal expected stress response for their surgery outcomes in critically ill patients is unknown.
(>150 mg/ day of hydrocortisone equivalent for There is agreement, however, that hypoglycemia
moderate/major surgery, or >50 mg/ day of is an undesirable complication of intensive insulin
hydrocortisone equivalent for minor surgery) also therapy and should be avoided. 
do not need additional coverage beyond their So, “FOR PERIOPERATIVE HYPERGLYCEMIA NO
current therapy, because acute NEED OF INSULIN” just IV saline!!!].
immunosuppressive doses are more than
sufficient to maintain cardiovascular stability
during the operation.
957. A 61yo man who had stroke 2y ago is
on aspirin. He has RA but suffers from
pain and can’t tolerate it. He is taking
senna for constipation.
What is the best med to relieve his pain?
Regular daily dose of more than 10 mg a. DMARDs
prednisolone within the last three months: b. Ibuprofen
c. Co-codamol
i) Minor surgery: 25 mg
d. Paracetamol
hydrocortisone at induction

ii) Moderate surgery Usual


Ans. The key is B. Ibuprofen. [DMARDs are not
pre-operative steroids
painkillers but prevents disease progression in
(Hysterectomy) + 25 mg hydrocortisone long term. Paracetamol is not enough to control
at induction severe pain. Co-codamol will cause constipation.
Patient is already taking laxative! So Co-codamol
+ 100 mg hydrocortisone/day
is not the option. Excluding other Ibuprofen
iii) Major surgery Usual seems to be the best option].
pre-operative steroids

Major trauma, prolonged + 25 mg 958. A young child was brought by his


hydrocortisone at induction mother to the OPD complaining that he
raised the vol of the TV and didn’t respond
surgery + 100 mg
hydrocortisone/day for 2 – 3 days to her when she called him. Exam:
tympanic membrane was dull greyish and
no shadow of handle of malleus. What is
961. A 4yo boy presents with recurrent
the most probable dx?
episodes of self limiting spontaneous
a. Chronic OM
bleeding. Coag test: PT normal, bleeding
b. Acute OM
time normal, APTT prolonged, Factor VIII
c. Secretory OM
d. Otitis externa
decreased. His father and uncle suffer
e. Cholesteatoma
from a similar illness. What is the most
likely dx?
Ans. The key is C. Secretory OM. a. Hemophilia A
b. Hemophilia B
c. Von willebrand’s disease
959. A 48yo woman always socially
d. ITP
withdrawn has stopped going out of the
e. TTP
house. She is afraid to socialize because
she fears that people will criticize her.
What is the most probable dx?
Ans. The key is A. Hemophilia A. [Prolonged APTT
a. Agoraphobia
and decreased factor VIII points towards the
b. PTSD
diagnosis of Hemophilia A].
c. Social anxiety
d. OCD
e. GAD
962. A 53yo lady presents with hot flash
and night sweats. Her LMP was last year.
She had MI recently. What is the most
Ans. The key is C. Social anxiety. [Social anxiety
appropriate management for her?
disorder is a type of complex phobia. This type of
a. Raloxifene
phobia has a disruptive or disabling impact on a
b. Estrogen
person's life. It can severely affect a person's
c. COCP
confidence and self-esteem, interfere with
relationships and impair performance at work or d. Evening primrose
school]. e. Clonidine

960. Post gastric ulcer got perforated Ans. The key is C. COCP. It is a wrong key! Correct
leading to bleeding involving the gastro- key should be E. Clonidine. [In a history of MI
Oestrogen and COCP better avoided. Evening
duodenal artery. Where would fluid
primrose is not used in post menopausal
accumulate in the cavity?
symptoms. Raloxifene is a SERM which rather
a. Left paracolic gutter
worsen hot flash and vasomotor syndrome. On
b. Pelvic cavity
the other hand clonidine is the best option which
c. First part of duodenum
improves hot flash and vasomotor symptoms].
d. Under the diaphragm
e. Retroperitoneal
963. A 73yo man who was a smoker has
quit smoking for the past 3yrs. He now
Ans. D. Under the diaphragm. [Correct option
presents with hoarseness of voice and
probably A. Left paracolic gutter].
cough since past 3wks. XR: mass is visible b. Refer to social services
in the mediastinum. What is the best inv c. Refer to psychology
to confirm the dx? d. CBT
a. Bronchoscopy
b. Thoracoscopy
Ans. The key is B. Refer to social services.
c. US
d. CT thorax
e. LN biopsy 966. A young girl presented to OBGYN
assessment unit with lower abdominal
pain and per vaginal bleeding after a hx of
Ans. The key is E. LN biopsy. hysterosalpingograph as a part of her
infertility tx. Observation:
BP=90/50mmHg, pulse=120bpm, exam
964. A 52yo man known DM presents to revealed rigid abdomen.
ED with sudden onset of pain in the left What is the most appropriate next inv?
loin and hematuria. Inv: 8mm stone in left a. CT
lower ureter. Nifedipine with steroids was b. XR erect and supine
prescribed as initial tx with supportive c. US abdomen
therapy. He returned complaining of d. Coag profile
worsening pain, vomiting with passing of e. CXR
2 stones. Renal function tests indicate
impending ARF. Ans. The key is C. US abdomen. [Likely cause of
How will you manage this pt? bleeding and shock is ruptured fallopian tube for
a. Continue same tx which appropriate next investigation is US
b. Start alpha blocker abdomen].
c. ESWL
d. Percutaneous nephrolithotomy
967. A 21yo woman who is on COCP had
e. Percutaneous nephrostomy
to take azithromycin. What should be
f. Open surgery
advised for her contraception?
a. Using 7d condoms after antibiotics and
avoid pill free break
Ans. The key is E. Percutaneous nephrostomy. b. Using 14d condoms after antibiotics and
[Percutaneous nephrostomy, or
avoid pill free break
nephropyelostomy, is an interventional
c. Using 7d condoms after antibiotics
procedure that is used mainly in the
d. No extra precaution
decompression of the renal collecting system.
e. Using 14d condoms after antibiotics
Percutaneous nephrostomy catheter placement
has been the primary option for the temporary
drainage of an obstructed collecting system. Here
impending ARF indicates obstructive uropathy]. Ans. The key is D. No extra precaution. [Before it
was thought that antibiotics like azithromycin
965. A lady who is alcohol dependent inhibits the enzyme and reduce the efficacy of
wants to quit but wants someone to COCP. But later it was established that practically
encourage her. What would you do? no significant changes occur and so no need of
a. Medication any extra precaution].
c. Increased risk of osteoporosis
d. Increased risk of ovarian ca
968. A 60yo woman presented with radial
fx and had a colle’s fx and supracondylar Ans. The key is A. Increased incidence of
fx in the past. What inv is req to detect her endometrial carcinoma.
possibility of having the same prb later?
a. Dexa scan
971. A 45yo male complains of tremors in
b. MRI
hands. Exam: tremors are absent at rest
c. Nuclear bone scan but present when arms are held
d. CT outstretched and persist on movement.
e. Bone biopsy What is the most probable dx?
a. Parkinsonism
Ans. The key is A. Dexa scan. [The likely cause of b. Benign essential tremor
these multiple fracture is osteoporosis (post c. Cerebellar disease
menopausal women) for which we should do d. Liver failure
Dexa scan to establish the diagnosis]. e. Stroke

Ans. The key is B. Benign essential tremor.


969. A 43yo woman presents with low [Tremors absent at rest and present on
mood, loss of libido, sleep disturbance, outstretched hand and persist on movement are
tiredness, palpitation, chest discomfort, benign essential tremor].
irritability and recurrent worries. What is
the most likely dx? 972. Pregnant lady had her antenatal
a. Seasonal Affective Disorder screening for HIV and Hep B.
b. Mod depression what more antenatal inf should she be
c. Dysthymia screened for?
d. GAD a. Rubella and syphilis
e. Bipolar disorder b. Toxoplasma and rubella
c. Syphilis toxoplasma
d. Hep C & E
Ans. The key is B. Moderate depression. [Mild e. Hep A & C
depression: i)Low mood ii) Anhedonia iii) Guilt iv)
Hopelessness v) Worthlessness vi) Inability to Ans. The key is A. Rubella and syphilis.
concentrate. Tx CBT
973. A young man has been found in the
Moderate depression: Features of mild + vii) Poor park, drunk and brought to the ED by
sleep viii) Poor Appetite ix) Poor libido x) Easy ambulance. He recently lost his job and
fatiguability. Tx Antidepressants
got divorced. He thinks nurses are plotting
Severe depression: Features of moderate + xi) against him. What is the most likely dx?
Suicidal intensions. Tx ECT]. a. Schizoid personality
b. Borderline personality
c. Schizophrenia
970. Which of the following is true for
d. Psychotic depression
tamoxifen? e. Paranoid
a. Increased incidence of endometrial
Ans. The key is C. Schizophrenia. [Patients disease
carcinoma
(schizophrenia) is responsible for his loss of job
b. Increased risk of breast ca
and divorce. He has persecutory delusion which 977. A 26yo political refugee has sought
supports the diagnosis of schizophrenia]. asylum in the UK and complains of poor
974. An elderly man who used to work in conc. He keeps getting thoughts of his
the shipyard industry presented with family whom he saw killed in a political
cough and SOB few weeks to months. He coup. He is unable to sleep and feels
was given salbutamol nebulization and hopeless about his survival. Because of
antibiotics and admitted to the ward. He this he is afraid to go out. What is the
died 3d later. CT: patchy infiltrates, most likely dx?
a. Acute stress disorder
pleural thickening and pleural effusion.
b. PTSD
Why is this a coroner’s case?
c. Social phobia
a. Pt got wrong dx or management
d. OCD
b. Pt died soon after admission
e. GAD
c. Death could be due to occupational
illness Ans. The key is B. PTSD. [Keep getting thoughts of
his family disaster, insomnia, to avoid similar
circumstances he is avoiding going out these are
Ans. The key is C. Death could be due to suggestive of PTSD].
occupational illness.
978. A 2yo boy presented with gradual
975. A 26yo lady came with abdominal swelling of feet and poor feeding. He has
pain, vaginal discharge and low grade gained weight and has dark urine. What is
fever. What is the most likely dx? the single most appropriate inv?
a. HELLP syndrome a. Serum albumin (2nd)
b. Acute PID b. 24h urinary protein (1st)
c. Ectopic pregnancy c. Serum calcium
d. Appendicitis d. BUE
e. Serum glucose

Ans. The key is B. 24 hr urinary protein. [A case of


Ans. The key is B. Acute PID.
nephritic syndrome. So the single most
976. A new screening test has been appropriate investigation is 24 hour urinary
devised to detect early stages of prostate protein].
ca. However, the test tends to dx a lot of 979. A 26yo lady presents with high fever,
ppl with no cancer, although they do have lower abdominal pain and purulent
cancer as dx by other standard tests. vaginal discharge. She looks very unwell.
What is this flaw called? What is the most appropriate
a. False +ve
management?
b. True +ve
a. Tetracycline 250mg QD
c. False –ve
b. Doxycycline 100mg BD and
d. True –ve
metronidazole 400mg BD
e. Poor specificity
c. IV Ceftriaxone 2g with doxycycline 100mg
d. IV ceftriaxone 2g with doxycycline 500mg
e. Ofloxacin 400mg BD and metronidazole
Ans. The key is C. False negative. 400mg BD
Ans. The key is C. IV Ceftriaxone 2g with e. V-ectopics
doxycycline 100mg. [The disease is severe
needing inpatient management with IV
Ceftriaxone 2g + IV doxycycline 100 mg (as Iv Ans. The key is D. Atrial fibrillation. [Ankle
swelling suggests alcoholic cardiomyopathy and
doxycycline is not licensed to use in uk probably
alcoholism is also a well known cause of atrial
oral doxycycline is given).
fibrillation].
980. A 39wk pregnanct woman came to
labor suite 3d after an obstructed labour 983. A young boy has acute scrotal pain
presents with pain and swelling of one leg. for a few hours. Exam: one testis is very
Exam: leg has blue mottline and is cold. painful to touch. He had this kind of pain
What is the dx? before but it was mild and resolved itself
a. DVT within 30mins. What would you do next?
b. Post phlebitis syndrome a. Urgent exploration
c. Embolus b. US
d. Varicose vein c. Antibiotics
e. Herpes gladiatorum d. IV fluids
e. Doppler US
Ans. The key is C. Embolus.

981. An 8yo boy has his tonsils and Ans. The key is A. Urgent exploration. [As sudden
adenoids removed. On the 7th post-op onset of severe pain likely diagnosis is torsion of
day, he comes back to the ED with testis. (infection takes a more prolonged course).
Next step is urgent exploration].
hemoptysis and fever. What is the most
appropriate management? 984. An 8wk pregnant woman presents
a. Admit for IV antibiotics
with persistent vomiting and weight loss.
b. Prescribe oral antibiotics and discharge
Exam: HR=110bpm. Dehydration was
c. Packing
corrected with NS infusion and K+. The
d. Surgery
e. Reassurance
condition didn’t improve so IM cyclizine
was given. She is still vomiting.
What is the next appropriate
Ans. The key is A. Admit for IV antibiotic. [Most management?
secondary hemorrhage occurs due to infection a. IV fluids
which erodes a vessel. So patient should be b. IV antiemetics
admitted for IV antibiotics]. c. IV steroids
982. A 50yo female had swelling in her d. Terminate pregnancy
e. Thiamine
ankles. She is a known alcoholic. Now she
presented with breathlessness and
palpitations.
What is the most likely cause of her Ans. The key is C. IV Steroid. [Steroids may be
used in patient's of hyperemesis gravidarum
condition?
refractory to standard therapy]. 
a. VT
b. SVT
c. A-flutter 985. A 28yo lady presents with
d. A-fib dyspareunia and dysmenorrhea. She is
very obese. She now wants reversible patients are tall males. So the likely diagnosis is
contraceptive method. Turner’s syndrome].
Which of the following will be most
suitable for her?
a. Minera
988. A 32yo woman wants reversible form
b. COCP
of contraception. She has one child
c. POP
delivered by emergency C-section. She
d. Copper T
also suffers from migraine and heavy
e. Barrier method
periods.
What is the most suitable form of
contraception for this lady?
Ans. The key is A. Mirena.
a. COCP
b. Mini pill
c. IUCD
986. A young lady who is 28wks pregnant d. Barrier method
presents with vaginal bleeding. She has e. Abstinence
lost about 200ml of blood. Exam: uterus is
tender. Resuscitation has been done.
What is the most imp inv to establish the Ans. The key is C. IUCD. [In migraine can not give
dx? COCP. In menorrhagia mirena is effective. So the
a. US answer is IUCD (mirena coil)].
b. CT
c. D-dimer 989. A 45yo known hypertensive man
d. Clotting profile presents with hematuria, proteinuria and
e. None edema. What is the definitive dx test for
him?
a. Urine protein
Ans. The key is A. US. [Antepartum hemorrhage. b. Renal biopsy
Most important investigation to establish the dx c. Renal function test
is US]. d. Urine microscopy
e. Serum protein

987. A 14yo girl presents with primary


amenorrhea and a short stature. What is Ans. The key is B. Renal biopsy. [In nephritic
syndrome glomerulus is affected. So renal biopsy
the most likely dx?
is the definitive diagnostic test].
a. Down’s syndrome
b. Klinefeltner’s syndrome
c. Turner’s syndrome 990. A 47yo man presents with
d. Fragile X syndrome proteinuria+, BP=160/95mmHg, small
e. Normal finding kidneys that have smooth renal pelvis.
What is the most probable dx?
a. GN
Ans. The key is C. Turner’s syndrome. [Down’s b. Chronic pyelonephritis
syndrome and Fragile x syndrome don’t have c. Unilateral renal artery stenosis
primary amenorrhea. Klienfelters syndrome d. Multiple myeloma
e. ARF d. Give paracetamol with future doses of
the same vaccine
e. Proceed with standard immunization
Ans. The key is A. GN. [In chr. Pyelonephritis US
schedule
will demonstrate renal scarring and urinalysis will
demonstrate pyuria which are absent here. So
the diagnosis is GN]. Ans. The key is B. Don’t give vaccine. [MMR is live
attenuated vaccine which should be avoided in
AIDS].
991. You are the HO in the hospital and
the lab report of a pt shows
glucose=4mmol/l, K+=5.2mmol/l, 994. A young man presents with sudden,
Na+129mmol/l. what is the most severe pain and swelling in the scrotum.
appropriate management? Exam: one testis seems higher than the
a. NS 0.9% other. What is the most probable dx?
b. NS 0.45% a. Varicocele
c. NS 0.9% and insulin b. Hematocele
d. Insulin c. Testicular tumor
e. Dextrose d. Epidiymo-orchitis
e. Testicular torsion

Ans. The key is A. NS 0.9%


Ans. The key is E. Testicular torsion. [Sudden
severe pain, scrotal swelling and higher position
992. A 27yo man presents with abdominal
of the affected testis is diagnostic of testicular
pain. He says his urine is dark. Exam: torsion. You may get further support from
BP=160/105mmHg. What is the most examining cremesteric reflex which is absent in
appropriate inv? testicular torsion!!].
a. US
b. Renal biopsy
c. CT 995. A 24yo male involved in RTA with XR:
d. Urine protein fx neck of humerus. What is the single
e. Urine microscopy most associated nerve injury?
a. Axillary nerve
b. Radial nerve
Ans. The key is A. US. [Abdominal pain, Dark urine c. Median nerve
(hematuria and hypertension suggests ADPKD. d. Ulnar nerve
The sensitivity of ultrasonography for ADPKD1 is
99% for at-risk patients older than 20 years]. Ans. The key is A. Axillary nerve.

993. A 12m child with AIDS is due for his 996. A 64yo man complains of increasing
MMR vaccination. What is the single most SOB and cough for the past 18m. He
appropriate action? coughs up a Tbsp of mucopurulent sputum
a. Defer immunization for 2wks with occasional specks of blood.
b. Don’t give vaccine What is the most likely underlying cause?
c. Give half dose of vaccine a. Acute bronchitis
b. Bronchiectasis
c. Chronic bronchitis b. Fluphenazine
d. Lung cancer c. Haloperidol
e. Pneumonia d. Paraldehyde
e. Risperidone

Ans. The key is C. Chronic bronchitis. [There may


be specks of blood in sputum in both Ans. The key is C. Haloperidol. It is a wrong key!
bronchiectasis , acute and chronic bronchitis. The correct key is E. Risperidone.
Duration of disease is 18 months, so it is not
[ Protocol of treating agitated violent patient.]
acute bronchitis. Again only one tablespoonful
1. Try to talk to the patient but don't touch him.
sputum does not justify the term copious sputum
2. If he agrees on oral meds. .give oral atypical
of bronhiectasis in which sputum will be much
antipsychotics olanzapine, risp or quetiapine
more in amount. So it is chronic bronchitis].
3. If refusing oral, give IV atypical antipsychotics
preferabbly its Olanzapine or Risperidone
4. If agitation not controlled give IM Lorazepam
997. A 55yo man who is hypertensive 5. If that doesn't work, give IV Haloperidol as last
suddenly lost his vision. The retina is pale resort.
and fovea appears as a bright cherry red 6. If acute sedation is required,for immediate
spot. What is the single most appropriate effect give IV Diazepam.
tx?
Courtesy Dr. Abeera Khan].
a. Pan retinal photocoagulation
b. Corticosteroids
c. Scleral buckling 999. A 65yo man complains of hematuria,
d. Surgical extraction of lens frequency, hesistancy and nocturia. He
e. Pressure over eyeball reports that on certain occasions he finds
it difficult to control the urge to pass
urine. Urine microscopy confirms the
Ans. The key is E. Pressure over eyeball. [In presence of blood but no other features.
central retinal artery occlusion (CRAO) retina What is the most porbable dx?
becomes pale and fovea becomes cherry red. a. BPH
Hypertension is a major cause of CRAO. Apply
b. Bladder ca
direct pressure for 5-15 seconds, then release.
c. Prostatic ca
Repeat several times. Ocular massage can
d. Pyelonephritis
dislodge the embolus to a point further down the
e. Prostatitis
arterial circulation and improve retinal perfusion].

998. A 32yo man with schizophrenia and a Ans. There are two keys A. BPH and C. Prostatic
carcinoma. [There are no constitutional features
hx of violence and distressing auditory
of carcinoma. Bleeding is more common in BPH
hallucinations was admitted to the ward
and occurs in a minority of cases and much less
with aggressive behavior and has already common in prostatic carcinoma. Features given
smashed his room. He is refusing any oral are of prostatism only which favours the
meds. What is the single most appropriate diagnosis of BPH].
injection?
a. Flupenthixol
1000. A 60yo man presents with mass in a. Thyrotoxicosis
the groin. Exam: mass lies below the b. Hyperthyroidism
midpoint of the inguinal ligament and is c. Vocal cord nodules
pulsatile. What is the most probable dx? d. Carcinoma bronchus
a. Direct inguinal hernia e. TB
b. Saphenavarix
Ans. The key is E. TB.
c. Femoral hernia
d. Irreducible hernia 1003. A 30yo woman presents with acute
e. Femoral aneurysm headache. She complains of seeing halos
especially at night. What is the single
Ans. The key is C. Femoral hernia. Controvertial
key! May be femoral aneurism . [Hernia below
most likely defect?
a. Paracentral scotoma
pubic tubercle (below inguinal ligament) is
b. Mono-ocular field loss
femoral hernia. But below mid-inguinal point and
pulsatile mass may be femoral aneurism as c. Tunnel vision
well!!]. d. Central scotoma
e. Cortical blindness
1001. An 82yo man has woken up with
incoherent speech and difficulty in finding Ans. The key is C. Tunnel vision. [The diagnosis is
the right words. Exam: otherwise normal, glaucoma (headache and haloes especially at
night). In glaucoma there occurs tunnel vision].
good comprehension.
Which anatomical site is most likely to be 1004. A 35yo man presents with a
affected? headache that worsens on bending his
a. Broca’s area head forward. What is the most likely dx?
b. Wernicke’s area a. Chronic sinusitis
c. Midbrain b. SAH
d. Parietal cortex c. Migraine
e. Pons d. Cluster headache
e. Tension headache
Ans. The key is A. Brocha’s area. [A person with
expressive aphasia will exhibit halting and Ans. The key is A. Chronic sinusitis.
effortful speech. Speech may only include
important content words. Word comprehension 1005. A 20yo man presents with painful
is preserved. The person may still be understood, swallowing. Exam: trismus and unilateral
but sentences will not be grammatical. This enlargement of his tonsils. The
contrasts with receptive or Wernicke’s aphasia, peritonsilar region is red, inflamed and
which is distinguished by a patient's inability to swollen.
comprehend language or speak with
What is the most appropriate tx?
appropriately meaningful words though fluency
a. Oral antibiotics
may be preserved].
b. IV antibiotics and analgesics
1002. A 25yo woman has a recent cough, c. I&D with antibiotics
hoarseness and swelling in the neck. There d. Analgesics with antipyretics
are several nontender swellings on both e. Tonsillectomy
sides of her neck. She has lost 13kgs. She
Ans. The key is C. I&D with antibiotics. [The dx is
takes recreational drugs. What is the most peritnsillar abscess and tx is I&D with antibiotics].
probable dx?
1006. A 40yo manual worker presents Ans. The key is C. Cerebellum. [Difficulty speech
with a swelling in the groin. Exam: mass is (cerebellar dysarthria) and nystagmus are
found to be just above and lateral to the suggestive of cerebellar lesion].
pubic tubercle. It is reducible. On applying 1009. A 75yo man presents with Bell’s
pressure on the internal ring there is no palsy. His PMH is significant for late onset
cough impulse seen. What is the most asthma and heartcfailure. He also reports
probable dx? to have consulted his GP for generalized
a. Direct inguinal hernia
rash prv. CXR: multiple soft shadows and
b. Indirect inguinal hernia
CBC: eosinophilia.
c. Femoral hernia
What is the single most likely positive
d. Strangulated hernia
antibody?
e. Femoral aneurysm a. P ANCA
Ans. The key is B. Indirect inguinal hernia. b. C ANCA
[Swelling in the groin; mass just above and lateral c. Anti Ro
to the pubic tubercle means inguinal hernia. It is d. Anti DS DNA
reducible. On applying pressure on the internal e. Anti centromere
ring there is no cough impulse; it means the
hernia enters through deep ring, and enters Ans. The key is A. p-ANCA. [The dx is Charg
scrotum passing through the superficial ring. That Strauss Syndrome (CSS).
means it is indirect inguinal hernia].

1007. A 34yo male presents with There are six criteria for dx of CSS:
headache and vomiting. Exam:
1. Asthma (wheezing, expiratory rhonchi).
temp=38.5C, neck stiffness, discharge 2. Eosinophilia of more than 10% in
from left ear and right sided hyper-reflexia peripheral blood.
with an extensor plantar response. What 3. Paranasal sinusitis.
is the most likely dx? 4. Pulmonary infiltrates (may be transient).
a. Cerebral tumor 5. Histological confirmation of vasculitis
b. Meningitis with extravascular eosinophils.
6. Mononeuritis multiplex or
c. Cerebellar tumor
polyneuropathy.
d. Cerebral abscess The presence of four out of six of these features
e. Normal pressure hydrocephalus has a high specificity and sensitivity for the
diagnosis of CSS.
Ans. The key is D. Cerebral abscess.
In the given case there are 1. Bell’s palsy
1008. A 26yo male presents with speech (mononeuritis multiplex) 2. Asthma 3. Multiple
difficulties. Exam: nystagmus. Which soft shadows on CXR (pulmonary infiltrates) 4.
anatomical site is most likely to be Eosinophilia. Presence of these 4 features are
affected? diagnostic of CSS. In CSS 70% patient is p-ANCA
a. Midbrain positive!].
b. Pons
c. Cerebellum 1010. A 50yo man complains of visual
d. Cerebrum prbs and dull pain in the left eye.
e. Vestibule cochlear nerve Fundoscopy reveals papilloedema. He was
dx with MS 2yrs ago. There is no
consensual light reflex of the right eye. Cefotaxime if <55yrs and Cefotaxime + Ampicillin
What is the single most likely defect? if >55yrs ].
a. Paracentral scotoma
1013. An 89yo pt has lung cancer. His
b. Mono-ocular field loss
Na+=122mmol/l. What is the tx for this?
c. Homonymous upper quadrantanopia
7. Demeclocycline
d. Central scotoma
8. Vasopressin
e. Homonymous lower quadrantanopia
9. Restrict fluids
Ans. The key is B. Mono-ocular field loss. 10. Reassure

1011. A 54yo pt wakes up with right sided Ans. There are two keys. A. Demeclocycline and
weakness. His current medication is C. Restrict fluids. *both are correct!![•
Asymptomatic chronic hyponatraemia fluid
bendroflumethiazide for HTN.
restriction is often suffi cient if asymptomatic,
Pulse=92bpm, BP=160/90mmHg. CT
although demeclocycline (ADH antagonist) may
shows left cerebral infarct. What is the be required. Ref: OHCM, 9th edition, page 686].
most appropriate tx?
a. Alteplase
b. Aspirin 1014. A 25yo woman who is 11wks
c. Clopidogrel pregnant had central abdominal pain for
d. Dipyridamole 36h. The pain is now colicky. There is no
e. Simvastatin vaginal bleeding. She has vomited once
and has had an episode of loose motion.
Ans. The key is A. Alteplase. It is a wrong key!
She looks ill, temp=37.8C and there is
Correct option is B. Aspirin. [In stroke window
rebound tenderness in the RIF. What is the
period to use alteplase is 4.5 hours. In the given
history we can no way certain the time of stroke most probable dx?
a. Salpingitis
and we can not proceed for alteplase unless
b. PID
window period is confirmed. Hence we can not
give alteplase but Aspirin]. c. Appendicitis
d. Ovarian torsion
1012. A 33yo man presented to the GP e. Uterine fibroid
with hx of headaches and photophobia.
The GP examines him and finds a rash and
Ans. The key is C. Appendicitis. [No vaginal
is now ringing you at the hospital for
bleeding and pain above pelvis makes pregnancy
advice.
complication less likely. Loose motion, low grade
What would you advice the GP? temperature and positive Mc Burney’s sign makes
a. Send pt home
the diagnosis of appendicitis more probable].
b. Start IV benzylpenicillin
c. Conduct LP
d. Start IV ceftriaxone 1015. A 42yo man presents with stroke.
He is not able to walk straight and his
Ans. The key is B. Start IV benzylpenicillin. [Before speech is slurred. What is the initial
hospitalization IV benzylpenicillin. In hospital appropriate inv?
Ceftriaxone can be given but not in calcium a. CT brain
containing fluid instead give Cefotaxime (NICE).
b. PET brain
According to OHCM hospital management is
c. MRI brain
d. Carotid angiography
e. Monitor for 24h What drug is the appropriate choice?
a. Disulfiram
b. Acamprosate
Ans. The key is C. MRI brain. [Ataxia and slurred
c. Vitamin supplement
speech are features of cerebellar lesion which is
d. Naloxone
posterior fossa organ. For posterior fossa lesion
MRI is the investigation of choice! So key is e. Naltrexone
correct. It is C. MRI brain].
Ans. The key is B. Acamprosate. This is a wrong
***1016. A 24yo woman has severe key! Correct key is A. Disulfiram. [Acramposate is
depression 3m after the birth of her first not a deterrent (which keeps away from drinking
by making it unpleasant) but Disulfirum!]
child. She is breastfeeding but is otherwise
unable to look after the baby and is 1019. A 68yo woman presents to the ED
convinced that her family is likely to kill with confusion. Temp=39.3C and
her. She has no interest in anything and productive cough. Sputum is rusty colored
keeps crying. after 2 days. CXR shows right lower lobe
What is the most appropriate tx? consolidation. What is the most likely
a. Fluoxetine organism?
b. Citalopram a. Streptococcus pneumonia
c. CBT b. Staphylococcus aureus
d. ECT c. Coxiella burnetti
e. Haloperidol d. Mycoplasma pneumonia

Ans. The key is D. ECT. [ [In question it is Ans. The key is A. Streptococcus pneumonia.
mentioned she has severe depression and also in [Typical presentation of streptococcal
history she has delusion of persecution which pneumonia].
makes likely diagnosis of psychotic depression!
However in both severe and psychotic depression 1020. A 70yo man with prostatic cancer
the tx is ECT]. has had severe acute back pain waking
him up at night for 6wks. What is the
1017. A 20yo woman with no prv hx of ear
most appropriate inv?
complains, presents with 1d hx of severe a. MRI spine
pain in the right ear which is extremely b. Radionuclide bone scan
tender to examine. What is the single c. DEXA scan
most likely dx? d. Serum ALP concentration
a. Chondromalasia e. Serum calcium concentration
b. Furuncle
c. Myringitis Ans. The key is B. Radionuclide bone scan. [[MRI
d. OE is good for soft tissue but not for bone. If it was
e. OM radiculopathy, spinal cord compression or
prolapsed disc creating pressure on nerve MRI
Ans. The key is D. OE. [Extreme tenderness to would be fine but not for bony metastasis. Here
examine indicate otitis externa]. investigation of choice is radionuclide bone scan].

1018. A couple has just finished their 1021. An asymptomatic 56yo man who
detox regime and wants a drug with a has never consumed alcohol came for a
pharmacological action to serve as a routine checkup. Exam: increased skin
deterrent when they take alcohol. pigmentation, spider angioma,
cardiomegaly, S3 gallop, liver firm with d. Dosulepin
8cm span, no ascites. He is in the risk of e. Diazepam
which condition?
a. Cerebellar degeneration
b. Werniecke’s encephalopathy Ans. The key is B. Citalopram. [Mild depression =
c. Renal failure CBT, Moderate depression = Antidepressant,
d. Hepatoma Severe depression & Psychotic depression = ECT,
e. Hepatic vein thrombosis Amitryptiline and Dosulepine causes urinary
retention (which is comperatively less in
Ans. The key is D. Hepatoma. [Increased skin citalopram) so not suitable in a patient with lower
pigmentation, features of chronic liver disease urinary symptoms. Hence Citalopram is the
(spider angioma), Cardiomyopathy (cardiomegaly, preferred option].
S3 gallop), Enlarged liver these are suggestive of
Haemochromatosis. (OHCM: Slate-grey skin
pigmentation; signs of chronic liver disease; 1024. A 48yo pt after surgical removal of
hepatomegaly; cirrhosis; dilated cardiomyopathy;
mandibular ca presents with perioral
osteoporosis; also endocrinopathies are features
paresthesia and severe pain which is not
of haemochromatosis). If cirrhosis, 22–30% get
hepatocellular cancer].
relieved by oral morphine.
What is the next step in treating this pt?
a. Oral amitryptiline (2nd line)
b. Oral oxycodone
1022. A 39yo male presents with visual
c. PCA
symptoms. Ophthalmoscopy shows
d. IV morphine
papilloedema. Which anatomical site is
e. Fentanyl patch
most likely to be affected?
f. Gabapentine (1st line)
a. Optic nerve
b. Optic disc
c. Optic radiation
Ans. The key is F. Gabapentine.
d. Occulomotor nerve
e. Optic chiasma

1025. A 34yo man was slapped over his


Ans. The key is B. Optic disc.
right ear in a fight. There is blood coming
from his external auditory canal and he
has pain, deafness and ringing in his ears.
1023. A 75yo man has been attending the What is the most appropriate initial inv?
a. CT
clinic for lower urinary tract symptoms.
b. MRI
His mood is very low and he says he feels
c. Otoscopy
unhappy, anxious and unable to sleep. He
d. Skull XR
has been dx with moderate depression.
e. Facial XR
What tx would be most effective for this
pt?
a. Amitriptyline
Ans. The key is A CT. It is a wrong key! Correct key
b. Citalopram
is C. Otoscopy. [Traumatic perforation of the
c. CBT
tympanic membrane (TM) can cause pain,
bleeding, hearing loss, tinnitus, and vertigo. 1028. A 27yo lady after C-section
Diagnosis is based on otoscopy. Treatment often developed epigastric pain after 8h.
is unnecessary. Antibiotics may be needed for What is the appropriate inv?
infection. Surgery may be needed for perforations a. ABG
persisting > 2 mo, disruption of the ossicular b. Coag profile
chain, or injuries affecting the inner ear.
c. Liver enzyme
Reference: Merck Manual (Prefessional version)
d. Liver biopsy
[www.merckmanuals.com/professional/ear,-
nose,-and-throat-disorders/middle-ear-and- Ans. The key is C. Liver enzyme. [Epigastric pain is
tympanic-membrane-disorders/traumatic- a warning sign of HELLP syndrome. So to rule out
perforation-of-the-tympanic-membrane]. HELLP syndrome Liver enzymes should be done].

1029. A 35yo woman presents with visual


1026. A 45yo man has developed an problems. CT brain reveals pituitary
annular rash with a scaly edge on his tumor. What is the single most likely
thigh. The rash has been spreading over defect?
the last 3wks. He has some general aches a. Homonymous hemianopia
b. Homonymous upper quadrantopia
and pains.
c. Bitemporal hemianopia
What is the single most useful
d. Cortical blindness
investigation?
a. ANA e. Homonymous lower quadrantopia
b. Biopsy lesion Ans. The key is C. Bitemporal hemianopia.
c. Lyme antibodies [pitutary tumour by pressing optic chiasma
d. Skin scrap for mycology causes bitemporal hemianopia].
e. Skin swab for bacteria
1030. A 45yo heroin addict was involved
in a car crash and is now paraplegic.
Ans. The key is C. Lyme antibodies. [Spreading During the 1st week of hospital stay he
annular rash suggests erythema migrans of lyme cried everyday because he couldn’t
disease]. remember the accident. What is the most
likely dx?
1027. A 80yo man with prostatic cancer a. PTSD
has confusion, thirst, abdominal pain and b. Severe depression
an abnormal ECG. What is the most c. Organic brain damage
appropriate inv?
a. MRI spine Ans. The key is C. organic brain damage.
b. Radionuclide bone scan
1031. A pt with T1DM has a fundus
c. DEXA scan
showing micro-aneurysm and hard
d. Serum ALP concentration
exudate.
e. Serum calcium concentration
What is the single most likely dx?
Ans. The key is E. Serum calcium concentration. a. Macular degeneration
[Confusion, thirst, abdominal pain and ECG b. Hypertensive retinopathy
changes (marked shortening of QT interval) are c. MS
features of hypercalcemia. So E. Serum calcium d. Diabetic background
concentration is the likely option]. e. Proliferative DM retinopathy
Ans. The key is D. Diabetic background malabsorption and distended abdomen
retinopathy. [Microenurism, hard exudates are (sometimes with everted umbilicus) with
seen in background diabetic retinopathy]. frequent offensive smelly stools are seen in
coeliac disease. Hence endomyseal/alpha gliadin
antibody should be done].
1032. A 62yo man has multiple liver mets
due to adenocarcinoma with an unknown 1034. A 78yo woman is brought to the
primary. He is deeply jaundiced and has hospital complaining of back pain and is
ascites with edema upto the buttocks. He referred to the surgeon. She has been
is now drowsy and his family are worried saying that her mother is due to visit her
that he is not drinking enough. His meds today and that somebody must have
include: haloperidol 1.5mg, lactulose broken her lower back as she is in agony.
10ml. Bloods taken 3d ago: electrolytes Labs: creatinine=295mmol/l,
normal, urea=6.5mmol/l, calcium=3.03mmol/l. Which inv is most
creatinine=89mmol/l, likely to lead to a dx?
calcium=2.04mmol/l, albumin=17g/L, a. US KUB
total bilirubin=189mmol/l. What is the b. XR Spine
single most appropriate management of c. IVU
d. Bence-Jones Protein
his fluid intake?
a. Albumin infusion e. Mental state exam
b. Crystalloids IV
Ans. The key is D. Bence-Jones protein. [Severe
c. Crystalloids SC back pain, high calcium level and renal
d. Fluids via NGT impairment are typical of multiple myeloma].
e. Fluids PO
1035. A 40yo woman presents with
dysphagia. Exam: febrile with erythema
Ans. The key is A. Albumin infusion. [Here serum and middle neck swelling. What is the best
albumin is 17. So the patient developed ascites management strategy?
and oedema secondary to reduced plasma a. IV antibiotics and drainage
colloidal osmotic pressure. Albumin infusion can b. Antipyretics
help rise the colloidal osmotic pressure and c. XR neck
improve the patients condition]. d. Endoscopic diverticulectomy
e. I&D
1033. A 2yo with atrophy of the buttocks,
distended abdomen with frequent Ans. The key is A. IV antibiotics and drainage.
offensive smelly stool. Choose the single [Neck abscess. Treated with IV antibiotics and
most likely inv? drainage].
a. Upper GI endoscopy
1036. A young lady presents with
b. Endomyseal/alpha glidin antibody
gradually worsening headaches, visual
c. Sweat test
d. Colonscopy
disturbance, and lack of energy. MRI
e. Stool culture
shows 15mm tumor in the pituitary fossa.
What is the tx of choice?
a. Radiotherapy
b. Octreotide
Ans. The key is B. Endomyseal/alphagliadin
c. Reassurance and f/u after 6m
antibody. [Atrophy of buttock due to fat
d. Surgery c. PCA
e. Chemotherapy d. IV morphine
e. Oral morphine
Ans. The key is D. Surgery. [Hormonically active
tumour, tumour causing pressure symptoms (like Ans. The key is E. Oral morphine.
headache, visual disturbance) and large size of
tumour are indications for surgical removal]. 1040. A 6wk baby with vomiting,
irritability and palpable mass in the
1037. A man with dementia has an abdomen on feeding. Choose the single
ulcerative lesion on his forehead. He most likely inv?
wants it removed so ‘it can help improve a. Upper GI endoscopy
his memory’. Wife says he is not fit to give b. Barium meal
consent. What will you do? c. US
a. Get letter signed from the GP d. CT abdomen
b. Get letter signed from the wife e. Barium enema
c. Get letter signed from the pt
d. Refer to psychiatrist to assess the mental
capacity to give consent Ans. The key is C. US. [Palpable mass in the
abdomen “ON FEEDING” (just after feeding the
Ans. The key is D. Refer to psychiatrist to assess
mass becomes more prominent and can be
the mental capacity to give consent
palpated more easily) is suggestive of pyloric
1038. A pt with flame shaped hemorrhage stenosis. US is used to diagnose this].
on long term tx with nifedipine. 1041. A 79yo man who is being treated
What is the single most likely dx? with GnRH antagonist for proven
a. Macular degeneration
adenocarcinoma of the prostate attends a
b. HTN retinopathy
follow up session.
c. MS
What is the most appropriate inv?
d. DM background
a. Serum AFP
e. Proliferative DM retinopathy
b. Serum PSA
f. SLE
c. Serum acid phosphates conc
Ans. The key is B. HTN retinopathy. [Stages of d. Serum ALP isoenzyme conc
hypertensive retinopathy: e. Trans rectal US

1. Grade 1: Silver wiring

2. Grade 2: 1 + AV nipping Ans. The key is B. Serum PSA. [Serum PSA is used
to assess the progress of treatment. (If a man’s
3. Grade 3: 2 + flame shaped
PSA level rises after prostate cancer treatment,
hemorrhage
his doctor will consider a number of factors
4. Grade 4: 3 + optic disc before recommending further treatment.
edema + macular star]. Additional treatment based on a single PSA test is
not recommended. Instead, a rising trend in PSA
1039. A pt whose pain is not relieved by level over time in combination with other
oral codeine. What is the best findings, such as an abnormal result on imaging
management? tests, may lead a man’s doctor to recommend
a. Oral oxycodone further treatment)].
b. Co-codamol
1042. A middle aged woman has some anorexia (BMI 15–17.5, no evidence of system
weakness of hand after an injury. Which failure) routine referral can be to
vertebra will be the lowest to be included the local community mental health team
on cervical XR to dx the injury? (CMHT)/adolescent unit or Eating Disorder Unit
a. C7/T1 (EDU) if available. So the key is A. Eating disorder
b. C8/T1 clinic]. OHCS, 9th edition, page-349.
c. C5/C6
d. C6/C7 1045. A 36yo woman has an injury to the
right external laryngeal nerve during a
Ans. The key is A. C7/T1. thyroid surgery. What symptom would be
expected in this pt?
1043. A 50yo man with a known hx of
a. Stridor
stroke. He can’t remember anything about b. Hoarseness
his life. What is the single most likely c. Aphonia
defect? d. Dysphonia
a. Homonymous hemianopia
e. Aphasia
b. Homonymous upper quadrantanopia
c. Bitemporal hemianopia
d. Binasal hemianopia Ans. The key is D. Dysphonia. [External laryngeal
e. Homonymous lower quadrantanopia nerve (smaller, external branch of the superior
laryngeal nerve) lesion causes mono toned voice
(loss of ability to produce pitched sound) that is
Ans. The key is B. Homonymous upper dysphonia].
quadrantanopia. [Memory processing occur in
temporal lobe. So as the patient lost memory his 1046. A 75yo woman has weakness of the
temporal lobe is involved. We know temporal left side of her face. She has had a painful
lobe lesion causes “homonymous upper ear for 48h. There are pustules in the left
quadrantanopia” which is the key here!] ear canal and on the eardrum. What is the
1044. An 18yo girl has been dx with single most likely dx?
a. Chronic serous OM
anorexia nervosa and has mild depressive
b. Herpes zoster infection
symptoms. She has cut down her food
c. Impacted earwax
intake for the last 18m and exercises 2h
d. Perforation of eardrum
everyday. Her BMI=15.5, BP=90/60mmHg.
e. Presbycusis
What would be the single most
appropriate management? Ans. The key is B. Herpes zoster infection. [A case
a. Refer to eating disorder clinic of Ramsay Hunt syndrome defined as an acute
b. Refer to psychodynamic therapy peripheral facial neuropathy associated with
c. Refer to acute medical team erythematous vesicular rash of the skin of the ear
d. Prescribe antidepressant canal, auricle (also termed herpes zoster oticus),
and/or mucous membrane of the oropharynx
caused by Herpes zoster infection].
Ans. The key is C. Refer to acute medical team.
This is a wrong key! Correct key is A. Eating 1047. An 8wk baby boy is noted to be
disorder unit. [Anorexia nervosa is moderate jaundiced. He is breast-feeding well and
when BMI is 15-17.5 as in given case. In moderate has gained 300g since birth. His stools are
yellow and his urine is pale straw colored.
What is the most likely dx? 8mm in size. Which tx modality will be
a. Galactosemia most effective?
b. Biliary atresia a. Fluids and alpha blockers
c. G6PD deficiency b. ESWL
d. Breast milk jaundice c. CCB
e. Congenital viral infection d. Dormier basket
e. PCNL
Ans. The key is A. Galactosemia. [Biliary atresia
causes obstructive picture where stools are pale Ans. The key is B. ESWL. [Though for 8 mm stone
and urine becomes dark. Hemolytic disorder like we can use medical expulsive therapy but for this
G6PD or spherocytosis causes appearance of patient with agonizing pain “most effective”
jaundice in 1st 24 hours. In breastmilk jaundice, therapy seems to be ESWL].
jaundice develops in 2nd week. Also congenital
viral infection (TORCH) occurs in 1st 24 hours. 1051. A 37yo woman believes that her
Here there is failure to thrive also. So likely cause neighbours have been using her shower
in the presented case is Galactosemia]. while she is away from home. Her 42yo
partner is convinced about this and calls
1048. A lady developed breast abscess
the police.
after delivery. What is the most likely
What term best describes this situ?
organism? a. Capgras syndrome
a. Staph aureus
b. Cotard syndrome
b. Staph albus
c. Delusion of persecution
c. GBS
d. Folie a deux
d. Strep pyogenes
e. Munchausen syndrome
e. Strep faecalis
Ans. The key is D. Folie a deux. [Folie a deux is
Ans. The key is A. Staphylococcus aureus. symptoms of a delusional belief
1049. A 32yo man suffering from MS and hallucinations that are transmitted from one
individual to another as here from wife to
presents with blurring of vision.
husband].
Ophthalmoscopy shows pallor of the optic
disc. Which anatomical site is most likely 1052. A 45yo woman has dull pain in her
to be affected? right ear which has been present for
a. Optic nerve several weeks. There is no discharge.
b. Optic disc Chewing is uncomfortable and her
c. Optic radiation husband has noticed that she grinds her
d. Trigeminal teeth during sleep. The eardrum appears
e. Oculomotor nerve normal.
Ans. The key is A. Optic nerve. [ MS--optic What is the single most likely dx?
neuropathy which affects optic nerve and long a. Dental caries
standing optic neuropathy can be seen as pallor b. Mumps
of optic disc in opthalmoscope]. c. OM
d. Temporomandibular joint pain
1050. A 23yo man presents with severe e. Trigeminal neuralgia
pain in the right flank radiating to his
groin. He is rolling about on the floor. An
IVU confirms a stone in the ureter which is
Ans. D. Temporomandibular joint pain. lesion also. These features only can be explained
[Temporomandibular joint pain may cause pain in by cranial nerve involvement in brainstem lesion].
ear and teeth grinding is a recognized cause of
this symptom]. 1055. A 30yo woman has experienced
restlessness, muscle tension and sleep
disturbance on most days over the last
1053. A 42yo lady had corrective surgery 6m. She worries excessively about a
for cyanotic congenital heart disease at number of everyday events and activities
the age of 3y, after a palliative operation and is unable to control these feelings
during infancy. There is a parasternal which are impairing her ability to hold
impulse and an early diastolic murmur. down her job. What is the most likely dx?
a. Panic disorder
What is the most probable dx?
a. Aortic regurgitation b. GAD
b. Ischemic mitral regurgitation c. Pheochromocytoma
c. Aortic stenosis d. Acute stress disorder
d. Pulmonary stenosis e. Social phobia
e. Pulmonary regurgitation
Ans. The key is B. GAD. [In GAD patient is worried
about different number of events every day.
Almost everything triggers the anxiety].
Ans. The key is E. Pulmonary regurgitation. [In the
present day, some patients with tetralogy of 1056. Which of the following is not a
Fallot have survived for longer than 15-20 years degenerative corneal disease?
after their first operation. The major problem
a. Band keratopathy
encountered by these individuals is the
b. Marginal dystropathy
development of pulmonary valvular regurgitation.
It appears that a number of these individuals c. Fatty/lipid degeneration
require pulmonary valve replacement (ref: d. Mooren’s ulcer
Medscape)]. e. Keratoconus

Ans. The key is D. Moorens ulcer. [All other


options are degenrative disease of cornea except
1054. A 45yo lady presents with hx of Mooren’s ulcer which is a type of ulcerative
double vision and facial numbness. keratitis].
Which anatomical site is most likely to be
affected? 1057. A 30yo man presents to hosp
a. Cerebral cortex complaining that his urine has been very
b. Trigeminal nerve dark recently, resembling coffee at worst.
c. Oculomotor nerve He has been under the weather 2wks back
d. Brain stem and had taken a few days off work with a
e. Basal ganglia sore throat and coryzal symptoms. Urine
dipstick in hosp returns highly positive for
Ans. The key is D. Brain stem. [Cerebral cortex will
blood and protein. He is admitted for
not cause features of cranial nerve lesions.
supportive management and is scheduled
Trigeminal nerve: ophthalmic and maxillary
divisions are pure sensory hence it is also not the
for a renal biopsy, which shows mesangial
answer. Oculomotor does not cause facial proliferation with a positive immune-
numbness. These features are not of basal ganglia
flurescence pattern. What is the most c. HSP
probable dx? d. ALL
a. Membranous glomerulonephropathy e. HUS
b. SLE
c. Wegener’s granulomatosis
d. Post – strep GN Ans. The key is C. HSP. [Rash over buttock,
e. IgA nephropathy abdominal pain and vomiting, blood in urine or
stool suggest HSP].

Ans. The key is D. Post- strep GN. [Nephritic


picture, H/O sorethroat 2 weeks before 1060. A man with hx of fall had confusion
presentation and biopsy reports are suggestive of and laceration mark on the head. Which is
post streptococcal glomerulonephritis]. the most appropriate vessel affected?
a. Basiliary artery
b. Middle meningeal artery
1058. A 65yo lady presents with a 6h hx of c. Vertebral artery
facial droop and weakness in the left sied d. Diploic vein
of her body. What single agent will she be
prescribed for her whole life?
a. Clopidogrel Ans. The key is B. Middle meningeal artery.
b. Altepase [Considerable head injury and lucid interval
c. Aspirin (several episodes of transient confusion)points
d. Labetalol towards extradural hematoma which occurs
frequently from bleeding from middle meningeal
artery. Subdural hematoma usually occur due to
bleeding vein].
Ans. The key is A. Clopidogrel. [Ongoing
symptoms are considered as stroke (unless
completely resolved within 24 hours). Hence
Clopidogrel is the answer here as they want life 1061. A 72yo lady is drowsy and her
long!]. relatives want to take her home. She has
been prescribed diazepam 2.5mg. What is
the best delivery route?
1059. A 10yo boy is brought to the hosp a. Oral
with a rash over his buttocks a/w b. IV
abdominal pain and vomiting. In the ED, c. IM
he is accompanied by his mother and d. Per rectal
stepfather. His mother had left him for the e. SC
weekend with the stepfather and was
called to come back from holiday as he
started to have some hematuria with the Ans. The key is D. Per rectal.
rash. Social services had been notified on
arrive to hospital. What is the most
probably dx? 1062. A nonsmoker who has worked in
a. NAI coal mines for 20yrs presents with
b. ITP gradually increasing SOB, limited exercise
tolerance and a dry cough. His CXR shows
round fibrotic tissue demonstrating a
1065. A 24yo schizophrenic has been
mixed restrictive and obstructive
under antipsychotic tx for the last 1 yr and
ventilator defect with irreversible airflow
now complains of ED Erectyl Dysfunction).
limitation and reduced gas transfer.
Which drug is most likely to have caused
What is the single most appropriate dx?
this?
a. Churg-strauss syndrome
a. Fluoxetine
b. Cryptogenic organizing
b. Citalopram
c. Extrinsic allergic alveolitis
c. Clozapine
d. Good pasture’s syndrome
d. Haloperidol
e. Progressive massive fibrosis
e. Risperidone
f. Molluscum

Ans. The key is D. Haloperidol. [Haloperidol, raises


Ans. E. Progressive massive fibrosis. [Coal miners
serum prolactin levels to 20~40 ng/ml in
pneumoconiosis].
therapeutic doses causing erectile dysfunction].

*1063. A pt was complaining of pain 1066. What is the most likely dx based on
within 6h after his appendectomy for
this ECG
gangrenous appendix. What med is the
best option for his pain relief?
a. IV morphine
b. Diclofenac per rectal
c. PCA
d. Tramadol

Ans. The key is D. Tramadol.

1064. A 62yo farmer presents with a


persistent firm irregular lesion on upper
part of pinna which grew over the last few
months. What is the most appropriate dx?
a. Basal cell
b. Squamous cell
a. Normal
c. Keratocanthoma
b. VT
c. Sinus Tachycardia
d. WPW syndrome
Ans. The key is A. Basal cell. [Any ulcer which is e. A-fib
located above the neck is always basal cell
carcinoma until proven otherwise. (samson
note)].
Ans. The key is C. Sinus Tachycardia.
Ans. The key is B. Secure airway. [Probable
epiglottitis. So airway should be secured].
1067. A 45yo woman has recently been dx
with MS and has been started on oral
steroids. She is brought to the hosp after 1070. A pt presents with hemoptysis 7d
having ingested 100 paracetamol tablets after tonsillectomy and adenoidectomy.
4h ago. She is refusing all med tx. What is What is the next step of management?
the next best step? a. Explore again
a. Observe b. Pack it
b. Refer to psychiatrist to assess pts ability c. Oral antibiotics and discharge
to refuse tx d. Admit and IV antibiotics
c. Gastric lavage e. Ice cream and cold fluid
d. Activated charcoal
e. Refer to social worker
Ans. The key is D. Admit and IV antibiotics.
[secondary hemorrhage can occur due to
Ans. The key is B. Refer to psychiatrist to assess infection causing sloughing out of part of arterial
pt’s ability to refuse tx. wall due to infection. It is managed by iv
antibiotics folloing admitting the patient to
hspital].
1068. A 44yo obese pt with findings:
FBS=6mmol/l, OGTT=10mmol/l. What is
the most likely dx? 1071. A 55yo man presents with swelling
a. Impaired glucose tolerance at the angle of the mandible which is
b. Diabetes insipidus progressively increasingcin size and it’s
c. T1DM mobile for 6m. What is the most probable
d. T2DM dx?
e. MODY a. Benign parotid
b. Mandible tumor
c. Tonsillar carcinoma
Ans. The key is A. Impaired glucose tolerance.
[Impaired glucose tolerance is 2-hours glucose
level 140-199 mg/dl (7.8 to 11.0 mmol/l) on the Ans. The key is A. Benign parotid. [Mandibular
75-g oral glucose tolerance test]. tumour or tonsillar carcinoma would be not be so
mobile after such time period and Benig parotid is
likely diagnosis].
1069. A child distressed with fever, stridor
and unable to swallow saliva. His
RR=40bpm. What is the initial step that 1072. A 61yo man, known smoker, comes
needs to be taken? to the hospital with complaints of painless
a. Examine throat hematuria, urgency and dysuria. He has
b. Secure airway been worried about his loss of weight and
c. Keep him laid flat reduced general activity. Which inv would
d. IV penicillin be diagnostic of his condition?
a. Urine microscopy
b. IVU ALP are normal. In osteomalacia Ca+ low, PO4³-
c. CT low and ALP is high. In Pagets disease Ca+ and
d. Cystoscopy PO4³- normal and ALP markedly raised].
e. US abdomen
f. KUB
g. Cystoscopy with biopsy 1075. A 54yo lady presents with sudden
h. Mid stream urine for culture severe pain in the left half of her skull. She
i. Trans rectal US also complains of pain around her jaw.
What is the most likely next step?
Ans. The key is G. Cystoscopy with biopsy. a. CT
[Painless hematuria in elderly man is due to Ca of b. MRI
bladder usually and in bladder cancer there are c. Fundoscopy
UTI like symptoms like urgency, dysuria. Also d. ESR
constitutional symptom like weight loss and e. Temporal artery biopsy
reduced genral activity is feature of carinoma].

Ans. The key is D. ESR. [Elderly lady with severe


pain in the left half of her skull and pain around
1073. An 8wk pregnant lady is brough to
jaw is suspicious of Giant cell or Temporal
the ED due to severe vomiting. She was
arteritis. ESR if markly raised supports the
administered IV fluids and oral anti- diagnosis of temporal arteritis].
emetics. She still can’t tolderate anything
orally. What is the next best tx?
a. IV feeding 1076. A 7yo school boy has been dx with
b. IV antiemetics meningococcal meningitis. What is the
c. Termination of pregnancy advice for schoolmates and staff?
d. PPI a. Rifampicin for the whole class and family
e. IV steroid b. Rifampicin for the whole school and
family
c. Meningococcal vaccine for the family
Ans. The key is IV antiemetics. [If oral antiemetics d. Benzylpenicillin
are not tolerated IV antiemetics are tried]. e. IV cefotaxime

Ans. The key is A. Rifampicin for the whole class


1074. A 48yo man presents with bone and family. [In contacts of meningococcal
pain. Labs: ALP=high, phosphate=normal. meningitis chemoprophylaxis is given with
What is the most likely dx? Rifampicin].
a. Osteoporosis
b. Osteomalacia
1077. A pt came with dyskaryosis to the
c. Paget’s disease
d. Fx
OPD. She is a heavy smoker and alcoholic.
e. Myeloma
Cervical smear shows abnormal cells.
What is the best advice for her?
a. Colposcopy
Ans. The key is C. Paget’s disease. [Bone pain, b. Biopsy
high ALP and normal phosphate are suggestive of c. Endocervical sample
pagets disease. In osteoporosis Ca+, PO4³- and d. Repeat after 4m
e. None 1080. A 64yo man with multiple myeloma
f. Cone biopsy has been vomiting since the past 2days.
Labs:
Ans. The key is A. Colposcopy. [Patient with
dyscariosis should go for colposcopy].
Ca2+=3.2mmol/l, K+=5mmol/l,
Na+=149mmol/l and PCV=55%.
What is the most appropriate next step?
a. IV insulin
1078. Pt with pain and swelling in left leg
b. IV calcium gluconate
and thigh up to the level of inguinal
c. IV fluids
ligament. Where is the level of occlusion?
d. IV bisphosphonates
a. Femoro-popliteal artery
e. Oral bisphosphonates
b. Left common iliac artery
c. Aortoiliac artery
d. Femoral artery
e. Profound femoral artery Ans. The key is C. IV fluids. [Multiple myeloma
itself is a cause of vomiting and also associated
hypercalcemia can cause sickness. As the patient
is vomiting for 2 days there may be considerable
Ans. There are two keys B. Left common iliac dehydration and also the hypercalcemia needs
artery. and D. Femoral artery. Correct key is B. treatment with IV fluid].
Left common iliac artery. [General rule is
occlusion occurs in proximal bifarcation. So the
correct key is B. Left common iliac artery. That is
occlusion in the bifarcation of left common iliac
1081. A 30yo man from Australia returned
artery (as femoral artery bifarcates more distally from a business trip to Indonesia 6d ago
to superficial and deep branches)]. presenting with complaints of fever, joint
and muscle ache and headache, in
particular behind the eye for the past 2
1079. A 65yo man presents with dyspnea days. What is the most probable dx?
3d after an MI. On auscultation he has a a. Malaria
pansystolic murmur at the apex radiating b. Chicken pox
to the axilla. What is the most likely dx? c. TB
a. Ruptured papillary muscle d. Lyme’s disease
b. Ventricular aneurysm e. Dengue
c. Pericarditis
d. Pericardial effusion Ans. The key is E. Dengue. [Fever, arthralgia,
e. VSD myalgia, headache these are common in dengue
fever. Particularly retro-orbital pain is well
recognized feature of dengue fever].
Ans. The key is A. Ruptured papillary muscle.
[Pansystolic murmur in apex radiating to axilla
1082. A lady came for OBGYN assessment
may be due to mitral regurgitation secondary to
papillary muscle rupture]. unit with hx of 8wk pregnancy and
bleeding per vagina for last 2 days. On
bimanual exam, uterus =8wks in size. On
speculum exam, cervical os is closed. How
do you confirm the viability of the fetus?
a. Transvaginal US 1085. A person doesn’t go outside the
b. Serum BHCG home because he thinks that people will
c. Urinary BHCG look at him and talk about him. He finds it
d. Abdominal US difficult to associate with his peers in a
e. Per speculum exam restaurant or under social settings. What
is the most likely dx?
a. Agoraphobia
Ans. The key is A. Transvaginal US. [Viability of
the fetus is better assessed with transvaginal US. b. GAD
TVU can be done throught pregnancy. It should c. Panic disorder
be avoided once the membrane is ruptured to d. Adjustment disorder
avoid chance of fetal infection. It also should be e. Social phobia
avoided in placenta previa].

Ans. The key is E. Social phobia.


1083. A 24yo lady has been low after the
death of her husband and had stopped
contacting her family. She was started on
1086. A 63yo man presented with sudden
SSRI tx and starts feeling better after a
onset of severe dyspnea, orthopnea,
few months. On discontinuating the meds
raised JVP and bilateral basal crackles 3d
she starts feeling that she has developed
after an episode of MI. A dx of acute
cancer just like her husband. What is the
congestive cardiac failure was made and
most appropriate next step?
a. Continue SSRI IV furosemide was started for this pt.
b. Add TCA What electrolyte abnormality is expected?
a. High Na+, Low K+
c. Neuropsychiatric analysis
b. Low Na+, High K+
d. CBT
c. Low Na+, Low K+
e. Antipsychotics
d. High Na+, High K+
e. Low Na+, Normal K+
Ans. The key is C. Neuropsychiatric analysis.

1084. A 24yo male who is sexually active Ans. The key is C. Low Na+, Low K+. [Frusemide
with other males with hx of discharge per causes hyponatremia, hypochloremic alkalosis,
hypokalemia, hypomagnesemia and
urethra. Dx of chlamydia has been made.
hypocalcemia].
What is the possible complication if left
untreated?
a. Orchitis
b. Balanitis
1087. A 70yo hypertensive white british
c. Epididymo-orchitis
man on thiazide diuretics needs a 2nd
d. Acute abdomen drug to control his BP. Which one of the
following is the best choice for him?
a. Amlodipine (CCB)
b. Enapril (ACEi)
Ans. The key is C. Epididymo-orchitis.
c. Propranolol (BB)
d. Increase dose of diuretic
e. Prazocin (Alpha blocker)
Ans. The key is B. Enalapril. [In patients >55 yrs Ans. The key is C. Mobitz type 1 block. [Gradual
CCB. If CCB not suitable (oedema, heart failure) prolongation of PR interval followed by a drop
thiazide is preferred. In patients <55yrs ACEI or beat].
ARB. Now if patient is on CCB or Thiazide 2nd
drug should be added it should be one of ACEI or
ARB and not one from CCB or Thiazide]. 1090. A 29yo woman presents with lid lag,
lid retraction and diplopia.
What is the most appropriate next step?
1088. A 74yo lady who has had a stroke in a. TFT
the past has an indwelling catheter for b. Tensilon test
10m. She presents with bluish-purple c. Fundoscopy
discoloration of the catheter bag. d. Autoantibodies
What is the most likely explanation for e. EMG
this?
a. Normal change
b. Catheter degradation Ans. The key is A. TFT. [Features are suggestive of
c. Acidic urine Grave’s disease hence TFT].
d. Alkaline urine
e. Bacterial colonization of the urinary tract
1091. A 41yo man presents with
longstanding foul smelling ear discharge
and progressive hearing loss. Otoscopy
Ans. The key is E. Bacterial colonization of the
urinary tract. [Purple Urine Bag Syndrome].
showed perforation of the pars flacida
and a mass in the upper part of the middle
ear. What is the most likely dx?
a. ASOM
1089. A 62yo man has slow palpitations
b. CSOM
and the following ECG. What is the most
c. Acquired cholesteatoma
likely dx?
d. Congenital cholesteatoma
e. Barotrauma

Ans. The key is C. Acquired cholesteatoma. [In


congenital cholesteatoma there is usually no
perforation. In acquired cholesteatoma
perforation is in pars flaccida].

1092. A 9yo child presented with a rash on


his skin which didn’t respond to
a. Sinus bradycardia
antibacterial ointment. What med should
b. 1st degree heart block
be added next?
a. Corticosteroid
c. Mobitz type 1 block
b. Antifungal
d. Mobitz type 2 block
c. Emollient
e. Complete heart block
d. Permethrin b. Corticosteroids
e. Coal tar c. Infliximab
d. Cyclosporine

Ans. The key is B. Antifungal.

Ans. The key is A. Mesalazine. [Features and tx


1093. A young boy has a hx of epistaxis. after colonoscopy suggests dx of ulcerative colitis
CBC=normal, except APTT=47s. What is for which initial treatment option is mesalazine].
the most likely dx?
a. Hemophilia
b. ITP 1096. A 52 yo male with poorly controlled
c. Sickle cell DM has now presented to his GP with pain
d. HUS in the ear. Exam: skin around the ear is
e. Thalassaemia black in color and there was foul smelling
discharge from the ear. Pt also had
Ans. The key is A. Hemophilia. [A young boy with conductive hearing loss. What is the most
epistaxis and prolonged APTT is a probable case probable dx?
of hemophilia. Factor VIII and IX should be a. Carbuncle
offerred to confirm the diagnosis]. b. Folliculitis
c. Malignant OE
d. Cholesteatoma
1094. A 29yo young man presents with e. Furuncle
complaints of recurrent attacks of
diarrhea. He says his stools contain blood
and mucos. Sometimes he has low grade Ans. The key is C. Malignant OE. [In some cases,
fever. What is the most appropriate inv otitis externa can spread to the outer ear and
surrounding tissue, including the bones of the jaw
for his condition?
a. Stool culture and face. This infection is known as malignant
otitis externa. Thogh the term malignant is used it
b. Plain abdominal XR
is not a cancer].
c. Per rectal exam
d. Barium enema

Ans. The key is D. Barium enema. [Features are 1097. A 55yo male has been admitted for
suggestive of IBD, so barium enema is the most elective herniorraphy.
relevant investigation among the given options]. Which among the following can be the
reason to delay his surgery?
a. Controlled asthma
1095. A 26yo young man presents with hx b. Controlled A-fib
of passing loose stools for the past 2m. He c. DVT 2yrs ago
says his stools contain blood and mucus d. DBP 90mmHg
and are a/w abdominal pain. He e. MI 2m ago
undergoes a colonscopy after which he
was started on tx. What is the most
appropriate tx for his condition? Ans. The key is E. MI 2m ago. [Following MI at
a. Mesalazine least 6 months should be elapsed for elective
operation. Prior this time surgery is associated progesterone=normal, LH=33.2. What is
with increased mortality]. the most probable dx?
a. PCOS
b. Pregnancy
1098. A 21yo female in her first pregnancy c. Cushing’s disease
at 38wks was brought to the ED with d. CAH
generalized tonic clonic seizure. IV MgSO4 e. POF
was given but fits was not controlled. She
is having fits again. What is the single
Ans. The key is A. PCOS. [Clinical features
most imp immediate management of this (obesity, hirsutism, amenorrhea etc.)are
pt? consistent with PCOS. We know in PCOS LH:FSH
a. IV MgSO4 ratio is 2:1 or 3:1. Here LH is 33.2 and FSH is 10.9
b. IV diazepam which also supports diagnosis of PCOS].
c. Immediate C-section
d. IV phenytoin
e. MgSO4 bolus 1100. A 17yo girl with a lump in her breast
f. IV lorezepam was seen in the clinic. Exam: the lump was
free and mobile and not attached to the
skin. Her mother wants further tests done.
Ans. The key is A. IV MgSO4. It is a wrong key! What should be the next step?
Correct key is E. MgSO4 bolus. [Treat a first a. CT
seizure with 4g magnesium sulfate in 100mL 0.9% b. US breast
saline IVI c. Punch biopsy
d. Reassure and send home
over 5min + maintenance IVI of 1g/h for 24h.
e. Stereotactic biopsy
Beware  respiration.
Ans. The key is B. US breast. [Described lump is
 If recurrent seizure give 2g IVI magnesium
sulfate over 5 min. fibroadenoma. Next investigation is either US
 Check tendon reflexes and respiratory breast or mammography].
rate every 15min.
 Stop magnesium sulfate IVI if respiratory
rate <14/min or tendon reflex loss, or 1101. A lady comes with a missing IUCD
urine output <20mL/h. Have IV calcium thread. Her LMP was 2wks ago. What is
gluconate ready in case of MgSO4
the single most appropriate next step in
toxicity: 1g (10mL) over 10 min if
respiratory depression. management?
 Use diazepam once if fits continue (eg 5– a. Abdominal US
10mg slowly IV). If seizures continue, b. Prescribe contraceptives
ventilate and consider other causes c. CT
(consider CT scan). OHCS, 9th edition, d. Serum BHCG
page-49]. e. Vaginal exam

Ans. The key is A. Abdominal US. [ LOST THREAD:


1099. A 24yo lady with BMI=30 complains Ultrasound should be arranged to locate the
of facial hair growth and hx of device.
amenorrhea. FSH=10.9, prolactin=400IU, If ultrasound does not locate the device and there
estradiol=177.8mmol/l, is no definite history of expulsion then abdominal
X-ray should be performed to look for an 1104. A 25yo woman presents with a
extrauterine device. single lump in the breast and axilla. The
Expulsion should not otherwise be assumed. lump is mobile and hard in consistency.
Hysteroscopy can be helpful if ultrasound is
The US, mammogram and FNA turn out to
equivocal.
be normal. What is the most appropriate
Surgical retrieval of an extrauterine device is
advised].
inv to confirm the dx?
a. FNAC
b. MRI
c. Punch biopsy
1102. A 32yo woman presents with hx of
d. Genetic testing and counselling
lower abdominal pain and vaginal
e. Core biopsy
discharge. She had her menses 4wk ago.
She has a temp of 38.6C. What is the most
suitable dx?
Ans. The key is E. Core biopsy. [The lump is
a. Acute appendicitis
suspicious but US, mammogram and FNA turn out
b. Acute PID
to be normal. So most appropriate investigation
c. Endometriosis
to confirm diagnosis is core biopsy. It is often the
d. Ectopic pregnancy preferred biopsy method because it is accurate
e. UTI and does not involve surgery].

Ans. The key is B. Acute PID. [Fever, lower *1105. A 37yo lady stopped taking COCP
abdominal pain and vaginal discharge are 18m ago and she had amenorrhea for
features of PID].
12m duration. Labs: FSH=8, LH=7,
prolactin=400, estradiol=500. What is the
cause?
1103. A 40yo female was on COCP which a. Hypothalamic amenorrhea
she stopped 6m ago. But she has not had b. PCOS
her periods since then. Labs: FSH=22, c. Prolactinoma
LH=24, prolactin=700, estradiol=80. d. Post pill amenorrhea
What is the most appropriate dx? e. POF
a. Hypothalamic amenorrhea
b. Post pill amenorrhea
c. Prolactinoma
Ans. The key is D. Post pill amenorrhea. [Post pill
d. Pregnancy amenorrhea= failure to resume menses within 6
e. Premature ovarian failure months but here initial 6 months there was
menses!! again in post pill amenrrhea LH and FSH
low also Oestrogen should be low but here
Ans. The key is E. Premature ovarian failure. [ FSH oestrogen is high!! In post pill amenorrhea
and LH are raised in ovarian failure; an FSH level prolactin raised but in hypothalamic amenorrhea
≥20 IU/l in a woman aged under 40 with prolactin normal and oestrogen low. Actually the
secondary amenorrhoea indicates ovarian given picture does not fit to any condition given
failure]. in options and probably a bad recall or erroneous
question!!!].
*1106. A lady with a firm smooth breast d. Adenomyosis
lump in outer quadrant had a FNAC done. e. Incomplete abortion
Results showed borderline benign
changes. She also has a fam hx of breast Ans. The key is C. Fibroids. [Regular heavy
menstruation in a uterus of 14 wk size is highly
cancer.
suggestive of fibroid].
What is the your next?
a. Mammography 1109. A 29yo at 38wks GA presents with a
b. US 2h hx of constant abdominal pain. She
c. Core biopsy then passes 100ml of blood per vagina.
d. Genetic testing and counselling What is the next appropriate inv?
e. Punch biopsy a. USS
b. CTG
c. Clotting screen
Ans. The key is D. Genetic testing and counselling. d. Hgb
Probably wrong key! Correct one should be C. e. Kleihauer Betke test
Core biopsy. [ Genetic testing is done for
asymptomatic patients in their 30s and if positive Ans. The key is A. USS. [Abdominal pain and
SERMs are given which cause significant bleeding makes the likely diagnosis of abruption
reduction in cancer . placenta for which USS is the next appropriate
Since patient has firm breast lump in upper outer investigation].
quadrant ( most common location for cancer) and
FNAC has shown borderline benign changes plus
1110. A 26yo woman had amenorrhea for
patient is high risk for cancer , so I must go for 10wks and is pregnant. She experiences
core biopsy to confirm whether it is cancer or hyperemesis. Now she presents with
not]. vaginal bleed. Exam: uterus=16wks,
closed os. What is the most probable dx?
1107. A pt presents with mild dyskaryosis. a. Thyrotoxicosis
1y ago smear was normal. b. Hyperemesis gravidarum
What is the most appropriate next step? c. Twins
a. Cauterization d. Wrong dates
b. Repeat smear e. Molar pregnancy
c. Swab and culture
d. Cone biopsy Ans. The key is E. Molar pregnancy. [In
e. Colposcopy hytaditidiform mole uterus becomes more in size
than actual gestational age and due to production
Ans. The key is E. Colposcopy. [In mild dyscariosis of large amount of gonadotrophin by moles
colposcopy should be done with HPV test]. patient suffers from severe vomiting i.e.
hyperemesis].
1108. An African lady presents with heavy but
regular periods. Her uterine size correlates to 1111. A pregnant woman of G2, GA 11wks
14wks presents with heavy vomiting, headache
pregnancy. What is the most appropriate dx? and reduced urine output. Urine analysis
shows ketonuria. Choose the next best
a. Blood dyscrasia
step?
b. Hematoma a. US
b. Oral fluid replacement
c. Fibroids
c. Serum BHCG e. Karyotyping
d. Parental anti-emetics
e. IV fluids Ans. The key is E. Karyotyping. [The likely
diagnosis is Klinefelter’s syndrome for which
Karyotyping should be done to make the
diagnosis established].
Ans. The key is E. IV fluids. [Hyperemesis
gravidarum with oliguria and ketonuria. IV fluid 1115. A woman who is on regular COCP
should be started to prevent renal failure]. presented to you for advice on what to do
as she has to now start to take a course of
1112. A pt had inflammatory changes on
7d antibiotics. What would you advice?
cervical smear. There is no vaginal
a. Continue regular COC
discharge, no pelvic pain and no fever. b. Continue COCP and backup contraception
What is the next step? using condoms for 2d
a. Repeat smear in 6m
c. Continue COCP and backup contraception
b. Take swab
using condoms for 7d
c. Treat with antibiotics
d. Continue COCP and backup contraception
d. Colposcopy
using condoms for 2wks
e. Cone biopsy
Ans. The key is D. Continue COCP and backup
contraception using condoms for 2wks. This is
Ans. The key is B. Take swab. wrong key! Correct key should be A. Continue
regular COCP. [Before it was thought that
1113. A 37yo infertile lady with 5cm antibiotics alter the gut flora and
subserosal and 3cm submucosal fibroid is ethinyloestradiol is not conjugated. There is more
trying to get pregnant. Which is the most ethinyloestradiol passed in the stool. So
suitable option? pregnancy and breakthrough bleeding can occur.
a. Clomifen therapy But later it was established that except for strong
b. IVF enzyme inducer like Rifampicin and Rifabutin
c. Myomectomy other antibiotics practically does not reduce
d. Hysterectomy potency of COCP and hence except only
e. IU insemination rifampicin or rifabutin for other antibiotics no
additional precautions are needed]. 
Ans. The key is C. Myomectomy. [Fibroids are
preventing from her being pregnant. So 1116. A lady presents with hot flashes and
myomectomy is the most suitable option to let other symptoms of menopause.
her get pregnant]. What is the tx option?
a. Raloxifen
b. HRT
1114. A young tall man and his wife are c. Bisphosphonate
trying for babies and present at the d. COCP
infertility clinic. On inv the man has e. Topical estrogen
primary infertilitiy and azoopermia.
What other inv should be done? Ans. The key is B. HRT
a. Testosterone 1117. A 28yo woman at 34wks GA for her
b. LSH
first pregnancy attends antenatal clinic.
c. FSH
d. Estradiol
Her blood results:Hgb=10.6, MCV=95, d. Danazol
MCHC=350. What do you do for her? e. Mirena
a. Folate f. IUCD
b. Dextran
Ans. The key is E. Mirena.
c. Ferrous sulphate
d. None 1120. A 32yo woman comes with
e. IV FeSO4 intermenstrual bleeding. Her last cervical
f. Explain this physiologic hemodynamic smear was 1y ago and was negative.
anemia What test would you recommend for her
g. Blood transfusion initially?
a. Colposcopy
Ans. The key is F. Explain this physiologic
b. Cervical smear
hemodynamic anemia. [According to NICE, cut
offs for iron supplements: c. Endocervical swab
at booking (8-10 weeks)- if less than 11  d. Transvaginal US
at 28 weeks and further- if less than 10.5 e. Pelvic CT
if less than these values=> give iron].
Ans. The key is B. Cervical smear. This is a wrong
1118. A 34yo woman who never had fits key! Correct key is C. Endocervical swab. [Smear
or high BP developed fits 6h after delivery can only be done if it is due or overdue, not in any
of a term healthy child. What is the most time or not as needed basis].
likely dx?
a. Eclampsia
b. Preeclampsia 1121. A 20yo woman has had abdominal
c. Epilepsy pain in the LIF for 6wks duration. Over the
d. Pulmonary embolism past 48h, she has severe abdominal pain
e. Pregnancy induced HTN and has a fever of 39.1C. Pelvic US shows
a complex cystic 7cm mass in the LIF.
Ans. The key is A. Eclampsia. What is the most likely dx?
[In pregnant women, the convulsive stage of pree a. Endometriosis
clampsia-eclampsia b. Dermoid cyst
syndrome; the convulsions are not c. Ovarian ca
attributable to other cerebral conditions such as e
d. Tubo-ovarian abscess
pilepsy. It can occur from 20 weeks of gestation
e. Ectopic pregnancy
to post partum period (usually 10 days after
delivery)]. Ans. The key is D. Tubo-ovarian abscess.
1119. A 30yo lady who already has one 1122. A woman is 16wk pregnant and she
child through a prv C-section demands a is worried about abnormal chromosomal
reversible contraception. She presently anomaly in her child. What is the
experiences heavy and painful periods. definitive inv at this stage?
What is the most appropriate a. Amniocentesis
contraceptive you will recommend for b. CVS (Chorionic Villous Sampling)
her? c. Parents karyotyping
a. COCP d. Coomb’s test
b. POP e. Pre-implantation genetic dx
c. Implanon
Ans. The key is A. Amniocentesis. [Amniocentesis ***1125. A 36wk pregnant woman
is done between 14 to 16 weeks of gestation. CVS presents with sudden onset of uterine pain
is done prior to 15 weeks. So the option is A. and bleeding, uterus is tender, no prv
Amniocentesis].
LSCS. What is the most appropriate
1123. A 28yo lady with a fam hx of CF cause?
comes for genetic counselling and wants a. Preeclampsia
the earliest possible dx test for CF for the b. DIC
baby she is planning. She is not in favor of c. Placental abruption
d. Placental previa
termination. What would you recommend
e. Ectopic pregnancy
for her?
a. CVS (Chorionic Villous Sampling) f. Missed abortion
b. Amniocentesis g. Ectropion
c. Pre-implantation genetic dx
d. Chromosomal karyotyping
e. Maternal serum test Ans. The key is C. Placental abruption. [Painful
f. Reassure bleeding at 36th week is placental abruption
(either revealed or mixed type)].

1126. A 28wk pregnant woman presents


Ans. C. Pre-implantation genetic dx. [Patient is with uterine bleeding after sexual
not yet pregnant but planning for pregnancy. intercourse. What is the most appropriate
Earliest possible diagnostic test for CF is Pre-
cause?
implantation genetic dx. Preimplantation genetic a. Preeclampsia
diagnosis (PGD) is a procedure used prior to
b. DIC
implantation to help identify genetic defects
c. Placental abruption
within embryos created through in vitro
d. Placental previa
fertilization to prevent certain diseases or
e. Ectopic pregnancy
disorders from being passed on to the child].
f. Missed abortion
1124. A 39yo woman in her 36th week GA g. Ectropion
with acute abdominal pain is rushed for
immediate delivery. Her report: Ans. The key is G. Ectropion. [Post coital bleeding
can be either placenta previa or cervical
BP=110/60mmHg, Hgb=low, bilirubin=22,
ectropion. But as ectropion is commoner in
AST=35, Plt=60, APTT=60, PT=30, pregnancy so it is the option here].
Fibrinogen=0.6. What is the cause?
a. Pregnancy induced hypertension 1127. A 6wk pregnant woman presents
b. DIC with abdominal pain. She has prv hx of
c. HELLP syndrome PID. What is the most likely dx?
d. Acute fatty live a. Preeclampsia
e. Obstetric cholestasis b. DIC
c. Placental abruption
d. Placental previa
Ans. The key is B. DIC. [It is not HELLP syndrome e. Ectopic pregnancy
as liver enzyme is not elevated. Acute abdominal f. Missed abortion
pain may indicate concealed abruption placenta g. Ectropion
which may lead to DIC].
Ans. The key is E. Ectopic pregnancy. [H/O PID is a then aspirin where no history of
recognized cause of ectopic pregnancy. Also pain thromboembolism and warfarine where history
without bleeding at 6th week support the of thromboembolism].
diagnosis of ectopic pregnancy].
1131. A 6yo child presents with hx of
1128. A 33wk pregnant woman presents recurrent jaundice. Between the episodes
with vaginal bleeding, low Hgb, low plt, he is totally fine. Mother gives hx of
increased bilirubin, AST normal, APTT & PT jaundice being bought about by ongoing
increased. What is the most likely dx? infections.
a. Preeclampsia What is the most likely dx?
b. DIC a. Hereditary spherocytosis
c. Placental abruption b. G6PD deficiency
d. Placental previa c. Thalassemia
e. Ectopic pregnancy d. Sickle cell disease
f. Missed abortion e. Congenital storage disorder
g. Ectropion
Ans. The key is B. G6PD deficiency. [There are
Ans. The key is B. DIC. some precipitating factors of hemolysis causing
jaundice in patients with G6PD deficiency among
1129. A 25yo lady at her 28th week GA which infection is a recognized one].
came for check up. Her BP=160/95mmHg,
protein in urine=6g/d. What is the most 1132. A 42yo woman who smokes 20
likely dx? cigarettes/d presents with complains of
a. Essential HTN heavy bleeding and prolonged menstrual
b. Gestational HTN period. What is the most appropriate tx
c. Chronic HTN for her?
d. Preeclampsia a. Tranexemic acid
b. COCP
Ans. The key is D. Preeclampsia. c. Mefenemic acid
1130. A 32yo woman has a hx of d. IUCD
spontaneous abortions at 6wks, 12wks, e. Norethisterone
and 20wks. She is now keen to conceive Ans. The key is D. IUCD [IUCD = copper-T + IIUS.
again. Which of the following would you Here it is IUS which should be given]. [In regular
prescribe for the next pregnancy? menstruation with menorrhagia: - i) LNG-IUS ii)
a. MgSO4 tranexamic acid or NSAIDs or COCP iii)
b. Aspirin Norethsterone or injectable long acting
c. Warfarin progestogen NICE].
d. Mefenemic acid
e. Heparin 1133. A 17yo senior school girl with
complain of prolonged irregular menstrual
Ans. The key is B. Aspirin. Key is not correct and period and heavy blood losses. What is
correct key Heparin + aspirin is not present in the most appropriate tx for her?
option. [Probable antiphospholipid syndrome. As a. Mefenemic acid
there is no history of previous thromboembolism b. COCP
but history of 3 fetal loss Heparin + aspirin should c. POP
be given (if previus thromboembolism heparin is d. IUCD
to be given). If not thinking to become pregnant
e. Mirena Ans. The key is C. Depot provera. [Hormone and
barrier methods are all acceptable choices but
Ans. The key is B. COCP. [In irregular period: COCP intrauterine devices are not recommended, as
except the contraindications for it and in that they may be associated with uterine bleeding and
case POP should be used]. infection. Depot contraceptive (Depo-Provera®) is
safe and has been found to improve the blood
1134. A 32yo presents with heavy blood
picture and reduce pain crises. Source:
loss, US: uterine thickness>14mm.
patient.info].
What is the most appropriate tx for her?
a. Mefenemic acid 1137. A 70yo woman is admitted with
b. COCP diarrhea, vomiting and dehydration.
c. POP Exam: yellow visual halos in her eyes,
d. IUCD ECG=bradycardia. She has a hx of chronic
e. IU system (mirena) A-fib.
Which drug causes the above mentioned
Ans. The key is E. IU system (mirena). [Simple
endometrial hyperplasia without atypia responds side effects?
a. Nifedipine
to high-dose progestogens, with repeat histology
after three months. This can be effectively b. Ramipril
delivered by the levonorgestrel intrauterine c. Atenolol
system (IUS). Source: patient.info]. d. Lithium
e. Digoxia
1135. A 37yo woman presents with heavy
bleeding. Inv show subserosal fibroid=4cm Ans. The key is E. Digoxin. [Digoxin toxicity causes
and intramural fibroid=6cm. Which is the diarrhea, vomiting, dehydration, xanthopsia
most appropriate tx? (yellow halos around light), bradicardia. History of
a. UAE atrial fibrillation is also a clue of digoxin use].
b. Abdominal hysterectomy
c. Hysteroscopic Myomectomy ***1138. A 33yo lady who is a drug addict
d. Vaginal Hysterectomy wants to quit. She says she is ready to
e. Abdominal myomectomy stop the drug abuse. She is supported by
Ans. The key is E. Abdominal myomectomy. [As
her friends and family. What drug tx
patient is young we should go for myomectomy. would you give her?
a. Benzodiazepines
As hysteroscopic myomectomy is suitable for
mainly submucosal fibroids. We should go for b. Diazipoxide
Abdominal myomectomy which will deal with c. Lithium
both subserosal and intramural fibroids]. d. Methadone
e. Disulfiram
1136. A woman with sickle cell disease
complains of heavy menstrual blood loss.
What is the most appropriate tx? Ans. The key is D. Methadone. [Methadone is an
a. COCP opioid medication. Methadone reduces
b. Mirena withdrawal symptoms in people addicted to
c. Depot provera heroin or other narcotic drugs without causing
d. Copper IUS the "high" associated with the drug addiction and
e. Transdermal patch thus used to help a drug addict to quit from drug
addiction].
1141. A 45yo female looking pale has
1139. A 50yo lady has been suffering from bluish discoloration of hands whenever
chronic RA and is on methotraxate and she goes out in the cold. She has also
naproxen. Her CBC shows microcytic noticed some reddish spots on her body.
anemia. What is the most likely cause? She has symmetrical peripheral
a. Anemia of chronic disease arthropathy for the last yr. What is the
b. GI hemorrhage most probable dx?
c. Menorrhagia a. RA
b. Osteosarcoma
c. Limited systemic sclerosis
Ans. The key is B. GI hemorrhage. [Anemia of d. Diffuse systemic sclerosis
chronic disease is mostly normocytic and e. Chondrosarcoma
methotrexate causes folate deficiency which may
lead to macrocytosis. So for this microcytic
anemia NSAIDs induced GI hemorrhage is the
Ans. The key is C. Limited systemic sclerosis.
most likely cause].
[Raynaud’s phenomena, telangiectasia,
arthropathy indicates to the dx of CREST
1140. A 15yo male noticed swelling on the syndrome or Limited systemic sclerosis].
left knee following a fall while playing.
The swelling has not subsided in spite of
rest and analgesia. Exam: full knee 1142. A 60yo female has pain and
movement with slight tenderness. He has stiffness in her right hip joint. Pain is not
painless palpable mass in left inguinal severe in the morning but increases as the
region. What is the most probable dx? day progresses. She has noticed some
a. Osteosarcoma nodules in her hands. Inv: Hgb=low. What
b. Ewing’s sarcoma is the most probable dx?
c. Chondrosarcoma a. RA
d. Lymphangiosarcoma b. Osteoarthritis
e. Osteodosteoma c. Gout
d. Pseudogout
e. Multiple myeloma
Ans. There are two keys i) Osteosarcoma ii)
Ewing’s sarcoma. But the likely correct key is
probably A. Osteosarcoma. [Both occurs in Ans. The key is B. Osteoarthritis. [Stiffness and
children and young adults but the occurrence of pain of right hip increasing as the day progresses,
painful symptom during playing is usually a nodules in hands (Hebarden or Bouchard’s
feature of osteosarcoma. In case of osteosarcoma nodes), anemia (probably secondary to prolonged
teenagers who are active in sports often complain NSAIDs use) are indicating dx of osteoarthritis].
of pain in the lower femur, or immediately below
the knee. If the tumor is large, it can present as
overt localised swelling. Sometimes a sudden 1143. A 30yo female has chronic diarrhea,
fracture is the first symptom, because affected mouth ulcers and skin tags. She complains
bone is not as strong as normal bone and
of visual prbs, low back pain and morning
may fracture abnormally with minor trauma].
stiffness. Inv: ESR & CRP=raised,
Hgb=10mg/dl. What is the most probable Ans. The key is B. Plasma RF. It is a wrong key.
dx? Correct key should be C. Joint fluid uric acid
a. SLE crystals. [Thiazide diuretics may increase uric acid
b. Reactive Arthritis levels in blood and joint fluid may show uric acid
c. Gout crystals].
d. Pseudogout 1146. A 60yo lady with a hx of HTN and
e. Seronegative arthritis suffering from RA since the last 10y now
presents with hot, swollen and tender
knee joint. What inv would you do for her?
Ans. The key is E. Seronegative arthritis. [Chronic a. XR
diarrhea, mouth ulcers and skin tags are features b. C&S of joint aspirate
of IBD. There is a well known association between c. US
IBD and seronegative arthritis (particularly AS). d. MRI
Here low back pain, visual problem, morning
e. CT
stiffness, raised ESR and CRP, low hemoglobin
indicates the diagnosis of Seronegative arthritis]. Ans. The key is B. C&S of joint aspirate. [RA itself
is a risk factor for joint infection and there are
possibilities to increase the chance of septic
1144. A 28yo woman has been on tx for arthritis with the use of probable steroid which
RA for 3yrs. She has gradual loss of vision lowers immunity and further facilitates infection
in both eyes. Her IOP is normal. Red reflex of joints for which C&S of joint aspirate should be
is absent in both eyes. What is the single done].
most likely dx? 1147. A 34yo man after an RTA was
a. Cataract
brought to the ED. He has BP=50/0mmHg
b. DM retinopathy
and chest wall with asymmetrical
c. Hypermetropia
movement, RR=34bpm. What would be
d. Macular degeneration
the initial action?
e. HTN retinopathy a. IV fluid infusion
b. Intubation and ventilation
c. CT chest
Ans. The key is A. Cataract. [Prolonged use of d. Transfer to ITU
steroid (here in RA) is a known cause of cataract].

Ans. The key is B. Intubation and ventilation.


1145. An elderly man with recently dx HF
has been treated with diuretics. He now
develops severe joint pain in his left ankle 1148. A 7yo presented with chronic cough
with swelling and redness. What is single and is also found to be jaundiced on
most likely inv? exam. What is the most likely dx?
a. XR of bone
a. Congenital diaphragmatic hernia
b. Plasma RF b. Congenital cystic adenomatoid
c. Joint fluid uric acid crystals
malformation
d. ESR c. Bronchiolitis
d. RDS
e. Alpha 1 antitrypsin deficiency
weeping lesions and some of them are
Ans. The key is E. Alpha 1 antitrypsin deficiency. crusted. What is the most probable dx?
[In those with unexplained liver disease with or a. Varicella
without respiratory symptoms should be b. Impetigo
evaluated for AATD].  c. Drug reaction
d. Contact dermatitis
e. Scabies
1149. A 65yo man had a bowel resection Ans. The key is B. Impetigo. This is probably a
5d ago. He is anuric and breathless. His wrong key! Likely correct key should be A.
BP=150/110mmHg. He has crackles at Varicella. [Please consider dx of impetigo if
both lung bases and sacral edema. specifically mention honey (or yellow or golden)
Bloods: K+=6.8mmol/l, urea=58mmol/l, coloured lesion or distribution described to be on
creatinine=600umol/l. What is the single face and limbs predominately. Otherwise
most appropriate immediate consider chickenpox. Though key is impetigo
management? here, it is wrong key. Likely correct option here is
a. Bolus of 20U insulin varicella].
b. Calcium resonium enema
1152. A pt comes with 6m hx of painless
c. Dextrose-saline infusion
bilateral swelling of the face which has
d. 5% dextrose infusion
been progressively increasing in size. On
e. 10U insulin, 50ml of 50% dextrose
routine CXR, he is found to have perihilar
infusion
lymphadenopathy. What is the most
Ans. The key is E. 10U insulin, 50ml of 50% probable dx?
dextrose infusion. [There is renal failure with a. Chronic sialadenitis
hyperkalemia. First we have to save heart from b. Thyroid adenoma
arrest or life threatening arrhythmia by shifting c. Carcinoma of salivary gland
K+ into cell. 10U insulin in 50 ml 50% dextrose will d. Adenoid cystic carcinoma
help doing this]. e. Mikulicz’s disease

1150. A 25yo woman presents with a Ans. The key is E. Mikulicz’s disease. [Mikulicz’s
painful shallow ulcer on the vulva. What disease (MD) is a well-known disorder
inv has to be done? characterized by enlarged lacrimal and parotid
a. HSV antibodies glands caused by infiltration with lymphocytes.
b. Syphilis serology When no specific cause is found it is called
c. Swab for hemophilus ducreyi Mikulicz’s disease; and if secondary to disease like
d. Urine culture sarcoidosis it is termed as Mikulicz’s syndrome].
e. Blood culture
1153. A woman has widespread
Ans. The key is C. Swab for hemophilus ducreyi. metastasis from a carcinoma. She
[Herpes simplex virus causes multiple painful presented with severe back pain. Where
vesicles and syphilis ulcer is painless. As here do you expect the cancer to be?
single painful ulcer probable dx is Chancroid a. Lungs
caused by Hemophilus ducreyi]. b. Cervix
c. Ovary
1151. A child was admitted with fever, d. Uterus
generalized skin lesion, some of them are e. Breast
Ans. The key is E. Breast. [In female breast and 1157. A 15m baby girl presented to the ED
lung cancer and in male prostate and lung cancer with difficulty in breathing. Exam: she has
are most common to metastasize to bone]. intercostals recessions and a wheeze.
1154. A 10yo child has got progressive Temp=normal. What is the most likely dx?
a. URTI
bilateral hearing loss. He has started to
b. Pneumonia
increase the TV volume. All other
c. Bronchiolitis
examination is normal. What is the most
d. RDS
likely dx?
e. Alpha 1 antitrypsin deficiency
a. Wax
b. Foreign body Ans. The key is C. Bronchiolitis. [Asthma and
c. Bilateral OM with effusion bronchiolitis has similar sign symptoms. The
d. SNHL difference is in pathology. In asthma there is
e. Meningitis due to meningococcus bronchoconstriction but in bronchiolitis there is
oedema of the airway wall. In infants and
Ans. The key is C. Bilateral OM with effusion. younger children bronchiolitis is commoner than
1155. A child had a patchy rash following asthma and it is seen that many of the children
with bronhiolitis usually followed by asthma].
tx for sore throat & cervical LN
enlargement. Which is the most 1158. An 8yo boy develops a seizure
appropriate antibiotic? affecting his right arm, seizure lasts for
a. Ampicillin several mins. He doesn’t remember
b. Erythromycin anything what happened. On his CT:
c. Cefuroxime lesion in left hemisphere. What is the most
d. Metronidazole probable dx?
e. Tetracycline a. Epilepsy
b. Space occupying lesion
Ans. The key is A. Ampicillin. [Infectious
mononeucleosis can present with sorethroat and c. Dementia
lymphadenothy like tonsillitis and if treated with d. Huntington’s chorea
ampicillin leads to eruption of patchy rashes]. e. Intracranial HTN

1156. A child with a hx of asthma is Ans. The key is B. Space occupying lesion. [This is
brought to ED with a cut on knee and complex partial seizure due to space occupying
sprained on her left wrist.cWhich is the lesion].
best analgesic for her? 1159. A 28yo female presented with
a. Paracetamol
complains of difficulties in swallowing
b. NSAIDs
liquids only. She also suffers from
c. Cocodemol
recurrent chest infection in the past few
d. Ibuprofen
months. What is the most probable dx?
Ans. The key is A. Paracetamol. [NSAIDs including a. Foreign body
ibuprofen can precipitate asthma and Cocodamol b. Plummer vinson syndrome
is also advised to avoid in asthma (due to its c. Achalasia cardia
codeine content). So paracetamol is the best d. Peptic stricture
option here]. e. Esophageal carcinoma
Ans. The key is C. Achalasia cardia. [Dysphagia to e. Warfarin
both solid and liquid or mostly to liquid are
common feature of achalasia. In achalasia there
may occur aspiration due to regurgitation during Ans. The key is B. DIC.
lying down to sleep which may result recurrent
chest infection and cough]. 1163. A study was done amongst 2 hosp
for the equal number of cancer pts. It was
1160. Mother having 2 children with CF.
noted that hosp A had the higher rate of
What is the risk of getting another baby?
mortality than hosp B for treated cancer
a. 1:2
b. 1:8
pts.
c. 1:4
What is the study done here classified as?
a. Retrospective
d. 1:16
b. Observational
e. 1:1
c. Cohort
d. Case study

Ans. The key is C. 1:4. [Cystic fibrosis is an Ans. The key is C. Cohort study. [A cohort is a
autosomal recessive disease. If both parents are group of people who share a common
carrier there is 1:4 chance of risk of occurrence of characteristic or experience within a defined
the disease for each child]. period (e.g., are born, are exposed to a drug or
vaccine or pollutant, or undergo a certain medical
1161. A 14yo boy has been dx with procedure].
nephrotic syndrome. 5d later he presents
with flank pain, hematuria and fluctuating 1164. A 17yo girl comes to see her GP
urea levels. A dx of renal vein thrombosis after having unprotected sex 2d ago. She
is made. What is the most likely cause for asks if her GP can explain to her how this
renal vein thrombosis? prescribed procedure would work by
a. Protein C deficiency helping her not to get pregnant.
b. Vasculitis a. It helps to prevent implantation
c. Loss of antithrombin III b. It helps in preventing or delaying
d. High estrogen levels ovulation
e. Stasis c. It causes an early miscarriage
d. It releases progesterone and stops
ovulation
Ans. The key is C. Loss of antithrombin III. e. It causes local enzymatic reaction

1162. A 36yo woman presented with


massive bleeding from multiple sites. Lab: Ans. The key is A. It helps to prevent
implantation. Probably key is suggesting IUCD
fibrin degradation products: +++, plt=30,
wich helps to prevent implantation and not the
bleeding time=prolonged, PT=prolonged, pills as both the available pill causes a delay in
APTT=prolonged. What is the most likely ovulation. [The time required for the egg to
dx? travel to the uterus and implant is usually
a. Hemophilia between 7 and 10 days]. [There are two types of
b. DIC pill:
c. ITP 1. 1.5 mg of levonorgestrel pill. It should be taken
within 72 hours (three days). It is thought to work
d. Factor V leiden
mainly by preventing or delaying the release of an
egg from your ovary, which normally happens 1167. A pt presented with jaundice, fever
each month (ovulation). and upper abdominal pain within 24h
2. Ulipristal acetate (brand name ellaOne®) is a
after removal of gallstone by ERCP. The
type of emergency contraceptive pill that was
launched in the UK in 2009. It is taken as one cholangiography was done and it was
single tablet. Take the pill as soon as possible patent. What is the possible cause of his
after unprotected sex. The earlier you take the complaints?
pill, the more effective it is. It can be taken up to a. Biliary infection
120 hours (five days) after having unprotected b. Acute pancreatitis
sex. It is a type of hormone which seems to work c. Perforation
by stopping or delaying release of an egg
(ovulation). Ans. The key is B. Acute pancreatitis. [Though
IUCD prevent implantation. As she is sexually biliary infection is possible but acute pancreatitis
active hymen is ruptured and IUCD can be given
is more common complication of ERCP and
to her. But with IUCD chance of ectopic
absence of any obstruction (stone or tumour
pregnancy is much more!].
which helps biliary infection to occur) makes
cholangitis less likely].
1165. A 2d baby’s mother is worried about
the baby’s hearing. Mother has a hx of 1168. A mother presents with her 14m
conductive hearing loss. What is the most child. He holds furniture and other things
appropriate test? to help him stand and walk. He can say
a. Brain stem evoked response ‘mama’ and ‘papa’. He makes eye contact
b. CT and smiles. He can transfer objects from
c. Fork test one hand to another. He responds to his
d. MRI name.
e. Reassure what do you interpret from his
development?
a. Delayed gross motor development
Ans. The key is A. Brain stem evoked response. [A b. Delayed fine motor development
BAER (brainstem auditory evoked response) test c. Delayed verbal development
can help to diagnose hearing loss and nervous d. Normal development
system disorders, especially in newborns, young e. Delayed social development
children, and others who may not be able to
participate in a standard hearing test]. Ans. The key is D. Normal development.

1166. A healthy 8yo boy had antibiotic tx 1169. A young child, 3yo, has presented
for meningitis. Initially he wasn’t with vomiting for 3d. Exam: mild-mod
resuscitated. What will be the outcome if dehydration. What is his ABG profile likely
he receives full tx? to show?
a. He will recover fully to his prv health a. pH low, PCO2 low
b. He will have hearing impairment b. pH low, PCO2 high
c. He will have brain abscess c. pH high, PCO2 low
d. He will have encephalitis d. pH high, PCO2 high

Ans. The key is D. pH high, PCO2 high. [Prolonged


vomiting causes loss of H+, Cl- and Na+. There is
Ans. The key is A. He will recover fully to his prv
also hypokalemia. There occur metabolic
health.
alkalosis. Compensation for metabolic alkalosis
occurs mainly in the lungs, which retain carbon b. Somatization
dioxide (CO2) through slower breathing, c. PTSD
or hypoventilation (respiratory compensation) d. Dissociation
leading to high PCO2]. e. GAD
1170. A 68yo woman has been admitted Ans. The key is A. Conversion reaction.
with poor appetite, weight loss, poor [Conversion  reaction is sometimes applied to
concentration and self neglect for 3wks. patients who present with neurological
She has not been eating or drinking symptoms, such
adequately and has rarely left her bed. as numbness, blindness, paralysis, or fits, which
She is expressive suicidal ideas and is are not consistent with a well-established organic
convinced that people are out to kill her. cause, and which cause significant distress. It is
thought that these symptoms arise in response to
She has been on antidepressant therapy
stressful situations affecting a patient's mental
for the past 3m with no improvement.
health].
What is the most appropriate tx?
a. Anti depressants 1173. A 25yo man has been suffering from
b. CBT breathlessness and wheeze for 3m. He has
c. Interpersonal therapy been taking salbutamol 2puffs as
d. ECT required. In the last 2 wks his symptoms
e. Antipsychotics have worsened and he has to take
Ans. The key is D. ECT. [Patient has symptoms of
salbutamol more frequently during the
severe depression with persecutory delusion day time. He also complains of excessive
making the diagnosis of psychotic depression dyspnea at night. What drugs or regimen
which is treated by ECT]. would you like to add?
a. Prednisolone
1171. A 78yo retired teacher was b. Fluticasone + salbutamol inhaled
admitted for a hernioplasty procedure. c. Beclomethasone inhaled
After the operation he became agitated, d. Montelukast PO
aggressive and confused. What is the e. Salmetrol PO
most appropriate management?
a. Diazepam Ans. The key is C. Beclomethasone inhaled.
b. Chlordiazepoxide [Patient was in step 1. As not controlled next step
c. Vit B is addition of inhaled corticosteroid].
d. Clozapine 1174. A 64yo man who was exposed to
e. Thiamine asbestos for 40yrs presents with weight
Ans. The key is B. Chlordiazepoxide. [Probable loss and chest pain. The dx of
delirium tremens]. mesothelioma has been made. He
develops SOB and XR=pleural effusion.
1172. A 25yo girl saw a tragic RTA in What is the most appropriate
which a young boy was killed. The night of management?
the event she couldn’tvsleep and the day a. Thoracocenthesis
after she suddenly lost her vision. She was b. Chest drain
prv fine and there was no hx of medical or c. Radiation therapy
psychological prbs. What is the dx? d. Pneumonectomy
a. Conversion e. Chemotherapy
Ans. The key is E. Chemotherapy. It is probably a 1177. A white English man with a past hx
wrong key! Correct key should be A. of MI is a known HTN and DM. He is
Thoracocentasis. [As mesothelioma responds very currently on aspirin, statin and metformin.
poor to chemotherapy and life expentancy is also
What would you add to the tx?
very poor Thoracocentasis is the appropriate a. ACEi
procedure to improve patients distress].
b. Diuretic
c. Insulin
1175. A 72yo presents with polyuria and d. Beta blocker
polydipsia. The fasting blood sugar is 8 e. CCB
and 10mmol/l. BP=130/80mmHg and the
level of cholesterol=5.7mmol/l. There is Ans. The key is A. ACEI.
microalbuminuria. What is the single most
appropriate next management?
a. ACEi and sulfonylurea 1178. A 57yo man who had MI a few
b. Statin and biguanide months ago has been having a low mood.
c. Statin and glitazone A dx of moderate depression has been
d. Insulin and ACEi established. Which medication is the best
e. Statin and ACEi tx for him?
a. SSRI
b. TCA
c. MAOi
Ans. The key is E. Statin and ACEI.
d. Benzodiazepam
1176. A 49yo woman presents to the OPD. e. Mood stabilizer
Her oral glucose test after 2h of glucose
intake vs plasma level in 2 different tests
are 6mmol/l and 10mmol/l. This situation Ans. The key is A. SSRI. [Among SSRIs Sertraline is
can be categoraized as the drug of choice. If SSRI cannot be used
a. Impaired glucose tolerance Mirtazapine is recommended as next
b. Impaired fasting glucose antidepressant].
c. T1DM
d. T2DM
e. Metabolic syndrome 1179. A 12yo presents with chest pain.
Exam: tachycardia, hypotension, dilated
neck veins and the trachea is not centrally
Ans. The key is A. Impaired glucose tolerance. placed. What is the next appropriate
[The WHO defines someone as having pre- management?
diabetes if they have: a. Portable XR
b. Needle thoracocenthesis
 A fasting blood glucose of less than 7
c. Chest drainage
mmol/L; AND
d. ABG
 A blood glucose of 7.8 mmol/L or more e. CTPA
but less than 11.1mmol/L after a two-
hour oral glucose tolerance test]. Ans. The key is B. Needle thoracocentesis. [The
case may be either pneumothorax or massive
pleural effusion. Thoracocentasis is However it does not shift or reduce potassium
recommended for both]. level but protects heart from arrhythmia and buy
time for definitive measure to take place!].
1180. A 7yo child is being inv for TB. His
parents don’t agree for taking a BAL. 1183. A 38yo man presents with acute
what other sample will show growth of infection of skin in the leg. Dx of cellutitis
the organism? has been made. What meds should be
a. Blood test prescribed?
b. Throat swab a. Penicillin + Flucloxacillin
c. Gastric washing b. Metronidazole + erythromycin
d. Mantoux test c. Vancomycin + metronidazole
e. CSF d. Ceftriaxone + terbinafine
e. Ceftriaxone + flucloxacillin

Ans. The key is A. Penicillin + Flucloxacillin.


Ans. The key is C. Gastric washing.
[Cellulitis is usually caused by bacteria, such as
1181. A 51yo man had a MI a few days staphylococci or streptococci that are commonly
ago. He developed breathlessness. Echo present on the skin. So Penicillin + Flucloxacillin
should be given].
was done and showed a pansystolic
murmur. What can be the cause of this 1184. A 72yo man presents to the ED with
symptom? chest pain. The following ECG was taken.
a. Ruptured papillary muscle What is the most likely dx?
b. Acute pericarditis
c. Dresslers syndrome
d. Malignant VT
e. Ventricular aneurysm

Ans. The key is A. Ruptured papillary muscle.


[Papillary muscle rupture causes mitral
regurgitation causing pancystolic murmur leading
to features of heart failure like breathlessness].

1182. A 61yo man was found with K+=7.5


and ECG with prolong QRS complex. What
is the best possible tx option?
a. Dialysis
b. IV calcium gluconate
c. IV insulin and dextrose
d. Salbutamol nebulizer a. Anterior MI
e. Loop diuretics b. Inferior MI
c. Lateral MI
Ans. The key is B. IV calcium gluconate. [In this d. Posterior MI
high level of potassium we have to shift this e. NSTEMI
potassium into cells to reduce plasma level and
save heart from arrest or life threatening
arrhythmia. For keep the heart safe option of first
Ans. The key is E. NSTEMI.
choice in such emergency is IV calcium gluconate!
1185. A 36yo woman has recently spent a investigation from the given options is A. Anti ds
lot of money on buying clothes. She goes DNA].
out almost every night with her friends. 1188. A 75yo man presents with back
She believes that she knows better than pain. Inv: plasma cells are found. What is
her friends, so she should choose the the most probable dx?
restaurant for eating out with her friends. a. Multiple myeloma
She gave hx of having low mood at 12y. b. AS
What is the dx? c. Disc prolapse
a. Mania d. Leukemia
b. Depression e. Myelofibrosis
c. Bipolar affective disorder
d. Borderline personality disorder Ans. The key is A. Multiple myeloma. [H/O back
e. Dysthymia pain with presence of plasma cells on inv. Are
highly suggestive of Multiple myeloma].
Ans. The key is C. Bipolar affective disorder. [In
bipolar disorder there is depressive disorder 1189. A 45yo woman presents with
alternate with mania and the present case is in complains of abdominal pain and blood in
manic stage of bipolar disorder]. stool. She brings the stool sample from
home but has never been able to produce
1186. A homeless lady presents with
a sample at the hospital. Her urine and
cough and fever. She complains of night
blood tests are normal. Exam: multiple
sweats and weight loss. CXR has been
scars on the abdomen consistent with
done and shows opacity. What is the next
laparoscopies and appendectomy. She
appropriate management?
insists on getting further inv although no
a. AFB
abnormalities are found. What is the most
b. Mantoux test
likely dx?
c. IFN gamma testing
a. Malingering
d. Bronchoscopy
b. Somatization
e. CT
c. Hypochondriasis
Ans. The key is A. AFB. [Cough and fever and night d. Conversion disorder
sweat and weight loss are very suggestive of TB. e. Munchausen syndrome
So we should do AFB as her next investigation].

1187. A 32yo woman presents with


Ans. The key is E. Munchausen syndrome.
malaise fatigue and fever. She complains [Munchausen syndrome is a psychiatric factitious
about weight loss. Exam: malar rash with disorder wherein those affected feign disease,
sparing of nasolabial fold can be seen. illness, or psychological trauma to
What is the most appropriate inv? draw attention,sympathy, or reassurance to
a. Anti ds DNA themselves].
b. Anti histone
c. Anti centromere 1190. A 36yo woman contacts the police
d. Anti Jo to notify them she was responsible for a
e. Anti Scl70 recent disastrous flood with loss of lives.
What kind of delusions is she suffering
Ans. The key is A. Anti ds DNA. [Features are from?
suggestive of SLE. So most appropriate a. Persecutory
b. Poverty e. 1:16
c. Guilt
d. Nihilistic Ans. The key is B. 1:2. [As the parents have a child
with cystic fibrosis and they are healthy both of
e. Reference
them are carrier. It is an autosomal recessive
disease where if both parents are carrier mode of
inheritence is as follows: Chance of being healthy
Ans. The key is C. Delusion of guilt. [Delusions of child 1:4, Chance of being diseased 1:4 and
guilt or sin (self-accusation): This type of chance of being carrier 1:2].
delusions involve feeling guilty or remorseful for
no valid reason.  An example would be someone 1193. A 64yo man believes a female
that believes they were responsible for a war in newscaster is communicating directly with
another country or hurricane damage in another him when she turns a page. What kind of
state.  In this case, the person believes that they delusions is he suffering from?
deserve to be punished for their sins and place a. Persecutory
full blame on themselves]. b. Control
1191. A 27yo man presents with c. Grandeur
d. Nihilistic
symptoms characterized by alternating
e. Reference
mood swings a/w flight of ideas, elation,
over activity and disinhibition, or low Ans. The key is E. Delusion of reference.
mood with lack of energy and social [Delusions of reference – A neutral event is
withdrawal. What is the most probable believed to have a special and personal meaning.
dx? For example, a person with schizophrenia might
a. Bipolar affective disorder believe a billboard or a celebrity is sending a
b. Dysthymia message meant specifically for them].
c. Mania
1194. A 7yo girl with allergy became
d. Hypomania
acutely unwell while visiting a friend’s
e. Cyclothymia
house and has been brought immediately
to the ED. She is fully conscious but has
got stridor, wheeze and erythematous
Ans. The key is A. Bipolar affective disorder.
[Flight of idea, elation, over activity and
rash. She is receiving oxygen. What is the
disinhibition are features of mania and low mood, single immediate management?
lack of energy and social withdrawal are features a. Check airway patency and prepare
of depression. Alternating mood swings with intubation
depression and mania are chracteristic of Bipolar b. Give 0.25ml in 1000U epinephrine IM
disorder]. c. Give 10mg chlorphearamine IM
d. Give 50ml hydrocortisone IM
1192. Healthy parents have 2 children, a e. Obtain secure IV access
child with CF and a healthy child. They
want to have another child. What are the Ans. The key is B. Give 0.25ml in 100U
chances of that child being a carrier? epinephrine IM. [Stridor is one of the indication
a. 1:4 of IM epinephrine in anaphylaxis].
b. 1:2
1195. A terminally ill pt with metastatic
c. 2:3
carcinoma presents with dysphagia and
d. 1:8
difficulty in swallowing. What is the best b. Dressing
possible tx? c. Burst blisters
a. Nystatin suspension d. Local antibiotics
b. Amphotericin B IV e. Refer to burn unit
c. PO fluconazole
Ans. The key is E. Refer to burn unit.
d. Cotrimazole
e. Analgesic 1199. A 28yo man presents with a 2h hx of
rapid palpitations. He feels a little light
Ans. The key is C. PO fluconazole. [Treatment of
carcinoma can predispose to development of
headed but is otherwise well. Exam:
oesophageal candidiasis which is treated as pulse=170bpm and regular,
follows: Oral fluconazole (200 – 400mg) daily for BP=100/68mmHg. He has had 2 similar
14 to 21 days (if needed IV fluconazole can also episodes in the past. What is the most
be given]. likely rhythm disturbance?
a. SVT
1196. A couple attends their GP because b. VF
of marital problems. The wife states that c. VT
her husband is having affairs although she d. V-ectopics
has no proof of this. The husband states e. A-fib
that she even had him followed by a
private detective and this is putting Ans. The key is A. SVT. [Palpitation, light
considerable strain on their marriage. headedness with a tachycardia of 170bpm that is
What is the most likely dx? regular are most likely a SVT].
a. Fregoli syndrome 1200. A child has hypothyroidism. What
b. Cotard syndrome
feature is a/w it?
c. Mood disorder
d. Ekbom syndrome a. Microglossia
e. Othello syndrome b. Prolonged neonatal jaundice
c. Undescended testis
Ans. The key is E. Othello syndrome. [Othello d. Anal tag
syndrome is delusion of infidelity (cheating, e. Left soft palate
adultery, or having an affair) of a spouse or
partner]. Ans. The key is B. Prolonged neonatal jaundice.
[Prolonged neonatal jaundice is a well known
1197. A 65yo lady who is on thiazide
feature of hypothyroidism].
suffers from falls in the morning. What is
the cause for her symptoms? 1201. A 2wk girl presents with E-coli
a. Orthostatic hypotension which is confirmed by urine culture. What
b. TIA is the most appropriate next inv?
c. Epilepsy a. US
b. IVU
Ans. The key is A. Orthostatic hypotensin. c. CT kidney
[Thiazide diuretic is associated with orthostatic
d. BUE
hypotension].
e. MCUG
1198. A boy was admitted with partial
Ans. The key is US. [Infants and children who have
thickness burn, what is your next step?
had a lower urinary tract infection should
a. Escharectomy
undergo ultrasound (within 6 weeks) only if they 1205. A 39yo man comes with umbilicated
are younger than 6 months or have had recurrent papules on his face. His CD4 count is
infections. NICE (Probably to exclude VUR wich is measured to be 35. What is the single
a cause of recurrent UTI)].
most appropriate option?
1202. A lady from Asia presented with a. Mycobacterium avium intercellular
lump in her neck. FNAC has been done and b. CMV
revealed lesions with caseous material in c. Streptokinase
d. Toxoplasmosis
the center surrounded by fibrosis. What is
e. Pneumocystis jerovici
the most probable dx?
a. Thyroid carcinoma f. Moluscum contagiosum
b. TB lymphadenitis Ans. The key is F. Molluscum contagiosum. [CD4
c. Lymphoma count 35 is too low indicating immunodeficiency
d. Inf Mono where molluscum contagiosum occurs more
e. Mesothelioma easily. Umbilicated papules are feature of
molluscum contagiosum].
Ans. The key is B. TB lymphadenitis.
1206. A 45yo man is admitted to ED with
1203. A 32yo woman has undergone a
excruciating pain in the right leg. Exam:
biopsy for a breast lump. The report says:
limb is pale and dorsalis pedis and
a well circumscribed lump with clear
posterior tibial pulses are absent.
margins and separated from the
Pulse=88bpm, irregular and he has a
surrounding fatty tissue. What is the most
pansystolic murmur at apex. What is the
appropriate interpretation of this report?
most probable dx?
a. Fibroadenosis
a. Thromboangitis Obliterans
b. Ca Breast
b. Sciatica
c. Mammary abscess
c. DVT
d. Fibroadenoma
d. Atherosclerosis
e. Fat necrosis
e. Embolus
Ans. The key is D. Fibroadenoma. [Fibroadenomas
Ans. The key is E. Embolus. [Pansystolic murmur
are the most common benign tumor of the
at apex indicates mitral regurgitation and
female breast. White , well-circumscribed mass is
irregular pulse indicates AF which can lead to
clearly demarcated from the surrounding yellow
thrombus formation what probably is responsible
adipose tissue].
for this embolus in leg].
1204. A young boy presented with peri-
1207. An 18yo man has a smooth, tender
oral blisters. Some of which are weeping
swelling extending from the ear to the
and others are crusted. What is the single
angle of the jaw of sudden onset.
most appropriate dx?
Temp=38.5C. What is the single most
a. Impetigo
b. Varicella zoster
likely dx?
a. Dental caries
c. Shingles
b. Mumps
d. Scabies
c. OE
e. Herpes simplex
d. OM
Ans. The key is A. Impetigo. e. Temporomandibular joint pain
Ans. The key is B. Mumps. present on examination of his peripheral
blood smear?
1208. A 6wk baby has a blue mark near
a. Hypersegmented neutrophils
coccyx since birth. His mother is worried.
b. Nucleated RBC
What would you do? c. Blasts
a. Reassure
d. Hypochromic, microcytic RBC
b. Coag profile
e. Schistocytes
c. Karyotyping
d. Skeletal survey
e. CT
Ans. No key is given! Probable key is A.
Ans. The key is A. Reassure. [This is a benign Hypersegmented neutrophils. [Only anemia with
condition known as “Mongolian spot” or macrocytosis is with normal examination findings
“Mongolian blue”. Mongolian spot refers to a makes Megaloblastic aneamia to be most likely
macular blue-gray pigmentation usually on the cause where PBF shows hypersegmented
sacral area of healthy infants. Mongolian spot is neutrophils].
usually present at birth or appears within the first
weeks of life. Mongolian spot typically disappears
1211. A 9yo girl with weekly abdominal
spontaneously within 4 years but can persist for pain and occasional headaches but not
life]. a/w vomiting or diarrhea. She maintains a
good appetite. Lab: normal. CBC, BUE, etc
1209. A man presents with inoperable are normal. Exam: no abnormality as
carcinoma and back pain. His pain has found and the abdomen was soft and non-
been well controlled with morphine but he tender. What would you do for her next?
develops vomiting. Morphine was stopped a. US abdomen
and he was started on metoclopramide b. CT thorax
and fentanyl patches. He then develops c. LFT
neck stiffness and fever. What is the cause d. Reassure
of these symptoms? e. Analgesics
a. Metoclopramide
b. Fentanyl
c. Morphine Ans. The key is D. Reassure. [Probable case of
d. Meningitis abdominal migraine a benign condition for which
e. Metastasis reassurance is appropriate].

Ans. The key is A. Metoclopramide.


[Extrapyramidal effects like neck stiffness is 1212. A 54yo male pt DM with BMI=33
common side effect of metoclopramide and also who has been treated using dietary
there may occur rise of temperature]. control up till now presents to his GP with
a fasting blood sugar of 14mmol/l and
1210. A 51yo man has become creatinine=90mmol/l. Urine shows
increasingly fatigued for the past 10m. PE: glycosuria. No other abnormalities are
no abnormal findings. Labs: Hgb=9.2, found. What is the best next step in
Hct=27.9%, MCV=132fl, plt=242, management?
WBC=7.59. Which of the following a. Biguanide
morphologic findings is most likely to be b. Sulfonylurea
c. Insulin
d. Sugar free diet Ans. The key is A. FNAC.
e. ACEi
1216. A 34yo man from Asia presented
with 5m hx of productive cough, night
Ans. The key is A. Biguanide. [Patient is obese sweats and weight loss. His CXR reveals
type2 diabetic with normal renal function for some shadowing in the left upper zone.
whom biguanide is the treatment of choice]. What is the single most discriminating
1213. What are the side effects of thiazide inv?
a. AFB for sputum
diuretics? b. CXR
a. Hypocalcemia c. CT
b. Hyponatremia d. TFT
c. Hypernatremia e. US abdomen
d. Hyperkalemia

Ans. The key is A. Sputum for AFB. [Features are


Ans. The key is B. Hyponatremia. [Thiazide causes suggestive of PTB for which most discriminating
hypercalcemia, hypokalemia and hyponatremia]. inv. Is Sputum for AFB.

1214. A 46yo man who is a heavy drinker 1217. A prv healthy 23yo presented a
is brought to the ED in a drowsy state. He week hx of bloody diarrhea and
is responding vaguely to questions. Exam: abdominal pain with cramps and fever.
nystagmus and hyperreflexia. MCV=103fl. Exam: tenderness in lower abdomen.
What is the most likely cause for his What is the most appropriate dx?
cognitive impairment? a. Celiac disease
a. B1 deficiency b. Colorectal polyps
b. B12 deficiency c. UC
c. Folate deficiency d. Laxative abuse
d. B6 deficiency e. Gastroenteritis
e. Alcohol withdrawal

Ans. The key is E. Gastroenteritis.


Ans. The key is A. B1 deficiency. [Presence of
confusion and nystagmus indicates Wernicke’s 1218. A 10yo boy presents with irritability,
encephalopathy caused by thiamin (vit. B1) sudeen onset of pain and discharge from
deficiency]. the right ear. Which antibiotic would be
1215. A 23yo female presented with a the 1st line of tx?
a. Amoxicillin
swelling of her neck that moved upwards
b. Ciprofloxacillin
on protrusion of tongue. What is the next
c. Flucloxacillin
appropriate inv?
d. Ceftazidime
a. FNAC
e. Benzyl penicillin
b. Punch biopsy
c. Core biopsy
d. MRI neck
e. Radioactive thyroid scan
Ans. The key is A. Amoxycillin. [For acute swelling increased after large meal as the saliva
suppurative otitis media 1st line antibiotic is produced during meal could not come out of the
Amoxycillin. Ref: patient.info]. gland due to stone].

1219. A 26yo man strongly believes that 1222. A 45yo man has had impaired vision
every elderly man he meets is his father. and pain on eye movement in his left eye
Although they look different, he is sure it over the last 5d. He also notes loss of color
is father wearing different disguises. What vision in the same eye. In the left eye, the
kind of delusions is this man suffering visual acuity is up to counting fingers.
from? When the pupil is stimulated with light, it
a. Delusion of persecution dilates. His fundus is normal. What is the
b. Erotomania single most appropriate clinical dx?
c. Delusion of grandeur a. Acute dacryocystitis
d. Delusion of doubles b. Acute iritis
e. Delusion of reference c. Papillitis
d. Retrobulbar neuritis
Ans. The key is D. Delusion of doubles. [The
e. Scleritis
Fregoli delusion, or the delusion of doubles, is a
rare disorder in which a person holds a delusional Ans. The key is D. Retrobulbar neuritis.
belief that different people are in fact a single [Presentation can be described by either optic
person]. neuritis or retrobulbar neuritis. In optic neuritis
there is disc pallor. As in this case disc is normal it
1220. A 26yo passed a 4mm stone in his
is more likely to be a case of retrobulbar neuritis.
urine. On US a 3mm stone is found in the
renal pelvis. What is the single most 1223. A 56yo pt has been dx with MS. She
appropriate management? presents with a positive Romberg’s test.
a. ESWL She also has weakness and loss of
b. None sensations in all her 4 limbs. Which site is
c. Open Surgery most likely to be affected?
d. Conservative a. Cerebral cortex
b. Cerebellum
Ans. The key is D. Conservative. [Increased fluid
c. Cervical spinal cord
intake is advised].
d. Thoracic spinal cord
1221. A 35yo man has had acute pain and e. Brain stem
swelling below the mandible on the left
side for 2h. The swelling occurred after
eating a large meal. What is the single Ans. The key is C. Cervical spinal cord.
most likely dx? [Quadriplegia is the feature of cervical cord
a. Laryngocele lesion].
b. Ranula
c. Neck abscess 1224. A 58yo man suddenly becomes
d. Parotid calculus
shocked several days after suffering an
e. Submandibular calculus
acute ant MI. His CXR shows a large
Ans. The key is E. Submandibular calculus. [Pain globular-shaped heart and clear lung
and swelling below mandible is due to stone in fields. What is the single most likely
submandibular salivary gland duct. Pain and explanation for the abnormal inv?
a. Acute pericarditis Ans. The key is C. Closed reduction of fracture.
b. Cardiac tamponade This is a wrong key! Correct option seems to be B.
c. Atrial thrombus Below elbow split plaster of paris. [Older people
d. Left ventricular aneurysm are less likely to go under closed reduction even if
e. Dressler syndrome the dorsal angulation is not anatomic (like 20
degrees!). Less than 10 degree dorsal angulation
does not require even closed reduction but only
immobilization. [Medscape]. As plaster cast is
Ans. The key is B. Cardiac tamponade. [Shock in a
often advocated B. Below elbow split plaster of
post MI patient with globular heart on X-ray and
paris seems to be the option of choice].
clear lung field indicate Cardiac tamponade].
1227. A 16yo girl who is normally fit and
1225. A 56yo alcoholic man who has well attends her GP complaining of heavy
increased the amount of alcohol he is and painful periods. She is requesting tx
using wants to attend his daughter’s for these complaints. She denies being
wedding that is in 2wks. He is now coming sexually active. Select the most
to you for help. How would you help him? appropriate management for her
a. Acamprosate menorrhagia?
b. Refer to clinical psychologist a. Antifibrinolytics (tranexamic acid)
b. COCP
c. Refer to GP
d. Despiramine c. Endometrial ablation
d. IUS progestrogens (mirena)
e. Refer to community mental health
support group. e. NSAIDS (mefenamic acid)

Ans. The key is A. Antifibrinolytics (tranexamic


Ans. The key is A. Acamprosate. [If the patient
acid). It is a wrong key. Correct answer is E.
stop alcohol without any supportive treatment
NSAIDs (mefenamic acid)
there will occur withdrawal symptoms. For the
presented situation Acamprosate can help by
stopping alcohol without producing withdrawal
symtoms (by restoring brain chemical 1228. A 67yo lady with an ulcer on the
derangement cased by alcohol which is anal margin. Which is the single most
responsible for withdrawal symptoms)]. appropriate LN involved?
a. External iliac LN
b. Pre-aortic LN
1226. An 80yo woman fell over at her
c. Aortic LN
nursing home. XR shows fx of radius with
d. Inguinal LN
<10degree of dorsal angulation. What is
e. Iliac LN
the single most appropriate tx?
a. Below elbow full plaster of paris
b. Below elbow split plaster of paris
Ans. The key is D. Inguinal LN. [Anal canal below
c. Closed reduction of fx
pectinate line is drained into superficial inguinal
d. Elasticated support bandage
lymph nodes].
e. Open reduction and internal fixation
1229. A branch of the dominant coronary
artery that supplies the inferior portion of 1231. A pt is on cancer tx with
the septum. What is the single most dexamethasone. According to her
appropriate option? biochemical results her K+=normal and
a. Septal branches her Na+=low. What is the dx?
b. Obtuse marginal branches a. Addisons
c. Circumflex artery b. Dexamethasone side effect
d. Left main stem, post descending artery c. Dilutional hyponatremia
e. Diagonal branch

Ans. The key is C. Dilutional hyponatremia.


Ans. The key is D. Left main stem, post
descending artery. [Here no option is
satisfactorily correct! By dominant coronary 1232. A diabetic has been prescribed a
artery we mean that coronary artery which gives long acting hypoglycemic in the morning
of the branch of posterior descending artery. and short acting in the evening. He takes
Mostly it is right coronary artery and if there is a regular lunch, but has been having
left coronary dominance, posterior descending
hypoglycemic attacks at around 4pm each
artery is the branch of circumflex artery and not
day. What is the most appropriate
direct branch of left main artery. However the
only option that goes nearer is D. Left main stem,
intervention?
a. Recommend a heavier lunch
post. descending artery].
b. Review morning drug
c. Review evening drug
d. Review both drug
1230. A 55yo female presented with
e. Reassure
anemia and dysphagia. There is a feeling
of something stuck in the throat. The
esophagus can’t be negotiated beyond the
Ans. The key is B. Review morning drug. [As there
crico-pharynx. What is the most probable
is hypoglycemic attacks at evening morning dose
dx? needs to be adjusted].
a. Foreign body
b. Plummer vinson syndrome
c. Pharyngeal carcinoma 1233. A male pt presented with blood and
d. Barret’s esophagus mucus in stool. He has also noticed weight
e. Esophageal carcinoma loss but has no hx of altered bowel habits.
What is the dx?
a. Carcinoma of cecum
Ans. The key is B. Plummer Vinson syndrome. b. Carcinoma of descending colon
[The picture fits two D/D. 1. Plummer Vinson c. Carcinoma of sigmoid colon
syndrome 2. Oesophagial carcinoma. Lower d. Carcinoma of rectum
oesophagial stricture is more common for Ca. So
presenting case is likely a case of Plummer Vinson Ans. The key is A. Carcinoma of cecum.
syndrome. It can be differentiated by the type of
anemia. If it is IDA dx is Plummer Vinson
syndrome].
1234. A 22yo man keeps having persistent
and intrusive thoughts that he is a dirty Ans. The key is D. OCD. [Obsessive compulsive
thief. No matter what he tries these disorder (OCD) is a mental health condition
thoughts keep coming to him. Any where a person has obsessive thoughts and
attempt to avoid these thoughts leads to compulsive activity.
serious anxiety. What is the most likely An obsession is an unwanted and unpleasant
thought, image or urge that repeatedly enters a
dx? person's mind, causing feelings of anxiety, disgust
a. Schizophrenia
or unease.
b. OCD A compulsion is a repetitive behaviour or mental
c. PTSD act that someone feels they need to carry out to
d. Mania try to temporarily relieve the unpleasant feelings
e. Psychotic depression brought on by the obsessive thought].

1237. A 6wk baby has been dx as HIV+ve.


Ans. The key is B. OCD. Which immunization plan will you opt for
him?
1235. A 45yo female comes to the ED a. Don’t give any vaccine
while having a generalized tonic clonic b. Give all vaccines except live attenuated
seizure and she has having difficulty vaccines
c. Give only BCG vaccine
breathing and is cyanosed. What is the tx
d. Give all vaccines except BCG vaccine
option for her?
a. Secure airways
b. IV diazepam
c. IV phenytoin Ans. The key is D. Give all vaccines except BCG
d. Oxygen mask vaccine.

Ans. A. Secure airways. [Breathing difficulty with 1238. A 36yo man has been dx with DI.
cyanosis may indicate saliva, blood, foreign body What electrolyte picture is expected to be
(as denture), fall back of tongue etc. for which 1st seen?
action is to secure airway]. a. High serum Na, low serum osmolarity,
high urine osmolarity
b. Low serum Na, low serum osmolarity,
1236. A 30yo man is becoming concerned high urine osmolarity
about the safety of his family. He has been c. Low serum Na, high serum osmolarity,
checking the locks of the door every hour high urine osmolarity
during the night. He becomes very anxious d. High serum Na, high serum osmolarity,
if his wife tries to stop him. What is the low urine osmolarity
e. Normal Na, normal serum osmolarity,
most likely dx?
a. Paranoid delusion normal urine osmolarity
b. PTSD
c. Social phobia
d. OCD Ans. The key is D. High serum Na, high serum
e. GAD osmolarity, low urine osmolarity.
and LOC. She is brought to the hosp where
she is found to be conscious and
1239. The artery that supplies the ant
completely alert. Exam: normal pulse and
right ventricular wall. What is the single
BP with no abnormal neurological sign.
most appropriate option?
a. Acute marginal branch What is the next step in her
b. Left ant descending artery management?
a. Admission for observation
c. Coronary sinus
b. CT brain
d. Circumflex artery
c. MRI head
e. Right coronary artery
d. Reassurance and discharge home
e. XR skull

Ans. The key is A. Acute marginal branch.

Ans. The key is B. CT brain. [To exclude any


possible intracranial bleeding].
1240. A 55yo male presents to the ED
after an RTA with breathlessness,
engorged neck veins and a dull percussion
1243. A 30yo woman is taking tx for
note on the right side of his chest. Exam:
asthma. She has a HR=130bpm and peak
pulse=140bpm, BP=80/50mmHg. What is
expiratory flow rate=400. What is the
the most likely dx?
a. Hemothorax
most appropriate management?
a. Atenolol
b. Hemopneumothorax
b. Digoxin
c. Tension pneumothorax
c. Review drugs
d. Simple pneumothorax

Ans. The key is C. Review drugs. [Oral beta


Ans. The key is B. Hemopneumothorax.
agonist frequently causes tachycardia and
palpitations].

1241. A 32yo woman presents with


complaints of having low back pain. She is 1244. A pt presents with a mask face. He
taking analgesics for it. All inv are normal. also has gait prbs. Which class of drug is
What will you advice her? causing this?
a. Bed rest
a. Anti-depressant
b. Physiotherapy b. Anti-psychotic
c. Advice to be more active
c. Anti-HTN
d. Admit

Ans. The key is B. Anty-psychotic. [Anti-psychotics


Ans. C. Advice to be more active. by reducing dopamine can precipitate or induce
parkinsonism].

1242. A 32yo woman suffers an episode of 1245. A 16yo boy came home from
severe occipital headache with vomiting boarding school with a cough. His CXR
showed bilateral consolidations. What is radiculopathy (pressure on spinal nerve root) can
the most likely organism which would cause this shooting pain which can be
have caused his symptoms? demonstrated by MRI spine].
a. Legionella pneumophilia
b. Mycoplasma pneumonia 1248. A young man returns to his hostel
c. Mycobacterium TB and gets headache and lethargy. Now
d. Pneumocystis jiroveci presents with fever. There are crepitations
e. Pseudomonas aeruginosa on the auscultation of lung. What is the
most likely organism which would have
caused his symptoms?
Ans. The key is B. Mycoplasma pneumonia. a. Legionella pneumonia
[Mycoplasma is common in military barrack, b. Mycoplasma
prison or boarding dwellers]. c. Staphylococcus
d. Streptococcus
1246. After an MI, a man presents with
Ans. The key is A. Legionella pneumonia.
pansystolic murmur which is radiating to [Legionella is common in hostel, hotel, hospital,
the axilla. What is the dx? nursing home where it spread through their
a. Tricuspid regurgitation water system, aircondition etc.].
b. Mitral regurgitation
c. Aortic stenosis 1249. A pt is about to undergo surgery.
d. Mitral stenosis Her Hgb=8.9g/dl and MCV=70. What is
the best option for her?
a. Inv and postpone the surgery
Ans. The key is B. Mitral regurgitation. [MI can b. Transfuse and proceed with surgery
lead to papillary muscle rupture causing mitral c. Transfuse and defer surgery
regurgitation]. d. Continue with surgery

1247. A 34yo laborer developed severe Ans. Key is A. Investigation and postpone the
pain in his lower back after lifting a sack surgery. [For elective operation only proceed if
of sand. He also complains of shooting Hb% > 10 g/dl. If Hb% < 10 g/dl then defer the
pain down his leg. The GP has prescribed operation and investigate first. If Hb% < 8 g/dl
him complete bed rest, with painkillers also must be transfused. Samson note].
and also scheduled an MRI for him. What
is the most likely dx? 1250. A 24yo male presents with
a. Peripheral vascular disease discomfort in the groin area and scrotal
b. Intervertebral disc prolapse swelling. Exam: scrotal skin is normal.
c. Hairline fx of the spine What would be the next best step?
d. Sprain of the back muscles a. Urgent US
e. Muscle injury b. Urgent surgery
c. OPD referral
d. Antibiotics
Ans. The key is B. Intervertebral disc prolapsed.
[Intervertebral disc prolapsed can cause severe
back pain following heavy lifting and
Ans. The key is C. OPD referral. [A case of inguinal to go home the following day. Which of
hernia. In both torsion or epididymo-orchitis the following does she require?
scrotal skin will be inflammed (erythematous and a. OPD referral to relationship counselor
oedematous). Should be referred to Surgery b. OPD referral to psychiatrist
OPD]. c. Inpatient referral to psychiatrist
d. Inpatient referral to psychologist
1251. A 22yo girl unhappy about her
weight with BMI=22. She likes to have her
dinner in an expensive restaurant. She Ans. The key is C. Inpatient referral to
does excessive shopping. K+=3.3. What is psychiatrist. [Suicidal ideation is abnormal
the dx? psychology and if discharged there is chance of
a. Anorexia nervosa repeated suicidal attempts. So inpatient referral
b. Bipolar to psychiatrist is needed for this patient].
c. OCD
d. Bulimia
1254. A 74yo man presents with sudden
onset of with right sided weakness and
Ans. The key is D. Bulimia. [BMI 22, even though slurred speech. He also has loss of
unhappy, hypokalemia, like to have dinner in an sensation over the right side of the body
expensive restaurant (probable binge eating)
and visual field defects. CT shows ischemic
suggest the diagnosis of bulimia].
stroke. What is the most appropriate
management?
1252. A 59yo pt has been dx with HTN. His a. Alteplase
BP has been >160/90mmHg on 3 separate b. Streptokinase
occasions. His biochemical profile is as c. Nimodipine
follows: Na+=145mmol/l, K+=6.2mmol/l, d. Aspirin
creatinine=112umol/l, urea=5.7mmol/l. e. Labetolol
What is the most appropriate anti-HTN
drug for him?
a. Amlodipine Ans. The key is A. Alteplase. [Patient has acute
b. Bendroflumethiazide onset of symptoms and time of onset is also
c. Ramipril known. So considering window period of 4.5
d. Lorsartan hours from onset of symptoms can be given
e. Propranolol alteplase].

Ans. The key is B. Bendroflumethiazide. [Thiazide


reduces Na+ and K+ level. So in this picture of 1255. The artery that runs along the left
upper normal Na+ with hyperkalemia thiazide
AV groove. What is the single most
seems to be appropriate].
appropriate option?
a. Left internal mammary artery
1253. A 22yo girl had a fight with her b. Left anterior descending artery
boyfriend and then took 22 tabs of c. Circumflex artery
paracetamol. She was commenced on N- d. Left main stem (LMS) post descending
acetyl cysteine and she was medically fit artery
e. Diagonal branch c. Ovarian ca
d. Breast ca
e. Vaginal ca
Ans. The key is C. Circumflex artery.

Ans. The key is B. Endometrial Ca. [Actually it is


1256. A 26yo man presents with painless endometrial thickness and in general 12 mm
hematuria. He has no other complaints means hyperplasia which may suggest
and on examination no other abnormality endometrial Ca. in this woman].
is found. What is the most appropriate
initial inv to get to a dx?
a. Cystoscopy 1259. A 30yo woman has PID which was
b. Midstream urine for culture treated with metronidazole and
c. Abdominal US cephalosporin. It is getting worse. What is
d. MRI spine the next best inv?
e. Coag screening a. Endocervical swab
b. US
c. Laparotomy
Ans. Ans. The key is C. Abdominal ultrasound. d. High vaginal swab
[Abdominal US to exclude polycystic disease,
Ans. The key is B. US. [Probable tubo-ovarian
malignancy, tumour, urolithiasis etc].
abscess].

1257. A pt, 50yo smoker and heavy 1260. A pregnant woman had hit her
drinker, presents with complaints of chest 3wks ago. Now she is 24wks
racing heart. A 24h EKG comes out pregnant and presents with left upper
normal. What is your next step in quadrant mass with dimpling. What is the
management? most probable dx?
a. Echo
a. Breast ca
b. Reassure
b. Carcinoma
c. Stress test
c. Fibroadenoma
d. Fibroadenosis
e. Fatty necrosis of breast
Ans. The key is B. Reassure. [Racing heart or
palpitation is a common phenomenon in
alcoholics which is not serious or harmful. So
Ans. The key is E. Fatty necrosis of the breast.
reassure the patient].

1261. A pregnant pt with Rh –ve who


1258. A 36yo woman came with uterine
hasn’t been prv sensitized delivers her first
bleeding. Vaginal US reveals uterine
baby without any prbs. What would be
thickness=12mm. what is the most
the latest time to administer anti-
probable dx?
a. Cervical ca sensitization?
a. 6h PP
b. Endometrial ca
b. 24h PP Ans. The key is D. Secondary PPH. [Secondary PPH
c. 48h PP occurs from 24 hours after delivery. Usually
d. 72h PP occurs between 5 and 12 days].
e. 5d PP
1264. A 22yo lady who is in her last
trimester of pregnant comes with hx of
exposure to a child dx with chicken pox 1d
Ans. The key is D. 72h PP.
ago. She was investigated and was +ve for
varicella antibody. What is the single most
1262. A 30yo primigravida who is 30wks appropriate management?
a. Give varicella Ig
GA presents to the L&D with absent fetal
b. Quarantine
movements. She also complains of severe
c. Give varicella vaccination
headache, heartburn and seeing floaters d. Oral acyclovir
before her eyes for the last few days. e. Reassure
Exam: BP=170/110mmHg, urine protein=+
+++, rock hard uterus, no visible signs of
fetal movements. Choose the single most Ans. The key is E. Reassure. [ If you have
likely dx? antibodies in your blood, this means you have
had chickenpox in the past, or have been
a. Abruption of placenta 2nd pre-eclampsia
immunised. No further action is then needed].
b. Antepartum hemorrhage
c. Placenta previa
1265. A 22yo woman who is 20wk
d. Primary PPH
pregnant came with pain and bleeding per
e. IUFD
f. Abruption of placenta due to trauma
vagina. Exam: os is not open. What is the
single most likely dx?
a. Threatened abortion
b. Missed abortion
Ans. 2 keys A. Abruption of placenta 2nd pre-
c. APH
eclampsia. E. IUFD.
d. Miscarriage
e. Inevitable abortion

1263. A 38yo woman, 10d post partum,


presents to her GP with a hx of passing Ans. The key is A. Threatened abortion.
blood clots per vagina since yesterday.
Exam: BP=90/40mmHg, pulse=110bpm, 1266. A 32yo lady G1, 28wks GA came to
temp=38C, uterus tender on palpation and her ANC with a concern about pain relief
fundus is 2cm above umbilicus, blood clots during labour. She has no medical
+++. illnesses and her pregnancy so far has
Choose the single most likely dx? been uncomplicated. She wishes to feel
a. Abruption of placenta 2nd preeclampsia
b. Concealed hemorrhage
her baby being born but at the same time
c. Primary PPH she wants something to work throughout
d. Secondary PPH her labour. What method of pain relief
e. Retained placenta best matches this lady’s request?
f. Scabies a. C-section
b. Pudendal block
c. Entonox pregnancy test is +ve and US scan shows
d. TENS an empty uterus. What is the next step?
e. Pethidine a. Laparoscopy
b. HCG measurements
Ans. The key is C. Entonox. [This is a mixture of
c. US
oxygen and nitrous oxide gas. Gas and air won't
d. Laparotomy
remove all the pain, but it can help to reduce it
e. Culdo-centhesis
and make it more bearable].

Ans. B. HCG measurements. This is a wrong key!


1267. A primipara at fullterm in labor has
Correct key is A. Laparoscopy. [Surgery should
passed show and the cervix is 3cm dilated.
What is the single most appropriate be offered to those women who cannot return
management for her labor? for follow-up after methotrexate or to those who
a. Repeat vaginal examination in 4h have any of the following:
b. CTG
c. IV syntocin drip  Significant pain.
d. Repeat vaginal examination in 2h  Adnexal mass ≥35 mm.
e. Induction of labour  Fetal heartbeat visible on scan.
 Serum hCG level ≥5000 IU/L.

Ans. The key is A. Repeat vaginal examination in


A laparoscopic approach is preferable. A
4h.
salpingectomy should be performed,
unless the woman has other risk factors
1268. A 36yo pregnant woman comes for
evaluation with her husband. Her husband for infertility, in which case a salpingotomy

has been complaining of morning should be undertaken].


sickness, easy fatiguability and even
[If HCG >6000IU/L and an intrauterine gestational
intermittent abdominal pain. What is the
sac is not seen, ectopic pregnancy is very likely, as
husband suffering from? is the case if HCG 1000–1500IU/L and no sac is
a. Ganser syndrome
seen on transvaginal Ultrasound].
b. Couvade syndrome
c. Pseudo-psychosis
d. Stockholm syndrome
1270. A 23yo woman who has had several
e. Paris syndrome
recent partners has experienced post-
coital bleeding on gentle contact. What is
Ans. The key is B. Couvade syndrome. [Cauvade the single most likely cause of her vaginal
syndrome, also called sympathetic pregnancy, is a discharge?
proposed condition in which a partner a. Cervical ca
experiences some of the same symptoms and b. Cervical ectropion
behavior of an expectant mother]. c. CIN
d. Chlamydial cervicitis
1269. A woman comes to the ED e. Gonococcal cervicitis
complaining of pain in the right side of the
abdomen, she has 7wks amenorrhea. Her
Ans. D. Chlamydia cervicitis.
Ans. The key is B. Chronnic pyelonephritis. [35%
of childhood UTI is associated with VUR and many
1271. A 68yo woman presents with post- of them develop renal scarring and chronic
pyelonephritis causing hypertension].
coital bleeding following her first episode
of sexual intercourse in 10yrs. What is the
single most likely cause that has led to 1274. A 24yo woman has had lower
post-coital bleeding? abdominal pain for 12h. She is otherwise
a. Endometrial ca well. She is at 10wks GA in a planned
b. Atrophic vaginitis pregnany. What is the single most
c. Endometrial polyp appropriate test to inv the cause of acute
d. Cervical ca abdomen in this lady?
e. Cervical ectropion a. Abdominal US
b. Anti-phospholipid screen
c. CBC
Ans. The key is B. Atrophic vaginitis. d. Transvaginal US
e. Laparoscopy

1272. A 28yo woman 8wks GA had PID


Ans. The key is D. Transvaginal US. [D/D: ectopic
treated prvly and now comes with vaginal pregnancy, renal colic, torsion of ovarian cyst,
bleeding, rigid abdomen, appendicitis etc. Transvaginal US will give better
BP=80/50mmHg, pulse=140bpm. What is results in case of lower abdominal pain].
the most probable dx?
a. Threatened abortion
b. Miscarriage 1275. A pt is at term and in labor, the
c. Missed abortion membranes have ruptured, the liquor
d. Tubal pregnancy contains meconium but the CTG is normal.
e. Inevitable abortion The cervix is 3cm dilated. What is the
single most appropriate action?
a. BP monitoring
Ans. The key is D. Tubal pregnancy. [Previous PUD b. CTG
is a risk factor for tubal pregnancy]. c. C-section
d. Fetal scalp blood sample
e. Internal rotation
1273. A 34yo primigravida who is 16wk
GA comes for routine antenatal check up. Ans. The key is D. Fetal scalp blood sample.
Her
BP=160/100mmHg. She has a hx of
1276. A pt is at term and labor. The head
repeated childhood UTI. What is the most
has been delivered and you suspect
likely cause of her high BP?
a. Essential HTN shoulder dystocia. What is the single most
b. Chronic pyelonephritis appropriate action?
a. C-section
c. Acute pyelonephritis
b. Episiotomy
d. Pre-eclampsia
c. External rotation
e. Chronic UTI
d. Fetal scalp blood sample a. Termination of pregnancy
e. Instrumental delivery b. TPN
c. Feeds via NGT
Ans. The key is B. Episiotomy. d. P6 acupressure
e. IV hydrocortisone

1277. A 29yo female at 28wks GA


presents to you with complains of hard Ans. The key is A. Termination of pregnancy. This
stools and constipation for last 2wks. CTG is wrong key. Correct key is E. IV hydrocortisone.
shows fetal tachycardia. What is the [Termination is the last resort! Before it IV
single most appropriate tx? hydrocortisone is tried].
a. Oral laxatives
b. Fiber diet
c. Phosphate enema
d. Lactulose
e. Reassure 1280. A young lady with primary
amenorrhea has normal LH, FSH, estradiol
and prolactin. Choose the single most
Ans. The key is B. Fiber diet. [Changes to diet and likely dx?
lifestyle are often recommended as the first a. PCOS
treatment for constipation. It includes high fiber b. POF
diet and plenty of fluid, regular exercise etc.]. c. Absent uterus
d. Absent ovaries
e. Turner’s syndrome
1278. A 16yo girl presents with heavy
bleeding. What is the most appropriate Ans. The key is C. Absent uterus. [Normal LH, FSH,
initial inv? estradiol and prolactin rule outs PCOS (increased
LH, increased FSH, normal oestrogen. LH:FSH
a. Endometrial sampling
ratio is 2:1 or 3:1), POF (in POF, LH & FSH raised,
b. Transvaginal US FSH > 20 IU/L), Absent ovary will lead to low
c. Hysteroscopy estradiol, high FSH and LH, Turner’s syndrome:
d. Pelvic US gonadal streaks, as absent ovaries].
e. Exam under anesthesia

1281. An obese lady presents with primary


amenorrhea. She has high LH, normal FSH
Ans. The key is D. Pelvic US.
and slightly high prolactin levels. Choose
the single most likely dx?
a. PCOS
1279. A woman who is 7wks pregnant
b. POF
presents with excessive and severe
c. Hypothyroidism
vomiting and put on IV fluids and anti- d. Pregnancy
emetic (ondansteron). She is complaining e. Primary obesity
of severe headache and can’t take oral
fluids. What is the most appropriate
management?
Ans. The key is A. PCOS. [High LH, high or normal a. HRT
FSH with slight rise in prolactin levels in an obese b. COCP
lady is suggestive of PCOS]. c. Estrogen gel
d. Testosterone gel
1282. A 38yo lady presents with
amenorrhea has very high LH and FSH
levels, normal prolactin and low estradiol. Ans. No key is given. Likely key is C. Estrogen gel.
[Seems to be atrophic vaginitis for which estrogen
Choose the single most likely dx?
a. PCOS gel can be given].
b. POF
c. Hypothyroidism
d. Pregnancy 1285. A 35yo lady with subserosal
e. Menopause fibroid=4cm and submural fibroid=6cm is
planning for a child. Which way will you
remove the fibroids?
Ans. The key is B. POF. [High LH and FSH, normal a. Laproscopy
prolactin and low estrogen in secondary b. Vaginal myomectomy
amenorrhea in a lady under age 40 is highly c. Abdominal myomectomy
suggestive of POF]. d. Drugs
e. Reassure
1283. A 77yo publican was admitted for
an appendectomy. Post-op he becomes
confused, agitated and starts to pick at Ans. The key is B. Vaginal myomectomy. Probably
things. He is then given an IV drug which wrong key! Correct key should be C. Abdominal
settles this confusion. Which of the myomectomy. [Subserosal fibroid is not suitable
following drugs was given for his to treat with vaginal myomectomy. Abdominal
myomectomy can deal with both subserosal and
confusion?
submural fibroid].
a. Diazepam
b. Chlordiazepoxide
c. Thiamine
d. Vit B 1286. A 32yo presents with heavy blood
loss, US: uterine thickness>14mm. What is
the best possible management for her?
a. COCP
Ans. The key is B. Chlordiazepoxide. [Patient is
b. UAE
alcoholic. On admission abstinence from alcohol
caused this withdrawal symptom. Agitation, c. Hysteroscopy myomectomy
confusion and pick at things are suggestive of d. Abdominal myomectomy
delirium tremens which is treated with e. Endometrial ablation
Chlordiazepoxide].

Ans. B. UAE. [COCP will not resolve the case.


1284. A 65yo lady presents with There is no fibroid so no myomectomy.
dyspareunia. What will you give her for Endometrial ablation may render the young lady
non fertile. So UAE is the only suitable option
her condition?
here].
1290. An 8yo boy with a BMI=28 was
1287. A pt comes with sudden loss of admitted to a surgical ward following a
vision. Exam: high BP. Fundoscopy: retina MVC. He was found to have glycosuria.
appears swollen. Which blood vessel When he recovered from his injury the
occlusion is involved? glycosuria resolved. What is the single
a. Branch RVO most appropriate follow-up inv?
b. Branch RAO a. Fasting blood glucose conc
c. CRAO b. Glycosylated hemoglobin - HbA1c
d. CRVO c. OGTT
d. Random blood glucose conc
e. Serum cortisol conc
Ans. The key is D. CRVO.

Ans. The key is A. Fasting blood glucose


1288. A 2yo girl has had a temp=39C, poor concentration. [The boy had glycosuria while
appetite, abdominal pain and urinary treating in hospital following a MVC (motor
frequency for 3d. What is the single most vehical crush). It may be due to stress related
appropriate inv? cortisol release which later became nil as the boy
a. Catheter specimen of urine for culture recovered from trauma. But yet to certain that he
is not diabetic fasting blood glucose
b. Clean catch urine specimen for culture
concentration should be done in follow up visit].
c. CBC
d. KUB US
e. Supra-pubic aspirate of urine for culture
1291. At birth, a baby boy at 38wks GA
weighs 1.8kgs. He has hepato-
splenomegaly and a rash. Blood test show
Ans. The key is B. Clean catch specimen of urine
for culture. raised level of bilirubin and liver enzymes.
What is the most likely dx?
a. Galactosemia
b. Biliary atresia
1289. A child with T1DM who is not
c. G6PD deficiency
compliant with meds and eats a lot. He
d. Rh incompatibility
thinks that he is short in his class. He is
e. Congenital viral infection
not happy. What would you do next?
a. Refer to psychologist
b. Refer to pediatrician
c. Refer to GP Ans. The key is E. Congenital viral infection.
[Congenital infections affect the unborn fetus or
d. Refer to social services
newborn infant. They are generally caused by
e. Change type of insulin
viruses that may be picked up by the baby at any
time during the pregnancy up through the time of
delivery. The more common viruses linked to
Ans. The key is A. Refer to psychologist. congenital infections include the Cytomegalovirus
(CMV), Herpes, Rubella (German measles),
Parvovirus, Varicella (chickenpox), and
Enteroviruses].
1295. A pt presents with headache,
1292. A 12yo boy with T1DM has poor
blurring of vision and acuity loss. On
long-term control. He is unconscious,
fundoscopy, dots and blots were noted
hyperventilating and dehydrated. His
with huge red swollen optic disc. What is
blood glucose is 28mmol/l. What is the
the most probable dx?
single most imp initial tx? a. CRAO
a. Albumin IV
b. Branch RAO
b. Bicarbonate IV
c. CRVO
c. Insulin IV
d. Optic atrophy
d. Insulin SC
e. Saline 0.9% IV

Ans. The key is C. CRVO.

Ans. The key is E. Saline 0.9% IV. [Probable


diabetic ketoacidosis. Initial treatment is IV fluid 1296. A 64yo DM has come for a routine
(0.9% saline)]. eye check up. Fundoscopy: new vessels all
over the retina. What is the most
appropriate management?
1293. A 30yo woman on OCP presents a. Strict sugar control
with dilated tortuous veins crossing her b. Regular eye check ups
abdomen to join the tributaries to SVC. c. Non urgent referral to specialist
What is the single most likely cause? d. Laser photocoagulation
a. Intra-abdominal malignancy e. Insulin
b. Ovarian cyst
c. Fibroids
d. Ascites Ans. The key is D. Laser photocoagulation.
e. DVT [Neovascularization suggests proliferative
diabetic retinopathy which is treated with laser
photocoagulation].
Ans. The key is A. Intra-abdominal malignancy.

1294. An 84yo woman with drusen and 1297. A 25yo primigravida of 8wk GA
yellow spots in the center of retina. What presents with severe lower abdominal
is the single most likely dx? pain, vaginal bleeding and passage of
a. Macular degeneration clots. The internal os is open. What is the
b. HTN retinopathy most likely dx?
c. MS a. Appendicitis
d. DM background b. Placental abruption
e. Proliferative DM retinopathy c. Ectopic pregnancy
d. Abortion

Ans. The key is A. Macular degeneration. [In early


disease, the macula shows yellowish-colored Ans. The key is abortion. [Up to 24 weeks
subretinal deposits called “drusen”]. termination of pregnancy is abortion. Ref:
patient.info]
1298. A man developed intense pain after BC=normal. An audiogram shows
using the end of a pencil to scratch his moderate hearing loss in both ears across
inner ear. He took out the pencil from his all frequencies. What is the single most
ear and realized the end of the pencil with likely dx?
the rubber part is still stuck in his ear. a. Acoustic neuroma
What is the most appropriate b. Menieres’ disease
management? c. Noise induced deafness
a. Remove with a hook d. Otosclerosis
b. Instill olive oil e. Presbyacusis
c. Remove GA
d. Remove with magnet instrument
e. Do syringing Ans. The key is D. Otosclerosis. [This patient has
conductive hearing loss. So it is not acoustic
neuroma. Meniere’s disease has symptoms like
Ans. The key is A. Remove with a hook. [Place a vertigo, tinnitus, fullness, along with hearing loss.
hook behind the object and pull it out]. There is no history of noise pollution not
presbycausis as his age is 45. So his deafness fits
more with otosclerosis].
1299. A 16yo boy presents with acute pain
in the right ear and little bleeding from
1301. The biological parents of a child
the same ear. He had been in a boxing
with CF come to you to know about the
match and had sustained a blow to the
chances of their future children with the
ear. There is little amount of blood in the
same disease. What would you say to
auditory canal and a small perforation of
them?
the eardrum. What is the most
a. There is a 1:4 chance that your future
appropriate management?
child will have this disease
a. Admission for parental antibiotics
b. Nasal decongestant b. All their unaffected children will be
c. Oral amoxicillin carriers of CF
d. OPD review c. Nothing can be predicted
e. Packing of ear d. It can 100% dx antenatally
f. Surgical intervention
g. Syringing ENT Ans. The key is A. Tnshere is a 1:4 chance that
your future child will have this disease. [It is
autosomal recessive disease. As one child is
Ans. The key is C. Oral amoxicillin. [Small
affected both parents are carrier. So in this case
perforations are usually heal by themselves
next child will be normal in 1:4 case, carrier in 1:2
within 6-8 weeks and doctors often prescribe
cases and diseased in 1:4 cases].
antibiotics to prevent infections during this
healing period].

1302. A 14yo boy presents with recurrent


1300. A 45yo man has noticed difficulty abdominal pain, malaise and weight loss
hearing on the telephone. He is concerned over 6m. Exam: vague mass is felt in RIF.
because his father has been moderately Colonoscopy shows transmural
hard of hearing since middle age.
inflammation and granulomata. What is
the most appropriate management?
1305. A 45yo male with epigastric
a. Sulfasalazine
discomfort has been given triple therapy.
b. Paracetamol
He has now returned after 4wks of
c. Metronidazole
d. Ibuprofen
epigastric discomfort. What inv would you
do for him?
Ans. The key is C. Metronidazole. It is a wrong a. ECG
key. Correct key should be A. Sulfasalazine. [Drug b. H.pylori breath test
of 1st preference is Steroids, then c. Endoscopy and biopsy
immunosuppressants, then amynosalicylates d. US
(such as sulfasalazine). Metronidazole is used if
there is infective complications like infected
perianal fistula Ref: patient.info].
Ans. The key is C. Endoscopy and biopsy. This is a
wrong key! Correct key is B. H. pylori breath test.

1303. A 62yo prv shipyard worker


complains of breathlessness and chest
pain for 6m. He has now developed a
large pleural effusion. Which is the single
best diagnostic inv?
a. ABG
b. Bronchoscopy
c. CXR
d. Pleural biopsy 1306. A 13yo boy with umbilical pain for
e. Transfer factor the last 12h presents with anorexia,
nausea and has not passed a bowel
motion 24h. What is your dx?
Ans. The key is D. Pleural biopsy. [Shipyard a. Acute appendicitis
worker is exposed to asbestos and the likely b. IBD
diagnosis here is mesothelioma for which single c. IBS
best investigation is pleural biopsy]. d. Meckel’s diverticulum
e. Muscle strain
f. Ovarian cysts
g. PID
1304. A 67yo man presents with a hx of h. Psoas hematoma
increasing confusion and drowsiness. He i. Pyelonephritis
had a fall 2wk ago. CT head reveals a j. Uretric calculus
chronic subdural hematoma. What is the
best management for this pt? Ans. The key is D. Meckel’s diverticulum.
a. Craniotomy
b. Burr hole drainage
c. Conservative management 1707. A 4yo baby has a generalized tonic-
d. Excision and biopsy
clonic convulsions and fever of 39C. His
Ans. The key is B. Burr hole drainage. mother informs you that this has
happened 3-4 times ebfore. What is the
most probable dx?
1310. A 10yo boy with lower abdominal
a. Febrile convulsion
pain for the last 10d presents with a hx of
b. Absence seizures
passing 6-8 loose stools. Temp=38.8C. He
c. Epilepsy
d. Partial complex seizure
is tender in the right lower quadrant and
has an anal fistula. Choose the single most
Ans. The key is A. Febrile convulsion. [Febrile likely cause of abdominal pain.
convulsion does not usually recur. It seems to be a. IBD
partial complex seizure]. b. IBS
c. Pyelonephritis
1308. A 24yo male is admitted with acute
d. Uretric calculus
severe asthma. Tx is initiated with 100%
e. Gastroenteritis
oxygen, nebulized salbutamol and
ipratropium bromide nebulizers and IV
hydrocortisone. Despite initial tx there is
no improvement. Which is the next step in
management? Ans. The key is A. IBD. [Likely diagnosis is crohn’s
a. IV aminophylline disease which is often associated with perianal
b. IV magnesium sulphate fistula formation].
c. IV salbutamol
d. IM adrenaline
e. IV adrenaline 1311. A 28yo woman with hx of drug
addiction wants to start a family and have
a baby. She would like to stop taking
Ans. The key is B. IV magnesium sulphate. heroin and asked for something to help
her stay away from it. What drug tx would
you give her?
1309. A 49yo man first presented with a. Naloxone
increasing difficulty in swallowing. Several b. Acamprosate
months later he developed weakness in c. Methadone
his right foot. Now he can no longer feed d. Chlordiazepoxide
himself, he chokes on food and has e. Naltrexone
become confined to a wheelchair. What is Ans. The key is C. Methadone. [Methadone is a
the most likely dx? powerful synthetic analgesic drug which is similar
a. Cerebral tumor to morphine in its effects but less sedative and is
b. Myasthenia gravis used as a substitute drug in the treatment of
c. Lambert-Eaton syndrome morphine and heroin addiction].
d. Motor neuron disease
e. Cerebro-vascular disease
1312. A pt with vesicles in the maxillary
divison of trigeminal nerve. Which area of
mucus membrane will be involved?
Ans. The key is D. Motor neuron disease. [The
a. Palate
picture is of amyotrophic lateral sclerosis with
b. Cheek
bulbar onset, so it has very bad prognosis].
c. Cornea
d. Conjunctiva 1315. A 32yo man presents with 3d of
scrotal pain. Exam: thickening o the left
Ans. The key is B. Cheek. testis and it is hot to touch. What is the
most appropriate management?
a. Analgesia
1313. A 52yo man presents with visual b. Reassurance
hallucinations and features of cognitive c. Antibiotics
impairment. What is the most likely dx? d. Referral to surgeon
a. Frontotemporal dementia
b. Lewy body dementia
c. Delirium tremens Ans. The key is C. Antibiotics. [scrotal pain of 3
d. Alzheimer’s disease days with thickening of testis which is hot to
e. Huntington’s disease touch suggests epididymoorchitis, which is
treated with antibiotics].

Ans. The key is B. Lewy body dementia. [Visual


hallucinations and cognitive impairment are 1316. A 34yo woman presents with
suggestive of Lewy body dementia. Hallucination truncal obesity, easy bruising,
is prominent feature of LBD than Alzheimers. hyperglycemia, high BP and depression.
Again cognitive impairment goes more with LBD Which of the following inv’s will be most
while prominent early memory loss symptom
helpful in localizing the cause for
suggests Alzheimer’s].
Cushing’s syndrome?
a. Serum cortisol
1314. A 40yo woman who has recently b. 24h urinary cortisol
returned from working in the middle east c. Low dose dexamethasone suppression
complains of thirst, episode of loin pain, test
urinary frequency, dysuria and has passed d. High dose dexamethasone suppression
a urinary stone. All inv are normal. She test
plans to return to the Middle East in a e. Overnight dexamethasone suppression
month’s time. What is the single best test
advice to prevent recurrent stone
Ans. The key is D. High dose dexamethasone
formation? suppression test. [High dose dexamethasone
a. Drink less milk
suppression test can identify whether pituitary
b. High fibre diet adenoma or ectopic ACTH producing tumour].
c. Increase fluid intake
d. Low calcium diet
e. Low protein diet
1317. A 32yo man develops hematuria
Ans. The key is C. Increased fluid intake. [ Risk 2wks after a sore throat. What is the dx?
factors for renal stones include being overweight,
a. Post infection nephritis
certain foods, some medications, and not drinking
b. IgA nephropathy
enough fluids].
c. Membranous nephritis
d. Glomerulonephritis
Ans. The key is A. Post infection nephritis. to gout. There can be sacroilitis and onycholysis.
[Hematuria 2 wks after sorethroat indicate post Ref: patient.info].
infection nephritis while hematuria after few days
of sorethroat indicate IgA nephropathy]. 1321. Which of the following conditions
requires operative management?
a. Cellulitis
1318. An elder man who has anorexia,
b. Dyshidrosi
prostate synptoms and HTN. There are
c. Erysipelas
small kidneys on US. What is the dx?
d. Fournier’s gangrene
a. Hypertensive renal disease
e. Lymphangitis
b. Prostate ca
c. BPH Ans. The key is D. Fournier’s gangrene. [Fournier's
gangrene: A horrendous infection of the genitalia
Ans. The key is A. Hyperensive renal disease.
that causes severe pain in the genital area (in the
penis and scrotum or perineum) and progresses
1319. A 55yo woman with breast ca which from erythema (redness) to necrosis (death) of
has spread to lung, liver and bone now tissue. Gangrene can occur within hours].
presents with increasing constipation,
weakness, thirst and anorexia for the past 1322. A 55yo pt presents with collapse
3d. Her only medication is haloperidol for and complains of abdominal pain that
hiccoughs. Today she is disorientated and radiates to the back. An expansile
has left sided weakness. What is the most abdominal mass is felt on examination
likely dx? and the pt is in shock. What is the single
a. Brain mets most likely dx?
b. Hypercalcemia a. Ruptured aortic aneurysm
c. Liver failure b. Renal colic
c. Trauma
d. Endocarditis
Ans. The key is A. Brain mets. [The patient has e. Atheroma
clinical hypercalcemia, but the neurological
features of disorientation and left sided weakness Ans. The key is A. Ruptured aortic aneurysm.
can only be explained with brain metastasis].

1320. A 22yo man presents with a red, 1323. A house-bound 78yo man with
hot, swollen, metatarsal phalangeal joint, severe COPD has had a gradual
sarcoilitis and onycholysis. What is the deterioration over recent months and is
single most likely cause of his condition? now breathless at rest. He is on maximal
a. Gout inhaled medical therapy. Result: pH=7.36,
b. RA PaCO2=5.9kPa, PaO2=6.9kPa. What is the
c. Reiter’s syndrome single most appropriate additional tx?
d. Psoriatic arthropathy a. Aminophylline PO
b. ACEi PO
c. Antibiotic PO
Ans. The key is D. Psoriatic arthropathy. [Psoriatic d. Oxygen
arthritis initially can affects the toes very similar e. Steroid PO
c. Allergy
d. Filariasis
Ans. The key is D. Oxygen. [In patients with
chronic hypoxaemia, LTOT should usually be
prescribed after appropriate assessment, when
the PaO2 is consistently at or below 7.3 kPa (55 Ans. The key is A. Lymphedema. [As during
mm Hg) when breathing air during a period of mastectomy axillary clearance is done, lymphatics
clinical stability. Clinical stability is defined as the on that upper limb is compromised. So there
absence of exacerbation of chronic lung disease ocurs lymphedema].
for the previous five weeks. The level of
1326. A homeless person is found
PaCO2(which may be normal or elevated) does
not influence the need for LTOT prescription].
wandering on the street. He had ataxic
gait, nystagmus and opthalmoplegia. He
looked unkempt and his clothes had a
1324. A 79yo man has a swelling of the sweaty odour. He had a dry mucous
right groin which was clinically dx to be membrane with a BP=118/70mmHg and
indirect inguinal hernia. What is the single PR=90bpm. Blood sugar level=8. Alcohol
feature of the hearnia sac that would breath test= -ve. What would the most
confirm the dx? imp initial inv?
a. Comes through femoral ring a. IV insulin
b. Doesn’t pass through the deep inguinal b. Vit B complex
ring c. Bolus IV 0.9%NS
c. Lies below and lateral to the pubic d. IV dextrose
tubercle e. Antibiotics
d. Only passes through the superficial
inguinal ring
e. Passes through the deep inguinal ring Ans. The key is B. Vit B complex. [This is
wernicke’s encephalopathy due to alcohol
withdrawal, so Vit B complex should be checked.
This occurs mainly due to thiamine deficiency].
Ans. The key is E. Passes through the deep
inguinal ring. [Ingunal hernia lies mostly above
and medial (occasionally lateral particularly when
small) to pubic tubercle. It first enters the inguinal 1327. A 34yo man has supra-orbital pain
canal through deep inguinal ring and then enters and tenderness and developed tenderness
the scrotum through the superficial inguinal ring]. over the maxilla. He also has mild fever.
What is the single likely cause for these
symptoms?
1325. A 56yo woman with hx of breast a. Acute sinusitis
cancer 10y ago has undergone radical b. GCA
mastectomy and axillary LN removal, now c. Trigeminal neuralgia
complains of swollen upper limb 3wks d. Maxillary carcinoma
after an insect bite. The bite site is better
but gross edema is still present. What is
the cause? Ans. The key is A. Acute sinusitis. [In frontal
a. Lymphedema sinusitis there is supraorbital pain in frontal
b. Breast Ca sinusitis and tenderness over maxilla in maxillary
sinusitis. Also mild fever may present in acute 1330. A young child dx with chicken pox.
sinusitis]. Usually goes to day care. What is the most
appropriate advice?
a. Child should be admitted to hospital
1328. A 51yo woman presents with straight away
painful tongue and complains of tiredness. b. Isolate the child from parents and siblings
She is pale and has angular stomatitis and at home
c. Advice that he can go back to nursery
a smooth red tongue. There is no when the rash is crusted over
koilonychea. Choose the single cell type
you will find on the blood film.
a. Numerous blast cells Ans. The key is C. Advice that he can go back to
b. Oval macrocytes nursery when the rash is crusted over. [At this
stage patient is no more infectious].
c. Spherocytes
d. Microcytic hypochromic
e. Mexican hat cells 1331. A 7yo boy is brought by his mother.
f. Erythrocytes There are multiple perioral and forehead
vesicles. Some vesicles are crusted and
some are not. The face is hot. What is the
Ans. The key is B. oval macrocytes. [Macrocytes most likely dx?
are two types, round and oval. Oval macrocytes a. Varicella zoster
are seen in megaloblastic anemia which occurs b. Herpes zoster
due to Vit B12and/or folic acid deficiency. c. Fungal infection
Glossitis (painful tongue), angular stomatitis and d. Impetigo
smooth tongue are though characteristic feature e. Psoriasis
of iron deficiency but also known feature of
megaloblastic anemia. Absence of koilonychea
also favours megaloblastic anemia].
Ans. The key is B. Herpes zoster.

1329. A 24yo woman presents with


1332. A 5yo boy is rescued from a burning
tingling and twitching of her fingers
building and is presented to the ED. He
followed by throbbing unilateral
has 5% partial thickness burns over the
headache. What is the most likely dx?
a. Tension headache arms and legs and had soot in the mouth
b. Migraine and nose. His breathing has become noisy.
c. Cluster headache What is the single most immediate
d. TIA management?
e. SAH a. Nebulized adrenaline
b. Nebulized salmetarol and oxygen
c. Needle cricothyrodotomy
d. Oropharyngeal airway
Ans. The key is B. Migraine. [The tingling and
e. Intubation of airway
twitching of her fingers are aura and the migraine
is with aura].

Ans. The key is E. Intubation of airway.


1333. A new born baby is borught with to the baby with Hep B. What would you
pansystolic murmur at sternal border but give to the baby?
the baby is not cyanosed. What is the dx? a. Hep B Ig only
a. VSD b. Hep B full vaccine and Ig
b. ASD c. Hep B vaccine only once
c. TOF d. Nothing until immune status is checked
d. PDA e. Hep B vaccine once and Ig

Ans. The key is A. ventricular septal defect. Ans. The key is B. Hepatitis B full vaccine and Ig.

1337. A man suffers from Herpes Zoster


affecting his face. Which of the following
1334. A woman complaining of diarrhea,
mucos membrane is to be affected?
abdominal pain and fatigue. All the tests
a. Cheek
are found to be normal. What is the b. Cornea
cause? c. Conjunctiva
a. Somatization
d. Oropharynx
b. Conversion
e. Palate
c. Hypochondriasis

Ans. The key is C. Conjunctiva. [Herpes zoster


Ans. The key is A. Somatization. [Somatization
ophthalmicus can cause following lesions
disorder is patients presenting with any physical
blepheritis, conjucktivitis, keratitis, anterior
symptom and frequent medical visits in spite of
uveitis etc. As question wants mucous membrane
negative investigations].
and among the mentioned lesions only
conjunctiva is mucous membrane the answer is
conjunctiva].
1335. A 26yo man has returned from NY
to the UK and noticed weight loss, night
sweats, temp=37.5C and cervical 1338. A 34yo man sustains a fx to shaft of
lymphadenopathy. He also has femur after falling from the roof of his
splenomegaly. What is the dx? house. Exam: distal pulses are not
a. TB palpable. Which vessel is damaged?
b. Lymphoma a. Femoral artery
c. Bronchial carcinoma b. Circumflex femoral artery
d. Bronchitis c. Profundafemoris artery
d. Popliteal artery
e. Obturator artery
f. Dorsalispedis artery
Ans. The key is B. Lymphoma.

Ans. The key is A. Femoral artery.


1336. A mother got infected with Hep B
during pregnancy. Her child is born and
1339. A 9yo child doesn’t play with his
she is worried about the risk of infection
peers and has collected 200 cars. He
doesn’t respond to any criticism. What is b. Legionella
the dx? c. Mycoplasma
a. Autism d. Streptococcus
b. Personality disorder
Ans. The key is C. Mycoplasma. [Rash, bilateral
c. Schizophrenia
consolidation favours mycoplasma pneumonia.
d. Rett syndrome
Mycoplasma is common in military barrack,
e. Social anxiety
prison or boarding dwellers].

Ans. The key is A. Autism.


1343. A 10yo boy is brought to the ED 10h
after injury to the foot. It was punctured
1340. A 63 yo man with vague but
with a metal spike that passed through his
persistent pain. On endoscopy: columnar
shoe. What is the next best step?
epithelium was found to be pouched into a. Ig
muscularis. What is the dx? b. Ig and vaccine
a. Adenocarcinoma
c. Vaccine only
b. Adenoma
d. Clean the wound
c. Peptic ulcer
e. Antibiotics
d. H. pylori infection

Ans. The key is A. Adenocarcinoma. Ans. The key is D. Clean the wond. [“NEXT” best
step is clean the wound].
1341. A 24yo man after a head injury
presents with difficulty dressing himself, 1344. A 56yo male presents with
difficulty in writing and inability to persistent watery diarrhea. What is the
differentiate the fingers of his hand. most likely dx?
Which part of the brain is most likely to be
a. Treponema pallidum
affected?
a. Frontal lobe b. Nesseria meningitides
b. Parietal lobe c. Cryptosporidium
c. Temporal lobe d. Staph aureus
d. Occipital lobe e. Pseudomonas aeruginosa
e. Brainstem

Ans. The key is C. Cryptosporidium.


Ans. The key is B. Parietal lobe. [Dressing apraxia, 1345. A 2yo girl has frequency, urgency
agraphia and finger agnosia are features of and burning micturition. She has some
parietal lobe lesions].
supra pubic tenderness. Which one of the
following is the most appropriate initial
inv?
1342. A 16yo boy in boarding school feels a. Supra pubic aspiration of urine for C&S
unwell. He developed cough and rash. His b. Clean catch of urine for C&S
CXR showed bilateral consolidations. c. USG
What is the cause of his symptoms? d. IVU
a. Staph aureus
e. MCUG lesion. What is her most common primary
ca?
a. Lung
Ans. The key is B. Clean catch of urine for C&S. b. Breast
[The features are consistent with lower UTI for c. Uterine
which clean catch of urine for c&s should be d. Brain
done].
Ans. The key is B. Breast. [In female breast and
lung cancer and in male prostate and lung cancer
are most common to metastasize to bone].
1346. An 89yo man presents with
carcinoma of posterior oropharynx. Which
is the single most appropriate LN 1349. A 6m infant has breast milk
involved? jaundice. He is otherwise feeding well and
a. Pre-aortic LN is not dehydrated. What would his LFTs
b. Aortic LN look like?
c. Submental LN a. Total bilirubin:40, conjugated
d. Submandibular LN bilirubin<5%
e. Deep cervical LN b. Total bilirubin:300, conjugated bilirubin
85%
c. Total bilirubin:500, conjugated
Ans. The key is E. Deep cervical LN. bilirubin>85%
d. Total bilirubin:400, conjugated
bilirubin<85%
1347. A young boy presented to the OPD
12wks after renal transplantation with
fever and pain in lower abdomen. Renal Ans. The key is A. Total bilirubin:40, conjugated
functions were deranged. Renal biopsy bilirubin<5%. [Brest milk jaundice is characterized
showed immune cell infiltrate and tubular by indirect hyperbilirubinemia].
damage. What is the most probable dx?
a. Pyelonephritis
b. Chronic graft rejection 1350. A 29yo man took a tour of Japan
c. Acute rejection and also travelled to other parts of Asia,
d. Drug toxicity developed fever, petechie and rash on his
e. Graft vs host disease body. He didn’t take malaria prophylaxis
prior to travel. What is the most likely dx?
Ans. The key is C. Acute rejection. [Acute a. Malaria
rejection usually occurs in the first few months b. HSP
after transplant, but may occur up to a year after
c. HIV
transplant. Features presented are typical of
d. Dengue fever
acute graft rejection].
e. ITP

1348. A 56yo lady presents with a


Ans. The key is D. Dengue fever.
pathological fx of T11 vertebra. There is
found to be an underlying metastatic
1351. A 4yo boy ingested his b. RF
grandmother’s medicine and has c. Clotting factors
developed dilated pupil. What is the
cause?
a. Amitryptiline Ans. The key is C. Clotting factors.
b. Paracetamol
c. Iron
d. Digoxin 1355. A 66yo man has renal colic. He has
also presented with acute onset pain in his
knee in the past. What is the single most
Ans. The key is A. Amitryptiline. [TCA causes likely cause for renal failure?
mydriasis]. a. SLE associated GN
b. Hypercalcemia
c. HTN
1352. A 46yo male presents with
d. Hyperuricemia
confusion and drowsiness. What is the
e. Hyperoxaluria
most likely dx?
a. Cryptococcus neoformans
b. Toxoplasma gondii
c. HSV Ans. The key is D. hyperuricemia. [Knee pain is
due to gout and renal colic may occur from uric
d. CMV
acid stone (urate stone)].
e. Candida albicans

Ans. The key is A. Cryptococcus neoformans. [The 1356. A boy with a hx of recurrent swollen
key is doubtful as Toxoplasmosis is most common tender joints on both knees and elbows
cns lesion in AIDS]. and not able to participate in sports. What
is the inv of choice to dx?
a. RF/ASO titre
1353. A child has developed rash after the b. Clotting factor
tx of penicillin. What will be the cause of c. ESR
rash?
a. Drug reaction
Ans. The key is B. Clotting factor. [Haemarthrosis
b. Kawasaki
causing disability].
c. Inf Mono
1357. A 26yo man is referred for
gastroscopy because of a hx of several
Ans. The key is A. Drug reaction. months of dyspepsia. He has routine
bloods checked and is found to have a
serum calcium level=3.2mmol/l with a
1354. A child comes with recurrent joint venous bicarbonate level of 33mmol/l.
pain, multiple bruises, swollen ankle and Renal and LFT are both mornal. CXR is
unable to move his legs. What is the inv of normal. What is the most likely cause of
choice? his hypercalcemia?
a. ESR a. Melanoma
b. Metastatic malignancy Ans. The key is E. Tracheal intubation. [If a burnt
c. Milk alkali syndrome patient gets soots in mouth and /or nose tracheal
d. Primary hyperparathyroidism intubation is done].
e. Sarcoidosis

1360. A 40yo man complains of severe


Ans. The key is C. Milk alkali syndrome. [In colicky loin pain that radiates to his
medicine, milk-alkali syndrome is characterized scrotum. He is noted to have microscopic
by high blood calcium caused by taking in too hematuria. No masses are palpated. What
much calcium and absorbable alkali; common is the single most likely cause?
sources of calcium and alkali are dietary a. Acute cystitis
supplements taken to prevent osteoporosis and b. Bladder ca
antacids. If untreated, milk-alkali syndrome may c. Renal vein thrombosis
lead to kidney failure or death]. d. Acute pyelonephritis
e. Ureteric calculus

1358. A 3yo boy presents with difficulty in


walking and skin lesions. What is the most Ans. The key is E. Ureteric calculus. [In ureteric
stone there is radiation of pain from loin to groin
likely causative agent?
a. Strep pyogenes or scrotum].
b. Rubella virus
c. Parvovirus 1361. A 55yo man is having slow growing
d. Papovirus ascites. When we tap the peritoneal fluid
e. Paramyxovirus the protein is <25 and it is clear and
yellow. What could be the origin for
ascites?
Ans. The key is C. Parvovirus. [Sometimes there a. Budd-Chiari
may occur arthropathy for which children gets b. Gastrinoma
difficulty to walk. There occurs macular c. Hepatoma
morbiliform rash in parvovirus infection following d. TB
disappearance of rash of the cheeks]. e. Pancreatitis

1359. A pt after his house fire came with Ans. The key is A. Budd-Chiari syndrome. [In
gastrinoma there is no ascites! In hepatoma there
hematemesis with erosion/ulcer of
will be exudative ascites as in TB and pancreatitis.
esophagus and on examination there is
Budd-Chiary syndrome causes transudative
55% burn and on endoscopy there is a ascites and the likely option here].
stomach/gastric erosion and soot in the
mouth. What is the tx?
a. PO PPI 1362. A 7yo boy presents with his mother
b. IV PPI to GP surgery. His mother describes he
c. PPI and antibiotic had presented this since 3wks ago. He had
d. H. pylori test not experienced any trauma. No other
e. Tracheal intubation symptoms a/w the condition. Exam: non
tender swollen ankles bilaterally. There is
no rash or lesion. He is otherwise well. onset rotational vertigo which comes on
Which single test would be the best as an with sharp movements of the head and
initial assessment? neck. Which of the following would be
a. Plasma electrolytes most helpful?
b. Albumin a. Caloric testing
c. Total serum protein b. Hallpikes maneuver
d. Anti-streptolysin c. Gutenbergers test
d. Meniere’s test
e. Otoscopy
Ans. The key is B. Albumin. [Likely case of ankle
oedema from hypoalbuminema].

Ans. The key is B. Hallpikes maneuver. [Vertigo


1363. In lyme disease, which complication which comes on with sharp movement of the
is most likely to lead to collapse? head and neck without hearing loss, nausea-
vomiting, tinnitus points towards the diagnosis of
a. Dilated CM
benign positional vertigo which can be
b. AV block demonstrated by Hallpikes maneuver].
c. Mild encephalitis
d. Meningitis
e. Myocarditis 1366. A man rescued from a building on
fire presented with unconsciousness
Ans. The key is B. AV block. without any evidence of burns or external
injury or soot. What would you do next?
a. 100% oxyen inhalation
1364. A 30yo pt came to the OPD with b. 24% oxygen by mask
complaint of breathlessness and dry c. Hyperbaric oxygen in a hyperbaric
cough. He has lost 5kgs in 2m. He is an IV chamber
drug abuser. Inv: CXR=bilateral interstitial d. Intubation
shadowing. What is the single most likely e. Refer to specialist unit
causative organism?
a. Klebsiella
b. TB Ans. The key is A. 100% oxygen inhalation.
c. Chlamydia pneumonia
d. PCP
e. Chlamydia psitacci 1367. A pt has had 1 ep of depression and
2 eps of mania over the last year and now
presents with depression. He is on anti-
Ans. The key is D. PCP. [IV drug abuser, weight depressants. What additional
loss, breathlessness and dry cough with pharmacological tx would now act as a
interstitial shadowing indicate PCP pneumonia in prophylaxis for his condition?
an AIDS patient]. a. Antidepressants
b. Antipsychotics
c. Mood stabilizers
1365. A 27yo female who had a RTA 7m d. No additions req
back now complaints of attacks of sudden
d. Heparin
e. Warfarin
Ans. The key is C. Mood stabilizers.
[Antidepressant alone can lead to manic attack Ans. The key is A. Alteplase.
and to prevent this a mood stabilizer is required].

1371. A young boy presents with fever


1368. A man presented with a purplish and cough. His father was dx with TB a
swelling at the anal area. It is acutely week ago. The parents don’t want him to
painful and he complains of constipation have a BAL under anesthesia. Which other
for the last 2m. What is the most samples can be taken for dx?
appropriate management? a. Urine
a. I&D b. Blood
b. I&D + antibiotics c. CSF
c. Reassure d. Gastric washing
d. Analgesia e. Sweat
e. Sclerotherapy

Ans. The key is D. Gastric washing.


Ans. The key is B. I&D + antibiotics.
1372. A 50yo man came to the hosp a few
1369. A pt came to the ED after he had months after he had a MI. Exam:
banged his car quite a few times on everything normal, S1 and S2 were heard
reversing. He was complaining of seeing on auscultation, but there is a new pan-
double while he tried to look back during sytolic murmur. What is the most
the process of reversing the car, he also appropriate inv of choice?
a. ECG
complains of double vision on looking at
b. 24h ECG
an outward gaze. Which nerve is
c. Echo
involved?
a. Abducent nerve d. CXR
b. Trochlear nerve e. CT
c. Oculomotor nerve
d. Optic nerve Ans. The key is C. Echo. [To rule out any valvular
e. Trigeminal nerve or septal lesion].

1373. A 73yo stroke pt has been on aspirin


Ans. The key is A. Abducent nerve.
for 2yrs. He now presents with epigastric
pain and is asking for a tx. What is the
1370. A pt had a stroke. Now, there is left most appropriate management?
sided weakness and right side facial a. Laparotomy
numbness. CT shows ischemic stroke. b. NSAIDs
Which one would you prescribe? c. Omeprazole
a. Alteplase d. Morphine
b. Aspirin e. Tramadol
c. Clopidogrel
Ans. The key is D. benign essential tremor. [In
Ans. The key is C. Omeprazole. benign essential tremor there is no rest tremor
but tremor occurs during movement of the
affected limb].
1374. A 2yo girl is brought to the ED by
her mother. The child is screaming that
there is something in her ear and she
1377. A 40yo woman with breast cancer
appears agitated. Exam: a plastic bead is
has back pain which keep her awake at
seen inside the ear. What is the best
night. She blames it on a gym session she
method of removal?
had 2wks ago. She now has difficulty in
a. Forceps
b. Hook
climbing stairs. There is tenderness over
c. Under general anesthesia
the right thoracic spine. She has
d. Syringing diminished fine touch and temp sensation
e. Magnet in her right foot. What is the single most
appropriate inv?
a. Bone density scan
b. CT head
Ans. The key is C. Under general anaesthesia.
c. MRI spine
d. Nuclear bone scan
e. XR thoracolumbar spine
1375. During antenatal visits, the
following tests are routinely offered to all
pregnant mothers apart from HIV and Hep
Ans. The key is C. MRI spine. [There is
B?
neurological deficit so to exclude spinal
a. Rubella and syphilis
involvement MRI is the single most appropriate
b. Syphilis and toxoplasmosis
investigation].
c. Hep C & thalassemia
d. CMV and rubella
e. Sickle cell anemia and Hep
1378. A pregnant lady at her 39wk GA
present with eclampsia. Soon after her
Ans. The key is A. Rubella and syphilis. arrival in the labour suit, IV MgSO4 and IV
hydralazine has been prescribed. The pt
then develops another fit in the hosp and
1376. A 32yo male complains of tremors maintenance dose of MgSO4 has been
everytime he tends to use his muscles and started. What is your next step in
when he is pointing at objects. No management?
complaints at rest. His father complained a. Mg SO4 bolus
of similar problems. What is the most b. Delivery of baby
probable dx? c. MgSO4 loading dose
a. Parkinsonism d. Diazepam
b. Lithium toxicity
c. Thyrotoxicosis
d. Benign essential tremor Ans. The key is A. MgSO4 bolus.
1379. A man suffering from Influenza A
since 5d ago. CXR: pneumonia. What Ans. The key is B. echo. [Sudden breathlessness
organism is responsible for pneumonia in and dyspnea suggest mitral regurgitation
this pt? secondary to papillary muscle rupture but on the
a. Hemophilius influenze otherhand pansystolic murmur all over the
b. Klebsiella precordium suggest VSD! However in either case
c. Staphylococcus aureus required investigation is echo].
d. Streptococcus pneumonia
e. Pseudomonas
1382. During a laparoscopic procedure, a
trochar is inserted halfway between the
Ans. The key is C. Staphylococcus aureus. [Post umbilicus and the ant superior iliac spine.
flue pneumonia is usually caused by What are the structures most likely to be
staphylococcus aureus]. pierced?
a. Rectus sheath
1380. A pt admitted due to repeated b. Linea alba
c. External oblique aponeurosis
attacks of pancreatitis presents with
d. Internal oblique and transverse
dementia and loss of proprioception in the
abdominal
legs. What is the most appropriate tx?
a. Thiamine e. Both C and D
b. Pyridoxine
c. Cobolamin
d. Lipase Ans. The key is D. Internal oblique and
e. Antibiotics transversus abdominis.

Ans. The key is C. Cobalamin. [Pancreatic 1383. A pt, a small child presented with
insufficiency can lead to malabsorption and B12 URTI and later developed fever, earache
(cobalamin) deficiency. Dementia and diminished and tympanic membrane is dull. What is
vibratory sensation and proprioception in the the likely dx?
lower extremities were the most common a. OM
objective findings of cobalamin deficiency]. b. OE
c. Glue ear
d. Perforation of the tympanic membrane
1381. A man after MI presented with
e. Referred ear ache
sudden breathlessness and dyspnea.
Exam: scattered pansystolic murmur all
over the precordium. What is the next inv
Ans. The key is A. OM.
that will lead to dx?
a. ECG
b. Echo
c. CT
1384. A 72yo male who is a regular
d. Blood culture smoker has come to the ED with
e. CXR complaints of loss of weight and loss of
appetite. He also complains of
odynophagia. Exam: actively bleeding started on flucloxacillin. What other meds
ulcer on right tonsil. What is the most will you add?
appropriate dx? a. Ciprofloxacin
a. Tonsillar ca b. Gentamicin
b. Vincent’s angina c. Metronidazole
c. Irritant ingestion d. Benzylpenicillin
d. Paracoccidiodmycosis e. Ceftriaxone
e. Herpes simplex infection

Ans. The key is D. Benzylpenicillin. [Abscess,


cellulitis, furuncle these lesions are usually caused
Ans. The key is A. Tonsillar ca. by Staphylococcus and streptococcus. So
Benzylpenicillin should be added with
flucloxacillin to cover streptococcus].
1385. A pt with regular episodes of SNHL,
vertigo and tinnitus lasting >30min.
1388. A 65yo man has incurable bronchial
Neurological exam=normal. What is the
cancer. He is unable to cough up his
likely dx?
a. Meniere’s disease
secretions. This is leading to a distressing
b. Acoustic neuroma cough. Which of the following drugs is
c. Otosclerosis most likely to help him?
a. Scopolamine
d. Benign positional vertigo
b. Xanomeline
e. Labrynthitis
c. Aceclidine
d. Pilocarpine
Ans. The key is A. Meniere’s disease. e. Cevimiline

1386. A pt with celiac disease from birth, Ans. The key is A. Scopolamine. [Any secretory
now as an adult presented with some condition of lung is reduced and improved with
abdominal symptoms. The biopsy shows scopolamine].
infiltration of the gastric epithelium by 1389. A pt presented after eating a seafood dish
lymphocytes. What is the most likely dx? at a local restaurant. He complains of difficulty in
a. Lymphoma breathing. His speech is slurred and his
b. Diverticular disease BP=85/55mmHg. What would be the most
c. Lynch syndrome appropriate next step?
d. Gastric TB a. IV adrenaline
e. Peritoneal tumor b. IM adrenaline
c. SC adrenaline
d. PO chlorpheniramine
Ans. The key is A. Lymphoma. [Coeliac disease e. IV chlorpheniramin
leads to an increased risk of both
adenocarcinoma and lymphoma of gut]. Ans. The key is B. IM adrenaline. [Patient is
getting anaphylactic shock with difficulty of
breathing and slurred speech (probably from
1387. A 55yo man presented with hot,
tongue swelling) which are indications of IM
raised, tender area of skin on his right leg. adrenaline].
He is febrile with rigors. He has been
Ans. The key is C. Hypnagogic hallucinations.
1390. A 7yo boy presents with proptosis [Hallucinations during going for sleep is
and periorbital edema. What is the hypnagogic and hallucinations during wake up
immediate action that needs to be taken? from sleep is hypnapompic hallucinations].
a. IV morphine and immediate
ophthalmoscopy
b. IV morphine 1393. A 32yo man on psychiatric meds
c. Observation only presents with coarse tremors and
diarrhea. What is the most likely alt to the
drug causing the prb?
Ans. The key is A. IV morphine and immediate a. Lithium
ophthalmoscopy. b. Diazepam
c. Haloperidol
d. Valproate
1391. A schizophrenic man complains that e. Citalopram
he can hear voices talking about him and
telling him to end his life by cutting his
throat. He only hears them when he Ans. The key is D. Valproate. [Diarrhea and coarse
wakes up from sleep and not at other tremors are well known side effects of valproate].
times. What type of hallucinations is he
having?
a. Somatic 1394. A man is brought to the ED after he
b. Kinesthetic was stabbed in the best. Chest is clear
c. Hypnogogic bilaterally with muffled heart sounds.
d. Hypnopompic BP=60/0mmHg, pulse=120bpm, JVP is
e. Lilliputian raised. What is the most probable dx?
a. Pulmonary embolism
Ans. The key is D. Hypnapompic hallucination.
b. Cardiac tamponade
[Hallucinations during going for sleep is
c. Pericardial effusion
hypnagogic and hallucinations during wake up
from sleep is hypnapompic hallucinations]. d. Hemothorax
e. Pneumothorax

1392. A 28yo woman complains of


hearing strange voices in her bedroom as Ans. The key is B. Cardiac tamponade. [Lungs
clear bilaterally excludes effusion and
she is falling asleep in the night. She says
pneumothorax. Muffled heart sound and low bp
there is no one in the room except her. On
with raised jvp points towards the diagnosis of
evaluation she has no other problems. cardiac tamponade].
What is she suffering from?
a. Delusion of persecution
b. Cotard syndrome
1395. A 64yo alcoholic who has been dx
c. Hypnogogic hallucinations
with liver cirrhosis presents with a massive
d. Lilliputian hallucinations
ascites. What is the mechanism of fluid
e. Schizophrenia
accumulation in a pt with liver disease?
a. Cirrhosis
b. Portal HTN
1398. A man feels mild discomfort in the
c. Hypoalbuminemia
d. Liver failure
anal region and purulent discharge in
e. Hepatic encephalopathy
underpants. What is the most likely dx?
a. Feacal incontinence
b. Anal abscess
c. Fistula in ano
Ans. The key is C. Hypoalbuminemia. This is a
d. Anal tags
wrong key! Correct key should be B. Portal
e. Rectal Ca
hypertension. [Portal hypertension is more
specific for ascites in liver disease while Ans. The key is C. Fistula in ano. [Abscess is with
hypoalbuminemia is cause for more generalized severe pain which may reduce after it bursts with
fluid accumulation]. purulent discharge. But here mild discomfort and
purulent discharge favours the diagnosis of
fistula].
1396. A 38yo man presented to ED with
severe pain in upper abdomen. He has
already taken course of triple therapy and 1399. A 38yo female presents with
now had elective endoscopy 2d ago. He is difficult in looking upward and on
in shock. What is the most probable dx? examination she was found to have lid lag
a. Ca esophagus as well. She also complains of her heart
b. Barret’s esophagus racing at times. Which test will help in dx?
c. Mediastinitis a. Tensilon test
d. Ca stomach b. 24h ECG
c. TFT
d. Schimmer test
Ans. The key is C. Mediastinitis. e. Young Helmholtz ophthalmoscopy

Ans. The key is C. TFT. [Difficulty in looking


upwards is due to superior rectus or inferior
1397. A 68yo man who is a known case of
oblique palsy and palsy of one or more extra-
liver cirrhosis has developed ascites. What
ocular muscle can occur in thyrotoxicosis which is
is the mechanism for the development of known as Ballett’s sign. There is lid lag in
ascites? thyrotoxicosis and racing heart may be due to
a. Portal HTN paroxysmal atrial fibrillation caused by
b. Hypoalbuminemia hyperthyroidism].
c. Congestive heart failure
d. Liver failure
1400. A young anxious mother of a 10m
boy comes to you and requests a test for
Ans. The key is A. Portal hypertension. This is CF. What is the most appropriate inv?
wrong key! Correct option should be B. a. Sweat test
Hypoalbuminemia. [Portal hypertension is more b. Heel prick test
specific for ascites in liver disease while c. Breath test
hypoalbuminemia is cause for more generalized
d. CXR
fluid accumulation].
naevi on his chest, BP=100/76mmHg,
Ans. The key is A. Sweat test. [The sweat test pulse=110bpm. He has a swollen
measures the concentration of chloride that is abdomen with shifting dullness.What is
excreted in sweat. It is used to screen for cystic your diagnosis?
fibrosis (CF). Due to defective chloride channels, a. Gastric ca
the concentration of chloride in sweat is elevated b. Mallory-weiss tear
in individuals with cystic fibrosis]. c. Esophageal ca
d. Esophageal varices
e. Esophagitis
1401. A 22yo Greek man presents with f. Peptic ulceration
rapid anemia and jaundice following tx of
Ans. The key is D. Esophgeal varices. [Spider nevi
malaria. He is noted to have Heinx bodies.
and ascites suggest advanced liver disease. Here
Choose the single most likely cause from in alcoholic middle aged man having large
the given options? hematemesis with above mentioned features are
a. G6PD deficiency very suggestive of bleeding from esophageal
b. Anemia of chronic disease varices secondary to portal hypertension in liver
c. Pernicious anemia cirrhosis].
d. IDA
e. Vit B12 deficiency
1404. A 23yo woman presents with a 1cm
small smooth, firm, mobile mass in her left
Ans. The key is A. G6PD deficiency. [G6PD
breast. She is very anxious. What is the
(glucose-6-phosphate dehydrogenase) deficiency most appropriate inv?
a. Mammography
exacerbated by administration of oxidant drugs
(e.g., primaquine, dapsone, quinidine) can also b. US breast
result in Heinz bodies]. c. FNAC
d. Mammography and US

1402. A 65yo has terminal cancer and his


pain is relieved by a fentanyl patch but he Ans. The key is B. US breast. [This is
now complains of shooting pain in his fibroadenoma of the breast (breast mouse) for
which US of breast is the appropriate
arm. Which of the following will add to his
investigation and if it reveal a fibrous lump we
pain relief?
shall go for FNAC].
a. Gabapentin
b. Radiotherapy
c. Amitryptiline 1405. A pt was admitted with abdominal
d. Morphine pain, diarrhea, pigmented palmar creases
and buccal mucosa. What is the most
Ans. The key is A. Gabapentin. [Shooting in the
probable dx?
arm indicates neuropathic pain for which both
a. Addison’s disease
amytriptiline and gabapentin can be used].
b. Cushing syndrome
c. Pheochromocytoma
d. Hyperthyroidism
1403. A 45yo male alcoholic presents after
e. Hypoparathyroidism
a large hematemesis. He has some spider
b. Glossopharyngeal nerve
c. Hypoglossal nerve
Ans. The key is A. Addison’s disease. [Abdominal
d. Optic nerve
pain, diarrhea, pigmented palmar creases and
buccal mucosa are well known features of e. Vagus nerve
addisons disease].

Ans. The key is A. Facial nerve. [Features are


1406. A 36yo pt came with diarrhea, suggestive of left sided facial nerve palsy].
bleeding, weight loss and fistula. What is
the single most likely dx?
a. Colorectal ca
1409. A 19yo female dx with trichomonas
b. Celiac disease
vaginalis. LMP was 10d ago. What is the
c. CD
best antibiotic tx?
d. UC a. Erythromycin
e. IBS b. Vancomycin
c. Metronidazole
d. Penicillin
Ans. The key is C. CD. [Bloody diarrhea can occur e. Clarithromycin
in both CD & UC. Also weight loss is common! but f. Doxycycline
g. Fluconazole
fistula formation is unique feature of CD].
h. Clotrimazole

1407. A 45yo man has undergone detox Ans. The key is C. Metronidazole.
and now wants a drug to stop him from
craving alcohol. What med would be that
drug of choice? 1410. A 35yo man has been given a dx of
a. Disulfiram allergic rhinitis and asthma. Exam:
b. Acamprosate peripheral neuropathy with tingling and
c. Thiamine numbness in a ‘glove and stocking’
d. Naloxone distribution. Skin elsions are present in the
e. Diazepam form of tender subcutaneous nodules. The
pt is responding well to corticosteroids.
What is the single most appropriate dx?
Ans. The key is B. Acamprosate. [Disulfirum is a a. AS
deterrent which does not reduce craving. b. Churg-strauss syndrome
Acramposate by restoring deranged brain c. Crytogenic organizing
chemical reduces craving]. d. Extrinsic allergic alveolitis
e. Tropical pulmonary eosinophilia

1408. A 68yo man awoke to find that he is


unable to close his left eye and dribbling Ans. The key is B. Churg-strauss syndrome. [The
saliva from the left angle of his mouth. American College of Rheumatology has identified
What is the single most appropriate six criteria for the diagnosis of CSS:[3]
option?  Asthma (wheezing, expiratory
a. Facial nerve rhonchi).
 Eosinophilia of more than 10% in Ans. The key is A. SVT. [Regular tachycardia
peripheral blood. responding to vagal maneuver is likely SVT].
 Paranasal sinusitis.
 Pulmonary infiltrates (may be
transient).
1413. A 43yo man with a hx of hospital
 Histological confirmation of
vasculitis with extravascular admissions talk about various topics,
eosinophils. moving from one loosely connected topic
 Mononeuritis multiplex or to another. What is the most likely dx?
polyneuropathy. a. Psychosis
The presence of four out of six of these features b. Mania
has a high specificity and sensitivity for the c. Schizophrenia
diagnosis of CSS]. d. Pressured speech
e. Verbal diarrhea

1411. A 28yo woman comes with sudden


onset vomiting and pain per abdomen. Ans. The key is B. Mania. [Flight of idea is
Exam: mobile swelling in the right iliac common in mania].
fossa. What is the most probable dx?
a. Ectopic pregnancy
b. Tubo-ovarian abscess 1414. An 18yo girl presents with rahs on
c. Acute appendicitis her trunk, abdominal pain, arthritis,
d. Ovarian torsion
proteinuria and hematuria. What is the
e. Diverticulitis
most probable dx?
a. TTP
b. ITP
Ans. The key is D. [As there is no history of c. HSP
amenorrhea ectopic pregnancy is less likely. So dx d. HUS
is possible ovarian torsion]. e. Measles

1412. A 68yo man on tx for an irregular Ans. The key is C. HSP. [Henoch-Schönlein
heart beat comes to the ED. He has purpura (HSP) is an acute immunoglobulin A
palpitations for the last 3h. Exam: (IgA)–mediated disorder characterized by a
pulse=regular, 154bpm. Carotid sinus generalized vasculitis involving the small vessels
massage settled his pulse down to 80bpm. of the skin, the gastrointestinal (GI) tract, the
What is the most likely rhythm kidneys, the joints, and, rarely, the lungs and the
disturbance? central nervous system (CNS). Rash on trunk,
a. SVT abdominal pain, arthritis, proteinuria and
b. V-fib hematuria are well known features of HSP].
c. VT
d. V-ectopics
e. A-fib 1415. A pt is on loop diuretics. What effect
do loop diuretics produce?
a. Low Na+, low K+
b. Low Na+, normal K+
c. Normal Na+, normal K+ 1418. A 49yo man comes with hx of cough
d. High Na+, low K+ and SOB. His CD4 count is measured as
e. High Na+, high K+ 350. CXR shows lobar consolidation. What
is the single most appropriate option?
a. Mycobacterium avium intercellular
Ans. The key is A. Low Na+, Low K+. [Loop b. CMV
diuretics causes hyponatremia and hypokalemia]. c. Streptococcus
d. Toxoplasmosis
e. Pneumocystis jerovici
1416. A 6yo girl is being investigated for
renal failure. She is found to have a
congenital abnormality of the insertion of Ans. The key is C. Streptococcus. [Features are
the ureters into the urinary bladder. What consistent with lobar pneumonia].
is the single most likely cause for renal
failure in this pt? 1419. A 32yo woman with prv hx of PID
a. SLE
now presents with severe abdominal pain.
b. PKD
Her LMP was 8wks ago. What is the most
c. Wilm’s tumor
probable dx?
d. Acute tubular necrosis a. Ectopic pregnancy
e. Reflux nephropathy b. Ovarian torsion
c. Hematometrium
d. Chronic PID
Ans. The key is E. Reflux nephropathy. [Reflux e. Cholecystitis
nephropathy is kidney damage (nephropathy)
due to urine flowing backward (reflux) from the
bladder toward the kidneys; the latter is called Ans. The key is A. Ectopic pregnancy. [PID is a risk
vesicoureteral reflux (VUR)]. factor for ectopic pregnancy].

1417. A 76yo man is in the CCU 2d after


1420. A 25yo who is 38wks pregnant
an acute MI. He tells you that he had an
presents to the labour ward with a hx of
episode of rapid pounding in the chest
fewer fetal movements than usual during
lasting for about 2mins. He remains
the evening. She also says that abdominal
conscious throughout. What is the most
contractions are coming every few
likely rhythm?
a. SVT
minutes and she is having a blood stained
b. VF show per vagina for the last few minutes.
c. VT Exam: cervix is fully affaced, 9cm dilated,
d. V-ectopics cephalic presentation and station is +1.
e. A-fib Choose the single most likely dx?
a. APH
b. Concealed hemorrhage
Ans. The key is C. VT. It is wrong key. Correct key c. Labour
should be D. V-ectopics. [Ref: OHCM] d. IUFD
e. IUGR
dose. So she should be admitted under mental
health act].
Ans. The key is C. Labour. [Abdominal
contractions coming every few minutes, having
blood stained show per vagina, fully effaced
cervix with dilatation of 9 cm, cephalic 1423. A 63yo male presents after having
presentation and station +1 suggests that the had a seizure. Exam: alert and oriented.
patient is in labour]. Exam: inattention on the left side and
hyperreflexia of the arm. What is the most
probable dx?
1421. A 30yo woman has a painless lump a. Cerebral tumor
in the outer aspect of her left breast. She b. Pituitary adenoma
has had a prv breast lump. Her c. Cerebellar abscess
grandmother had breast cancer at 70yrs. d. Huntington’s chorea
She has a 1cm smooth, firm, discrete, e. Parkinsonism
mobile lump in the other quadrant region
of the left breast.
What is the single most likely dx? Ans. The key is A. Cerebral tumour.
a. Breast abscess
b. Breast carcinoma
1424. A 70yo lady on Raloxifene for
c. Breast cyst
osteoporosis has recently to the UK from
d. Fibro-adenoma
Australia. She now presents with severe
e. Sebaceous cyst
chest pain, SOB and suddenly collapsed in
the ED. What is the single most
appropriate dx?
Ans. D. Fibroadenoma. [H/O previous brest lump a. MI
and presently with a lump of 1 cm size which is
b. Aortic dissection
smooth, firm (not hard), discrete and mobile (not
c. Pulmonary embolism
fixed) suggests fibro-adenoma].
d. Costochondritis
e. Pneumothorax

1422. A 38yo woman is in the ED


following an OD of her meds. She doesn’t
Ans. The key is C. Pulmonary embolism.
need med tx for the OD. She says she
[Prolonged air travel is a risk factor for pulmonary
wishes to be discharged. What is the embolism].
single most appropriate management?
a. Community psychiatric nurse visit
b. Psychiatric OPD review the next day
1425. A 35yo woman complains of
c. Prescribe anti-depressants
hoarseness of voice 3h after partial
d. Admission under the mental health act
thyroidectomy. She had no hx of
e. Discharge and allow to go home
phonation probs before the surgery. What
is the single most appropriate inv?
a. Laryngoscopy
Ans. The key is D. Admission under the mental
b. Bronchoscopy
health act. [There is every chance of repeat over
c. CT neck
d. CXR decreased Na+, bi-basal consolidation are well
e. Barium swallow known features of legionnaires disease].

Ans. The key is A. Laryngoscopy. [Probable 1428. A 20yo student came to the OPD
diagnosis is recurrent laryngeal nerve palsy]. with complains of headache, malaise, dry
cough, joint pain and vomiting. Exam:
temp=39C. CXR: patchy consolidation.
1426. A 40yo pt came to OPD with What is the single most likely causative
complaint of fever, pleuritic chest pain, organism?
productive cough and painful vesicles a. Pneumococcal pneumonia
around the lips. Exam: temp=38C. He has b. Mycoplasma
a hx of splenectomy last yr. What is the c. Klebsiella
single most likely causative organism? d. Streptococcus
a. Pneumococcal pneumonia e. PCP
b. Staphylococcus
c. Klebsiella
d. Streptococcus Ans. The key is B. Mycoplasma.
e. Chlamydia psitacci

1429. A 45yo man presented to his GP


Ans. The key is A. Pneumococcal pneumonia. with vague symptoms of headache,
[painful vesicles around the lips is well known proximal muscle weakness and nocturia.
association of pneumococcal pneumonia]. Test results show him to be severely HTN
(230/130mmHg) and hypokalemic. What
is the most probable dx?
1427. A 37yo male pt who recently a. Addison’s disease
returned back to UK from UAE attends the b. Conn’s disease
OPD with complaint of dry cough, c. Familial hyperaldosteronism
breathlessness and anorexia. According to d. Cushing’s disease
him he had flu like symptoms a week ago. e. Cushing’s syndrome
He is slightly confused. Inv: lymphopenia
& decreased Na+. CXR: bi-basal
consolidation. What is the single most Ans. The key is B. Conn’s disease. [High BP is often
likely causative organism? the only presentation of Conn’s syndrome. Loss of
a. Legionella K+ in urine leads to hypokalemia which in turn
b. Chlamydia pneumonia causes muscle weakness and polyuria particularly
c. PCP nocturia].
d. Viral pneumonia
e. Chlamydia psitacci 1430. A man says his insides are rotting
and nobody has buried him. Which term
best describes his condition?
Ans. The key is A. Legionella. [H/O travel (staying a. Delusion of nihilism
in AC and watersystem of hotel), lymphopenia, b. Delusion of guilt
c. Delusion of persecution reflex and reduced hearing in the left ear.
d. Incongruent affect What is the most likely dx?
e. Clang association a. Meniere’s disease
b. Acoustic neuroma
c. Cerebral abscess
Ans. The key is A. Delusion of nihilism. d. Pituitary tumor
e. Gentamicin

1431. A man with chronic cough presents


with copious purulent sputum. What is the
Ans. The key is B. Acoustic neuroma. [nystagmus,
single most dx?
loss of corneal reflex, hearing loss, balance
a. Bronchitis
problem are well known feature of acoustic
b. Bronchiectasis
neuroma].
c. COPD
d. Pneumonia
e. Emphysema
1434. A 22yo man reports a 2d hx of
hoarseness of voice. He denies any weight
loss but he has been smoking for 4yrs.
Ans. The key is B. Bronchiectasis.
What is the single most appropriate inv?
a. None
1432. A 32yo man working in a shipyard b. Laryngoscopy
comes with SOB. Exam: dullness on left c. Bronchoscopy
side of the chest, pain in left side of chest, d. BAL
pleuritic rub and crackles been heard on e. CXR
the same side. What is the single most
likely dx?
a. Pericarditis Ans. The key is B. Laryngoscopy. It is a wrong key!
b. Pleurisy Correct key is A. None. [If horseness is of >3
c. Pleural effusion weeks in man >50 yrs and smoker and heavy
d. CCF drinker to rule out cancer do CXR and\or
e. TB laryngoscopy- NICE guideline].

Ans. The key is C. Pleural effusion. This is a wrong 1435. A 34yo IVDA (intravenous drug
key. The correct key should be B. Pleurisy. [The addict) presents with a 4m hx of
likely diagnosis is mesothelioma where there is productive cough. He has lost 10kgs.
pleural thickening and dullness. As there is pleural What is the single most appropriate inv?
rub still pleurisy is present and once effusion will a. Sputum for AFB
occur the rub will be lost and absent]. b. Laryngoscopy
c. Bronchoscopy
d. CT neck
1433. A 67yo female presents with e. CXR
balance prbs. Exam: nystagmus on left
lateral gaze, a loss of the left corneal
Ans. The key is A. Sputum for AFB. [In IVDA cavitation. What is the single most likely
immunity becomes low and easily gets infected causative organism?
with TB]. a. Legionella
b. Mycoplasma
c. Staphylococcus
1436. A 25yo pt came to the OPD with d. Klebsiella
complaint of fever, malaise, e. Streptococcus
breathlessness, cough and anorexia. His gf
has got similar symptoms. He had hx of
sore throat and ear discharge a month Ans. The key is D. Klebsiella. [Upper lobe
ago. What is the single most likely cavitation favours Klebsiella pneumonia].
causative organism?
a. Legionella 1439. A 20yo man complains that all his
b. Mycoplasma movements are being watched.
c. Chlamydia pneumonia Sometimes he feels as though his actions
d. PCP are being controlled by his radio. At other
e. Chlamydia psitacci times he is aware of voices describing
what he is doing. What is the most
probable dx?
Ans. The key is C. Chlamydia pneumonia. a. Mania
b. Drug induced psychosis
c. Delusion of control
1437. A 72yo male presents with acute d. Schizophrenia
confusion. He has been in the hosp for e. Korsakoff psychosis
2wks having been treated for a DVT. The
nurses have noticed that he became
increasingly drowsy. Exam: small scalp Ans. The key is D. Schizophrenia.
laceration, a GCS of 8 and bilateral up-
going plantar response.
a. Infection toxicity 1440. A 35yo is agitated and euphoric. He
b. Delirium tremens claims to be helping the prime minister
c. Extradural hematoma with economic policy, although this is not
d. Subdural hematoma true when checked. What is the most
e. Electrolyte imbalance likely dx?
a. Mania
b. Schizophrenia
Ans. The key is D. Subdural hematoma. [Even c. Hypomania
trivial head trauma can lead to subdural d. Drug induced personality disorder
hematoma. Presence of small scalp laceration, e. Delusion of grandeur
confusion and becoming increasingly drowsy
Glasgow coma scale of 8 are suggestive of
subdural hematoma].
Ans. The key is E. Delusion of grandeur. Key is
1438. A 50yo DM pt came to the OPD wrong! Correct key should be mania. [Agitated,
euphoric and delusion of grandiosity makes the
with complaint of fever, muscle ache, dry
likely dx to be “Mania”. Agitation and euphoria
cough and anorexia. Inv: CXR=upper lobe
are not feature of delusion of grandiosity but d. Dermatitis herpetiformis
mania]. e. Hyperthyroidism
f. Primary biliary cirrhosis

1441. A 20yo student who recently visited


Asia came to the OPD with complains of
Ans. The key is F. Primary biliary cirrhosis.
low grade fever, night sweats, anorexia [Pruritus, skin pigmentation (increased amounts
and productive cough. Inv: CXR=cavitatory of melanin, widely dispersed throughout both
lesions in upper lobes. What is the single epidermis and dermis), raised ALP, and presence
most likelt causative organism? of AMA are almost diagnostic of primary biliary
a. Mycoplasma cirrhosis].
b. Klebsiella
c. TB 1444. A 60yo man complains of tiredness,
d. PCP lethargy and itching that is severe after a
hot bath. He also has nocturia, polyuria
and nausea and vomiting. Exam: pallor,
Ans. The key is C. TB. [Low grade fever, night pigmentation and generalized edema.
sweats, anorexia, dry (initially) or productive What is the single most likely dx?
cough, and upper lobe cavitary lesions are highly a. Hyperthyroidism
suggestive of TB]. b. Lichen planus
c. Lymphoma
d. Eczema
1442. A 35yo man with T1DM is e. Liver failure
dehydrated with BP=90/50mmHg. What is f. CRF
the single most appropriate initial inv?
a. ABG
b. CBC Ans. The key is F. CRF. [Given picture is typical of
c. HbA1c CRF].
d. LFT
e. BUE
1445. A 30yo man complains of vague
pain in the loin with BP=140/90mmHg. He
is found to have proteinuria and
Ans. The key is A. ABG. [The likely dx is DKA. hematuria. What is the inv to confirm the
Among the given options ABG is the most
dx?
appropriate inv. To demonstrate acidosis].
a. Abdominal US
b. ANCA
c. ANA
1443. A 45yo woman presents with d. Urine microscopy and culture
pruritis. Exam: skin pigmentation. Inv: e. Stool culture
raised ALP and presence of anti-
mitochondrial antibodies. What is the
single most likely dx? Ans. The key is A. Abdominal US. [The likely dx is
a. Psoriasis
ADPKD for which US is diagnostic investigation].
b. Scabies
c. Atopic eczema
1446. A 54yo man comes with sudden
onset of palpitations and breathlessness. Ans. The key is B. Left ant descending artery.
His HR=164bpm. What is the single most
appropriate tx in the acute phase?
a. Adenosine
1449. Which virus is transmitted by the
b. Metaprolol
fecal-oral route?
c. Verapamil a. Hep C
d. Amiodarone b. Coxsackie virus
c. Dengue
d. None of the above
Ans. The key is A. Adenosine. [Common
arrhythmias we encounter are AF, SVT, VT in
exams. Here no suggestive feature for AF and SVT
Ans. The key is B. Coxsackie virus.
is the commonest presentation as in described
case. So first we shall give adenosine to establish
the diagnosis].
1450. A 40yo woman presented with
generalized itching and tiredness for few
1447. A 29yo woman has developed an months. She gave a hx of heavy menstrual
itchy scaly rash particularly over her wrist periods. Exam: pallor. What is the single
with fine white streaks overlying the most likely causative factor?
a. IDA
lesion. Her nails have ridges and her
b. Lichen planus
buccal mucosa is lined with a lacy white
c. Dermatitis herpitiformis
pattern. What is the single most likely dx?
d. Eczema
a. Psoriasis
e. Uremia
b. Scabies
c. Urtericaria
d. Dermatitis herpetiformis Ans. The key is A. IDA. [IDA is one of the cause of
e. Hyperthyroidism pruritus. Heavy periods, pallor and tiredness
f. Lichen planus further supports the diagnosis].

1451. A 7yo child presents with lesions on


Ans. The key is F. Lichen planus. [“Lacy white
the trunk. Exam: some lesions are
pattern” is used as a diagnostic description of
weeping and others are crusted with a red
lichenplanus].
base. What is the causative organism?
a. Herpes simplex
b. Varicella zoster
1448. The artery that runs in the ant inter- c. Rubella virus
ventricular groove. What is the single d. Herpes zoster
most appropriate option?
a. Acute marginal branch
b. Left ant descending artery Ans. The key is B. Varicella zoster.
c. Coronary sinus
d. Circumflex artery
e. Right coronary artery
1452. An 87yo woman with a hx of HTN Ans. The key is B. Ewing’s sarcoma.
has acute breathlessness. She has a
RR=32bpm, widespread lung crackles, 1455. A 76yo man presents with sore
pulse=120bpm, BP=160/90mmHg and throat, local irritation by hot food,
elevated venous pressure. Her peripheral dysphagia and a sensation of a lump in his
O2 sat=85%. What is the single most throat. He has a 20y hx of smoking. What
appropriate initial management? is the single most likely dx?
a. IV antibiotics a. Nasopharyngeal ca
b. IV furosemide b. Pharyngeal ca
c. Nitrate infusion c. Sinus squamous cell ca
d. Neb. Salbutamol d. Squamous cell laryngeal ca
e. 100% oxygen e. Hypopharyngeal ca

Ans. The key is E. 100% oxygen. [Oxygen Ans. The key is B. Pharyngeal Ca.
saturation is low. So we have to give oxygen
initially].
1456. A 42yo female who is obese comes
with severe upper abdominal pain and
1453. A 25yo man presented with painless
right shoulder tip pain with a temp=37.8C.
cervical lymphadenopathy with lethargy,
She has 5 children. What is the most
night sweats and itching. What is the
probable dx?
single most likely causative factor? a. ERCP
a. Lymphoma
b. LFT
b. Polycythemia
c. Serum amylase
c. IDA
d. MRCP
d. Uremia
e. US abdomen
e. Drug induced

Ans. The key is US abdomen. [5 “f”. Fat, female,


Ans. The key is A. Lymphoma. [Cervical
fair, fourty, fertile = cholecystitis].
lymphadenopathy, lethargy, night sweats and
itching are well known features of lymphoma].
1457. A 37yo laborer comes with hx of
1454. A 25yo male presents with fever redness of left eye with foreign body
and pain in the right lower thigh of 1m sensation in the same eye. What is the
duration. Exam: lower third of his thigh is single most appropriate option?
red, hot and tender. The XR showed new a. Ciliary body
bone formation. What is the most b. Sclera
probable dx? c. Conjunctivitis
a. Osteosarcomoa d. Cornea
b. Ewing’s sarcoma e. Iris
c. Tuberculus arthritis
d. Exotosis
e. Fibrosarcoma
Ans. The key is D. Cornea.
1458. An 11yo boy came to the hospital Ans. The key is B. Game keeper thumb.
with pain after falling off his bicycle. XR= [Gamekeeper's thumb is an insufficiency of the
fx at distal radius with forward ulnar collateral ligament (UCL) of the
metacarpophalangeal (MCP) joint of the thumb].
angulations. What is the single most
probable dx?
a. Dinner fork deformity 1461. A 25yo male had an injury to the
b. Cubitus valgus knee while playing football. XR=condylar
c. Gun stock deformity fx of tibia. What is the single most
d. Garden spade deformity probable deformity?
e. Genu valgus a. Dinner fork deformity
b. Gibbus
Ans. The key is D. Garden spade deformity.
c. Cubitus valgus
d. Garden spade deformity
1459. A middle aged man with a lump in e. Genu valgus
front of his neck which moves up while
he’s swallowing. US shows a mass
Ans. The key is E. Genu valgus.
replacing the left lobe of thyroid. And
spread to the sternocleidomastoid and
adjacent muscles. What is the most 1462. A 50yo man presents with itching
probable dx? after hot shower with dizziness, chest pain
a. Thyroid ca after exercise. Exam: splenomegaly. What
b. Pharyngeal pouch is the single most likely causative factor?
c. Bronchus ca a. ALL
d. Thyroid cyst b. Lymphoma
e. Larynx ca c. Polycythemia
d. Scabies
e. Eczema
Ans. The key is A. Thyroid Ca.
Ans. The key is C. Polycythemia. [Itching after hot
1460. A 28yo male complains of severe shower; dizziness and angina due to
pain while trying to grasp any object. It hyperviscosity and splenomegaly are well known
features of polycythemia].
started since he participated in skiing and
had a fall and caught his thumb in the
matting. Exam: rupture of the ulnar 1463. A man presented with carcinoma of
collateral ligament of MCP joint of the the bladder and has been working in
thumb. What is the single most probable factories. He wants to know what dye has
deformity? caused it. What is the single most likely
a. Dinner fork deformity cause?
b. Game keeper thumb a. Aniline
c. Mallet finger b. Asbestos
d. Gun stock deformity c. Latex
e. Garden spade deformity d. Silica
Ans. The key is A. Aniline. [Aniline dye is a well a. Diazepam
known cause of bladder cancer]. b. Disulfiram
c. Fluoxetine
d. Lithium
1464. A 62yo man presents with left sided
e. Olanzapine
hearing loss and tinnitus. He also
complains of vomiting and headache.
Exam: papilledema and SNHL in the left
Ans. The key is E. Olanzapine. [Auditory
ear. What is the single most likely dx?
a. Meningioma hallucination, social withdrawal, thought block
are features of schizophrenia. So olanzapine is
b. Nasopharyngeal ca
the drug to be prescribed from the given option].
c. Acoustic neuroma
d. Pharyngeal ca
e. Meniere’s disease
1467. A middle aged Asian presents with
episodes of fever with rigors and chills for
Ans. The key is C. Acoustic neuroma. [SNHL, last 1yr. Blood film: ring form of
tinnitus, papilledema (raised intracranial plasmodium with schuffners dots in RBCs.
pressure) are suggestive of acoustic neuroma]. What is the drug to eradicate this
infection?
1465. A HIV +ve 55yo man presents with a. Doxycycline
painless lymphadenopathy, fever, night b. Mefloquine
sweats and weight loss. What is the most c. Proguanil
probable dx? d. Quinine
a. Hodgkin’s lymphoma e. Artesunate
b. NHL
Ans. The key is B. Mefloquine which is a wrong
c. ALL key! None of the given option is correct!!
d. AML [Shuffners dot indicate either vivax or ovale
e. CML infection and the hepatic cycle only can be
eradicated by primaquine. So none of the given
drugs are the option!! It is a bad recall].
Ans. The key is B. NHL. [NHL is more likely
diagnosis in AIDS or immunodeficient state].
1468. A 50yo man presents with flight of
ideas which are rambling and disinhibited.
1466. A 22yo man says that he can hear He is distractable, confused and
the voice of his deceased uncle telling him overactive. What is the most likely dx?
that he is being spied on. The pt is a. Dementia
distressed by this becoming low in mood b. Mania
and anxious and has not left the house for c. Schizophrenia
2wks. He is starting to drink increasing d. Psychosis
quantities of alcohol. He is noticed to have e. Acute confusional state
thought-block and passivity phenomena.
What is the single most suitable med to
treat his symptom?
Ans. The key is B. Mania. [Flight of ideas,
dysinhibition, distractibility, confusion and
Ans. The key is B. Pneumonia. [Purulent sputum is
overactivity are features of mania].
the clincher for pneumonia here].
1469. A pt presents with a lid lag, bulging
eyes, opthalmoplegia and thyroid bruit. 1472. A 60yo pt recovering from a surgery
What inv will you do? for toxic goiter is found to be hypotensive,
a. TFT
cyanosed in the recovery room. Exam:
b. Eye sight
neck is tense. There is oozing of blood
c. Tensilon test
from the drain. What is the most probable
d. US
dx?
e. FNAC
a. Thyroid storm
b. Reactionary hemorrhage
c. Secondary hemorrhage
Ans. The key is TFT. [TFT should be done. Why not
d. Primary hemorrhage
FNAC? Thyrotoxicosis is benign and not
e. Tracheomalacia
malignsant].

1470. A 30yo lady complaining of right ear Ans. No key is given. Correct key is B. Reactionary
deafness with decreased corneal reflex hemorrhage. [Hemorrhage within 1st 24 hours
and past pointing. Acoustic analysis shows which usually occurs due to dislodgement of clot
SNHL. What is the next most appropriate or slippage of a ligature].
inv to do?
a. CT brain
b. CT acoustic canal 1473. A 40yo woman has had varicose
c. MRI brain vein surgery, planned as a day pt. After
d. MRI acoustic canal the op, she is distressed by repeated
e. PET brain retching and vomiting. Her pain is
currently well controlled. What is the best
management strategy?
Ans. The key is MRI brain. This is wrong key! a. Tramadol
Correct key is D. MRI acoustic canal. b. Co-codamol
c. IM morphine
d. IV ondansetron
1471. A 29yo woman who returned from
e. PO ondansetron
Egypt 2wks ago now presents with
difficulty in breathing, chest pain, cough
and purulent sputum with an episode of
Ans. The key is D. IVondensatron. [As there is
blood staining. She is on COCPs. What is
repeated retching and vomiting pt. cannot keep
the most likely dx? oral medication down. So IV ondansatron].
a. Pulmonary embolism
b. Pneumonia
c. Lung abscess
1474. A pt with renal failure has serum
d. Pneumothorax
K+=7.5, raised creatinine and broad
e. Pulmonary edema
complex tachycardia. What is the most
appropriate management? Ans. The key is D. Barium meal. [Probable
a. Calcium gluconate diagnosis is Crohn’s disease. Can be
b. Sodium bicarbonate demonstrated by barium meal].
c. Dialysis
d. Furosemide
e. Sotalol 1477. An 83yo elderly woman presented
in the ED with cough, fever and sneezing.
Tx was given but she became confused
Ans. The key is A. Calcium gluconate. [Calcium and again presented with above said
gluconate don’t shift K+ to cell or reduce symptoms. What is the cause of her
potassium level but it prevents arrythmogenic condition?
action of raised K+ till definitive measure is a. Aspiration due to confusion
taken]. b. Alveolar damage due to drugs
c. Drug toxicity
d. Pneumothorax
1475. An 18yo lady in her 30th wk of
pregnancy is brought to the hospital in an
altered sensorium. She is taking slow, Ans. The key is A. Aspirin due to confusion.
shallow breaths and her breath has a
fruity smell. An ABG: pH=7.20, urine 1478. A 37yo man presents with some
ketones: +ve. What is the most probable raised lesions on the shin. He came with
dx? cough and also complains of arthralgia.
a. HONK
Exam: bilateral hilar lymphadenopathy
b. DKA
and erythema nodosum is present. What
c. HELLP syndrome
is the single most likely cause?
d. PIH
a. CD
e. GDM
b. UC
c. Sarcoidosis
d. Streptococcal infection
Ans. The key is B. DKA. [Shallow breath and fruity e. TB
smell with acidosis in ABG and positive ketone
body in urine suggests the diagnosis of DKA].
Ans. The key is B. UC. It is a wrong key! Correct
key is C. Sarcoidosis. [Cough, arthralgia, bilateral
1476. A 26yo man presented with hilar lymphadenopathy and erythema nodosum
abdomen distension and pain. His stools are suggestive of sarcoidosis].
have been mucoid and sometimes blood
stained. What is the most appropriate 1479. A young lady with cervical ectropion
inv? bleeds on touch. What is the most
a. Stool C&S
appropriate next inv?
b. Gastroscopy
c. IgG tissue transglutaminase a. Transvaginal US
d. Barium meal b. Cervical smear
e. Jejunal biopsy c. Punch biopsy
d. Serum estradiol need basis. In such case if investigation is needed
e. Colposcopy colposcopy can be done].

Ans. B. Cervical smear. Wrong key! [Screening


*1482. A 7yo boy presents with epistaxis
test cervical smear is only done in scheduled time
and not in on demand basis. So if it is scheduled
of 2h duration. The bleeding has been
now then it can be the option otherwise controlled. Inv: Plts=210, PT=13, APTT=42,
Colposcopy should be the key!! This is definitely bleeding time=normal.
an incomplete question]. Which of the following is the most likely
dx?
a. Hemophilia
1480. A 28yo man with recent onset of b. Von willebrand disease
dyspepsia after eating spicy food and c. ITP
alcohol consumption. H. pylori fecal d. Vit K deficiency
antigen was negative. He returns after 1m e. Liver disease
with similar symptoms despite being given f. Anatomical defect
omeprazole 40mg. What is the single best
initial inv?
a. Hydrogen breath test Ans. The key is F. Anatomical defect.
b. Gastroscopy
c. Barium meal
d. None 1483. A pregnant woman returns from
Sudan, now presenting with intermittent
fever, rigor and seizures. What is the dx?
Ans. The key is B. Gastroscopy. a. TB
b. Malaria
1481. A 35yo woman who usually has 4 c. Meningitis
days mid-cycle bleeding, had her period d. Lyme disease
10d ago. She has now presented with
spots of blood. Her smear was normal 6m
ago. Exam: cervical ectropion which Ans. The key is B. Malaria. [Intermittent fever is
doesn’t bleed on touch. What would you seen in malaria. In meningitis fever is not
do? intermittent].
a. Cervical smear
b. Endocervical swab
c. US guided biopsy 1484. A pt is unresponsive and cyanosed.
d. Laparotomy
e. Transvaginal US
What is the most definitive 1st step in
f. Punch biopsy management?
g. Serum estradiol a. Chest compressions
h. Colposcopy b. Check airway
c. Call 999
d. Mouth to mouth
Ans. The key is A. Cervical smear. Wrong key!
Correct key is colposcopy! [Cervical smear can e. Recovery position
only be done on scheduled time and not in on
Ans. The key is B. Check airway. 1488. A child came to the ED with severe
asthma and not responding to salbutamol
nebulizer and vomiting many times. What
1485. A man was bitten by a drug addict is the most appropriate management?
and comes to the hosp with a wound. a. Salmetrol
What inv should be undertaken? b. Montelukast
a. Hep C c. Prednisolone
b. Lyme disease d. Budesonide inhaler
c. Hep B e. Oxygen
d. Syphilis f. IV salbutamol
e. Hep A

Ans. The key is A. Salmeterol which is a wrong


Ans. The key is C. Hepatitis B. key!! The correct option is F. IV salbutamol. [In
acute attack there is no place for salmeterol. In
the given case most appropriate management is
1486. An 18yo woman says that she can’t IV salbutamol].
walk around as she is very big for that
room. What is the most likely
hallucination? 1489. A 73yo woman with skeletal and
a. Extracampine visual hallucinations brain mets from breast ca has worsening
b. Liliputian visual hallucinations low back pain and blurring of vision. She
c. Alice in wonderland syndrome has weakness of her legs, minimal knee
d. Hypnagogic hallucinations and absent ankle tendon reflexes, a
palpable bladder, a power of 2/5 at the
hip, 3/5 at the knee and ankle, and
Ans. The key is B. Lilliputian visual hallucination. tenderness over the 2nd lumbar vertebra.
There is reduced sensation in the
1487. A middle aged lady presented with
perineum. She has been started on
fever, altered sensorium, bleeding gums
dexamethasone 16mg daily.What is the
and jaundice. Labs: deranged renal
single most likely cause of her weakness?
function tests, normal PT/APTT, a. Paraneoplastic neuropathy
fragmented RBCs and low plts. What’s the b. Progression of brain tumor
most likely dx? c. PID at L2/L3
a. Cholesterol emboli
d. Spinal cord compression
b. HUS e. Steroid induced myopathy
c. TTP
d. Hepatorenal syndrome
e. Sepsis
Ans. No key is given! The likely correct option is D.
Spinal cord compression. [Brain metastasis
induced cerebral oedema can explain blurring of
Ans. The key is C. TTP. [Fever and altered vision secondary to raised intracranial pressure.
sensorium suggest the diagnosis of TTP]. Rest of the features including weakness can well
explain spinal cord compression].
d. Diuretic

1490. A 78yo woman presents with


unilateral headache and pain on chewing. Ans. The key is C. Chemotherapy. It is a wrong
ESR=70mm/hr. She is on oral steroids. key! Correct key is A. Aspiration. [Respond to
chemotherapy and life expectancy is poor in
What is the appropriate additional
mesothelioma. To improve symptoms aspiration
therapy? should be done].
a. Bisphosphonates
b. HRT 1493. An 88yo woman is a known smoker.
c. ACEi She had an attack of MI 2y back and is
d. IFN known to have peripheral vascular
e. IV steroids disease. She presents with an irreducible
herniation over the incision region of a
Ans. The key is A. Bisphosphonates. [Oral steroid surgery which she underwent in her
can lead to decrease in bone mineral density. To childhood. What is the most appropriate
overcome this Bisphosphonate can be tx?
administered]. a. Truss
b. Elective herniorrhaphy
c. Urgent herniorrhaphy
1491. A 48yo woman is admitted to the
d. Elective herniotomy
ED with a productive cough and mod
e. Reassure
fever. She often has central chest pain and
she regurgitates undigested food most of
the time but doesn’t suffer from acid Ans. The key is B. Elective hernioraphy. [Truss can
reflux. These symptoms have been present not be used as hernia is irreducible; urgent
for the last 3.5m which affects her daily herniorrhaphy in strangulation; elective
food intake. CXR: air-fluid level behind a herniotomy- in herniotomy the hernia will not
subside or recur; only reassure is not an option as
normal sized heart. What is the single
irreducibe hernia may become strangulated or
most likely dx?
may develop intestinal obstruction].
a. Pharyngeal pouch
b. Hiatus hernia
c. Bulbar palsy
d. Achalasia
1494. A 72yo woman who is taking loop
e. TB diuretics for left ventricular failure. She
now is suffering from palpitations and
muscle weakness. What is the electrolyte
Ans. The key is B. Hiatus hernia. imbalance found?
a. Na+=130mmol/L, K+=2.5mmol/L
1492. A retired ship worker has pleural b. Na+=130mmol/L, K+=5.5mmol/L
c. Na+=140mmol/L, K+=4.5mmol/L
effusion and pleural thickening on right
d. Na+=150mmol/L, K+=3.5mmol/L
side with bilateral lung shadowing. What
e. None
would you do to improve his symptoms?
a. Aspiration
b. Chest drain
c. Chemotherapy
Ans. The key is A. Na+=130mmol/L, b. Diazepam IV
K+=2.5mmol/L. [Loop diuretics causes c. IV fluid
hyponatremia and hypokalemia]. d. Hydralazine IV
e. Anti-epileptic

1495. A young woman who is a marathon


runner comes with secondary Ans. The key is A. MgSO4 IV. [Eclampsia, tx is IV
amenorrhea. Inv: normal LH, FSH and MgSO4].
estradiol, prolactin=600. What is the most
likely dx?
1498. A 23yo woman presents with
a. Hypothalamic amenorrhea
b. Pregnancy
offensive vaginal discharge. Vaginal
c. PCOS
pH=4.5. What is the most likely organism?
a. Gardenella
d. Prolactinoma
b. Trichomonas
e. Anorexia
c. Candida
d. Mycoplasma

Ans. The key is D. Prolactinoma. It is a wrong key.


Correct key is probably A. Hypothalamic
amenorrhea. [Generally in hypothalamic Ans. The key is A. Gardenella.
amenorrhea there is slight low level of LH, FSH
and Oestrogen and mild elavation of prolactin]. 1499. A 62yo man has had ano-rectal pain
aggravated by defecation for 3d. Rectal
exam: purple, tender lump at the anal
1496. A 4yo child comes with a sprain in verge. Flexible aigmoidoscopy: normal
his foot. Hx reveals that the child has had rectal mucosa and hard feces. What is the
recurrent admissions to the hosp due to best management strategy?
severe asthma. What is the most a. Anal hematoma
appropriate analgesic? b. Anal fissure
a. Diclofenac sodium c. Rectal ca
b. Ibuprofen d. Diverticulitis
c. Paracetamol e. Angiodysplasia
d. Codeine

Ans. The key is A. Anal hematoma.


Ans. The key is C. Paracetamol.
1500. A 43yo presents with severe vertigo
on moving sidewards whilst sleeping.
1497. A 34yo pregnant woman, 38wk GA What test would you do to confirm the
is in labor. She had a long 1st stage and dx?
troublesome 2nd stage, has delivered a a. Hallpikes maneovure
baby. After her placenta was delivered she b. Romberg’s test
had a convulsion. What is the most c. Trendelenburg test
probable management? d. Heel-shin test
a. MgSO4 IV
e. Na+=130, K+=1.5

Ans. The key is A. Hallpike maneovure. [Hallpike


maneovure is the preferred method to detect
benign positional vertigo]. Ans. The key is A. Na+=130, K+=6.5. [In Addison’s
disease there is low Na+, low Cl-, low CO2 and
1501. A 23yo man is having difficulty in raised K+].
speaking following a stab wound to the
right of his neck. On being asked to
1504. A 10yo boy develops nasal bleeding.
protude his tongue, the tip deviated to the
What is the best way to stop the bleeding
right. Which anatomical site is most likely
from the nose?
to be affected? a. Pressure over base of the nose
a. Facial nerve
b. Ice packs
b. Hypoglossal nerve
c. Pressure over the soft tissues
c. Vagus nerve
d. Nasal packing
d. Trigeminal nerve
e. Surgery
e. Glossopharyngeal nerve

Ans. The key is C. Pressure over the soft tissues.


Ans. The key is B. Hypoglossal nerve. [In
1505. A pt came to the hosp with a
hypoglossal nerve palsy the tongue will be
complaint of severe chest pain lasting for
curved toward the damaged side, combined with
the presence of fasciculations or atrophy]. >1h. Following ECG test, pt revealed to
have ST depression. He was already on
aspirin.
1502. A girl presents with signs of What is the most specific tx for this pt?
hyperventilation. What is the most likely a. GTN
ABG derangement? b. Simvastatin
a. pH increased, PCO2 increased c. Clopidogrel
b. pH decreased, PCO2 increased d. BB
c. pH increased, PCO2 decreased e. LMWH
d. pH decreased, PCO2 decreased

Ans. The key is B. Simvastatin. It is a wrong key.


Ans. The key is A. pH increased, PCO2 increased. Correct key should be LMWH. [Specific treatment
This is a wrong key. Correct key should be C. pH means treatment particularly adapted to the
increased, PCO2 decreased. special disease being treated. LMWH is vital to
prevent further events or deterioration and
1503. A pt presents with skin seems to be the most specific drug in the given
pigmentation, diarrhea, vomiting, scenario].
abdominal pain and postural hypotension.
What electrolyte abnormality is likely to
occur? 1506. A 69yo woman presents with a
a. Na+=130, K+=6.5 sudden onset of weakness of her right arm
b. Na+=130, K+=2.5 and leg. She is known to be hypertensive.
c. Na+=13, K+=6.0
There has been no headache, LOC, visual,
d. Na+=140, K+=8
speech or sensory symptoms. Exam: H. Pylori infection which can be demonstrated by
BP=180/90mmHg, pulse=100 and regular C13 urea breath test. Serum antibody persist
heart sounds, no carotid bruit. Higher (IgG) for long and not reliable for recheck].
mental function tests are normal. No
apraxia or neglect. Speech, swallowing
and sensation are normal. There are no 1508. A girl with sickle cell anemia has
visual field defects. There is a mild facial painful bleeding and vaso-occlusive crisis
weakness sparing the forehead. The right during her periods. What is the best
arm and leg are flaccid and weak. possible management for this pt?
a. COCP
Reflexes and tone are normal. There is a
b. Tranexamic acid
right extensor plantar response.
c. Copper IUS
What is the most likely cause of this pt’s
d. UAE
symptoms?
e. Depot provera
a. Cardioembolic stroke
b. Lacunar stroke
c. Right internal carotid artery
atheroembolic stroke Ans. The key is E. Depot provera. [Hormone and
barrier methods are all acceptable choices but
d. Right internal carotid artery dissection
intrauterine devices are not recommended, as
e. Right vertebral artery atheroembolic
they may be associated with uterine bleeding and
stroke
infection.
Ans. The key is B. Lacunar stroke. [Weakness of Depot contraceptive (Depo-Provera®) is safe and
right arm and leg. So lesion is on left side. Hence has been found to improve the blood picture and
C,D and E can not be the option. Features are reduce pain crises].
very much consistent with lacunar infarct].

1509. A 70yo pt comes with swelling in


1507. A 34yo man has an intermittent the parotid region for the last 10y. Exam:
epigastric pain for 3wks. It is worse by gland is soft and cystic. Choose the most
food but helped by some tablets he probable dx?
obtained from the pharmacy. He had a a. Pleomorphic adenoma
similar episode 3yrs ago and his doctor b. Carcinoma of the salivary glands
gave him a course of 3 types of tablets at c. Mikulicz’s disease
the time. d. Adenoid cystic carcinoma
What is the most appropriate next inv? e. Parotid duct stones
a. Abdomen US
b. Barium meal
c. Serum H.Pylori antibodies Ans. The key is D. Adenoid cystic carcinoma.
d. C13 urea breath test
e. Upper GI endoscopy
1510. A 74yo man has been admitted
unconscious with no hx. He has a GCS=6
Ans. The key is D. C13 urea breath test. [Patient and a dilated left puil which becomes
was allright for 3yrs after eradication therapy.
Now symptoms again may indicate recurrence of
insensitive to light. What is the single Ans. The key is C. Huntington’s disease. [A person
most likely dx? with Huntington's disease may appear to have a
a. Extradural hematoma lack of drive, initiative and concentration.
Involuntary jerking or writhing movements
b. Meningitis
(chorea). Typical presentation is between 35-55
c. Opioid OD yrs].
d. Pontine hemorrhage
e. SAH 1513. A 54yo man has collapsed suddenly
following a headache. He has
Ans. The key is E. SAH. [Extradural hematoma is
very rare in elderly and there occurs lucid
hypertension and takes warfarin for
interval; Fteatures are not consistent with prosthetic heart valve. GCS=4 and dilated
meningitis. Opioid and pontine hemorrhage left pupil.
causes myosis. So likely dx here is SAH]. What is the single most likely dx?
a. Ant circulation stroke
b. Post circulation stroke
1511. A 27yo man presents to the ED with c. Intracerebral hemorrhage
2d hx of severe headache and pyrexia d. Intracerebellar hemorrhage
(38.9C). CT: petechial hemorrhage in the e. Pontine hemorrhage.
temporal and inf frontal lobes. What is the
most likely dx? Ans. The key is C. Intracerebral hemorrhage.
a. Brain abscess [Headache, collapse, and warfarine use makes the
b. Meningococcal meningitis dx of intracerebral hemorrhage most likely.
c. Cerebral malaria “Pointers to bleeding (unreliable!): Meningism,
d. Herpes simplex encephalitis severe headache, and coma within hours.”
e. New variant CID OHCM].

1514. A 5wk breast fed baby whose birth


Ans. The key is D. Herpes simplex encephalitis. weight was 3.5kg and is now 4.5kg is
[Petechial hemorrhage in the temporal and thriving well but is deeply jaundiced. What
inferior frontal lobes are characteristic of Herpes
is the most likely dx?
simplex encephalitis].
a. Galactosemia
b. Breast milk jaundice
1512. A 44yo woman with memory loss,
c. Thalassemia
poor concentration and inability to
d. Sickle cell disease
recognize household projects. She has
e. Congenital storage disorder
right-handed involuntary writhing
movement. There is strong fam hx of
similar complain. What is the single most Ans. The key is B. Breast milk jaundice. [Breast
likely dx? milk jaundice is a type of neonatal jaundice
a. Pic’s dementia associated with breastfeeding. It is characterized
b. Wilson’s disease by indirect hyperbilirubinemia, presents in the
c. Huntington’s disease first or second week of life, and can persist for as
d. HIV associated dementia long as 12 weeks before spontaneous resolution.
e. Fronto-temporal dementia There is normal thrive and weight gain. In
galactosemia there will be lethargy, vomiting,
diarrhea and failure to thrive].
1515. A 71yo man with no prv immediate Ans. The key is A. COCP.
hx is brought to the ED by his wife who
1518. A pt of tuberculous abscess with the
says he has become progressively more
hx of prv abscess drainage presented with
forgetful, tends to lose his temper and is
fever and tenderness between L2/L3
emotionally labile. There is no hx of
vertebra. Which is the best inv for this pt?
infectious disease or trauma. What’s the a. XR
single most likely dx? b. CT
a. Pick’s dementia
c. US
b. Fronto-temporal dementia
d. MRI
c. Huntington’s disease
e. Blood culture
d. Alzheimer’s disease
e. Vascular dementia

Ans. The key is D. MRI.

Ans. The key is D. Alzheimer’s disease.


1519. A 4yo child presents with repeated
chest infections. He has yellow
1516. A 38yo woman with hemophilia discoloration of sclera and the mother
who received several blood transfusions a gives a hx of diarrhea as well. What is the
few years ago presents with irritability single inv most likely to lead to a dx?
and increasing memory deficit. She is a. Sweat chloride test
unable to speak properly. He is on anti-TB b. Anti-endomysial antiboides
tx. What is the single most likely dx? c. LFT
a. Creutzfeldt Jacob disease d. Jejunal biopsy
b. Drug toxicity e. TFT
c. Vascular dementia
d. HIV associated dementia
e. Space occupying lesion Ans. The key is A. Sweat chloride test. [Probable
cystic fibrosis, for which Sweat chloride test can
be done].
Ans. The key is D. HIV associated dementia.
[Blood transfision is the clue for HIV transmission. 1520. An 82yo woman has been admitted
Immunodeiciency is also responsible for TB]. from a nursing home with dense
hemiplegia and homonymous
hemianopia. She is dysphasic. What vessel
1517. An 18yo girl has menorrhagia and is most likely to be involved?
dysmenorrhea and requires contraception. a. Ant cerebral artery
What drug will you give her? b. Mid cerebral artery
a. COCP c. Post cerebral artery
b. Mirena coil d. Internal carotid artery
c. Copper T e. Post inf cerebellar artery
d. UAE
e. Depo provera
Ans. The key is B. Mid cerebral artery. b. Calcium gluconate IV
[Hemiplegia, homonymous hemianopia, c. Insulin subcutaneously
dysphasia these are common features of mid d. Furosemide IV
cerebral artery stroke].
Ans. The key is B. Calcium gluconate IV. [Calcium
gluconate does not shift potassium into cells or
reduce its level but prevents its arrythmogenic
1521. A pt is dx with SIADH. Choose the effect on heart and buys time till definitive
appropriate biochemical change. measures are taken].
a. Plasma Na+ decrease and urine
osmolarity increase
b. Plasma Na+ decrease and urine 1524. A 25yo man attended in urological
osmolarity decrease OPD has single testis. He was inv and
c. Plasma Na+ increase and urine osmolarity other testis was located in the abdomen.
decrease What is the best management plan for
d. Plasma Na+ increase and urine osmolarity this pt?
increase a. Short trial of HCG
b. Orchidectomy
c. Orchidopexy
Ans. The key is A. Plasma Na+ decrease and urine d. Reassurance
osmolarity increase. e. IV testosterone

1522. A newborn that is electively


intubated at birth and is due for surgery Ans. The key is C. Orchidectomy. [Ectopic testis is
48h after birth. The condition was prone to develop testicular cancer and therefore
suspected on antenatal US on CXR. What it should be surgically removed].
is the most likely dx?
a. CF
b. Congenital diaphragmatic hernia
*1525. A 56yo male who presented with
c. Congenital cystic adenomatoid
epilepsy like symptoms has been dx with
malformation an intracranial space occupying lesion. He
d. RDS now complains of thirst and mild
e. Alpha 1 antitrypsin deficiency dehydration. His blood glucose is also
increased. What is the single most
appropriate immediate tx?
a. Insulin
Ans. The key is B. Congenital diaphragmatic
b. IV fluids
hernia.
c. Stop dexamethasone
d. Stop sodium valproate and change to
1523. A 63yo male undergoes abdominal another anti-epileptic
surgery. On Monday morning, 3d post-op,
repeat samples confirm serum
K+=7.1mmol/l. His ECG shows broad QRS Ans. The key is B. IV fluids.
complexes. Which one of the following
can be used as an effective tx for this pt’s 1526. A mother brings her newborn to the
hyperkalemia? hosp concerned about a blue patch on the
a. Calcium chloride IV buttocks. The newborn is of mixed race
and was delivered normally. What is the Ans. The key is B. Atrial flutter. [Sawtoothlike
most appropriate management? waves and regular R-R interval are diagnostic of
a. Reassurance atrial flutter].
b. CBC
c. XR 1529. A man brings his wife into the ED
d. Plt count after finding her unconscious at home. He
says at breakfast time she had
complained of sudden severe headache.
Ans. The key is A. Reassurance. [The diagnosis is What is the most appropriate inv?
mongolian blue or spot. Mongolian spot refers to a. MRI
a macular blue-gray pigmentation usually on the
b. XR
sacral area of healthy infants. Mongolian spot is
usually present at birth or appears within the first c. CT brain
weeks of life. Mongolian spot typically disappears d. Carotid Doppler
spontaneously within 4 years but can persist for
life. It is a benign condition requiring no
intervention]. Ans. The key is C. CT brain. [Probable SAH. Among
the given option most appropriate is CT brain].

1527. The ECG of a 65yo shows absent P


waves, narrow QRS complex, ventricular 1530. A 68yo lady with T2DM. Which drug
rate of 120bpm and irregular R-R interval. should be prescribed?
What is the most probable dx? a. Biguanides
a. A-fib b. Sulphonyl urea
b. A-flutter c. Insulin
c. SVT d. Lifestyle modifications
d. Mobitz type 1 2nd degree heart block
e. Sinus tachycardia
Ans. The key is A. Biguanide. [This is an
incomplete question and BMI is needed to decide
Ans. The key ia A. Atrial fibrillation. [In A- whether biguanide or sulphonylurea be
fibrillation fibrillatory f waves replaces p waves prescribed].
and R – R interval are irregular].

1531. In a laparoscopic mesh repair for


1528. The ECG of an 80yo pt of IHD shows hernia, when the trochar is inserted at
sawtooth like waves, QRS complex of midpoint between umbilicus and ischial
80ms, ventricular rate of 150bpm and spine. What structure will be pierced?
regular R-R interval. What is the most a. Linea alba
probable dx? b. Rectus muscle
a. A-fib c. Conjoint tendon
b. A-flutter d. External and internal oblique muscles
c. SVT e. Inguinal ligament
d. Mobitz type 1 2nd degree heart block
e. Sinus tachycardia
Ans. The key is D. External and internal oblique
muscles.
e. Glycogen storage disease
*1532. A 48yo man has intermittent left
sided lower abdominal pain and feels
generally unwell. He has lost his appetite Ans. The key is D. Amyloidosis [Amyloidosis is a
and has lost weight. Temp=38.3C and he common cause of organomegaly].
has BP=190/100mmHg. What is the single
inv most likely to lead to dx”?
a. Colonoscopy 1535. A 75yo man has urinary symptoms
b. Endomysial antibodies of hesitancy, frequency and nocturia.
c. Fasting serum glucose conc Rectal exam: large hard prostate. What is
d. TFT the most appropriate inv?
e. US abdomen a. CA 125
b. CA 153
c. CA 199
Ans. The key is E. US abdomen. d. CEA
e. PSA

1533. A man with DM comes to the ED


after he collapsed at home. His GCS=10.
Ans. The key is E. PSA. [Urinary symptoms and
What should be the next initial inv for this
hard prostate on PR suggests prostatic cancer
man? forwhich PSA should be done].
a. Capillary blood sugar
b. MRI head
c. CT head
1536. A child suffering from CF developed
d. Serum electrolytes
pneumonia. Which organism is
responsible for this pneumonia?
Ans. The key is A. Capillary blood sugar. [It may be a. H. influenza
either hypoglycemic or hyperglycemic coma b. Klebsiella
which can be detected by checking capillary blood c. S. aureus
sugar]. d. S. pneumonia
e. Pseudomonas
1534. A 60yo DM pt presented with easy
fatigability, weakness and numbness of
hands and swollen feet. Exam: pedal Ans. The key is E. Pseudomonas.
edema, sensory neuropathy and palpable
liver and spleen. Urine: proteinuria. US
abdomen: enlarged kidney. Renal biopsy: 1537. An obese woman with hx of
amorphous homogenous substance that migraine presented with heavy bleeding
stained red with congo-red. What is the during menstruation which is painful and
dx? needs contraception too. What is the best
a. DM retinopathy possible management for this pt?
b. Sarcoidosis a. COCP
c. Wilms tumor b. Mirena coil
d. Amyloidosis c. Copper T
d. UAE
e. Depo provera c. Achalasia cardia
d. Esophageal rupture
e. Esophageal ca 1
Ans. The key is B. Mirena coil.

1538. A 2yo fell on outstretched hand on Ans. The key is A. Foreign body. [Sudden onset of
playground. He presents with pain on dysphagia to both liquid and solid and recent
base of the thumb. XR=no fx. What is the history of fitting denture suggests foreign body
single most likely dx? (denture) in oesophagus].
a. Colles fx
b. Head of radius
c. Mellet finger 1541. A 62yo man with chronic
d. Scaphoid fx schizophrenia presents with a mask like
e. No fx face and involuntary pill rolling movement
in both hands. He complains of chronic
cough and forgetfulness. He is on long
Ans. The key is D. Scaphoid fx. [Scaphoid fracture term antipsychotic meds. What is the
is often missed in initial x-ray].
single most likely dx?
a. Shy drager syndrome
1539. A pt was admitted with increased b. Parkinsonism
frequency of passing urine, increased c. Huntington’s chorea
thirst, weakness and muscle cramps. d. Tardive dyskinesia
What is the most probable dx? e. Akathisia
a. Conn’s syndrome
b. Cushing’s syndrome
c. Pheochromocytoma Ans. The key is B. Parkinsonism. [Antypsychotic
d. Hyperthyroidism drugs can cause parkinsonism].
e. Hypoparathyroidism

1542. A 34yo female presented with


Ans. The key is A. Conn’s syndrome. [Increased vomiting preceded by an occipital
frequency of passing urine and increased thirst headache of acute onset. Exam: conscious
are from nephrogenic DI resulted from and alert with photophobia but no neck
hypokalemia and hypokalemia also causes stiffness. CT: normal. What is the most
weakness and muscle cramps]. appropriate further management?
a. CT brain with contrast
1540. A 69yo male presented with sudden b. Repeat CT brain in 24h
onset of dysphagia. He is neither able to c. CSF exam
d. Cerebral angio
swallow liquid nor solid, he recently had a
e. MRI brain
denture fitting. What is the most probable
dx?
a. Foreign body
b. Plummer vinson syndrome Ans. The key is C. CSF exam. [Probable
subarachnoid hemorrhage. CT may not show any
abnormaly in early stage in som instances and in 1545. A 19yo man with known hx of OM
that case we shall do CSF examination 12 hours presents with headache, lethargy,
later to see xanthochromia]. sweating and shivering. What is the single
most likely dx?
a. Furuncle
1543. A lady with post ileo-colectomy b. Meningitis
closure of stoma has a small 4cm swelling c. Myringitis
around the stoma. What is the most d. Nasopharyngeal tumor
appropriate management of the swelling? e. OM
a. Local exploration of swelling
b. Exploratory laparotomy
c. Open laparotomy and re-closure Ans. The key is B. Meningitis.
d. Abdominal binder
e. Truss
f. Laparotomy with mesh repair 1546. A 46yo woman has gained weight.
She has sensitivity to cold. Her pulse =
regular at 50bpm and heart=enlarged.
Ans. The key is A. Local exploration of swelling. What is the single most likely underlying
mechanism for this condition?
a. Autoimmune
1544. A 64yo woman has been brought by b. Degenerative
her son for psychiatric evaluation. She c. Congenital
says that she has stopped living with her d. Infective
husband because she is convinced it is e. Nutritional
someone else posing to be him. What kind
of delusions is she suffering from?
a. Delusion of reference Ans. The key is E. Nutritional. [The diagnosis is
b. Delusion of control hypothyroidism (in hypothyroidism heart is often
c. Delusion of guilt gets enlarged) which may occur from iodine
d. Delusion of persecution deficiency (nutritional)].
e. Delusion of doubles

1547. A 70yo man presents with a


Ans. The key is B. Delusion of control. This is a punched out ulcer between his toes. He is
wrong key! The correct key is E. Delusion of a heavy drinker and smoker. Exam: ulcer is
doubles. [Delusion of control is a false belief that yellow and the foot turns red when
another person, group of people, or external dangling off the bed. What is the single
force controls one's general thoughts, feelings,
most likely dx?
impulses, or behavior. The delusion of doubles, is
a. Arterial ischemia ulcer
a rare disorder in which a person holds a
b. Malignancy
delusional belief that different people are in fact
c. Neuropathic ulcer
a single person who changes appearance or is in
d. Pressure ulcer
disguise].
e. Venous stasis ulcer
Ans. The key is A. Arterial ischemic ulcer.

1550. A 76yo woman presents with deep


1548. A 65yo woman complains of a stroke 6h ago. What would the immediate
painful discharging ulcer above her ankle tx be?
on the inner side of her left lower leg. a. Aspirin 75mg
Exam: the base of the ulcer is red and b. Aspirin 300mg
covered by a yellow fibrous tissue. The c. Streptokinase
border is irregular. The skin is tight. What d. IV heparin
is the single most likely dx? e. Dipyridamole 200mg
a. Arterial ischemia ulcer
b. Malignancy
c. Neuropathic ulcer Ans. The key is B. Aspirin 300mg.
d. Pressure ulcer
e. Venous stasis ulcer
1551. A 19yo man accuses his friend of
making his right arm swing out at a
Ans. The key is E. Venous stasis ulcer. [Some stranger. What is the best term to
people with venous insufficiency develop stasis describe his condition?
dermatitis. Blood pools in the veins of the lower a. Control
leg. Fluid and blood cells leak out of the veins into b. Persecution
the skin. It is usually seen above ankle and inner c. Guilt
side of leg]. d. Reference
e. Grandeur

1549. A 55yo woman suffered from an


acute MI 5d ago. While she was in the Ans. The key is A. Control. [Delusion of control is a
hosp the pt developed features of false belief that another person, group of people,
pulmonary edema and heart failure. What or external force controls one's general thoughts,
is the most probable cause of her present feelings, impulses, or behavior].
condition?
a. VSD 1552. A 26yo man with hx of hereditary
b. Ruptured papillary muscle hemorrhagic telengectasia is planning to
c. Pericarditis start a family. What is the mode of
d. A-fib inheritance?
e. Re-infarction a. AD with incomplete penetrance
b. Autosomal co-dominant
c. AR with incomplete penetrance
Ans. The key is B. Ruptured papillary muscle. d. AD
[Post MI rupture of papillary muscl is responsible e. AR
for valve failure (mitral regurgitation) causing
pulmonary oedema and heart failure].
Ans. The key is D. AD [Hereditary hemorrhagic
telangiectasia is of autosomal dominant mode of
inheritance].
d. Antifungals
1553. A 50yo man with a known hx of e. I&D
stroke is unable to get out of his house
because he can’t find where the door is.
He refuses help from his wife and says he Ans. The key is D. Antifungal. [Features are
is not blind. What is the single most likely consistent with oesophageal candidiasis which is
defect? treated with Oral fluconazole (200-400 mg daily)
a. Paracentral scotoma or IV fluconazole for 14-21 days].
b. Tunnel vision
c. Total blindness 1556. A 43yo woman has suffered with
d. Central scotoma
heavy periods for many years and has
e. Cortical blindness
tried many medical tx without success.
She is constantly flooding and at times
Ans. The key is E. Cortical blindness. [Cortical can’t leave her house due to heavy
blindness is the total or partial loss of vision in a bleeding. She has completed her family of
normal-appearing eye caused by damage to the 5 children and her last blood test showed
brain's occipital cortex]. Hgb=8.9g/dl. She feels that she can’t cope
with the bleeding anymore and her
husband is asking for a tx that can
1554. An elderly lady presents with guarantee success. What is the most
confusion. She is apyrexial but complains appropriate management to improve
of dysuria for 2d duration. What is the def menorrhagia in this pt?
dx inv? a. Endometrial ablation
a. Blood culture b. Hysterectomy
b. Urine nitrates c. Hysteroscopic/Laser resection of fibroids
c. CT head d. Myomectomy
d. ECG e. UAE
e. IVU

Ans. The key is B. Hysterectomy. [As family is


complete hysterectomy is the best option to stop
Ans. The key is B. Urine nitrates. [Dysurea is
bleeding and it also helps not to get endometrial
suggestive of uti which is further supported by
carcinoma].
urine nitrates. UTI also can lead to confusion].
1557. A man on antipsychotic meds
1555. A 40yo woman on chemotherapy develops features of retinitis pigmentosa.
for metastatic breast carcinoma now Which drug is most likely to cause these
presents with painful swallowing. Exam: symptoms?
a. Thioridazine
she has white plaques on top of friable
b. Haloperidol
mucosa in her mouth and more seen on
c. Chlorpromazine
esophagoscopy. What is the most
d. Risperidone
effective tx for this pt?
a. Antispasmodic
b. H2 blocker
c. Antibiotics
Ans. The key is A. Thioridazine. [Thioridazine and a. Amoxicillin
other antipsychotics (neuroleptics, dopaminergic b. Tetracyclin
antagonists) can cause degenerative c. Erythromycin
retinopathies with histological,
d. Clarithromycin
electrophysiological and symptomatological
e. Penicillin
features similar to those of primary retinitis
pigmentosa].
Ans. The key is A. Amoxicillin. This is a wrong key!
1558. Pt with low Hgb, MCV=76, angular Correct key is D. Clarithromycin. [OHCM, 9 th
stomatitis, red tongue, and koilonichea. edition].
What is the most probable dx?
a. Folate def
b. B12 def
1561. A 50yo man complains of dysphagia
c. Iron def
after eating bread. Barium swallow
d. Vit E def
reveals a lower esophageal ring. What is
e. Hemolytic anemia
the most appropriate tx?
a. Reassurance
b. Antispasmodics
Ans. The key is C. Iron deficiency anemia. [Low c. Dilatation of the LES
MCV, angular stomatitis, red tongue and
d. Endoscopic diverticulectomy
koilonichea are characteristic of iron deficiency
anemia]. e. I&D

1559. A pt with sudden severe eye pain,


red eye, visual blurring, acuity of only Ans. The key is C. Dilatation of the LES.
finger counting, nausea, vomiting with a
shallow ant chamber that is hazy on
shining a torch. What is the dx? 1562. A 48yo nulliparous woman feels
a. CRVO tired all the time. Her periods are regular
b. Acute closed angle glaucoma but have always lasted for at least 10d.
c. Uveitis Choose the single most appropriate intial
d. Iritis inv?
e. Open angle glaucoma a. High vaginal swab
b. Serum Hgb conc
c. TFT
Ans. The key is E. Open angle glaucoma. This is a d. None
wrong key! The correct key is B. Acute closed e. Abdominal US
angle glaucoma. [Sudden severe eye pain, red
eye, visual blurring, acuity of only finger counting,
nausea, vomiting, shallow anterior chamber that
Ans. The key is B. Serum Hb conc. [Feeling tired all
is hazy on shining torch are all classic
the time and prolonged period suggest anemia.
presentation of acute closed angle glaucoma].
So serum Hb should be done initially].
*1560. A pt who works in a pet shop has
temp=37.5C, dyspnea, chest pain and
cough. CXR: patchy consolidation. What is 1563. A man got his hand caught in
the most suitable tx? machinery at work. The fingers are
swollen but the XR shows no fx. What is
the most appropriate management? Ans. The key is B. C ANCA. [ARF and recurrent
a. Splint epistaxis and hemoptysis suggests Wegener’s
granulomatosis for which C ANCA is most
b. Put in plaster
specific].
c. Broad arm sling for 1wk
d. Elevate in high sling for 2d
e. Neighbor strapping 1566. A woman is admitted to the hosp
for elective abdominal hysterectomy. 2m
ago she was dx with DVT and pulmonary
Ans. The key is D. Elevate in high sling for 2d. embolism and was started on warfarin.
[Extravasetion of blood caused this swelling along What is the most appropriate preop
with inflammation for which best option is measure you will take on this occasion?
elevate in high sling for 2d which will help a. Continue warfarin
improve symptom by gravitational shift of fluid b. Stop warfarin
from hand]. c. Stop warfarin and start heparin
d. Increase warfarin dose e. Add heparin

1564. A 39yo woman presents with


symptoms recurring annually Ans. The key is C. Stop warfarin and start heparin.
characterized by depressed mood, being
socially withdrawn hypersomnia, lack of 1567. This condition affects middle aged
enjoyment in life, last for several months. women more than men and is
What is the most likely dx? characterized by low mood, early morning
a. Seasonal Affective Disorder waking, loss of libido, tiredness and
b. Mod depression suicidal intention last for at least 2wks.
c. Dysthymia What is the most probable dx?
d. GAD a. Bipolar affective disorder
e. Bipolar disorder b. Dysthymia
c. Major depressive disorder
Ans. The key is A. Seasonal Affective Disorder. d. Schizoaffective disorder
[Depression associated with late autumn and e. Recurrent brief depression
winter and thought to be caused by a lack of
light].
Ans. The key is C. Major depressive disorder.
[Given case is severe depression (major
1565. A 75yo man presents with ARF. He depressive disorder).
has been troubled by recurrent epistaxis
Mild depression: i)Low mood ii) Anhedonia iii)
but over the last 3wks he reports to have Guilt iv) Hopelessness v) Worthlessness vi)
coughed up blood too. What is the single Inability to concentrate. Tx CBT
most likely positive antibody? Moderate depression: Features of mild + vii) Poor
a. P ANCA sleep viii) Poor Appetite ix) Poor libido x) Easy
b. C ANCA fatiguability. Tx Antidepressants
c. Anti Ro Severe depression: Features of moderate + xi)
Suicidal intensions. Tx ECT
d. Anti DS DNA
e. Anti centromere
Psychotic depression: Features of severe + xii) solids and liquids over 18m but denies
Hallucinations xiii) Delusions xiv) Guilt xv) weight loss. Chest is clear. What is the
Nihilistic delusion. Tx ECT].
most likely dx?
a. Achalasia
1568. A 10yo boy has fallen from a tree b. Pharyngeal carcinoma
and injured his right chest. He has pain c. Esophageal spasm
and difficulty breathing. He is tachypenic d. Esophageal stricture
and tender with an area of paradoxical
chest wall movement on the right side. Ans. The key is C. Esophageal spasm. This is a
What is the single most likely dx? wrong key. Correct key should be A. Achalasia.
a. Diaphragmatic rupture [Dyspagia for both solids and liquids or dysphagia
b. Flail chest to mostly liquids are features of Achalasia.
c. Fx ribs Increasing dysphagia is characteristic of achalasia
d. Hemothorax (Esophagial spasm does not cause progressive but
e. Tension pneumothorax intermittent dysphagia].

Ans. The key is B. Flial chest. [In flial chest 1571. A 70yo man presents with a
multiple adjacent ribs are broken in multiple fluctuant swelling of the scrotum which
places, separating a segment, so a part of the feels like worms when he is standing but
chest wall moves independently. Pain, difficulty
regresses when he lies down. What is the
breathing, tachypnea and tender area of chest
wall movement suggests flial chest]. most probable dx?
a. Varicocele
1569. A 37yo woman had an elective LSCS b. Hematocele
1d ago. You are called to see her as she c. Testicular ca
becomes SOB with left sided chest pain d. Epidydimal cyst
and a cough. She has had 3 children, 2 e. Saphena varix
born by LSCS. Exam: she has reduced air
entry at left lung base. Her observations
include sat=92% on air, BP=105/84mmHg, Ans. The key is A. Varicocele. [Fluctuant swelling
of the scrotum which feels like worms are
pulse=120bpm, temp=37.2C. Choose
suggestive of varicocele].
among the options which C-section
complications has she developed?
a. Aspiration pneumonia
b. Aspiration pneumonitis
1572. A 52yo woman has had a swelling in
c. Spontaneous pneumothorax
the neck, hoarseness and stridor-both
d. Pulmonary embolism inspiratory and expiratory for 2m. What is
e. DVT the most probable dx?
a. Ca larynx
b. Ca thyroid
c. Vocal chord nodules
Ans. The key is D. Pulmonary embolism.
d. Ca bronchus
e. Thyrotoxicosis
1570. A pt presents with increasing
retrosternal pain and dysphagia for both
Ans. The key is B. Ca thyroid.  Bruit over 1 or both subclavian arteries or
the abdominal aorta
 Arteriographic narrowing or occlusion of
the entire aorta, its primary branches, or
1573. A woman became acutely SOB in
large arteries in the upper or lower
the recovery bay and is coughing after GA. extremities that is not due to
Auscultation: reduced air entry at the arteriosclerosis, fibromuscular dysplasia,
right lung base and diffuse wheeze. or other causes].
Observation: HR=88bpm,
BP=112/76mmHg, temp=37.8C and 1575. A 35yo woman presents with mass
sat=91% in air. Choose among the options
in the groin. Exam: mass found just below
which C-section complication has she and lateral to the pubic tubercle. There is
developed?
no cough impulse and it is irreducible.
a. Aspiration pneumonitis
What is the most probable dx?
b. Spontaneous pneumothorax
a. Direct inguinal hernia
c. Endometritis
b. Strangulated hernia
d. Pulmonary embolism
c. Femoral hernia
e. Tension pneumothorax
d. Saphenavarix
e. Femoral aneurysm

Ans. The key is A. Aspiration pneumonitis.

Ans. The key is C. Femoral hernia. [Mass below


and lateral to the pubic tubercle is suggestive of
1574. A 23yo female presents with femoral hernia].
paresthesias and loss of distal pulses in
her arms. She is noted to be hypertensive. 1576. A 30yo woman has injured her left
She describes feeling unwell a month prior lower chest in a RTA. She has
with fever and night sweats. What is the BP=80/50mmHg, pulse=120bpm.
most probable dx? Auscultation of chest=bowel sounds
a. Kawasaki disease present. What is the single most likely dx?
b. Takayasu arteritis a. Diaphragmatic rupture
c. Buerger’s disease b. Flail chest
d. Embolism c. Fx ribs
e. Raynaud’s phenomenon d. Ruptured esophagus
e. Tension pneumothorax
Ans. The key is B. Takayasu arteritis [Takayasu
arteritis is a granulomatous inflammation of the
aorta and its major branches. Criteria:
Ans. The key is A. Diaphragmatic rupture. [Bowel
 Age of 40 years or younger at disease sound present on auscultation of chest following
onset RTA causing lower chest injury is suggestive of
 Claudication of the extremities diaphragmatic rupture].
 Decreased pulsation of 1 or both brachial
arteries 1577. A lady presents with a swelling
 Difference of at least 10 mm Hg in systolic below the groin crease that can be
blood pressure between arms reduced. There is no med hx of note. What
is the most probable dx?
a. Inguinal hernia herniorrhaphy. Herniotomy = only resection of
b. Strangulated hernia sac; Herniorrhaphy = resection of sac + repair of
c. Testicular tumor posterior wall of inguinal canal; Hernioplasty =
d. Epidydimal cyst resection of sac + posterior wall repair + Mesh
e. Femoral hernia reinforcement].

Ans. The key is E. Femoral hernia. 1580. A 25yo man present with a mass in
the groin after heavy lifting. Exam: mass is
found just above and medial to the pubic
1578. A 32yo woman of 38wks GA attends tubercle. It is reducible. On applying
the antenatal day unit with pain in the pressure on the internal ring, cough
suprapubic area that radiates to the upper impulse is still present. What is the most
thighs and perineum. It is worse on likely dx?
a. Direct inguinal hernia
walking. Her urine dipstick showed a trace
b. Indirect inguinal hernia
of protein but no white cells, nitrates or
c. Femoral hernia
blood. What’s the most likely dx?
a. Braxton hicks contractions d. Strangulated hernia
b. Round ligament stretching e. Femoral aneurysm
c. Symphasis pubis dysfunction
d. Labor
e. Complicated femoral hernia Ans. The key is A. Direct inguinalhernia. [On
occlusion of deep inguinal ring if cough impulse
still palpable (actually more appropriate is visible)
on medial to occluded ring it is direct inguinal
Ans. The key is C. Symphysis pubis dysfunction.
hernia].

{Cough impulse negative means after occluding


1579. A 45yo mechanic presents with a deep ring there will be no visible cough impulse
reducible swelling in the groin, impulse on and positive means there will be visible cough
impulse. Negative cough impulse=indirect hernia;
coughing is present. He has mild dragging
positive cough impulse=direct inguinal hernia]
pain in the abdomen, otherwise he’s
hope it is clear to you now}.
normal. What is the best management
strategy?
a. Truss
1581. A 35yo woman presents with a
b. Elective herniorrhaphy
swelling in the neck. The swelling has
c. Urgent herniorrhaphy
increased in size gradually over the last
d. Elective herniotomy
e. Reassure
two years and the patient feels she has
difficulty with breathing. Exam: mass
measures 8cm by 10 cm, soft and not
warm to touch. It moves with deglutition.
Ans. The key is D. Elective herniotomy. This is
wrong key! Correct key is B. [Elective Which is the most appropriate
herniorrhaphy. [elective herniotomy is done in management of this mass?
young children preferably at 3-6 months of age. a. Partial thyroidectomy
Presenting case should be treated by elective b. Oral thyroxine
c. Oral propylthiouracil Ans. The key is B. Peroneal. [Inability of dorsiflex
d. Excision biopsy and foot eversion with sensory loss over front
and outer half of leg and dorsum of foot are seen
in peroneal nerve injury].
Ans. The key is A. Partial thyroidectomy.
[Gradually increased swelling in the neck which
moves with deglutition is thyroid enlargement 1584. A 46yo woman presents with
and as it is causing pressure symptom like sudden episode of abdominal pain which
difficulty in breathing a partial thyroidectomy started about 2h ago. The pain is located
should be performed]. in the epigastrium and radiates to her
back. She has vomited twice since the
onset of attack. The pain is made worse by
*1582. A 46yo laborer reports swelling in lying flat on her back and she is more
the right groin. The non-painful swelling is comfortable sitting up and bending
observable in both the erect and the forwards. She was informed of the
recumbent positions. Exam: non-tender presence of gallstones in her gall bladder
irreducible 4 cm mass in the right groin four weeks earlier when she reported pain
below and on the medial side of the in the right hypochondrium. The oral
inguinal ligament. Which is the most likely temp=39C, BP=120/80mmHg and the
dx in this pt? radial pulse=118/min. There is no jaundice
a. Indirect inguinal hernia but there is marked tenderness in the
b. Femoral hernia epigastrium both on deep and superficial
c. Saphenous vein varicocoele palpations. Which is the most appropriate
d. Hydrocoele
inv for the cause of the patient’s pain?
a. Plain abdominal X-ray
Ans. The key is C. Saphenous vein varicocele.
b. Serum Amylase
c. Serum bilirubin
d. Barium Swallow
1583. A camel rider sustained a kick to the
lateral side of his right leg just below the
knee caused by the camel stick. The site is
Ans. The key is B. Serum amylase. [Epigastric pain
slightly bruised and tender to touch.
radiating to back, worse on lying flat and comfort
During physical examination, he is unable
on bending forward are classic presentation of
to either dorsiflex or evert the foot. There acute pancreatitis in which serum amylase is
is loss of sensation over the front and increased].
outer half of the leg and dorsum of the
foot. If these observations are the result of
damage to a nerve bundle, which is the 1585. A 75yo Japanese woman reports
most likely nerve affected? repeated episodes of vomiting of
a. Lateral popliteal
undigested food mixed with blood. She
b. Peroneal
has lost 5 kgs in weight over the last one
c. Tibia
month. Clinical exam: shows a frail
d. Sural
woman with mild conjuctival palor. Exam:
non-tender slightly mobile mass in the
epigastric region. Which is the most likely Ans. The key is D. Uppergastrointestinal
dx? endoscopy. [The likely diagnosis is bleeding
a. Colon cancer oesophageal varices which should be diagnosed
b. Gastric cancer by endoscopy and if needed stappling can be
c. Gall bladder cancer done with endoscope].
d. Oesophageal cancer

1587. A 42yo woman reports to the


surgeon that she is worried about a lump
that she feels the right breast. The
surgeon observes a 2 cm by 3 cm mass in
Ans. The key is B. Gastric cancer. [Non-tender
the right lower quadrant of the breast.
mass in epigastrium, conjunctival pallor (anemia),
There are no associated skin changes and
weight loss, vomiting of undigested food mixed
with blood due to pyloric obstruction by cancer the mass has limited mobility. There is no
mass and particularly Japanese (highest incidence discharge from the nipple. There is no
of gastric cancer due to taking smoked fish) are axillary lymph node enlargement.
almost diagnostic of gastric cancer]. Examination of the left breast and axilla
was completely normal. A mammogram
report suggests the presence of
microcalcifications. Which is the most
1586. A 45yo man, known to be appropriate next step in the management
chronically addicted to alcohol, presents in of this pt?
the ED and reports two episodes of a. Observation for one year and repeat the
vomiting fresh bright red blood in the mammography
previous 6h. He estimated the volume b. A needle-guided biopsy of the breast
blood vomited at each bout to be more c. Excision biopsy of the breast
than 500mls. Clinical exam: the radial d. Partial mastectomy
pulse=120/min, BP=90/60mmHg. There is
no mass or tenderness in the epigastrium.
The liver is palpable for 3 cm below the Ans. The key is B. A needle-guided biopsy of the
costal margin and not tender. The patient breast.
is not jaundiced. The physician
resuscitates the patient with oxygen by
face mask, rapid infusion of intravenous
normal saline while he requests for 1588. A 45yo man presents with a mass
haemoglobin level and whole blood for on the right side of the face. The mass was
transfusion. Which is next appropriate first observed three months ago but has
step in management? recently become visibly larger. He feels
a. Barium Swallow pain over the mass and is unable to blow
b. Exploratory laparotomy a whistle. Clinical examination shows that
c. CT scan of the abdomen the mass is likely to be the parotid gland.
d. Upper gastrointestinal endoscopy An oral examination shows a foul smelling
discharge from the duct of the gland and
gentle probing shows that it is stenosed at
the meatus. Which of the following c. Fluid resuscitation and antibiotics IV
features suggests that the mass might be d. Anteroposterior & lateral neck x-ray
malignant?
a. Presence of pain
b. Recent enlargement
c. Facial nerve palsy
Ans. The key is A. Intubation under general
d. Stenosed duct meatus
anesthesia. [Acute epiglottitis. Should intubate to
save from closure of airway].

Ans. The key is C. Facial nerve palsy. [Due to 1591. A 6yo boy has been noticed to have
malignant infiltration]. problems with co-ordinating his voluntary
movements over the last two years. He
has a waddling gait and needs to support
1589. A 6yo boy presents with jaundice himself on his hands when rising from the
following treatment with sulphathiazole. floor. He has larger calves than other boys
Investigations suggest that the jaundice is but he runs more slowly. Which is the
due to haemolysis caused by G6DP most likely dx?
deficiency. Which is true regarding a. Myotonia
etiology of G6DP deficiency? b. Myasthenia gravis
a. Inherited as autosomal dominant c. Duchenne muscular dystrophy
condition d. Muscular atrophy
b. Inherited as sex-linked dominant
condition
c. Inherited as sex-linked recessive Ans. The key is C. Duchenne muscular dystrophy.
condition [The child having difficulty with walking, running,
d. Results from auto-antibodies to red cell jumping and climbing stairs. Walking may look
antigens different with a 'waddling' type of walk. The boy
may be late in starting to walk (although many
children without DMD also walk late).

When you pick the child up, you may feel as if he


'slips through your hands', due to looseness of
Ans. The key is C. Inherited as sex-linked recessive the muscles around the shoulder.
condition. The calf muscles may look bulky, although they
are not strong.

As he gets older, the child may use his hands to


1590. A 5yo previously healthy child has a
help him get up, looking as if he is 'climbing up his
1-day history of severe pain in the throat, legs'. This is called 'Gower's sign'].
breathing difficulties and fever. On
examination you find an anxious, septic-
looking child with drooling of saliva and 1592. A previously healthy, 10m female
stridor. Which is the most appropriate child presents to your clinic with a 1-day
initial management? history of high fever, runny nose and
a. Intubation under general anaesthesia conjunctivitis. The child looks unwell and
b. Insertion of nasogastric tube is irritable. Exam: child's oropharynx
shows that it is inflammed and there are sediment. Which is the most likely cause
small white spots on the oral mucosa. of this complication?
Which is the most likely dx? a. Diabetic nephropathy
a. Kawasaki disease b. Malignant hypertension
b. Parvovirus infection c. Acute tubular necrosis
c. Herpes zoster d. Interstitial nephritis
d. Measles

Ans. The key is C. Acute tubular necrosis.


Ans. The key is D. Measles. [Koplick’s spots are [Hypotension even for some minutes or few
characteristic of measles]. hours can readily lead to acute tubular necrosis
which is evident here by uremia and further
supported by brown granular cast in the urine
1593. A 3d term, breast-fed infant is sediment].
brought by the mother who reports that
the child has not been active and not
feeding well. She also notices jaundice, 1995. A 78yo pt is diagnosed with
which was not present at birth and is metastatic lung cancer; there is no cure
increasing. Exam: the temp=35.4°C, and for his condition. His son tells the
the liver is palpable 2 cm below the costal physician that in the case of a diagnosis of
margin. Which is the most likely dx? cancer, the physician must not tell his
a. Rhesus isoimmunisation father. He wishes that his father does not
b. Inadequate breast milk suffer any psychological distress caused by
c. Congenital biliary tract obstruction. the knowledge of a terminal diagnosis.
d. Sepsis Which one of the following ethical
principles supports the son’s request?
a. Patient autonomy
Ans. The key is D. Sepsis. [Not active, not feeding b. Beneficence
well, increasing new onset jaundice and c. Justice
hypothermia are suggestive of neonatal sepsis]. d. Non-maleficence

1594. A 65yo woman with DM, HTN and


normal kidney function underwent a total Ans. The key is D. Non-maleficence.
right hip replacement. She had massive [Nonmaleficence means non-harming or inflicting
haemorrhage during the operation and the least harm possible to reach a beneficial
was given 8 units of packed RBC. The outcome. Harm and its effects are considerations
blood pressure dropped to 60/40 mm Hg and part of the ethical decision-making process in
for about two hours before it was the NICU. Short-term and long-term harm,
corrected with blood transfusion. Two though unintentional, often accompany life-
days after the surgery the serum saving treatment in the NICU.
creatinine level rose to 4.2 mg/dl (normal
<1.5 mg/dl), BUN was 50 mg/dl (normal
10-20 mg/dl) and potassium 5.0 mmol/L 1596. A 23yo single male was brought to
(normal 3.5-5.0 mmol/l). There were Emergency exhausted and frightened. His
brown granular casts in the urine father tells you that his son, who was
previously healthy, had, for no apparent Which of the following is the most likely
reason, a sudden attack of fear, dizziness, dx?
sweating, palpitations and the feeling that a. Inevitable abortion
his heart is going to stop beating. The b. Threatened Abortion
symptoms started to decrease gradually c. Incomplete abortion
after about 10 minutes. Which is the most d. Missed Abortion
likely dx?
a. Panic attack
b. Delirious state Ans. The key is A. Inevitable abortion. [when os is
c. Alcohol withdrawal phenomena closed threatened and when os is opened
d. Social phobia inevitable abortion (No tissue has been
expelled)].

Ans. The key is A. Panic attack. [a sudden feeling


of acute and disabling anxiety; often fear of
1599. A 46yo woman comes for a routine
death].
gynaecological visit. On pelvic
examination, a 1-cm red, granular lesion
is noted on the posterior cervical lip,
1597. A 30yo woman, G2P1, at 37 weeks
which is firm and bleeds on contact.
gestation mentions that her 3-year-old
Which is the next best step for
son has just developed chickenpox. She is
establishing a dx?
not certain whether she has had the a. Cervical cytological smear
disease herself. Which is the next step in b. Punch biopsy
management? c. Transvaginal ultrasound
a. Administration of varicella-zoster
d. Colposcopy
immune globulin IM
b. Measurement of varicella IgM level
c. Acyclovir tablets orally
Ans. The key is B. Punch biopsy.
d. Measurement of varicella IgG level

Ans. The key is D. Measurement of varicella IgG 1600. A 31yo woman, G5P4, who has
level. [If previous infection is doubtful do varicella amenorrhoea for 12 weeks and a positive
IgG level]. pregnancy test presents to the ED with
vaginal bleeding. Symphysial-fundal
height measurement corresponds to 22
1598. A 24yo primigravida presents to the weeks gestation. Ultrasound examination
ED with a history of 8-week amenorrhoea reveals bilateral cystic masses. No fetal
followed by heavy vaginal bleeding and parts are seen during the examination.
severe, crampy abdominal pain. Exam: The cervix is closed. Which is the most
HR=110/min and BP=120/80mmHg. The likely dx?
uterus is bulky. The cervix is dilated and a. Tubal pregnancy
there is active bleeding from the cervical b. Endometriosis
os, but no tissue has been expelled. c. Hydatidiform mole
d. Threatened abortion
Risk factors:

Ans. The key is C. Hydatidiform mole. [In molar  Fertility treatments and intrauterine
pregnancy uterus is more enlarged than contraceptive devices (IUCDs) are the
gestational age and on US no fetal part but cystic most important associated risk factors.
masses are seen].  Pelvic inflammatory disease may cause
complete tubal occlusion or delay the
transport of the embryo so that
implantation occurs in the tube.
The final set (1601-1700) is done by DR. ARIF
Adhesions from infection and
SIDDIQUI. Thanks for Dr. Siddiqui for his kind inflammation from endometriosis may
contribution! play a part.
 Ectopic pregnancy has been reported in
tubes that have been divided in a
1601. A married 25yo woman presents sterilisation operation and where they
with 6h hx of abdominal pain located in have been reconstructed to reverse one.
 Ectopic pregnancy can occur in the
the LIF. The pain is persistent, of
treatment of infertility.
increasing intensity and not radiating first  Right-sided tubal pregnancy is more
experienced while she was lying down. common than left-sided. This is thought
She feels giddy when she tries to stand to be from spread of infection from
erect. The last menstrual period was 6 appendicitis.
weeks ago. The radial pulse=130/min and  Presentation:
 Symptoms and signs of ectopic pregnancy
BP=80/40 mmHg. Pelvic US shows free
can resemble those of other more
intra-peritoneal fluid. What is the most common conditions, including urinary
appropriate next step in management? tract infections and gastrointestinal
a. Immediate laparoscopy. conditions.
b. Immediate laparotomy. The most common symptoms are:
c. Pregnancy test (urine or serum).
* Abdominal pain.
d. Observation for 24 hours in the ICU * Pelvic pain.
* Amenorrhoea or missed period.
* Vaginal bleeding (with or without clots).
Key: Pregnancy test (urine or serum) (C) * Other symptoms may include:
* Dizziness, fainting or syncope.
Reason: This is a classic case of ruptured Ectopic * Breast tenderness.
pregnancy (though most rupture occurs a bit late * Shoulder tip pain.
that is 7/8 week) and the best step would * Urinary symptoms.
pregnancy test to establish the diagnosis. The * Passage of tissue.
immediate next step if the patient is * Rectal pain or pressure on defecation.
hemodynamically unstable would be an •There may be a history of a previous ectopic
immediate laparoscopy. Observation wouldn’t pregnancy. After one ectopic pregnancy the
help anyone and a laparotomy would be chance of another in the other tube is much
considered in third-world countries. increased.

Discussion: •If the ectopic pregnancy has ruptured, bleeding


is profuse and there may be features of
The majority of ectopic pregnancies occur in the hypovolaemic shock, including feeling dizzy on
ampullary or isthmic portions of the Fallopian standing. Most bleeding will be into the pelvis
tubes. and so vaginal bleeding may be minimal and
misleading.
•Diarrhoea and vomiting are possible, atypical e. Traumatic rupture of aorta
clinical features of ectopic pregnancy.

Tests:
Key: Traumatic rupture of the aorta (E)
The most accurate method to detect a tubal
pregnancy is transvaginal ultrasound. (A traumatic aortic disruption is caused by a rapid
acceleration (or deceleration) causing a tear in
* This can identify the location of the pregnancy the aorta. Normally this is immediately fatal, but
and also whether there is a fetal pole and those who survive may show a widened
heartbeat. mediastinum on CXR. This can be confirmed with
* Human chorionic gonadotrophin (hCG) levels CT scan or angiography of the aorta and requires
are performed in women with pregnancy of prompt surgical correction. Stable the
unknown location who are clinically stable. haemodynamics and surgical correction. Note: pts
with diaphragmatic rupture usually presents days
* hCG levels are taken 48 hours apart. If there is a after trauma with vague symptoms, history is
change in concentration between 50% decline vital)
and 63% rise inclusive over 48 hours then the
woman should be referred for clinical review in
an early pregnancy assessment service within 24 Reason: The wide mediastinum and the radio-
hours femoral delay make Traumatic rupture of the
Management: Arch of the Aorta the most likely diagnosis in this
case. Tension Pneumothorax will not present this
* Admit as an emergency if the diagnosis of way and a rib fracture would be less severe than
ectopic pregnancy is considered a possibility. A the patient is currently. Diaphragmatic rupture
bedside pregnancy test should be performed on would have bowel sounds in the chest and
all women of childbearing age presenting with cardiac tamponade would have Beck’s triad –
lower abdominal pain where pregnancy is even
Muffled heart sounds, distended neck veins and
the remotest possibility.
decreased blood pressure.
* Anti-D rhesus prophylaxis should be given (at a
dose of 250 IU) to all rhesus negative women who
have a surgical procedure to manage an ectopic
pregnancy. 1603. A 36yo woman presents with
* Conservative management may be appropriate swelling in the groin. Exam: swelling is
if the levels of hCG are falling and the patient is diffuse and soft and lies below the
clinically well. Repeat hCG levels are performed in
inguinal ligament. It empties with minimal
these cases.
pressure and refills with release. There is a
cough impulse and it disappears on lying
1602. A 40yo man has fallen off a roof. He down. On the calf of the same leg there
is shocked and has chest pain. There is a are varicosities on the medial aspect.
delay between the radial and femoral What is the most likely dx?
pulse. His CXR=widening of the a. Varicose vein
mediastinum. What is the single most b. Varicocele
likely dx? c. Saphena varix
a. Cardiac tamponade d. Femoral hernia
b. Diaphragmatic rupture e. Inguinal hernia
c. Fx ribs f. Key: Saphena Varix (C)
d. Tension pneumothorax
Reason: The emptying on lying down and the ache with time. Bulge can often be seen in the
coexistence of varicose veins make a saphena lower abdomen with the patient erect and
varix the most likely answer to this question. It is straining. This can be reduced by pressure with a
a dilatation of the saphenofemoral junction due 'gurgling' noise and then the hernia orifice can
to incompetent valves when the saphenous vein often be felt. However, the defect may not be
drains into the femoral vein. seen as a swelling palpable or a bulge may be found distant from
the site. This needs prompt repair.
around 2-4cm inferio-lateral to the pubic
tubercle). This may be mistaken for a hernia, but
a Saphena Varix has a bluish tinge
1605. A 70yo man presents with acutely
Femoral and Inguinal hernia not possible because painful, pale paralysed and pulseless left
the swelling is diffuse and empties with minimal
pressure. Varicose veins are present but they are
leg. He is noted to have a-fib. What is the
not the choice because the saphena varix is most probable dx?
immediately below the inguinal ligament. a. Intermittent claudication
b. Cardiovascular syphilis
c. Buerger’s disease
1604. A man presents with a swelling d. Chronic limb ischemia
above the groin crease in the abdomen. e. Acute limb ischemia
He has not had any med problems of note.
What is the most probable dx?
a. Inguinal hernia
Key: Acute limb ischemia (E)
b. Spigelian hernia
c. Testicular tumor Reason: This is acute presentation of limb
d. Epidydimal cyst ischemia because the atrial fibrillation has thrown
e. Irreducible hernia a clot which has blocked arteries in the leg
leading to this acute picture. Chronic ischemia
would present with amputation or gangrene plus
lipodermatosclerosis of limbs, intermittent
Key: Spigelian Hernia (B) Reason: A Spigelian claudication would present with pain on walking
hernia (or lateral ventral hernia) is a hernia and no pain on rest, Buerger’s disease would
through the spigelian fascia, which is the need a smoking history and syphilis of the CVS
aponeurotic layer between the rectus abdominis would have aneurysms of the large arteries rather
muscle medially, and the semilunar line laterally. than small vessel involvement leading to limb
There is a common misconception that they symptoms.
protrude below the arcuate line owing to Acute limb ischaemia is most often due to either
deficiency of the posterior rectus sheath at that acute thrombotic occlusion of a previously
level, but in fact the defect is almost always partially occluded, thrombosed arterial segment,
above the arcuate line. These are generally or to embolus from a distant site. Without
interparietal hernias, meaning that they do not lie surgical revascularisation, complete acute
below the subcutaneous fat but penetrate ischaemia leads to extensive tissue necrosis
between the muscles of the abdominal wall; within six hours.
therefore, there is often no notable swelling. All Presentation: The affected part becomes pale,
the other options lie below the groin crease i.e. pulseless, painful, paralysed, paraesthetic and
the inguinal ligament. 'perishing with cold' ('the 6 Ps')
Discussion:

Spigelian: this is a hernia through the linea


semilunaris muscle. Initially this causes localised Investigations:
pain exacerbated by straining and coughing, but
the pain may become less localised and more an
•Hand-held Doppler ultrasound scan may help thrombosis of an arterial graft, then thrombolysis
demonstrate any residual arterial flow. is the first step.
•Blood tests
•FBC (ischaemia is aggravated by anaemia). •For patients with acute arterial emboli or
•ESR (inflammatory disease - eg, giant cell thrombosis, treatment with immediate systemic
arteritis, other connective tissue disorders). anticoagulation with unfractionated heparin has
•Glucose (diabetes). been recommended. This should be followed by
•Lipids. long-term warfarin in patients with embolism.
•Thrombophilia screen.
•If diagnosis is in doubt, perform urgent •An arteriogram is performed and the catheter
arteriography. advanced into the thrombus. Streptokinase,
•Investigations to identify the source of embolus: urokinase or tissue plasminogen activator (tPA)
•ECG. should be combined with heparin and
•Echocardiogram. thrombolysis continued for 48 hours or until clot
•Aortic ultrasound. lysis.
•Popliteal and femoral artery ultrasound.
Management •Fibrinolysis usually takes between 6 and 72
hours to achieve clot lysis and so patients with
•Urgent admission - this is an emergency and limb-threatening ischaemia are not candidates for
often requires urgent open surgery or local fibrinolysis, and require emergent
angioplasty. Objective sensory loss requires embolectomy. Local thrombolytic therapy is
urgent treatment. Heparinization is needed therefore reserved for patients with non-life-
immediately (this may double the limb salvage threatening limb ischaemia.
rate), and provide analgesia.
•If a limb is irreversibly ischaemic, amputation
•The limb must be checked for evidence of will be required.
compartment syndrome and, if necessary, a
fasciotomy should be performed.

•If the occlusion is embolic, the options are 1606. A 50yo woman complains of several
surgical embolectomy (Fogarty balloon months hx of weakness and difficulty
embolectomy catheter) or local intra-arterial climbing stairs. Exam: fissuring of the skin
thrombolysis: of her hands. CXR: pulmonary fibrosis.
What is the single most likely positive
antibody?
•If embolectomy with a Fogarty catheter fails, an a. Anti Jo1
on-table angiogram is performed and bypass graft b. Anti Scl 70
or intraoperative thrombolysis considered.
c. Anti Ro
Routine intraoperative angiography for arterial
d. Anti dsDNA
thromboembolectomy has been shown to be
e. Anti centromere
beneficial.

•After successful embolectomy, anticoagulation


with heparin is needed to prevent recurrence. Key: Anti Jo1 (A)
Many surgeons postpone heparin for six hours
after surgery to reduce the risk of a haematoma. Reason: Anticentromere would be present in
limited scleroderma and Anti-Scl 70 would be
•If the occlusion is due to thrombotic disease the present in diffuse scleroderma. Anti-DSDNA
options are intra-arterial thrombolysis, would be positive in SLE, along with Anti-Ro. Anti-
angioplasty or bypass surgery. If due to Ro is also positive in Sjogren’s syndrome and
scleroderma. This is a case of Polymyositis
because none of the others would have the Reason: This is a case of possibly testicular torsion
muscular weakness that is present in this case. (although TT may occur at any age but usual age
The antibody of choice would be Anti Jo1 is 12y-16y), if in any doubt, refer immediately to
antibody. surgery/urology. Testicular torsion presents with
inflammation of one testes which is tender, hot??
ERYTHEMATOUS and swollen. It may lie high
1607. A 65yo woman complaining of and/or transversely. Analgesics, IV fluids and
symptoms suggestive of Raynaud’s antibiotics may have a role but none of those
things would be initial management. Immediate
phenomenon and difficulty in swallowing.
consent is sought for orchidectomy but first it is
Exam: painful lesions on her finger tips
tried to unwind the testicle to avoid surgery.
and facial telangiectasis. What is the Reassurance is an asinine choice.
single most likely positive antibody?
a. Anti Jo1
b. Anti Scl 70
1609. A 60yo man is brought to the ED in
c. Anti Ro
an agitated state. He is lashing out
d. Anti ds DNA
violently. Which drug in low dosage due to
e. Anti centromere
its relative lack of autonomic side effects
is a drug of choice in the tx of agitation in
this pt?
Key: Anti-Centromere (E)
a. Haloperidol
Reason: Anti-centromere antibody would be b. Diazepam
present in CREST syndrome which is also called c. Fluoxetine
Limited scleroderma. All the others are already d. Clozapine
accounted for and would be present in other e. Chlorpromazine
connective tissue diseases.

Key: Haloperidol (A)


1608. A 6yo boy presented about 4h ago
with acute severe pain on the testis with Reason: Acute psychosis’s drug of choice is always
Haloperidol. Diazepam would sedate the patient
the left half slightly higher than the right.
but would not take care of his psychotic
Pain was not relieved by any strong
symptoms, fluoxetine is an SSRI and would take 2
analgesic. What is the initial weeks to act, clozapine and chlorpromazine are
management? not drugs of choice in psychotic episodes due to
a. Give strong analgesic their adverse effects.
b. IV NS and monitor vital signs
c. Reassure
d. Immediate surgical referral 1610. A 32yo woman of 40wks gestation
e. Cover with antibiotics
attends the antenatal day unit with
sudden onset epigastric pain with nausea
and vomiting. She is clinically jaundiced.
Her biochemistry results show a raised
Key: Immediate surgical /UROLOGICAL referral bilirubin, abnormal liver enzymes, high
(D) uric acid and hypoglycemia. What’s the
most likely dx?
a. Acute fatty liver of pregnancy * The white cell count is often elevated. There
b. Obstetric cholestasis may also be neutrophilia and thrombocytopenia.
c. Cholecystitis * Liver transaminases are moderately high.
* Raised serum bilirubin.
d. HELLP syndrome
* Abnormal clotting with coagulopathy
e. Acute hepatitis (prolongation of prothrombin and partial
thromboplastin times with depression of
fibrinogen levels).
Key: Acute Fatty Liver of Pregnancy (AFLP) (A) * Biopsy would be diagnostic but coagulation
problems often preclude it. CT/MRI scanning may
When jaundice is present in pregnancy, AFLP show reduced attenuation in the liver.
should be high on the differential. Management:
Reason: Pain, nausea, vomiting, jaundice, fever Consider early delivery, as the condition usually
with elevated liver enzymes and bilirubin is resolves afterwards with complete recovery.
clinically indicative of AFLP. Also can have Supportive ITU care is frequently required.
elevated INR, TLC and hypoglycaemia. It isn’t Complications:
cholestasis because there is no pruritis, AFLP is a life-threatening condition with a
cholecystitis isnt the answer because there is no reported 1.8% maternal and 23% fetal mortality
history of evidence of gallstones, HELLP isn’t the rate. Serious complications include: *
answer because there isn’t any hemolysis or Disseminated intravascular coagulation (DIC) and
thrombocytopenia and acute hepatitis would gastrointestinal bleeding.
present subclinically or with very less symptoms * Hepatic coma. * Acute kidney injury. *
like diarrhoea and vomiting alongwith clinical Pancreatitis. * Hypoglycaemia.
history of food poisoning.
Discussion:
1611. A 24yo man believes his bowels are
Epidemiology: blocked and his life is in ruin. What kind of
* It is a rare condition with an incidence of 5 in delusion is he suffering from?
100,000 pregnancies. (in USA, AFLP affects 1 in a. Persecutory
7000 to 1 in 16,000 deliveries; more in nullipara b. Factitious
and women with multiple gestations) c. Guilt
* Acute fatty liver of pregnancy (AFLP) tends to d. Nihilistic
occur in late pregnancy.
e. Hypochondriacal
* Risk factors include first pregnancies, pre-
eclampsia, twin pregnancies and male fetuses.
* It may be associated with a mutant gene Key: Nihilistic (D)
producing a defect in mitochondrial fatty acid
oxidation and infants born to mothers with AFLP Reason: The man believes his bowels are blocked
should be screened for defects in this system. and his life is ruined. This is a presentation of
Presentation: nihilistic delusions whereupon the individual
believes his life or his organs are useless and
* This usually presents acutely with nausea, leading to his death, destruction or ruin.
vomiting and abdominal pain, fevers, headache Persecutory delusions involve the belief that
and pruritus, beginning typically at about 35 people are out to get him or her and factitious
weeks of gestation but can occur much earlier. It delusions are when the person himself is lying,
may also appear immediately after delivery. guilt is an emotion and requires a history of
* Jaundice appears soon after onset of symptoms inciting factor while hypochondriacal delusions
and can become intense in a large proportion of are delusions that are disease oriented and
patients. Fulminant liver failure may follow. targeted to receiving medicines or care from a
Investigations: doctor.
1612. A 75yo man with declining vision, abnormality is found on physical exam.
cornea and pupils are normal, fundus What is the most appropriate initial inv
shows obscured margins. What is the which is helpful to get a dx?
single most likely dx? a. Coag screening
a. Macular degeneration b. MSU
b. HTN retinopathy c. Cystoscopy
c. MS d. MRI spine
d. DM background e. Abdominal US
e. Proliferative DM retinopathy

Key: Macular degeneration (A)


Key: Abdominal USG (E)
Reason: The age of the patient, normal
Though some may went for MSU initially but the
opthalmological examination and obscured
catch is …the most appropriate initial inv which is
margins of the fundus all point towards age
related macular degeneration. HTN would have helpful to get a dx
other findings on the ocular exam, MS presents Reason: Painless hematuria in a young male
with optic neuritis and RAPD +ve with red colour without any other findings on history or
blindness developing, DM and proliferative DM examination is likely Polycystic Kidney Disease
would present with other fundus findings which frequently presents with painless gross
according to degree and stage of diabetic hematuria. Coagulation screening in a 24y old
retinopathy. man is only helpful with a history of blood
dyscrasias, mid stream urine will not help us in
establishing a diagnosis in this case, Cystoscopy is
1613. A man under psychiatric tx develops
used to visualize the urinary tract, better to
GI distress and tremors. Which drug is perform in older age group to rule out CA and
most likely to cause these symptoms? MRI spine will not help us at all.
a. Lithium
b. Diazepam
c. Citalopram 1615. A 29yo woman presents to her GP
d. Clozapine with troublesome heavy periods. The med
e. Imipramine tx that she has tried have made little
difference. She is known to have large
Key: Lithium (A) uterine intramural fibroids. You confirm
Reason: Lithium causes these symptoms along that she is currently trying for more
with Diabetes insipidus. children. Select the most appropriate
management for menorrhagia in this pt?
Lithium can cause nausea, diarrhea, dizziness, a. Danazol
muscle weakness,fatigue, and a dazed feeling. b. Endometrial ablation
These unwanted side effects often improve with
c. Hysterectomy
continued use. Fine tremor, frequent urination,
d. Hysteroscopic resection of fibroids
and thirst can occur and may persist with
e. Myomectomy
continued use. Weight gain and swelling from
excess fluid can also occur

Key: Myomectomy (E)


1614. A 24yo man presents with painless Reason: She is currently trying for more children,
hematuria. No other complaint and no hysterectomy will not be useful in this case
obviously. OHCS pg. 276 states that chance of d. OM
subsequent pregnancies is better after e. Perforation of eardrum
myomectomy and it is the best treatment in this
case. Endometrial ablation will not affect the
fibroids and danazol causes a post-menopausal
state which would not help her in conceiving. Key: Otitis Externa (C)

Reason: The swimming history, irritation in both


1616. A 30yo schizophrenic female attacks ears and ear being hot, red, swollen and painful
indicates inflammation of the external acoustic
her mother believing that aliens have
meatus called Otitis Externa. It isn’t otitis media
replaced her with an exact double. What
because of the lack of Tympanic membrane signs,
condition is she suffering from? perforated eardrum would present with just pain
a. Capgras syndrome
and deafness, impacted earwax would also
b. Ganser syndrome present with pain and conductive deafness.
c. Todd syndrome Foreign body would have history of something
d. Fregoli syndrome being used near or inside the ear and would be
e. Cotard syndrome seen on examination of the ear canal. Mainly
Pseudomonas /staph aureus.

Key: Capgras Syndrome (A)


1618. A healthy 2yo boy is brought to the
Reason: Capgras syndrome is an irrational belief
that a familiar person or place has been replaced ED having cut his hand playing in the
by a duplicate. garden. He has a 2cm clean laceration. He
Ganser syndrome is a fictitious disorder in which has not received any routine
a patient deliberately acts as if he has a physical immunizations as his parents are
or mental illness when he doesn’t have it. concerned about possible side effects.
Todd syndrome/Alice In Wonderland
syndrome/Lilliputian syndrome is a disorienting
There are no contraindications to
neurological condition affecting human immunizations. What is the single most
perception of size, shape and time. appropriate follow up inv?
Fregoli syndrome is a delusion of doubles, a a. Courses of DPT vaccine
delusional belief that different ppl are infact a b. Courses of DT
single person in disguise or change appearance. c. Single inj of DPT vaccine
Cotard’s syndrome/Nihilistic delusions is ‘walking
d. Single inj of DT
corpse syndrome’, the person think they are dead
e. Single inj of tetanus Ig
or that one of their organs has stopped
functioning.
Key: Courses of DPT Vaccine (A)

Reason: The child is unimmunized and has no


1617. A 38yo man has just returned from contraindication to vaccination. Keeping in mind
a holiday where he went swimming his age, single injections would be useless and
everyday. For the last few days he has had would not help the patient. Courses of DPT
irritation in both ears. Now his right ear is vaccine would be the best choice in this case.
hot, red, swollen and acutely painful.
What is the single most likely dx?
a. Foreign body 1619. A 6wk child has hx of frequent
b. Impacted earwax vomiting which became worse during the
c. OE last weeks. He has no fever, recently he
has passed stool only once every 2-3d. a. Diazepam
What inv will you do to confirm the dx? b. ECT
a. Abdominal US c. Imipramine
b. Barium meal d. Lithium
c. Erect XR abdomen e. Antipsychotics
d. Feed test
Key: Diazepam
e. Reassure
Reason: Diazepam is the mildest option available
Key: Abdominal USG (A) here, ECT is very unnecessary, Imipramine and
Reason: Barium studies in this case are always the other TCAs are severely contraindicated with a
wrong answer, Abdominal USG is going to give history of MI, Lithium causes MI as an adverse
you a definitive diagnosis as soon as is realistically effect and antipsychotics are used for psychosis.
possible. The age, symptoms point towards
pyloric stenosis as the likely cause, X-ray
Abdomen will not show pyloric stenosis at all, 1622. A pt. comes back from India and
Feed test is a useless answer and reassuring the presents with night sweats and
patient’s parents will not work because this is lymphadenopathy. XR: Cavitation. What
something that needs intervention, possibly investigation should be done next?
surgically by Ramstedt’s pyloromyotomy. a. CT scan
b. AFB stain
c. Blood culture
1620. A 30yo woman had an IUCD d. Bronchoscopy
inserted 8-9m ago. Now on routine follow
up the thread is missing. Uterine US Key: AFB Stain (B)
showed no IUCD in the uterus. What is the Reason: The symptoms and arrival from an
best management? endemic area for Pulmonary TB suggests the best
a. Laparoscopy course of action would be to go for AFB staining
b. Pelvic CT via ZN stain. CT Scan would not help in initial
c. Laparotomy diagnosis, blood culture isn’t first line for TB,
d. Pelvic XR sputum culture clearly is and bronchoscopy is
unnecessarily invasive.
Key: Pelvic XR (D)
1623. A 45yo woman has been extensively
Reason: Thread is missing and it isn’t seen in the
investigation for a lump she believes to be
uterus, Xray is the logical choice. Laparotomy and
laparoscopy are not needed unless it perforates
cancer. She doesn’t think doctors take her
an organ which is highly unlikely. USG is the first seriously and demands another referral.
choice which has been performed, Xray after that What term best describes her condition?
to check location and then advice surgical a. Munchausen syndrome
retrieval if needed. b. Munchausen’s by proxy
c. Hypochondriasis
d. Malingering
1621. A pt comes with weight loss and e. Phobia
sleep disturbance has mild depression. He Key: Hypochondriasis (C)
has a hx of MI. What is the single most
appropriate tx? Reason: This patient, with her history of extensive
investigations or her “fat folder syndrome”
warrant nothing other than the label of being a This is because the levels of growth hormone in
Hypochondriac who wants treatment for the body fluctuate a lot throughout the day in all
imaginary illnesses she has even after being people. * The diagnosis of acromegaly is made by
counselled about the severity of her illness. a glucose tolerance test. In this test you drink a
sugar drink containing 75 grams of glucose. You
Muchausen’s syndrome describes a patient who then have a series of blood tests over two hours.
lies vividly, is addicted to institutions and goes The glucose should lower the blood level of
from hospital to hospital feigning illnesses hoping growth hormone. However, if you have
for laparotomy or mastectomies. acromegaly, the growth hormone level remains
high.
Munchausen’s by proxy defines injury to a * A blood test to measure the level of IGF-1 (see
dependent person by carer to gain medical above) may be measured if acromegaly is
attention. Malingering is the creation of a suspected. This may also be used as an index of
fictitious illness without even the lump which is disease activity to assess how well treatment is
present in this case. Phobia is a fear of something working. * A magnetic resonance imaging (MRI)
irrational. scan can show the size of any tumour.
* Eye and visual tests can assess if the tumour is
pressing on the optic nerve.
* If you are confirmed as having acromegaly,
1624. A 15yo man presents with other tests will be needed to see if the tumour is
bitemporal hemianopia and spade-like causing a lack or excess of other hormones made
hands. What is the definite test to confirm by the pituitary. * Other tests may include chest
the dx? X-ray, electrocardiogram (ECG) and X-rays of
a. Early morning growth hormone some of your joints.
b. Insulin tolerance test What are the treatments for acromegaly?
c. OGTT with growth hormone The aim of treatment is to reduce the level of
measurements growth hormone in the blood to normal, and to
d. Random insulin-like growth factor (IGF-1) reduce the size of an enlarged tumour. Many of
e. Short ACTH test the symptoms and features of acromegaly will
reverse or improve with successful treatment
Key: OGTT with growth hormone measurements (apart from any fixed extra bone growth that had
(C) occurred).
Surgical treatment
Reason: The bitemporal hemianopia and spade-
like hands point towards this being acromegaly. The most common treatment is to remove the
The best initial test is insulin like growth factors adenoma by surgery. This is done using very fine
but the definitive test that confirms the diagnosis instruments. There are two different ways to
operate on the pituitary gland:
is OGTT with serial growth hormone
o The first method is known as endonasal trans-
measurements. Early morning growth hormone
sphenoidal surgery. This involves the surgeon
would be raised anyway, it is released in a
reaching your pituitary gland through a small cut
pulsatile manner, Short ACTH is used for (incision) in wall of one of your nostrils. In the
Cushing’s disease and Insulin tolerance test is not other way, the surgeon approaches the pituitary
used for growth hormone or acromegaly gland through a small incision behind your upper
assessment. lip, just above your front teeth. The instruments
are passed through the base of your skull - the
Discussion:
sphenoid bone. The aim is to remove the
How is acromegaly diagnosed? adenoma, but to leave the rest of the pituitary
gland intact.
* A blood test can measure the level of growth o The operation is successful, with no further
hormone. However, a single test is not reliable. treatment needed, in around 9 out of 10 cases
with smaller tumours. The operation is less Therefore, although many of the symptoms of
successful in those with larger tumours. However, growth hormone excess will be eased, it does not
sometimes it is not possible to remove all the reduce the size of the tumour and headaches are
cells of the tumour. If not all is removed and your not eased.
growth hormone level remains high following
surgery, other treatments listed below are likely
to work. * Radiotherapy * Radiotherapy is an option to
o Your surgeon will advise on the possible reduce the size of the tumour and hence reduce
complications which can sometimes occur. For the production of growth hormone. Radiotherapy
example, sometimes the operation may damage focuses high-intensity radiation at your pituitary
some other parts of the pituitary gland. This may tumour to destroy the abnormal cells. It may be
cause a reduced production of some other used if you are not able to have surgery, or if
hormones. If this occurs, you will need to take surgery was only partially successful. However, it
replacement hormone therapy. can take months or years after the radiotherapy is
Medication given for the level of growth hormone to reduce
to normal. You can take medication whilst waiting
Medication can be used if surgery is not possible, for the effects of radiotherapy to work.
or not wanted. It is also used whilst waiting for A possible side-effect of pituitary radiotherapy is
surgery or radiotherapy. It can also be used in damage to other normal pituitary cells. This can
cases where surgery fails to remove the tumour cause a reduced level of some other hormones.
totally and the level of growth hormone remains However, if this occurs you can take replacement
high. hormone therapy.

* Somatostatin analogues (octreotide and Acromegaly and bowel cancer screening


lanreotide) reduce the level of growth hormone
to normal in over half of cases, and reduce the As mentioned earlier, people with acromegaly
have an increased chance of developing bowel
size of the tumour in about 8 in 10 cases.
However, these medicines need to be given as an (colonic) polyps and bowel cancer. Therefore, if
you are diagnosed with acromegaly and are aged
injection. They work in a similar way to
somatostatin (described above) which is a 40 or more you will normally be offered a routine
colonoscopy every 3-5 years. A colonoscopy is a
hormone that prevents growth hormone from
being released from pituitary cells. These test where an operator (a doctor or nurse) looks
into your large bowel (colon) with a flexible
medicines used to be injected several times a day.
However, longer-acting preparations are now telescope. It can diagnose bowel problems such
as polyps and bowel cancer. The aim is to detect
available as monthly or fortnightly injections.
Side-effects are not common with these those people who develop cancer as early as
possible (before symptoms develop) when the
medicines. Some people develop tummy
(abdominal) pains and diarrhoea, but these chance of a complete cure is high.However, you
should always tell you doctor if you develop any
usually wear off with time. Gallstones can also
occur but rarely cause problems. * Dopamine new symptoms from your bowel such as
persistent diarrhoea, passing mucus, passing
agonists (such as cabergoline, bromocriptine and
quinagolide) can be taken as tablets. They work blood, or tummy (abdominal) pain.
by preventing the release of growth hormone
from tumour cells. However, they only work well 1625. A 22yo man has had an acute,
in about 1 in 5 cases. Side-effects such as feeling
sick and dizzy are also quite common. painful, red right eye with blurring of
vision for one day. He had a similar
* Pegvisomant (Somavert®) is taken as a daily episode 1y ago and has had episodic back
injection. However, unlike the other medicines pain and stiffness relieved by exercise and
listed above, it does not act directly at the diclofenac for four years. What is the
pituitary. Pegvisomant works by blocking the
SINGLE most likely cause of his red eye?
action of growth hormone on your body's cells.
a. Chorioretinitis patients have Iritis. Conjunctivitis and Episcleritis
b. Conjunctivitis would not have blurring of vision and Keratitis
c. Episcleritis wouldn’t be associated with systemic symptoms.
d. Iritis
e. Keratitis 1626. A 40yo divorced man with bipolar
affective disorder attends hospital
Ans)D
following an OD of 30 TCA tablets. His
Key)The symptoms described are characteristic of new partner has left him and he has
ankylosing spondylitis.(lower back pain and stopped taking his medicine and begun
stiffness which gets better after moving around drinking heavily. He appears depressed,
and taking NSAIDS) the extra articular
feels hopeless and is ambivalent about
manifestations of AS are uveitis.
being alive. He is now fit for discharge
Symptoms: from the medical ward and acknowledges
Anterior uveitis:eye the benefits of previous tx. What is the
pain,photophobia,redness,visual loss,pupil shape SINGLE most appropriate next
changes,symptoms develop in few hours or management?
days,maybe single eye is affected a. Admission to the psychiatry ward
Intermediate uveitis:painless blurred b. Arrange psychiatric outpatient follow-up
vision,FLOATERS,both eyes normally affected c. Discharge to the care of the general
Posterior uveitis:painless blurred vision,severe
practitioner
vision loss,FLOATERS and scotomas, take longer
d. Referral to local alcohol treatment team
to develop.
the stem of the question talks about the e. Referral to clinical psychologist
symptoms of anterior uvitis, hence the answer is
Ans)A
iritis. (chorioretinitis is excluded as it comes under
the symptoms of posterior uvitis) Key) the trigger (partner leaving) precipitated an
acute attack of maniac depression in the patient.
in episcleritis, the sclera may look blue and visual
there is no other option BUT for him to be
acquity is NORMAL. conjunctivitis will present
admitted in a psychiatric unit with his actions
with similar symptoms as well but there will be a
being constantly observed.the stem of the
discharge and the question will talk about eyes
question says he stopped taking his medication.
sticking together.Keratitis will be identified with a
abrupt cessation of Lithium precipitates mania in
white patch on the cornea.
50% of the patients as well and people with
Treatment of uvietis mania arent likely to adhere to their medications
in the first place either.Discharging the patient
1. Steroid eye drops
with an SSRI/antidepressant isnt a very good
2. cycloplegic eye drops to relieve pain
move since SSRI’s take at least 2-4 weeks for their
3. dark sun glasses
4. painkillers effect to build up fully. therefore, proving that the
best possible answer to the question is still A.

Reason: These symptoms point towards Reason: The patient is high risk on the suicide risk
Ankylosing Spondylitis with the episodic back pain assessment scale and doesn’t warrant out-patient
that is relieved by exercise and NSAIDs for four follow up or discharge to be seen by a GP. He
years. The presentation is that of Uveitis which is should be admitted and treated in the Psychiatry
associated with autoimmune diseases, specifically ward as needed. Referral to the local alcohol
Iritis with acutely painful red eye. Chorioretinitis treatment team or clinical psychologist is never
would present in the same way, but 1/3 or AS the right answer.
Neutrophils=0.1, plt=14. No abnormal
white cells were seen on the blood film.
1627. A healthy baby boy is born at term
She was transfused and given IV
to a woman who was unwell with
antibiotics and her condition improved.
confirmed acute hep B during pregnancy.
3wks later her blood count has returned
The mother is very concerned that she
to a similar picture. What is the SINGLE
may have infected the baby with hep B.
most likely underlying dx?
What SINGLE preventative intervention
a. ALL
should be given to the baby?
b. AML
a. Full course of hepatitis B vaccine
c. Aplastic anemia
b. Hepatitis B immunoglobulin alone
d. CML
c. Hepatitis B vaccine and hepatitis B
e. Pernicious anemia
immunoglobulin
d. Hepatitis B vaccine as single dose
e. None until hepatitis B status confirmed
Key: Aplastic Anaemia (C)
Ans)C
Reason: The age of the patient and pancytopenic
Key)Babies born to mothers infected with picture give us a clinical diagnosis of Aplastic
hepatitis B have a high risk of acquiring infection, anemia. Normal WBC morphology rules out ALL,
which can be prevented by vaccination at birth.All AML and pernicious anaemia while the age rules
babies with seropositive mothers should have the out CML as a diagnosis.
full primary course of hepatitis B immunisation
and most should also have HBIG within 24 hours
of birth. 1629. An 83yo woman admitted with a
chest infection becomes confused with
impaired attention and poor
Reason: If the mother has active confirmed acute concentration. She is restless and
Hep B, give Immunoglobulins and vaccinate the
frightened. She is verbally abusive and has
baby at birth. Also perform serology of the baby
perceptual abnormalities. There is no
at 12-15 months (1 year). If Hep B Antibodies +ve
and HbSAg -ve, do nothing. Full course of the significant prv psychiatric hx. What is the
vaccine is unnecessary because the SINGLE most likely dx?
aforementioned course will provide sufficient a. Delirium
coverage, Ig alone and Vaccine alone will not be b. Drug induced psychosis
enough for the baby. Full course with c. Lewy body dementia
immunoglobulin will be given at BIRTH, 1 d. Multi-infarct dementia
MONTH, 2 e. Psychotic depression

MONTHS. (E) isn’t a good option because the


baby needs immediate coverage in case the
infection has been transmitted at delivery. Key: Delirium (A)

Reason: Drug induced psychosis would require a


drug history, dementia and psychotic depression
1628. A previously well 15yo girl had an would have a significant previous psychiatric
acute onset of fever, sweating, bruising history. Delirium or Acute Confusional States
and petechiae. A blood count showed: happen in the elderly in response to stressors like
Hgb=63g/L, WBC=1.1mg/L,
acute infections and this is most likely brought on Progestogen implants would not help the
by the chest infection that has developed. menorrhagia. LNG-IUS (Mirena) is the hormone
releasing device that is most suitable in this
1630. A town has a population of 500,000. patient and would be the contraceptive of choice.
In a five year period there are 1250 cases
of bladder cancer diagnosed at the only
hospital. During the same period the 1632. An 8yo girl has had left earache for
occupational health department 2d. The earache subsided about 2h ago
diagnosed a further 500 cases. What is with the onset of a purulent discharge
the annual incidence per million of which relieved the pain. Her temperature
bladder cancer in this population? is 39.2C. What is the SINGLE most
a. 2100
appropriate antibiotic?
b. 1750 a. Amoxicillin
c. 1400 b. Ciprofloxacin
d. 700 c. Clindamycin
e. 350 d. Erythromycin
e. Flucloxacillin

Key: 700 (D)

Reason: There are 1750 cases a 5 year period in Key: Amoxicillin (A)
this scenario. The number of cases in 1 year in the Reason: This is the picture of Acute Otitis Media
0.5 million people in this town are 350 cases/0.5 which has led to tympanic membrane
million people. To calculate the incidence per perforation. PO Amoxicillin for 7 days is the
million of bladder cancer, we would simply treatment of choice with appropriate analgesics.
double the number to get an estimate which Amoxicillin +/- Clavulanate is the first line drug
would give us 350 x 2 = 700 cases/million. because it fits the organisms responsible which
are Strep pneumonia, Moraxella catarrhalis and
H. influenza.
1631. A 28yo woman who has had a prv
pulmonary embolism in pregnancy wishes
to discuss contraception. She has 1633. A 38yo man has disturbing thoughts
menorrhagia but is otherwise well. What about his house being infected by germs.
is the SINGLE most suitable contraceptive He is anxious about safety and checks the
method for this patient? locks of his doors repeatedly before going
a. COCP to bed. For the last 8wks he has been
b. Copper IUCD washing his hands every time he touches
c. Levonorgestrel intra-uterine system the lock, 20-30 times a day. What is the
d. Progestogen implant
SINGLE most appropriate management?
e. POP a. Antidepressant
b. Antipsychotic
c. Anxiolytic
Key: Levonorgestrel Intra-Uterine System (C) d. CBT
Reason: The woman has a history of e. Psychodynamic psychotherapy
thromboembolic disease, which essentially rules
out COCPs. POPs, Copper IUCD (Copper – T) and
Key: CBT (D) She has two young children. What is the
Reason: This scenario describes a case of OCD for SINGLE most likely dx?
which the best management is CBT followed by a. Psoriasis
SSRIs or TCAs. The first line treatment is always b. Reactive arthritis
CBT, not pharmacological therapy. Psychotherapy c. Rheumatoid arthritis
is indicated in depression, psychosomatic d. Sarcoidosis
disorders, dissociative or conversion disorders, e. SLE
personality disorders, relationship problems or
grief. key)B

Ans)

Reactive arthritis is caused when a joint reacts to


1634. A 65yo man had closure of
an infection elsewhere in the body. The infection
colostomy performed 5d ago. He is not which triggers reactive arthritis is not actually in
systemically unwell. There is a tender, the joint, Most often, these bacteria are in the
localised fluctuant swelling 4 cm in genitals (Chlamydia trachomatis) or the bowel
diameter in the wound. What is the (Campylobacter, Salmonella, Shigella and
SINGLE most appropriate management? Yersinia)..it can also occur after viral infections
a. Abdominal support causing sore throat skin rash etc.
b. Antibiotics
c. Laparotomy and re-suture wound
d. Local exploration of wound Fever is not usually a feature of the other
e. Observation diseases mentioned here.

1636. A 16yo girl has had an enlarging


Key: Local exploration of wound (D) mass in the right side of her neck for the
Reason: The patient is NOT systemically unwell, last 6wks. She has had no other
he has had colostomy performed 5 days back. The symptoms. She has a 2 x 2 cm enlarged LN
swelling is tender, localised and fluctuant. Local in the anterior triangle of the neck with
exploration with I&D if needed is the single most several smaller associated LN palpable.
appropriate management option. Abdominal
support is unnecessary, so are antibiotics if the
Oropharyngeal examination shows
patient is systemically well. Laparotomy is tonsillar membranes. What is the SINGLE
indicated if there is a bleed or some surgically most likely dx?
correctable issue going on, observation is not the a. Infectious mononucleosis
right answer here because intervention is b. Leukaemia
warranted. c. Lymphoma
d. Sarcoidosis
e. Tuberculosis
1635. A 32yo woman has had a febrile key)A
illness and swelling of the small joints of
her hands, feet, wrists and knees for two Ans)infectious mononucleosis is seen primarily in
days. She has a maculopapular rash and a young teenagers and college going students (16
year old girl). the rest of the options are excluded
few palpable, small cervical lymph nodes.
because there is no mention of weight loss(all
She was previously well. There is no
other 4 options), chest involvement
history of relevant travel outside the UK.
(sarcoidosis,TB), skin involvement • Patient is having reactive arthritis because
(leukemia=bruises,sarcoidosis=skin rash), night of any underlying infection, most commonly
sweats (TB,lymphoma) urethral infection.
• Reactive arthritis is caused when a joint
reacts to an infection elsewhere in the body. The
symptoms of infectious mononucleosis: infection which triggers reactive arthritis is not
actually in the joint, but is usually in the gut or
• Sore throat; tonsillar enlargement is
urethra.
common, classically exudative and may be
• This is because they are most at risk of
massive
urethral infection from sexually transmitted
• Fine macular non-pruritic rash, which
diseases. However, it can occur at any age and in
rapidly disappears.
anyone.
• Lymphadenopathy, especially neck
Treatment:
glands.
• Antibiotics for infection
• Arthralgias and myalgias low grade fever
• NSAIDs for joint pain
• Later signs include:
o Mild hepatomegaly and
splenomegaly 1636. A 16yo girl has had an enlarging
o Jaundice mass in the right side of her neck for the
investigations :detection of heterophile last 6wks. She has had no other
antibodies by the monospot test or the paul symptoms. She has a 2 x 2 cm enlarged LN
bunnell test in the anterior triangle of the neck with
EBV is also associated with: several smaller associated LN palpable.
Oropharyngeal examination shows
• Burkitt's lymphoma. tonsillar membranes. What is the SINGLE
• B-cell lymphomas in patients with
immunosuppression.
most likely dx?
• Undifferentiated carcinomas - eg, cancer a. Infectious mononucleosis
of the nasopharynx and cancer of the salivary b. Leukaemia
glands. c. Lymphoma
treatment is supportive. d. Sarcoidosis
e. Tuberculosis
1635. A 32yo woman has had a febrile
illness and swelling of the small joints of
Key is A: infectious Mononucleosis
her hands, feet, wrists and knees for two
days. She has a maculopapular rash and a • Infectious mononucleosis (IM) is usually a
few palpable, small cervical lymph Nodes. self-limiting infection, most often caused by
She was previously well. There is no Epstein-Barr virus (EBV)
• The incubation period is between 1 and 2
history of relevant travel outside the UK.
months
She has two young children. What is the • Clinical infection is most common in
SINGLE most likely dx? populations with many young adults
a. Psoriasis
b. Reactive arthritis Presentation:
c. Rheumatoid arthritis • Low-grade fever, fatigue and prolonged
d. Sarcoidosis malaise.
• Sore throat; tonsillar enlargement is
e. SLE
common
Key is B: Reactive Arthritis • Fine macular non-pruritic rash, which
rapidly disappears.
• Transient bilateral upper lid oedema. e. IV fluids; LP
• Lymphadenopathy, especially neck glands.

this man is showing septicemic meningitis and


1637. A 60yo man has had increasing pain needs ICU CARE before which fluids resus +Ab (be
in both buttocks, thighs and calves on ready for intubation, inotropes and vasopressors)
walking for three months. He has also ohcm 9th ed pp 833
recently developed impotence. Femoral Key is D: IV Fluids; IV antibiotics
and distal pulses are absent in both limbs.
What is the SINGLE most likely site of As his BP is low and has presented with nech
arterial obstruction? stiffness so, IV fluids and V antibiotics should be
a. Aorto iliac given, after giving him oxygen.
b. External iliac 1639. A 16yo boy was brought to hospital
c. Femoropopliteal in a comatose state having taken
d. Internal iliac
methadone belonging to his sister. He was
e. Tibial
given naloxone and rapidly became alert.
Some hours later, he gradually becomes
semi-conscious again.What is the SINGLE
Key is A: Aorto iliac (saddle embolus; surgical
most likely reason for this patient
intervention)
becoming semi-conscious again in
Page 718 OHCM 9th edition. hospital?
a. Methadone hepatotoxicity has caused
Patient is suufering from leriche’s syndrome.
acute liver failure
Classic triad of: b. Methadone is eliminated from the body
more slowly than naloxone
• Pain and claudication of buttock and thighs
(pale cold leg) c. Naloxone is a partial agonist at the
• Erectile dysfunction from aorto iliac central nervous system opioid receptor
occlusive disease d. The pt has misused another substance
• Absent femoral and distal pulse. that has caused an intracranial bleed
e. The pt has misused another substance
that is absorbed more slowly than
1638. A 78yo man has collapsed. He has methadone
had a severe headache for 12 hours and Key is B: Methadone is eliminated from the body
had an URTI 3d ago. He has a more slowly than naloxone (short t1/2)
temp=39.2C, pulse=122bpm,
BP=84/60mmHg and RR=34bpm but his All other options are not favoring the scenario
chest is clear. He has a GCS=10 and some
neck stiffness. He has been started on
high-flow oxygen. What is the SINGLE 1640. A 27yo woman who takes the COCP
most appropriate immediate has had painless vaginal spotting and
management? discharge for 3 days. Her last menstrual
a. IV antibiotic; CT brain scan period, which lasted four days, finished 10
b. IV antibiotic; LP days ago. Her last cervical smear two
c. IV fluids; CT brain scan years ago was normal. Abdominal and
d. IV fluids; IV antibiotic vaginal examinations are normal apart
from a mild ectropion with contact appropriate management the night before
bleeding. What is the SINGLE most the procedure?
appropriate initial inv?
a. Cervical smear
a. Bisacodyl tablets
b. Colposcopy
b. Glycerine suppository
c. Endocervical swab
c. Lactulose syrup
d. Endometrial biopsy
d. Magnesium citrate (orally)
e. Pelvic US
e. Senna tablets

Key is C: Endocervical swab


Key is D: Magnesium Citrate Orally
As her cervical smear and examination of
Colonoscopy is a test to assess your colon (large
abdomen and vagina are normal, next would be
intestine).
to exclude a STD for which Endocervical swab is
taken. What preparation do I need to do?

• Chlamydia is an sexually transmitted • The colon needs to be empty so that the


infection caused by a germ (bacterium) operator can obtain a clear view. You will be
calledChlamydia trachomatis instructed on how to take a special diet for a few
• About 1 in 20 sexually active women in the days before the test. You will also be given some
UK are infected with chlamydia. It is most laxatives to take.(magnesium citrate is the least
common in women aged under 25. (About 1 in 12 harmful)
women aged 20 are infected with chlamydia.)
Presentation: • You will need somebody to accompany you
• Vaginal discharge. This is due to the neck of home, as you will be drowsy with the sedative
the womb (cervix) becoming inflamed.
• Pain or burning when you pass urine. Its feb 20 2016 and I couldn’t find magnesium
• Vaginal bleeding or spotting between citrate recommended by NHS. Senna is there but
periods. In particular, bleeding after you have sex. not Magnesium citrate. They routinely use
• Pain or discomfort in the lower tummy sodium picosuflate or KLEAN-PREP/MOVI-PREP
(abdomen) area (the pelvic area) (potassium chloride, sodium chloride, sodium
Diagnosis bicarbonate, macrogol 3350, sodium sulfate,
• Chlamydia can be confirmed by a swab anhydrous). However In The Given Choices
taken from the neck of the womb (cervix) in Magnesium Citrate May Be Correct
womenhave sex
Treatment
• A short course of an antibiotic medicine
usually clears chlamydial infection 1642. A 19yo woman has had progressive
• Note: antibiotics can interfere with the bilateral iliac fossa pain and dyspareunia
combined oral contraceptive pill (COCP). If you for 3days. She has an offensive vaginal
take the COCP you should use alternative discharge and feels unwell and feverish.
methods of contraception until seven days after Her temp=39C. An initial antimicrobial
finishing a course of antibiotics
regimen is commenced. What SINGLE set
of organisms are the most appropriate for
the antimicrobial regimen to cover?
1641. A 72yo man being investigated for a. Neisseria gonorrhoeae and Candida
anaemia is booked for a colonoscopy in 24 albicans
hours. What is the SINGLE most
b. Neisseria gonorrhoeae and Candida antidepressants as well (e.g., amitriptyline,
albicans and Gardnerellavaginalia doxepine.
c. Neisseria gonorrhoeae and Chlamydia
trachomatis
d. Neisseria gonorrhoeae and Chlamydia 1644. A 4yo girl has had a temp=38.5C for
trachomatis and Candida albicans 2days and has not wanted to eat her food.
e. Neisseria gonorrhoeae and Chlamydia Yesterday she developed a sore throat and
trachomatis and Gardnerellavaginalis small, painful ulcers inside her mouth.
Today she has small blisters on the palms
of her hands and soles of her feet which
Key is C: Neisseria gonorrhoeae and Chlamydia are painful but not itchy. What is the
trachomatis SINGLE most likely underlying cause?
a. Coxsackie virus
• Most common antibiotic given for
UNCOMPLICATED STDs is azithromycin and it b. Herpes simplex virus
covers gram negative bacteria which are neisseria c. Staphylococcus aureus
and chlamydia. d. Streptococcus pneumonia
• Candida albicans is a fungal infection e. Varicella zoster virus
• Gardnerella is a gram variable bacteria and
normal commensal.

Key is A: coxsakie virus

1643. A 48yo man with renal cancer had Patient is suffering from HFMD (Hand,Foot and
radiotherapy for metastatic spinal cord Mouth Disease)
compression at the 11th thoracic vertebra • HFMD is due to an infection that usually
4wks ago. He has retained sensation but is causes a typical illness, including a typical rash. It
unable to stand. He has pain in a band is most commonly caused by the Coxsackie A16
around his lower trunk controlled by virus
• HFMD most commonly affects children
regular oral morphine. He is distressed by
under 10 years of age
increasingly frequent episodes of painful • This might include a high temperature
muscle spasms in his right leg. What is the (fever). After this, a sore throat commonly occurs,
SINGLE most appropriate management of quickly followed by small spots that develop
his symptoms? inside the mouth. These soon progress into small
a. Amitriptyline mouth ulcers
b. Baclofen • In many cases, spots also develop on the
skin. This is typically a day or so after the mouth
c. Fentanyl patch
ulcers develop. The spots are small lumps that are
d. Gabapentin a few millimetres in diameter and usually appear
e. Increase morphine dose on the hands and feet, they are not usually itchy
but sometimes they can be a little bit sore.
Treatment:
Key is B: Baclofen FOR MUSCLE RELAX • There is no treatment that will take away
the virus
Spinal metastases, often Also cause neuropathic
pain with a radicular component (sensory deficit,
burning pain, painful shock-like sensations) which
1645. A 32yo woman has had 3 episodes
requires the additional administration of of slurred speech and 2 episodes of
anticonvulsants (e.g., gabapentin) and perhaps transient weakness of both legs in the
past 5yrs. Each episode has resolved in • Weakness or paralysis of some muscles.
3m. What is the SINGLE most likely dx? Mobility may be affected.
a. Meningioma • Problems with passing urine.
• Inability to have an erection in men.
b. Migraine
• Difficulty with speaking.
c. Multiple sclerosis Investigation and Treatment:
d. Stroke • A firm diagnosis of MS is often not made
e. Transient ischaemic attack until two or more relapses have occurred
• A magnetic resonance imaging (MRI) scan
of the brain is a useful test. This type of scan can
detect small areas of inflammation and scarring in
Key is C: Multiple sclerosis
the brain which occur in MS.
• At present, although there is no cure for
MS.
Points in favor: symptoms have a relapsing-
remitting form

• MS is thought to be an autoimmune
disease 1646. An 8yo girl is complying with her
• About 1 in 600 people in the UK develop asthma treatment of low-dose inhaled
MS. It can affect anyone at any age, although it is
corticosteroid prophylaxis and short-
rare in young children. It most commonly first
develops around the age of 30. MS is the most acting bronchodilators as required. Her
common disabling illness of young adults in the inhaler technique is good. She now has a
UK. It is twice as common in women as in men. frequent night cough and mild exercise-
symptoms include: induced wheeze. What would be the
• Visual problems SINGLE most appropriate change in her
The first symptom of MS for around one in four
treatment?
people with MS is a disturbance of vision.
a. Add leukotriene antagonist
Inflammation (swelling) of the optic nerve can
occur. This is called optic neuritis. This can cause b. Add oral theophylline
pain behind your eye and also some loss of your c. Add regular long-acting bronchodilator
vision. This usually only affects one eye. d. Increase dose of inhaled corticosteroid
• Muscle spasms and spasticity e. Short course of oral corticosteroid
Tremors or spasms of some of your muscles may
occur. This is usually due to damage to the nerves
that supply these muscles. Some muscles may
shorten (contract) tightly and can then become Key is D: Increase dose of inhaled corticosteroid
stiff and harder to use. This is called spasticity. A common treatment plan for a typical person
• Pain
with moderate asthma is:
There are two main types of pain that may occur
in people with MS: • A preventer inhaler (usually a steroid
Neuropathic pain inhaler), taken each morning and at bedtime. This
Musculoskeletal pain usually prevents symptoms throughout the day
• Fatigue and night.
Extreme tiredness (fatigue) is one of the most • A reliever inhaler (short acting
common symptoms of MS. bronchodilators) may be needed now and then if
Other symptoms which may occur include: breakthrough symptoms occur. For example, if
• Numbness or tingling in parts of the skin. symptoms flare up when you have a cough or
This is the most common symptom of a first cold.
relapse.
• If exercise or sport causes symptoms then a 1648. An 84yo woman with Alzheimer's
dose of a reliever inhaler just before the exercise dementia has recently become incontinent
usually prevents symptoms.
and more confused than usual. What is
• The dose of the preventer inhaler may
need to be increased for a while if you have a the SINGLE most likely dx?
cough or cold, or during the hay fever season a. Detrusor overactivity
b. Neuropathic bladder
c. Nocturnal enuresis
1647. A 38yo man with longstanding d. UTI
alcohol dependence has vertigo and a e. Uterine prolapse
tremor every morning. What is the SINGLE
most likely dx?
a. Anxiety
Key is D: UTI
b. Benign positional vertigo
c. Cerebellar degeneration
d. Optic neuritis
1649. A 4yo boy complains of pain around
e. Temporal lobe epilepsy
his right eye. He is unwell, febrile and also
suffers from pain on the right side of his
Key is C: Cerebellar degeneration face. What is the most probable dx?
a. Allergic reaction
• Cerebellar degeneration is a process in
b. Furuncle
which neurons in the cerebellum - the area of the
brain that controls coordination and balance - c. Folliculitis
deteriorate and die d. Foreign body
Associated diseases: e. Periorbital cellulitis
• ischemic or hemorrhagic stroke, when
there is lack of blood flow or oxygen to the
cerebellum
Key is E: Periorbital cellulitis (also called preseptal
• cerebellar cortical atrophy, multisystem
atrophy, and olivopontocerebellar degeneration, cellulitis)
progressive degenerative disorders in which • preseptal cellulitis occurs at younger ages
cerebellar degeneration is a key feature (80% of patients are under 10 years of age and
• Friedreich’s ataxia, and other most are younger than 5 with a mean age of 21
spinocerebellar ataxias, which are caused by months)
inherited genetic mutations that result in ongoing Preseptal cellulitis:
loss of neurons in the cerebellum, brain stem, • Acute onset of swelling, redness, warmth
and spinal cord and tenderness of the eyelid.
• transmissible spongiform encephalopathies • Fever, malaise, irritability in children.
(such as Creutzfeldt-Jakob disease) in which • Ptosis
abnormal proteins cause inflammation in the • Diagnosis: is usually made based on the
brain, including the cerebellum clinical findings
• multiple sclerosis, in which damage to the Management:
insulating membrane (myelin) that wraps around • Most children are initially admitted to
and protects nerve cells can involve the hospital (even for preseptal cellulitis) unless there
cerebellum is good reason not to. This may be just for 24
Other diseases that can cause cerebellar hours. Children should be considered to have
degeneration include: orbital cellulitis until proven otherwise (ie
• chronic alcohol abuse that leads to repeated examinations normal, good response to
temporary or permanent cerebellar damage antibiotics in first 24 hours and normal CT scan).
• Oral co-amoxiclav may be used for both movements of both legs. Based on the
adults and children as long as there is no allergy likely clinical dx, which one of the
to penicillin. Clinical improvement should occur
following genetic patterns is most likely?
over 24-48 hours.
a. AD inheritance with anticipation
b. AD with variable penetrance
1650. A pt presents with irregularly c. AR
irregular pulse of 162bpm. What drug is d. X-linked
most useful initially? e. Mitochondrial disorder
a. Amiodarone
b. Digoxin Key is A: AD inheritance with anticipation
c. Bisoprolol o Patient is suffering from Huntingtons disease
d. Warfarin and that is autosomal dominant with
e. Heparin anticipation which means a genetic disorder
is passed on to the next generation, the
Key is C: Bisoprolol (most likely its acute AF and
symptoms of the genetic disorder become
1st line is verapamil/bisoprolol, 2nd line
digoxin/amiodarone) apparent at an earlier age with each
generation.
Bisoprolol is Beta blocker used as
antihypertensive and to control the heart rate Huntingtons Disease:
aswell.
• It is an inherited (genetic) condition that
Bisoprolol slows down the activity of your heart affects the brain and nervous system. It can
by stopping messages sent by some nerves to interfere with movements of your body, can
your heart. It does this by blocking tiny areas affect your reasoning, awareness, thinking and
(called beta-adrenergic receptors) where the judgement (cognition) and can lead to a change in
your behavior
messages are received by your heart. As a result,
• This faulty gene is carried on chromosome
your heart beats more slowly and with less force.
4.
• HD affects between 5-10 people per
100,000 in the UK.
1651. A 59yo man has shown a change in Presentation
his mood and personality over a 9m The symptoms of HD can be grouped into three
period. He has subsequently developed main areas:
difficulty with memory and conc, and then • Problems with movement
progressive fidgety movements of his • Problems with cognition
limbs and facial musculature. By the time • Mood and behavioural problems
Treatment:
of medical assessment he has frank
choreiform movements and a mini-mental • At present there is no cure for HD. Also,
state exam of 21/30. Other exam is there is no treatment that has been found to
normal. He was adopted and therefore no delay the onset of symptoms or to delay the
progression of symptoms
information on his famhx is available. He
has 3 adult children (27, 30, 33) of whom
the 2 youngest are asymptomatic. 1652. A 35yo pt has been dx with
However, the oldest son has recently been schizophrenia. He mimics the doctors and
inv by the neurology dept for slightly attendants – doing the same physical
erratic behavior and fidgety restless
actions as them. What symptom does this • Schizophrenia develops in about 1 in 100
pt have? people. It can occur in men and women. The most
a. Echopraxia common ages for it first to develop are 15-25 in
men and 25-35 in women.
b. Echolalia
Presentation:
c. Perseveration • Delusions
d. Apraxia • Hallucinations.
e. Anosognosia • Disordered thoughts.

1653. A pt has loss of sensation on the tip


Key:A
of her tongue and the inner aspect of the
Ans)Echopraxia is the involuntary repetition or lip. Which nerve is most likely to be
imitation of another person's actions. Similar to involved?
echolalia, which is the involuntary repetition of a. Vagus nerve
sounds and language. Echopraxia has long been b. Glossopharyngeal nerve
recognized as a core feature of Tourette
c. Lingual nerve
syndrome, and is considered a complex tic, but it
also occurs in autism spectrum disorders, d. Buccal nerve
schizophrenia and catatonia.Apraxia is a motor e. Facial nerve
disorder caused by damage to the brain
(specifically the posterior parietal cortex), in Key is C: Lingual Nerve
which someone has difficulty with the motor a. Vagus nerve: It leaves the skull through the
planning to perform tasks or movements when jugular foramen, passes within the carotid sheath
asked, provided that the request or command is in the neck, through the thorax supplying the
understood and he/she is willing to perform the lungs, and continues on via the oesophageal
task. Apraxia is an acquired disorder of motor opening to supply the abdominal organs.
planning, but is not caused by incoordination, b. Glossopharyngeal nerve: Passes across the
sensory loss, or failure to comprehend simple posterior fossa, through the jugular foramen and
commands (which can be tested by asking the into the neck, supplying tonsil, palate and
person to recognize the correct movement from a posterior third of tongue.
series)Anosognosia is a deficit of self-awareness,
a condition in which a person who suffers a c. Lingual nerve: lingual nerve is a branch of the
certain disability seems unaware of the existence mandibular division of the trigeminal nerve (CN
of his or her disability. V3), which supplies sensory innervation to
thetongue. It also carries fibers from the facial
Key is A: Echopraxia nerve, which return taste information from the
anterior two thirds of the tongue, via thechorda
Echopraxia is involuntary imitation of the tympani
movements and is a feature for the diagnosis of
schizophrenia. d. Buccal nerve:brach of mandibular division of
trigeminal nerve, supplies the skin and mucous
Schizophrenia: membrane of the cheek
• Schizophrenia is a serious mental health
e. Facial nerve: Mainly motor (some sensory
condition that causes disordered ideas, beliefs
fibres from external acoustic meatus, fibres
and experiences. In a sense, people with
controlling salivation and taste fibres from the
schizophrenia lose touch with reality and do not
anterior tongue).
know which thoughts and experiences are true
and real and which are not.
1654. A 51yo woman complains of may develop within seconds or minutes of
difficulty swallowing and also reddish dots contact with the trigger factor and is potentially
fatal.
on her skin. A pic of her hand is seen.
• Possible triggers can include skin or
What is the most appropriate term for the airborne contact with particular materials, the
condition you would expect to see? injection of a specific drug, the sting of a certain
a. Sclerodactyly insect or the ingestion of a food such as peanuts
b. RA
c. Swan neck deformity
1656. A 65yo pt who had MI 1yr ago now
d. Polydactyly
comes to the ED complaining that his
e. Ulnar deformity
neighbor is conspiring against him. When
Key is A: sclerodactyly his son is asked, he denies it and also
narrates that sometimes his father says
Patient is suffering from crest syndrome,
sclerodactyly its one of the features.
that everybody in his office is always
talking about him, which is not the case.
(page 554 OHCM) What is the most appropriate med?
Limited systemic sclerosis(CREST syndrome is a a. TCA
part of which) Calcinosis(sub cu tissue), Raynauds b. Clozapine
phenomenon, esophageal and gut dysmotility, c. Olanzapine
Sclerodactyly and telengectesia. Skin involvement d. Lorazepam
is limited to the face, hand and feet. It is
Key is C: Olanzapine
associated with anti-centromere antibodies in 70-
80%. Pulmonary hypertension is often present TCA is not given at this age
subclinically and become rapid life threatening.
Clozapine needs strict monitoring and can only be
given under the supervision of a psychiatrist

1655. A 37yo female working as a Olanzapaine is antipsychotic which is most


healthcare assistant in a nursing home suitable in this case
comes to the ED with complaints of severe
1657. You suspect Cushing's disease in a
itching all over her body. On asking she
50yo woman who has attended clinic with
replies that she had applied cream on the
glycosuria, HTN and a suggestive body
body of a resident in the nursing home
habitus. Initial inv point you towards a dx
who had similar itches. What is the
of Cushing's disease. Which of the
mechanism of itching?
a. Allergic reaction
following findings would be against this
b. Inflammation of keratinocytes
dx?
a. A normal 8am cortisol
c. Allergic reaction developed due to use of
b. Failure to suppress morning cortisol with
topical steroid creams
dexamethasone
d. Subcutaneous bleeding
c. HTN requiring >2 antihypertensive agents
e. None
d. Impaired growth hormone response to
glucose loading
e. Unilateral adrenal enlargement
Key is A: allergic reaction
Key is E: Unilateral adrenal enlargement
• A severe allergic reaction (anaphylaxis) will
affect the whole body; in susceptible individuals it
Cushing’s disease • The presentation: is variable and includes
dyspnoea (the most common presenting
Bilateral adrenal hyperplasia from an ACTH symptom), chest pain, palpitations and syncope.
secreting pituitary adenoma. • Examination: Classic examination findings
are a forceful apex beat, with double
Peak age 30-50 years, male female ratio 1:1
impulse(pulsusbisferiens) if the left ventricular
A low dose dexamethasone test leads to no outflow tract is obstructed and a late ejection
change in plasma cortisol but 8 mg may be systolic murmur, which can be augmented by
enough standing or Valsalvamanoeuvre and diminished
by squatting.[2]
1658. Which finding, on clinical • Investigation: Electrocardiogram (ECG):
examination of the pulse, suggests a most patients have an abnormal ECG, although
electrocardiographic features are nonspecific and
diagnosis of hypertrophic obstructive
include LVH, ST segment changes and T-wave
cardiomyopathy (HOCM)? inversion.
a. Irregularly irregular pulse suggesting A-fib
1659. A 60yo male is admitted with a 2d
b. Pulsusalternans
hx of lower abdominal pain and marked
c. Pulsusbigeminus
vomiting. On examination he has
d. Pulsusbisferiens
abdominal swelling, guarding and
e. Pulsusparadoxus
numerous audible bowel sounds. What is
Key is D Pulsusbisferiens the likely dx?
a. Gallstone ileus
pulsusbisferiens, is a sign where, onpalpation of
b. Ischemic colitis
the pulse, a double peak per cardiac cycle can be
c. Large bowel obstruction
appreciated. Bisferious means striking twice.
d. Sigmoid volvulus
Classically, it is detected when aortic insufficiency
e. Small bowel obstruction
exists in association with aortic stenosis,[1] but
may also be found hypertrophic obstructive Key is D: sigmoid volvulus
cardiomyopathy.
History is not suggestive of ischemic colitis.
Hypertrophic Cardiomyopathy:
There are no bowel sounds heard in gallstone
• Hypertrophic cardiomyopathy (HCM) is an ileus, large bowel obstruction and small bowel
autosomal dominant genetic disorder
obstruction, hence it is sigmoid volvulus.
characterised by left ventricular hypertrophy
(LVH), impaired diastolic filling, and abnormalities Sigmoid Volvulus:
of the mitral valve. These features can cause
dynamic obstruction of the left ventricular In sigmoid volvulus, a large sigmoid loop full of
outflow tract, diastolic dysfunction, myocardial faeces and distended with gas twists on its
ischaemia, and an increased risk of mesenteric pedicle to create a closed-loop
supraventricular and ventricular obstruction.
tachyarrhythmias.
• HCM is the most common genetic Risk factors:
cardiovascular disease. • The elderly.
• Epidemiology:The prevalence of HCM is • Chronic constipation.
about one in 500 and it tends to affect men and • Megacolon, large redundant sigmoid colon
black people more often. The obstructive form is and excessively mobile colon.[2]
seen in 25% of cases. • It is more common in men.
• Hypertrophy can occur in any part of the
left ventricle, although it is most common in the
anterior ventricular septum.
Presentation: • Scabies is common. In the UK, about 1 in
• lower abdominal pain associated with gross 1,000 people develop scabies each month.
abdominal distension and a failure to pass either Scabies is more common in town (urban) areas, in
flatus or stool. women and children, in the winter, and in the
• Abdominal examination reveals a North of the country.
tympanitic, distended (but usually non-tender) • The skin-to-skin contact needs to be for a
abdomen and a palpable mass may be present. reasonable time to catch the mite. You usually
Investigation and Management: need to be in skin contact for 15-20 minutes to
• Characterstic Abdominal X-Ray with an catch scabies.
inverted U loop of bowel that looks a bit like a • Scabies symptoms usually take 2-6 weeks
coffee bean. to occur after you are first
• Often managed by sigmoidoscopy and Treatment:
insertion of a flatus tube. Sigmoid colectomy is Scabies can stay in your skin for ever if not
sometimes required treated. Treatment is needed for:
1660. A 17-year-old boy is diagnosed with • Anybody who has scabies; AND
scabies. Which of the following • All household members, close contacts,
and sleeping/sexual partners of the affected
statements regarding scabies is correct?
person - even if they have no symptoms. This is
a. Is best treated by salicylate emulsion
because it can take up to six weeks to develop
b. It can be spread by a droplet infection symptoms after you become infected. Close
c. It causes itchiness in the skin even where contacts may be infected, but have no symptoms,
there is no obvious lesion to be seen and may pass on the mite.
d. It is caused by Staphylococcus aureus • Apply 5% permethrin over whole body
e. Typically affects the face including scalp, face (avoid eyes), neck and ears.
Do not forget the soles; wash off after 8-12 h,
Key is C: It causes itchiness in the skin even where repeat after 7 days.
there is no obvious lesion to be seen

Excluded Points:

a. The usual scabies treatment is with permethrin


1661. An anemic young man is found to
cream. Permethrin is an insecticide that kills the
mites have a macrocytosis of 90%. The most
likely cause is?
b. You need close skin-to-skin contact with an a. Zieve’s syndrome
infected person to catch scabies. This is because b. Thalassemia minor
the scabies mite cannot jump or fly c. Chronic renal disease
d. IDA
d. Scabies is caused by a mite (like a tiny insect) e. Folate def
called Sarcoptesscabiei. The mite is a parasite, f. Chronic liver disease
meaning it lives off the host (a human) with no g. HUS
benefit to the host h. Cytotoxic chemotherapy
i. Phenytoin
e. Itching is often severe and tends to be in one
place at first (often the hands)
Ans: Floate Def
Scabies:

• Scabies is caused by a mite (like a tiny


insect) called Sarcoptesscabiei. The mite is a Zieve's syndrome is an acute metabolic condition
parasite, meaning it lives off the host (a human) that can occur during withdrawal from prolonged
with no benefit to the host. alcohol abuse. It is defined byhemolytic anemia
(with spur cells and acanthocytes),
hyperlipoproteinaemia (excessive blood b. E.coli
lipoprotein), jaundice, and abdominal pain.[1] c. Bacteroides fragilis
The underlying cause is liver delipidization d. Staphylococcus aureus
e. Staphylococcus epidermis
f. C.perfringens
g. Pseudomonas aeruginosa
1662. An association with HPV is a most h. Streptococcus fecalis
characteristic feature of? i. Streptococcus pneumonia
a. Torus j. Brucella melitensis
b. Exotosis
c. Pleomorphic adenoma
d. Verruca vulgaris Ans: Staph epidermidis is most common cause of
e. Fibroma infections in prosthesis.
f. Epulis fissuratum
g. Mucocele
h. Pyogenic granuloma 1664. A primigravida in the 17th week of
i. Parulis her symptomless gestation is found, on
j. Ranula US, to have evidence of placental tissue
covering the cervical os. By the end of her
Ans.D, it is most commonly associated with warts pregnancy she is likely to develop?
or verruca vulgaris. a. Placental migration
b. Uterine myoma
c. Uterine rupture
d. Choriocarcinoma
Exotosis: benign outgrowth cartilaginous tissue e. Chorangioma
Plemorphic Adenoma: A Locally invasive benign f. Vasa previa
tumor g. Subplacental abruption
Fibroma: benign tumors composed of fibrous h. placenta
tissue i. Subchorionic abruption
Epulis Fissuratum : benign hyperplasia of fibrous j. placenta
connective tissue which develops as a reactive k. Placenta accrete
lesion to chronic mechanical irritation produced l. Placenta previa
by the flange of a poorly fitting denture
Mucocele: a benign swelling containing mucin.
pyogenic granuloma: Pyogenic granulomata are Ans: A
common benign vascular lesions of the skin and
mucosa.a reactive inflammatory mass of blood In 90% of pregnancies, an initial low lying
vessels and a few fibroblasts within the dermis of placenta will be pulled upwards by the growing
the skin. uterus and assume a normal position in the upper
Parulis: an elevated nodule at the site of a fistula segment. This phenomenon is referred to as
draining a chronic periapical ABSCESS Migration
Ranula : A ranula is a type of mucocele found on
the floor of the mouth 1665. An elderly lady with COPD has
chronic SOB. She is listed for cataract
extraction. What is the anaesthetic of
1663. For the following type of surgery
choice?
what is the most likely agent that may a. Facial nerve block
cause post-operative infection -- aorto- b. Bupivacaine infiltration of the peri-orbital
iliofemoral reconstruction with a Dacron skin
vascular prosthesis? c. IV midazolam
a. Proteus d. Peribulbar acupuncture
e. Peribulbar lignocaine He admits to a moderate regular
f. infiltration consumption of alcohol. Exam: noted to
g. Topical xylocaine
have rhinophyma. The most likely dx is?
h. IV alfentanil
a. Eczema
i. Epidural anesthesia
b. Herpes simplex
j. General anesthesia
c. Epidermolysis bullosa
k. Retrobulbar xylocaine Inj
d. Dermatomyositis
e. Tinea versicolor
Ans:E
f. Pemphigus vulgaris
The most used mode of anaesthesia in
g. Acne rosacea
ophthalmic feild is peribulbar using lingocaine
h. Malignant melanoma
i. Psoriasis
1666. A 55yo chronic alcoholic with known j. Atopic dermatitis
hepatic cirrhosis has been on a heavy bout
of alcohol the night before and was
Ans: Acne Rosacea
brought home by friends after falling
several times in the pub. While being Reason: Acne Rosacea is characterised by
taken up the stairs to his bedroom he falls recurrent episodes of facial flushing with
down the flight of 5 steps but sustains no persistent erythema, telangiectasia, papules and
pustules.It is a chronic acneform disorder of the
obvious injury. His wife calls the ED the
facial pilosebaceous glands with an increased
next day because she could not rouse him reactivity of capillaries to heat, causing flushing
in the morning. He is brought in a and eventually telangiectasia.Rhinophyma is an
comatose state and both pupils appear enlarged nose associated with rosacea which
dilated. Skull vault XR appears normal. occurs almost exclusively in men.
a. Hepatic encephalopathy
b. Intracerebral hematoma Management: Reassurance, benign disease,
c. Brain stem injury
sunscreens, facial massage reduces oedema.avoid
d. Extradural hematoma
topical steroids, avoid astringents.
e. Chronic subdural hemorrhage
f. Despressed skull fx Topical .75% MTZ firstline.
g. Vertibrobasilar ischemia Azaleic Acid 15% gel.
h. Acute subdural hematoma Oral oxytetracyline, doxycyline or erythromycin.
i. SAH 1% Ivermection cream (better than MTZ)
j. Severe migraine attack Topical Brimonidine for flushing

ANS: H 1668. A 60yo man who presented with


metastatic adenocarcinoma of unknown
Acute subdural hemotoma, typical history of source. He developed rapidly progressive
alcholics, falls and usually debilliated or elderly,
weakness of his arms and was found to
hepatic cirrhosis increases coagulopathy and
have a deposit of tumour in his cervical
chances for bleed.
spine. This was emergently treated with
radiation. He developed considerable
1667. A 58yo man complains of nose nausea and vomiting during his therapy
disfigurement. He has a hx of facial and at the end of the course began to
erythema particularly of the cheeks and have bloody vomiting. Following
nose. Papules and pustules have been rescusitation with 6 units of blood, what is
erupting at intervals over the last 10yrs. the next test of choice?
a. Apt test Leukonychia: areas of white nail plate due to foci
b. Neck, chest, abdominal XR of parakeratosis within the body of the nail plate.
c. 24h esophageal pH probe test
d. CT abdomen
e. US abdomen
f. MRI abdomen 1670. A young man develops nonfluent,
g. Barium swallow
effortful speech with dysarthria. He is able
h. Angiography
i. Nuclear scan to understand speech. He fails to repeat
j. Endoscopy the sentence. What would you do next?
a. XR skull
b. Non-contrast CT brain
Ans: Endoscopy
c. Contrast CT brain
Reason: whenever there hemetemesis endoscopy d. Contrast MRI optic nerves
should be carried out immediately if the patients e. 4-vessel cerebral angiogram
condition allows or it should be delayed till f. Single vessel cerebral
resucitation ,Underlying cause for hemetemesis g. angiogram
needs to be sorted out.( High dose radiation is a h. Cerebral angiography
cause Ulceration and any active bleeders must be i. MRI frontal lobe
treated) j. MRI pituitary gland
k. MRI temporal lobe
1669. A pt has fine nail pitting, small
yellow-brown areas of discoloration in the
nail bed involving the nails on both hands. Ans: MRI Frontal lobe (Brocas area)
These findings are commonly associated
with?
a. Yellow nail syndrome Production (Broca's) dysphasia/aphasia - lesions
b. Leukonychia are located in the left pre-central areas. This is a
c. Onychomycosis non-fluent or expressive aphasia since there are
d. Lichen planus deficits in speech production, prosody and
syntactic comprehension. Patients will typically
e. Pellagra
exhibit slow and halting speech but with good
f. Thallium toxicity
semantic content. Comprehension is usually
g. Contact dermatitis
good. Unlike Wernicke's aphasia, Broca's patients
h. Zinc deficiency
are aware of their language difficulties. Prosody is
i. Hypoalbuminemia the study of the meter of verse. Here it means
j. Psoriasis the rhythm of speech

Ans: Psoriasis Sensory (Wernicke's) dysphasia/aphasia - lesions


are located in the left posterior perisylvian region
Characteristic nail changes include pitting,
and primary symptoms are general
discolouration ,subungual hyperkeratosis,
comprehension deficits, word retrieval deficits
crumbling of the nail plate, and onycholysis.
and semantic paraphasias. Lesions in this area
Oil drop or salmon patch: translucent yellow-red damage the semantic content of language while
discoloration in the nail bed. leaving the language production function intact.
The consequence is a fluent or receptive aphasia
in which speech is fluent but lacking in content. i. Oral contraceptives
Patients lack awareness of their speech j. Megaloblastic anemia
difficulties. Semantics is the meaning of words.
Semantic paraphrasia is the substitution of a
H: Iron deficiency Anemia
semantically related but incorrect word.
Reason: S/S pallor, koilonychia,angular cheilitis,
atrophic glossitis, IN marked Anemia ( Cardiac
enlargemnet,Flow Murmurs,ankle oedema and
heart failure)

Inv: FBC : shows microcytic hypochromic anemia

1671. A pt being sedated with fentanyl Serum ferrtitin Level reduced, normal 12-15
develops severe respiratory depression. mcg/L, ( serum ferritin is falsely raised during
This is best reversed using? infections)
a. Ethanol Anisocytosis and poikilocytosis
b. Naloxone
c. Phyostigmine Total iron binding capacity is increased
d. Atropine
e. Methylene blue
f. Diphenhydramine Treatment: Iron supplememtation with B12 and
g. Calcium disodium ethylene
folic acid
h. diamine tetra-acetic acid
i. Deferoxamine mesylate
j. Flumazenil
k. Folic acid 1673. A 20yo prv healthy woman presents
with general malaise, severe cough and
Ans: Naloxone breathlessness which has not improved
with a seven day course of amoxycillin.
Opioid Antagonist, reverses the effects of There is nothing significant to find on
fentanyl, though it has to be administeres for a examination. The x-ray shows patchy
longer period of time due long half life of
shadowing throughout the lung fields. The
fentanyl.
blood film shows clumping of red cells
with suggestion of cold agglutinins.
a. Mycobacterium avium complex
1672. A pt presented with the following b. Coxiella burnetii
blood work, MCV: Decreased c. Escherichia coli (Gram -ve)
Serum ferritin: Decreased Total ,iron d. Haemophilus influenza
binding capacity: Increased ,Serum iron: e. Legionella pneumophila
Decreased f. Strep pneumococcus
g. TB
Marrow iron: Absent. What is your dx? h. Mycoplasma pneumonia
a. Thalassemia trait i. PCP
b. Hypoparathyroidism j. Staph aureus
c. Hereditary sideroblastic anemia
d. Protein energy malnutrition
e. Chronic renal failure Ans: Mycoplasma pneumonia
f. Anemia of chronic disease
g. Acute blood loss Reason : inablity to respond to a seven day course
h. IDA of amoxicillin suggests atypical pneumonia,
patchy shadows throughout lung fields and cold symptoms. After a few wks, each of the 7
agglutination points towards mycoplasma. people completely recovers and they
replace their caterer. What is a likely dx?
a. Pancreatic ca b. Hemochromatosis
M. pneumoniae:[9] b. Laennec’s cirrhosis
c. Hep A
Vague and slow-onset history over a few days or
d. HCC
weeks of constitutional upset, fever, headache, e. Rotor’s syndrome
dry cough with tracheitic ± pleuritic pain, myalgia, f. Primary biliary cirrhosis
malaise and sore throat. g. Gilbert’s syndrome
h. Hep B
This is like many of the common viral illnesses but
i. Hemolysis
the persistence and progression of symptoms is
what helps to mark it out.
Ans: Hepatitis A
In otherwise healthy individuals, it usually
resolves spontaneously over a few weeks.

The hacking, dry cough can be very persistent. Symptoms of Hepatitis A range from mild nauseas
to liver failure (very rare).Spread is normally by
Extra-respiratory features include rashes such as
the faecal-oral route although there are
erythema multiforme, erythema nodosum and
occasional outbreaks through food sources.Hand
urticaria; neurological complications like Guillain-
washing and good hygiene around food and drink
Barré syndrome, transverse myelitis, cerebellar
prevent spread of infection.Increasing age is a
ataxia and aseptic meningitis; haematological
direct determinant of disease severity.
complications such as cold agglutinin disease and
haemolytic anaemia; joint symptoms like
arthralgia and arthritis; cardiac complications
S/S The incubation period is 2-6 weeks with a
such as pericarditis and myocarditis; rarely, may
mean of 4 weeks.
cause pancreatitis
There is a prodrome of mild flu-like symptoms
TReatment: Macrolides, Fluroquinolones
(anorexia, nausea, fatigue, malaise and joint pain)
preceding the jaundice. Smokers often lose their
taste for tobacco. Diarrhoea can occur,
particularly in children.

This can progress to the icteric phase with:Dark


urine (appears first).Pale stools (not always).
Jaundice occurring in 70-85% of adults with acute
HAV infection. Abdominal pain occurring in 40%
1674. An 18yo male works in a company of patients. Itch or pruritus. Arthralgias and skin
where lunches are often catered. One day, rash.Tender hepatomegaly, splenomegaly, and
lymphadenopathy
the water at the company facility is not
working, but they manage to have the Inv:IgM antibody to HAV is positive with onset of
lunch anyway. 2wks later, he becomes symptoms (usually about 3 to 4 weeks after
sick. He develops anorexia, nausea, exposure but up to 6 weeks). The test is sensitive
malaise and jaundice. During the course and specific. It remains positive for between 3
of the next 4wks, 7 people who shared in and 6 months (up to 12 months). It remains
positive in relapsing hepatitis.
the lunch become ill with similar
IgG antibody to HAV appears soon after IgM and e. one and a half years
persists for many years. In the absence of IgM it f. two years
indicates past infection or vaccination rather than g. four years
acute infection. IgG remains detectable for life h. seven years
LFTs i. ten years
j. fourteen year
Mangement: Supportive, avoid alcohol.

Ans:6 months

Reason : holding head and raising chest upto 90


degrees from a surface while prone (4months)

1675. A 35yo 1st time donor suddenly social smile 6 weeks, becomes increasingly
socially responsive.
passes out as she is donating blood.
Which of the following steps would be palmer and rooting reflexes are primitive reflexes
least useful in managing this adverse
event?
1677. A mother is concerned because her
a. Ensure donor is adequately hydrated and
1m boy has a swelling in his scrotum. He
has not skipped a meal
was born prematurely. On examination
b. Elevating the donor's legs as this is
usually due to a vasovagal syncope
the swelling is seen to transilluminate. The
c. Haemoglobin of the donor meets the
likely cause is?
a. Lymphogranuloma Venereum
minimum requirement for donation
b. Testicular Torsion
d. The donation is usually continued along c. Hydrocele
with simultaneous normal saline infusion d. Epididymitis
e. The donor should be encouraged to e. Seminoma
mobilise after they have recovered f. Mature teratoma
g. Varicocele
h. Lymphoma
i. Orchitis
Ans: c j. Spermatocele
Option c has no role after such an event has
Ans: Hydrocele
occured.
Reason: 1-2% neonates present with congenital
hydrocele which disappears by 1-2 years.
1676. An infant is being examined as part
of a routine examination. The child can
hold its head up and lifts its chest off a Spermatoceles:Smooth, extratesticular, spherical
table. He has a palmer and rooting reflex cysts in the head of the epididymis are not
as well as a social smile. He is not afraid of uncommon in adult men. They are benign and do
not usually require treatment.Epididymal cysts
strangers. What is the most likely age of
usually develop in adults around the age of 40.
this child?
Epididymal cysts are rare in children and when
a. neonate
they occur, usually present around puberty.
b. 2 months
c. 6 months
d. one year
1678. A 2m girl has an ante-natal through the first year of life, and their parents
diagnosis of right hydronephrosis. counseled regarding the risk of urinary tract
infections. The management of patients with
Postnatal serial US exams revealed
pelviureteric junction or vesicoureteric junction
increasing dilatation of the right obstruction depends on clinical features and
pelvicalyceal system. No reflux was results of sequential ultrasonography and
demonstrated on a MUCG. Appropriate radionuclide renography. Surgery is considered in
management should include? patients with increasing renal pelvic APD and/or
a. Surgical repair an obstructed renogram with differential renal
b. Intermittent catheterization function <35-40% or its subsequent decline.
c. Diuresis renography Further studies are necessary to clarify the role of
d. Anticholinergic agents prenatal intervention, frequency of follow up
e. Phenylpropanolamine investigations and indications for surgery in these
f. Gellhorn pessary patients.
g. Biofeedback-assisted behavioral 1679. Jean is a 72yo woman with
h. treatment recurrent bowel cancer following a hemi-
i. Oral Estrogen therapy colectomy 2y ago. She is known to have
j. Vaginal Estrogen therapy
both local recurrence and liver mets and
k. Ring pessary
her pain has been under control on MST
90mg bd. She has had quite severe pain in
Ans : c. Diuresis renography the RUQ for the past hour despite having
taken her normal dose of MST. You find
that she has an enlarged liver which is
hard and irregular. There is marked
Understanding the pathophysiology of prenatally localised tenderness over the right lobe of
diagnosed RPD allows different interpretations of her liver. Her abdomen is otherwise soft
the diuretic renogram and may affect the and non-tender and the bowel sounds are
treatment of these children. Bju 2001 normal. She is apyrexial. The tx of choice
Revised Guidelines on Management of Antenatal would be?
Hydronephrosis a. Oral NSAIDs
b. TENS
Widespread antenatal screening has resulted in c. radio therapy to the liver
increased detection of anomalies of the kidneys d. IM diamorphine
and urinary tract. The present guidelines update e. Paracetamol
the recommendations published in 2000. f. Prednisolone
Antenatal hydronephrosis (ANH) is transient and g. Physiotherapy
resolves by the third trimester in almost one-half h. epidural anaesthetic
cases. The presence of oligohydramnios and i. Pitocin
additional renal or extrarenal anomalies suggests j. Aspirin
significant pathology. All patients with ANH
should undergo postnatal ultrasonography; the
intensity of subsequent evaluation depends on Ans: I/M diamorphine . Pain ladder
anteroposterior diameter (APD) of the renal
pelvis and/or Society for Fetal Urology (SFU) 1680. Titubation is a feature of disease
grading. Patients with postnatal APD exceeding involving the?
10 mm and/or SFU grade 3-4 should be screened a. Cerebellum
for upper or lower urinary tract obstruction and b. Basal ganglia
vesicoureteric reflux. Infants with vesicoureteric c. Corpus callosum
reflux should receive antibiotic prophylaxis d. Pons
e. Temporal lobe Investigations: Hepatitis B surface antigen is
f. Occipital lobe positive in 30%.
g. Optic chiasma
h. 3rd ventricle • The p-ANCA test is usually negative in
i. Hypothalamus PAN.
• There is a prominent acute phase
response but this is nonspecific.
Ans: Cerebellum • FBC shows leukocytosis with raised
neutrophils.
titubations (head/body nodding) mostly occur • Hypergammaglobulinemia occurs in 30%.
due to cerebellar lesions. • Biopsy
Arteriography shows aneurysms
1681. A 50yo farmer complains of pain in
his left arm. Exam: he appears to have a
neuropathy affecting isolated nerves in TREATMENT: Corticosteroids..RELAPSE add
multiple, random areas of his left arm. He Cyclophosphamide.=>
also has a palpable purpura and tender
Azathioprine useful in maintenance therapy.
nodules on both of his upper and lower
limbs. A likely diagnosis is? (IV-Ig) and aspirin are effective in childhood PAN.
a. Carpal tunnel syndrome
b. Polyarteritis nodosa
c. Angina Pectoris 1682. A patient with chronic neutropenia
d. Gout
e. Cellulitis develops a chronic cough. A CXR reveals a
f. Rheumatoid arthritis cavitating intrapulmonary lesion
g. Erysipelas containing a movable rounded ball lesion.
h. Fascitis A likely dx is?
i. Reiter's Syndrome a. Tuberculosis
b. Bronchiectasis
Ans: PAN c. Cystic fibrosis
d. Pulmonary hemosiderosis
e. Mitral stenosis
PAN is necrotising arteritis of medium or small f. Aspergillosis
arteries without glomerulonephritis or vasculitis g. Wegener’s granulomatosis
in arterioles, capillaries, or venules, and not h. Goodpasture’s syndrome
i. Pulmonary embolism
associated with antineutrophil cytoplasmic
j. Non-SCLC
antibodies (ANCAs).It can affect any organ but,
for unknown reasons, it spares the pulmonary
and glomerular arteries. Ans: Aspergillosis.

Presentaion:Peripheral nerves and skin are the mostly affects people with reduced immunity,
most frequently affected tissues. reduced neutrophil count is also predilection for
PURPURA,LIVEDOID,SUBCUTANEOUS NODULES aspergillosis. five clinical types of Aspergillosis
and NECROTIC ULCERS. Neurologically,
APBA
MONONEURITIS MULTIPLEX>...involvemnet of
CNS,Git,kidneys and heart means higher Severe Asthma with fungal sensitization
mortality.RENAL
INVOLVEMENT:hypertension,AKI, Aspergilloma : (fungal ball in already caivitated
GIT:necrosis,perforation.Myalgia space due to TB, Sarcoidosis)
Invasive aspergillosis A rough barking cough with croup

Chronic Necrotizing Pulmonary Aspergillosis. hoarsness and wheezing

labored breathing

Aspergilloma usually presents with massive runny nose, fever, cough


hemoptysis, cough and fever are rare, Discovered
decreased appetitie, diarrhea.
occasionally in asymptomatic patiets on xray
showing a mass in upper lobe surrounded by air
border. CT demonstrated fungal structure more
accurately. Investigation: Viral culture of secretions.

Surgical removal Treatment: Symptoatic supportive treatment

Long term itraconazole therapy Antibiotics incase of secondary infection.

Instillation of amphotericin
1684. INR:Normal, APTT:Elevated,
hemoptysis needs to be treated with bronchial Thrombin time:Elevated, Plt
artery embolization. count:Normal, Bleeding time: Normal. a
likely aetiology is?
a. Waldenström's
1683. A mother brings her 1yo infant to b. macroglobulinaemia
her pediatrician. She describes that c. Heparin
following a common cold her child's voice d. Sézary cell leukaemia
e. Pelger-Hüet anomaly
has become hoarse and has developed a
f. von Willebrand's disease
cough that sounds harsh and brassy and g. Haemophilia
was worse at night. Exam: the child was h. HIV infection
noted to have trouble drawing air into its i. DIC
lungs between coughs and had trouble j. Acanthocytosis
drawing air into its lungs. There was k. Vit K deficiency
visible stridor on inhalation. The cause is
most likely to be? Ans: Heparin
a. EBV
b. Rhinovirus
c. Parainfluenza PT-test for extrinsic system
d. Flavivirus INR- 0.9-1.2 (PT Control), Increased INR in
e. HIV warfarin , Vitamin K and liver disease
f. Rotavirus
g. CMV APTT- intrinsic system, increased PTT (Heparin,
h. Kemerovo Haemophillia (Factor 8 affected))
i. Creutzfeld-Jacob
j. Rubella Thrombin Time- 10-15 seconds, increased in
heparin, increased in DIC

Ans: Parainfuenza Bleeding Time (normal 7 min) - commonest cause


Von willbrand disease
Parainfluenza virus causes several respiratory
problms, laryngitis, bronchitis, pneumonia and
bronchiolitis,
1685. An infant has diarrhea for 3d with intravenous isotonic fluid boluses (20-60 mL/kg)
weight loss from 10 kg to 9 kg. Exam: he is until perfusion improves.
noted to have dry mucous membranes,
poor skin turgor, markedly decreased
EXAMPLES OF ISOTONIC SOLUTIONS
urine output, and tachycardia. His
BP=normal and compression-release of • 0.9% Saline (aka Normal Saline)
the nail beds shows satisfactory refilling. • Ringer’s Solution
Appropriate treatment would include? • Lactated Ringers
a. Plasmapheresis and plasma infusion • 5% Dextrose in 0.225% Saline
b. 0.5% Normal Saline • 5% Dextrose in Water (technically,
c. Lactated Ringer's injection Isotonic, but physiology hypotonic
d. Packed cells
e. Whole blood
1686. A 4yo boy has the sudden onset of
f. Platelets
g. FFP bone pain. He begins experiencing
h. double strength Normal Saline bleeding of his gums and frequent bloody
i. 5% dextrose in 0.5N saline noses. His mother takes him to his
j. solution pediatrician. Exam: he is pale and has
k. IV heparin numerous petechiae over his body, with
lymphadenopathy and
ANS: C hepatosplenomegaly. He has
1-5% body weight loos Mild dehydration
WBC=100,000mm and numerous
circulating blast cells. He is admitted to
skin turgor decreassed, mucous membrane dry, the hospital. A bone marrow biopsy=35%
urine low, HR raissed, BP normal,perfusion blast cells. Which of the following is most
normal,skin colour pale,irritable
likely?
GIVE ORS a. Mantle cell lymphoma
b. Infectious lymphocytosis
c. Waldenstrom’s
d. macroglobulinemia
5-10% body weight loss Moderate Dehydration
e. CML
skin turgor decreased,mucous menbrane very f. CLL
dry,urine oligouric,HR raised,BP normal,Perfusion g. Burkitt lymphoma
prolonged capilllary refill,skin colour grey, h. ALL
i. Mycosis fungoides
lethargic,
j. Hairy cell leukemia
GIVE ORS 50-100 mL/kg body weight over 2-4 k. AML
hours,again starting with 5 mL every 5 minutes. if AML --THIS IS AN ACUTE EMERGENCY IN AML.
ors fails then give IV Bolus of 20 ml/kg Isotonic Leukostatsis causing bone pain. Other s/s are
solution. suggestive of AML short Hx, wbc 100000,
petechiae, with lymphadenopathy, very high blast
0 cell count and hepatosplenomegaly.
10-105% body weight loss Severe Dehydration

skin turgor poor,m ucous membrane


1687. A 63yo male has anal canal
parched,urine anuric,HR raised, Bp Decreased,
carcinoma with no evidence of spread to
perfusion prolonged , skin colour mottledd,
Comatose.
the pelvic wall, pelvic muscles or lymph g. Deficiency of folate
nodes. This is typically managed by? h. Bone marrow failure
a. Resection of the sigmoid colon i. Inability to manufacture heme
b. Right hemicolectomy
c. Left hemicolectomy Ans : b. Failure of beta chain production
d. Transverse colectomy
e. Internal sphincterotomy  1689. A 30yo caucasian man presented
f. CT guided drainage with a 2wk hx of gradually worsening
g. Diverticulectomy
h. Transverse colostomy vision in his left eye. The patient had been
i. Chemotherapy and seen once by a neurologist 2yrs prv for
j. radiatherapy flashes. At that time a head CT was
k. Abdominal perineal resection
normal. The patient was lost to follow up
with the neurologist, but the flashes had
this is T1 N0 M0 that is stage 1 cancer. continued for the 2yr period. The patient
(http://www.cancer.gov/types/anal/hp/anal-
did not experience visual changes with
treatment-pdq#section/_45).
activity or movement. The patient
reported continued decreasing vision.
BUT we don’t have tumor size here and sphincter Goldmann visual fields were done and
information. However surgery can be avoided To showed a central scotoma. A MRI was
preserve sphincter. Chemo radiation is preferred done at this time and showed
ohcm 633; 9th ed
inflammation of the left optic nerve. A
likely diagnosis is?
http://www.cancer.gov/types/anal/hp/anal- a. Pseudotumor
treatment-pdq#section/_45 Continued b. Orbital teratoma
surveillance with rectal examination every 3 c. Optic neuritis
months for the first 2 years and endoscopy with d. Sarcoidosis
biopsy when indicated after completion of e. Optic glioma
sphincter-preserving therapy is important. f. Lymphangioma
g. Rhabdomyosarcoma
h. Retinal vascular shunts
i. Retinoblastoma
1688. A 2m baby develops a life- j. Mucormycosis
threatening anemia. Blood tests show a
normal serum iron, ferritin and TIBC.
Hemoglobin electrophoresis reveals a
markedly decreased Hemoglobin A Ans : c. Optic neuritis possibly due to MS.
content and an increased hemoglobin F Scotoma, 2yr Hx, age race are suggestive except
content. This baby's anemia is likely to be more in females than males
secondary to?
a. Failure of alpha chain production
b. Failure of beta chain production
c. Deficiency d. Lead poisoning of B12 1690. A pregnant woman in an early stage
d. IDA of labour expresses the wish to have pain
e. Presence of hemoglobin S relief during labour. The anesthetist
f. Presence of hemoglobin M describes that if the patient wishes he can
use medication as a local anesthetic to 1692. A 22yo has had recent chickenpox.
block the pain sensations of labour. Into He now presents with confusion. He is
which space should the local anaesthetic noted to have low urine output and large
be normally injected? petechiae all over his body. CXR: a large
a. Anterior pararenal space patch of consolidation is seen. The
b. Aryepiglottic space management of choice should be :
c. Vestibule space a. Ventilatory support
d. Epidural space b. Open surgical debridement
e. Sub-arachnoid space c. Resection of superficial
f. Space of Disse d. petechiae with wide margin
g. Middle ear e. Booster vaccine
h. Posterior pararenal space f. TENS
i. Supraglottic space j. Lesser sac g. Lontophoresis
h. Nephrostomy
Ans : d. Epidural space i. Oral Corticosteroids
j. Brivudin
k. IV acyclovir

• although ABC is important and airway


management should be done first in patients with
1691. A 29yo Afro-Caribbean man altered mental status.
presents with a non-productive cough acyclovir should be considered for healthy
mild aches in the ankles. The symptoms persons at increased risk of severe varicella
have been present for 2m. His ESR is infections, most notably patients older than 12
elevated. Ca: 2.69 mmol/l; PO4 3-: 1.20 years. Oral acyclovir therapy in this population
mmol/l; ALP: 80 iu/L. Serum 25(OH) D: (800 mg 5 times/d for 7 d), begun within 24 hours
180 nmol/l of onset of symptoms, has been shown to
decrease the duration of lesions and pyrexia,
while reducing other symptoms and disease
Normal values for Calcium: 2.12-2.65mmol/l;
duration.
Phosphate: 0.8-1.45mmol/l; ALP 30-300iu/L;
Serum 25(OH) D: 20-105nmol/l; Urea: 2.5-
6.7mmol/l; Creatinine: 70-120μmol/l

a. Osteoporosis
b. Thiazide diuretics 1693. A young girl with a psychiatric hx on
c. Skeletal metastases med tx is brought to the dermatologist by
d. Primary hyperparathyroidism her mother because of recurrent patchy
e. Hypoparathyroidism hair loss. Exam: the hair shafts revealed
f. Osteomalacia twisting and fractures. This suggests the
g. Multiple myeloma
h. Paget's disease of bone
following pathology:
a. Infection with Trichophyton tonsurans
i. Sarcoidosis
b. Infection with Microsporum canis
j. Hyperthyroidism
c. Alopecia areata
d. Telogen Effluvium
Ans : i. Sarcoidosis e. Androgenetic Alopecia
f. Lichen planus
g. Traction Alopecia
h. Alopecia totalis 1696. A 7yo boy with frequent episodic
i. Trichorrhexis nodosa asthma is on tx with sodium
j. Trichotillomania
cromoglycate. His physician wants to add
a non-steroid preventer. The mother of
Ans : j. Trichotillomania the boy, a teacher, has just read about a
1694. Syphilis typically causes nonsteroidal medication which acts on the
a. Lymphogranuloma Venereum mast cells, stopping them from releasing
b. Testicular Torsion harmful chemicals. Her physician agrees
c. Hydrocele to add this medication to the boy's drug
d. Epididymitis regimen. Which medication is the
e. Seminoma physician most likely to add to the boy's
f. Mature teratoma
g. Varicocele
treatment?
h. Lymphoma a. Inhaled short acting
i. Orchitis b. bronchodilator
j. Spermatocele c. SC adrenaline
d. Nedocromil Sodium
e. Inhaled long acting
Ans : i.Orchitis f. bronchodilator
g. Inhaled sodium cromoglycate
Chlamydia and gonorrhea are implicated in a h. Inhaled steroids
number of above conditions but syphilis is kind of i. Inhaled SABA
eccentric disease j. Oral steroids
k. Nebulised bronchodilators
1695. A middle aged woman has severe l. Oral theophylline
collapse of the right femoral head
requiring replacement. The removed
Key : C
femoral head is sent for pathology and is
found to contain enlarged fat cells. The Investigations for asthma
pathologist explains that this is the likely Spirometry:
cause of the patient's femoral head
collapse. A likely aetiology is? FEV1:FVC ratio, A low value indicates that you
a. Septic emboli have narrowed airways which are typical in
b. Impaired venous drainage asthma Therefore, spirometry may be repeated
c. Hgb SS disease after treatment. An improvement in the value
d. Steroid use after treatment with a bronchodilator to open up
e. Alcoholism the airways is typical of asthma.
f. Gaucher's disease
g. missed fracture PEFR: morning readings are usually lower than
h. Cushing's disease evening readings in asthmatics
i. Radiation
j. Vasculitis

Gaucher's disease fits here in the scenario 1697. A 3yo boy is playing with his brother
although common causes are Steroid use and
when he falls. He cries immediately and
trauma.
refuses to walk. His mother carries him to
hospital. He had a full term NVD with no
neonatal complications. His
immunisations are up to date. Exam: looks vomiting. He was connected to vital
well and well-nourished, no dysmorphic monitors which were not reassuring.
features. He has slight swelling, warmth What is the management?
and discomfort on the lower 1/3 of the left a. Oral antacids
tibia, and refuses to weight bear. AP and b. IV PPI
lateral x rays of the tibia are normal. c. Oral PPI
What is the most likely dx? d. Endoscopy
a. Ankle fx e. Analgesia
b. Ankle sprain
c. Fibular fx
key)D
d. Knee dislocation
e. Tibial fx Ans) long term use of an nsaid predisposes to
peptic ulcers. the symptoms described are of a
Key)E possibly perforated peptic ulcer or acute upper Gi
bleed caused by the ulcer. The guidelines suggest
Ans)Toddler's fracture
that
• Undisplaced spiral fractures of the tibial
shaft in children under 7 years old often follow Endoscopy is the primary diagnostic investigation
minimal trauma and may not be visible on initial in patients with acute UGIB
X-ray.[8] • Endoscopy should be undertaken
• Can be difficult to diagnose but should be
immediately after resuscitation for unstable
suspected whenever a child presents with a limp
patients with severe acute UGIB.
or fails to bear weight on the leg.
• Treatment consists of immobilisation for • Endoscopy should be undertaken within
a few weeks to protect the limb and to relieve 24 hours of admission for all other patients with
pain. UGIB.
• Subperiosteal bone formation is usually
apparent on X-rays by two weeks.
1700. A 68yo man presents with bruising
and hx of falls. He is found to have a
1698. Which one of the following
mask-like face, pillrolling tremor and
electrocardiographic changes is found in
shuffling gait. EEG=normal. Which of the
hypercalcaemia?
a. Increased QRS interval following conditions is he most likely
b. Prolonged Q-T interval being treated for?
a. HTN
c. Short P-R interval
b. DM
d. Short Q-T interval
c. Psychosis
d. TIA
e. Complex partial seizure
key)D-- Short Q-T interval secondary to a
shortened ST segment
key) C
Ans)Fact
Ans) features are classic for parkinson’s disease.
1699. An elderly male pt with prior hx of and people with PD usualy go on to develop
hematemesis is having hx of long term use parkinson related dementia or psychosis.
Symptoms of Parkinsons:
of aspirin and other drugs, now presents
with severe epigastric pain, dysphagia and
• Slowness of movement (bradykinesia). banging his head on the steering wheel,
For example, it may become more of an effort to which revealed no abnormality. 6m
walk or to get up out of a chair. This is a 'shuffling'
following this episode his symptoms have
walk with some difficulty in starting, stopping,
and turning easily. resolved. What did his original symptoms
• Stiffness of muscles (rigidity), and muscles likely represent?
may feel more tense. Also, your arms do not tend a. Conversion disorder
to swing as much when you walk. b. PTSD
• Shaking (tremor) is common, but does c. Somatization disorder
not always occur. It typically affects the fingers, d. GAD
thumbs, hands, and arms, but can affect other
e. Post-concussion syndrome
parts of the body. It is most noticeable when you
are resting. It may become worse when you are
anxious or emotional. It tends to become less
when you use your hand to do something such as Ans. The key is E. Post concussion syndrome.
picking up an object. [Post concussion syndrome is a set of symptoms
that may continue for weeks, months, or a year or
it is diagnosed clinically and treatment includes more after a concussion – a minor form of
traumatic brain injury].
1. levodopa plus a dopa decarboxylase
inhibitor 1703. A 34yo man had a 4mm ureteric
2. dopamine agonists stone which he passed in urine. This time
3. anticholinergics (tremor) he presents withh 3cm stone in the right
4. MAO inhibitors kidney. Single most appropriate
treatment?
1701. A 45yo woman presents with easy a. No treatment
fatigability, even on no exertion, chronic b. ESWL
headaches and body aches and severe c. Laparotomy
physical and mental exhaustion. She has d. Observe
no underlying conditions and all inv are e. Operative stone removal
non-conclusive. What is the most likely Ans. The key is E. Operative stone removal.
dx?
a. Somatization [Stones < 5mm: pass spontaneously, Inc fluid
b. Chronic fatigure syndrome intake
c. Polymyalgia rheumatic Stones 5mm-1cm /pain not resolving: medical
d. GCA e. Depression expulsive therapy---> Nifedipine or
Tamsulosin(and/or prednisolone)
Stones 1cm-2cm: ESWL or Ureteroscopy using
dormia basket
Ans. The key is B. Chronic fatigue syndrome.
Stones > 2cm/large/multiple/complex:
[Chronic fatigue syndrome (CFS) causes persistent
Percutaneous nephrolithotomy].
fatigue (exhaustion) that affects everyday life and
doesn't go away with sleep or rest].
1704. An 18m girl who has had single UTI
1702. A 23yo male presents to his GP is seen in the OPD. She has fever and
2wks after a RTA concerned about vomiting but these improved with course
increasing anxiety lethargy and headache. of trimethoprim. Subsequently, MCUG
At the time he had a CT brain after
showed bilateral vesicoureteric reflux.
Single most appropriate mgmt?
a. Prophylactic antibiotics
b. Reassure
c. No treatment
d. Ureteric surgery

Ans. The key is D. Ureteric surgery. [For plab,


vur=surgery and recurrent uti=prophylactic
antibyotics].

1705. A 22yo says she has taken about 40


tabs of paracetamol 3h ago. Her
HR=110bpm, BP=110/80mmHg and
RR=22bpm. What’s the initial
management?
a. Activated charcoal
b. N-acetyl cysteine
c. Gastric lavage
d. Wait for 4h paracetamol level

Ans. The key is D. Wait for 4h paracetamol level.


This is a wrong key. Correct key is A. Activated
charcoal. [If comes before 4 hours give Activated
charcoal].

1706. A 35yo man skidded on a wet road


while riding his motorbike at a speed of
70mph. He has a large hematoma on
temporal scalp, some bruises on chest
wall and abdomen and a deformed thigh.
GCS 11/15. High flow oxygen via mask
given. Most immediate radiological inv
required during initial resuscitation
phase?
a. CXR
b. CT brain
c. CT abdomen
d. XR femur

Ans. The key is A. CXR. [CXR may reveal possible


chest structure trauma requiring urgent attention
which may be life saving while brain lesion and
fracture femur may be managed taking more
time].

You might also like